Π‘ΠΎΠ΄Π΅Ρ€ΠΆΠ°Π½ΠΈΠ΅

Π€ΠΎΡ€ΠΌΡƒΠ»Π° эквивалСнтного сопротивлСния рСзисторов: расчСт

Π§Ρ‚ΠΎΠ±Ρ‹ Π»ΡƒΡ‡ΡˆΠ΅ ΠΏΠΎΠ½ΡΡ‚ΡŒ элСктродинамику ΠΈΒ Ρ„ΠΈΠ·ΠΈΠΊΡƒ, ΡΠ°ΠΌΠΎΡΡ‚ΠΎΡΡ‚Π΅Π»ΡŒΠ½ΠΎ Π²Ρ‹ΠΏΠΎΠ»Π½ΡΡ‚ΡŒ ΠΏΡ€ΠΎΡΡ‚Π΅ΠΉΡˆΠΈΠ΅ Ρ€Π°Π±ΠΎΡ‚Ρ‹ Π²Β Π΄ΠΎΠΌΠ΅, Π½ΡƒΠΆΠ½ΠΎ Π·Π½Π°Ρ‚ΡŒ, ΠΊΠ°ΠΊΠΎΠ²Π° Ρ„ΠΎΡ€ΠΌΡƒΠ»Π° эквивалСнтного сопротивлСния ΠΈΒ Ρ‡Ρ‚ΠΎ ΠΎΠ±ΠΎΠ·Π½Π°Ρ‡Π°Π΅Ρ‚ это понятиС. ΠžΠ±Β ΡΡ‚ΠΎΠΌ ΠΈΒ Π΄Ρ€ΡƒΠ³ΠΎΠΌ Π΄Π°Π»Π΅Π΅ Π²Β ΡΡ‚Π°Ρ‚ΡŒΠ΅.

Π§Ρ‚ΠΎ Ρ‚Π°ΠΊΠΎΠ΅ эквивалСнтноС сопротивлСниС рСзисторов

Π’ΠΎΡ‡Π½ΠΎΠ³ΠΎ понятия Π²Β Ρ„ΠΈΠ·ΠΈΠΊΠ΅ нС сущСствуСт. Π•Π³ΠΎ ΠΌΠΎΠΆΠ½ΠΎ вывСсти Ρ‡Π΅Ρ€Π΅Π· ряд Π΄Ρ€ΡƒΠ³ΠΈΡ… Ρ‚Π΅Ρ€ΠΌΠΈΠ½ΠΎΠ² ΠΈΒ Ρ„ΠΎΡ€ΠΌΡƒΠ»ΠΈΡ€ΠΎΠ²ΠΊΡƒ Π·Π°ΠΊΠΎΠ½Π° Ома. Π’Β Ρ€Π΅Π·ΡƒΠ»ΡŒΡ‚Π°Ρ‚Π΅ получится, Ρ‡Ρ‚ΠΎ эквивалСнтноС сопротивлСниС рСзисторов — это суммарноС прСпятствиС взаимозамСняСмых пассивных элСмСнтов элСктричСской сСти, Ρ‡Ρ‚ΠΎΠ±Ρ‹ заряд ΠΏΡ€ΠΎΡ…ΠΎΠ΄ΠΈΠ» Π²Β ΠΏΡ€ΠΎΠ²ΠΎΠ΄Π½ΠΈΠΊ.

Π‘ΠΎΠΏΡ€ΠΎΡ‚ΠΈΠ²Π»ΡΠ΅ΠΌΠΎΡΡ‚ΡŒ рСзисторов

К свСдСнию! Один ΠΏΠΎΠΊΠ°Π·Π°Ρ‚Π΅Π»ΡŒ Π΄Π°Π΅Ρ‚ Π½Π°Β Π²Ρ‹Ρ…ΠΎΠ΄Π΅ Π·Π½Π°Ρ‡Π΅Π½ΠΈΠ΅ сопротивляСмости Π±Π΅Π· воздСйствия Π½Π°Β Π½Π΅Π³ΠΎ ряда посторонних ΠΌΠΎΠΌΠ΅Π½Ρ‚ΠΎΠ².

ΠŸΠΎΠ΄Ρ€ΠΎΠ±Π½ΠΎΠ΅ объяснСниС эквивалСнтному ΡΠΎΠΏΡ€ΠΎΡ‚ΠΈΠ²Π»Π΅Π½ΠΈΡŽ

Как ΠΎΠΏΡ€Π΅Π΄Π΅Π»ΠΈΡ‚ΡŒ эквивалСнтноС сопротивлСниС

Если в элСктричСской сСти находится нСсколько рСзистивных источников, Ρ‚ΠΎ, Ρ‡Ρ‚ΠΎΠ±Ρ‹ ΠΏΠΎΠ΄ΡΡ‡ΠΈΡ‚Π°Ρ‚ΡŒ силу Ρ‚ΠΎΠΊΠ°, напряТСния ΠΈΒ ΠΌΠΎΡ‰Π½ΠΎΡΡ‚ΡŒ, слСдуСт ΠΈΡΠΏΠΎΠ»ΡŒΠ·ΠΎΠ²Π°Ρ‚ΡŒ ΠΎΠ΄ΠΈΠ½ взаимозамСняСмый физичСский ΠΏΠΎΠΊΠ°Π·Π°Ρ‚Π΅Π»ΡŒ сопротивлСния элСктричСской Ρ†Π΅ΠΏΠΈ.

Π›ΡŽΠ±ΠΎΠΉ ΠΏΠΎΠΊΠ°Π·Π°Ρ‚Π΅Π»ΡŒ ΠΏΠΎΡΠ»Π΅Π΄ΠΎΠ²Π°Ρ‚Π΅Π»ΡŒΠ½ΠΎΠ³ΠΎ ΠΈΠ»ΠΈ ΠΏΠ°Ρ€Π°Π»Π»Π΅Π»ΡŒΠ½ΠΎΠ³ΠΎ ΠΏΠΎΠ΄ΠΊΠ»ΡŽΡ‡Π΅Π½ΠΈΡ Π²ΠΎΠ·ΠΌΠΎΠΆΠ½ΠΎ ΠΏΡ€Π΅ΠΎΠ±Ρ€Π°Π·ΠΎΠ²Π°Ρ‚ΡŒ, ΠΈΡΠΏΠΎΠ»ΡŒΠ·ΡƒΡ эквивалСнтный рСзистор ΠΈΒ ΠΎΠ΄ΠΈΠ½ источник элСктродвиТущСй силы. Π‘ΠΎΠΏΡ€ΠΎΡ‚ΠΈΠ²Π»ΡΠ΅ΠΌΠΎΡΡ‚ΡŒ Π²Β Π΄Π°Π½Π½ΠΎΠΌ случаС Π±ΡƒΠ΄Π΅Ρ‚ Ρ€Π°Π²Π½Π° суммС всСх прСпятствий пассивных устройств заряду элСктричСской сСти. ЭлСктродвиТущая сила взаимозамСняСмого источника Π±ΡƒΠ΄Π΅Ρ‚ Ρ€Π°Π²Π½Π° суммС всСх источников, ΠΊΠΎΡ‚ΠΎΡ€Ρ‹Π΅ входят Π²Β Ρ†Π΅ΠΏΡŒ.

Π€ΠΎΡ€ΠΌΡƒΠ»Π° опрСдСлСния показатСля

ΠžΠ±Ρ€Π°Ρ‚ΠΈΡ‚Π΅ Π²Π½ΠΈΠΌΠ°Π½ΠΈΠ΅!Β Π‘Π²ΠΎΡ€Π°Ρ‡ΠΈΠ²Π°Π½ΠΈΠ΅ΠΌ Ρ†Π΅ΠΏΠΈ, ΠΈΡΠΏΠΎΠ»ΡŒΠ·ΡƒΡ прСобразования ΠΏΠΎΡΠ»Π΅Π΄ΠΎΠ²Π°Ρ‚Π΅Π»ΡŒΠ½ΠΎ ΠΏΠΎΠ΄ΠΊΠ»ΡŽΡ‡Π΅Π½Π½Ρ‹Ρ… ΠΈΠ»ΠΈ ΠΏΠ°Ρ€Π°Π»Π»Π΅Π»ΡŒΠ½Ρ‹Ρ… ΠΏΡ€ΠΎΠ²ΠΎΠ΄Π½ΠΈΠΊΠΎΠ²Ρ‹Ρ… ΠΏΡ€ΠΈΠ±ΠΎΡ€ΠΎΠ², ΠΌΠΎΠΆΠ½ΠΎ по максимуму ΡΠ΄Π΅Π»Π°Ρ‚ΡŒ ΠΏΡ€ΠΎΡ‰Π΅ дальнСйший расчСт в любой схСмС. Π˜ΡΠΊΠ»ΡŽΡ‡Π΅Π½ΠΈΠ΅ΠΌ Π±ΡƒΠ΄ΡƒΡ‚ Π²Ρ‹ΡΡ‚ΡƒΠΏΠ°Ρ‚ΡŒ Ρ†Π΅ΠΏΠΈ, ΠΊΠΎΡ‚ΠΎΡ€Ρ‹Π΅ содСрТат ΡΠΎΠΏΡ€ΠΎΡ‚ΠΈΠ²Π»ΡΠ΅ΠΌΠΎΡΡ‚ΡŒ по схСмС Π²Β Π²ΠΈΠ΄Π΅ Π·Π²Π΅Π·Π΄Ρ‹ ΠΈΒ Ρ‚Ρ€Π΅ΡƒΠ³ΠΎΠ»ΡŒΠ½ΠΈΠΊΠ°.

ΠŸΠ°Ρ€Π°Π»Π»Π΅Π»ΡŒΠ½ΠΎΠ΅ ΠΈΒ ΠΏΠΎΡΠ»Π΅Π΄ΠΎΠ²Π°Ρ‚Π΅Π»ΡŒΠ½ΠΎΠ΅ соСдинСниС элСмСнтов

Π’Β Ρ€Π°Π·Π΄Π΅Π»Π΅ элСктротСхники присутствуСт нСсколько Π²Π°Ρ€ΠΈΠ°Π½Ρ‚ΠΎΠ² Ρ‚ΠΎΠ³ΠΎ, ΠΊΠ°ΠΊ ΠΏΠΎΠ΄ΠΊΠ»ΡŽΡ‡ΠΈΡ‚ΡŒ Π΄Π΅Ρ‚Π°Π»ΠΈ Π²Β ΡΠ»Π΅ΠΊΡ‚Ρ€ΠΈΡ‡Π΅ΡΠΊΡƒΡŽ Ρ†Π΅ΠΏΡŒ. Π•ΡΡ‚ΡŒ ΠΏΠ°Ρ€Π°Π»Π»Π΅Π»ΡŒΠ½ΠΎΠ΅ ΠΈΒ ΠΏΠΎΠΏΠ΅Ρ€Π΅ΠΌΠ΅Π½Π½ΠΎΠ΅ подсоСдинСния. Π˜Ρ…Β ΠΎΠ±ΡŠΠ΅Π΄ΠΈΠ½ΡΠ΅Ρ‚ смСшанная схСма, которая прСдставлСна Π½ΠΈΠΆΠ΅.

ΠŸΠΎΡΠ»Π΅Π΄ΠΎΠ²Π°Ρ‚Π΅Π»ΡŒΠ½ΠΎΠ΅ ΠΏΠΎΠ΄ΠΊΠ»ΡŽΡ‡Π΅Π½ΠΈΠ΅Β β€” это ΠΊΠΎΠ³Π΄Π° всС источники ΡΠΎΠ΅Π΄ΠΈΠ½ΡΡŽΡ‚ΡΡ Π΄Ρ€ΡƒΠ³ с другом ΠΏΠΎΡΠ»Π΅Π΄ΠΎΠ²Π°Ρ‚Π΅Π»ΡŒΠ½ΠΎ. ΠŸΠΎΠ»ΡƒΡ‡Π°Π΅ΠΌΠ°Ρ Ρ†Π΅ΠΏΡŒ Π½Π΅Β ΠΎΠ±Π»Π°Π΄Π°Π΅Ρ‚ Π½ΠΈΠΊΠ°ΠΊΠΈΠΌΠΈ развСтвлСниями. Π‘ΠΈΠ»Π° Ρ‚ΠΎΠΊΠ° Π²Β Π΄Π°Π½Π½ΠΎΠΌ случаС ΠΏΡ€ΠΎΡ…ΠΎΠ΄ΠΈΡ‚ Ρ‡Π΅Ρ€Π΅Π· ΠΊΠ°ΠΆΠ΄Ρ‹ΠΉ источник. Она постоянная, ΠΎΠ±Ρ‰Π΅Π΅ напряТСниС ΠΎΠ΄ΠΈΠ½Π°ΠΊΠΎΠ²ΠΎΠ΅.

В случаС прСпятствия рСзисторов заряду ΠΏΡ€ΠΈ ΠΏΠΎΡΠ»Π΅Π΄ΠΎΠ²Π°Ρ‚Π΅Π»ΡŒΠ½ΠΎΠΌ ΠΏΠΎΠ΄ΠΊΠ»ΡŽΡ‡Π΅Π½ΠΈΠΈ получится, Ρ‡Ρ‚ΠΎ ΡΠΎΠΏΡ€ΠΎΡ‚ΠΈΠ²Π»ΡΠ΅ΠΌΠΎΡΡ‚ΡŒ Π±ΡƒΠ΄Π΅Ρ‚ Ρ€Π°Π²Π½Π° суммС всСх взаимозамСняСмых пассивных элСмСнтов Ρ†Π΅ΠΏΠΈ. Рассчитывая ΠΏΠ°Ρ€Π°ΠΌΠ΅Ρ‚Ρ€Ρ‹ элСктротСхничСской схСмы, Π½Π΅Β Π½ΡƒΠΆΠ½ΠΎ ΠΏΡ€ΠΈΠΌΠ΅Π½ΡΡ‚ΡŒ частныС ΠΏΠ°Ρ€Π°ΠΌΠ΅Ρ‚Ρ€Ρ‹ устройств. Π˜Ρ…Β ΠΌΠΎΠΆΠ½ΠΎ Π·Π°ΠΌΠ΅Π½ΠΈΡ‚ΡŒ ΠΎΠ΄Π½ΠΈΠΌ Π·Π½Π°Ρ‡Π΅Π½ΠΈΠ΅ΠΌ, ΠΊΠΎΡ‚ΠΎΡ€ΠΎΠ΅ Ρ€Π°Π²Π½ΠΎ их суммарному ΠΏΠΎΠΊΠ°Π·Π°Ρ‚Π΅Π»ΡŽ.

ΠžΠ±Ρ€Π°Ρ‚ΠΈΡ‚Π΅ Π²Π½ΠΈΠΌΠ°Π½ΠΈΠ΅! Польза взаимозамСняСмости ΠΊΠΎΠΌΠΏΠΎΠ½Π΅Π½Ρ‚ΠΎΠ² Π·Π°ΠΊΠ»ΡŽΡ‡Π°Π΅Ρ‚ΡΡ в возмоТности Π·Π°ΠΌΠ΅Π½Ρ‹ Π½Π΅ΡΠΊΠΎΠ»ΡŒΠΊΠΈΡ… пассивных элСмСнтов элСктричСской сСти ΠΎΠ΄Π½ΠΈΠΌ.

Π‘ΠΎΠ΅Π΄ΠΈΠ½Π΅Π½ΠΈΠ΅ элСмСнтов

ΠŸΠ°Ρ€Π°Π»Π»Π΅Π»ΡŒΠ½ΠΎΠ΅ ΠΏΠΎΠ΄ΠΊΠ»ΡŽΡ‡Π΅Π½ΠΈΠ΅Β β€” это Ρ‚Π°ΠΊΠΎΠ΅ подсоСдинСниС источников, Π²Β ΠΊΠΎΡ‚ΠΎΡ€ΠΎΠΌ Π²Ρ…ΠΎΠ΄Ρ‹ всСх устройств находятся Π²Β ΠΎΠ΄Π½ΠΈΡ… мСстах, Π°Β Π²Ρ‹Ρ…ΠΎΠ΄Ρ‹Β β€” Π²Β Π΄Ρ€ΡƒΠ³ΠΈΡ…. Π­Ρ‚ΠΈΠΌΠΈ мСстами слуТат ΡƒΠ·Π»Ρ‹.

В случаС эквивалСнтного прСпятствия заряду ΠΏΡ€ΠΈ ΠΏΠ°Ρ€Π°Π»Π»Π΅Π»ΡŒΠ½ΠΎΠΌ соСдинСнии ΠΎΠΏΡ€Π΅Π΄Π΅Π»ΠΈΡ‚ΡŒ Π΅Π³ΠΎ ΠΌΠΎΠΆΠ½ΠΎ благодаря Π·Π°ΠΊΠΎΠ½Ρƒ Ома с прСобразованиСм Ρ„ΠΎΡ€ΠΌΡƒΠ»ΠΈΡ€ΠΎΠ²ΠΊΠΈ подсчСта. Π’Π°ΠΊ, ΡΠ΄Π΅Π»Π°Ρ‚ΡŒ Π½Π΅ΠΎΠ±Ρ…ΠΎΠ΄ΠΈΠΌΡ‹ΠΉ расчСт ΠΌΠΎΠΆΠ½ΠΎ, ΠΎΡΠ½ΠΎΠ²Ρ‹Π²Π°ΡΡΡŒ Π½Π°Β ΡΠ»Π΅Π΄ΡƒΡŽΡ‰Π΅ΠΉ Ρ„ΠΎΡ€ΠΌΡƒΠ»Π΅: R Β· RΒ / NΒ·R = RΒ / N.

Если это соСдинСниС Π½Π΅ΡΠΊΠΎΠ»ΡŒΠΊΠΈΡ… ΠΈΠ½Π΄ΡƒΠΊΡ‚ΠΈΠ²Π½Ρ‹Ρ… ΠΊΠ°Ρ‚ΡƒΡˆΠ΅ΠΊ, Ρ‚ΠΎΒ ΠΈΡ…Β ΠΈΠ½Π΄ΡƒΠΊΡ‚ΠΈΠ²Π½Ρ‹ΠΉ ΠΏΠΎΠΊΠ°Π·Π°Ρ‚Π΅Π»ΡŒ сопротивляСмости Π±ΡƒΠ΄Π΅Ρ‚ Ρ€Π°ΡΡΡ‡ΠΈΡ‚Ρ‹Π²Π°Ρ‚ΡŒΡΡ ΠΏΠΎΒ Ρ‚ΠΎΠΉΒ ΠΆΠ΅ Ρ„ΠΎΡ€ΠΌΡƒΠ»Π΅, Ρ‡Ρ‚ΠΎ для рСзисторных устройств.

Π’Π°ΠΆΠ½ΠΎ! В случаС ΡΒ ΠΏΠ°Ρ€Π°Π»Π»Π΅Π»ΡŒΠ½Ρ‹ΠΌ ΠΏΠΎΠ΄ΠΊΠ»ΡŽΡ‡Π΅Π½ΠΈΠ΅ΠΌ ΠΎΠ±Ρ‰ΠΈΠΉ ΠΏΠΎΠΊΠ°Π·Π°Ρ‚Π΅Π»ΡŒ Π±ΡƒΠ΄Π΅Ρ‚ мСньшС любого показатСля рСзистора. ΠŸΡ€ΠΈ ΠΏΠΎΡΠ»Π΅Π΄ΠΎΠ²Π°Ρ‚Π΅Π»ΡŒΠ½ΠΎΠΌ подсоСдинСнии всС Π½Π°ΠΎΠ±ΠΎΡ€ΠΎΡ‚.

Как ΠΏΡ€Π°Π²ΠΈΠ»ΡŒΠ½ΠΎ Ρ€Π°ΡΡΡ‡ΠΈΡ‚Π°Ρ‚ΡŒ ΠΏΡ€ΠΈ смСшанном соСдинСнии устройств

Π‘ΠΌΠ΅ΡˆΠ°Π½Π½Ρ‹ΠΌ ΠΏΠΎΠ΄ΠΊΠ»ΡŽΡ‡Π΅Π½ΠΈΠ΅ΠΌ устройств называСтся Ρ‚Π°ΠΊΠΎΠΉ Ρ‚ΠΈΠΏ, ΠΏΡ€ΠΈ ΠΊΠΎΡ‚ΠΎΡ€ΠΎΠΌ Ρ‡Π°ΡΡ‚ΡŒ взаимозамСняСмых ΠΊΠΎΠΌΠΏΠΎΠ½Π΅Π½Ρ‚ΠΎΠ² ΠΏΠΎΠ΄ΠΊΠ»ΡŽΡ‡Π°Π΅Ρ‚ΡΡ ΠΏΠΎΡΠ»Π΅Π΄ΠΎΠ²Π°Ρ‚Π΅Π»ΡŒΠ½ΠΎ, Π°Β Ρ‡Π°ΡΡ‚ΡŒΒ β€” ΠΏΠ°Ρ€Π°Π»Π»Π΅Π»ΡŒΠ½ΠΎ. ΠŸΡ€ΠΈ смСшанном подсоСдинСнии устройств ΠΎΠΏΡ€Π΅Π΄Π΅Π»ΠΈΡ‚ΡŒ эквивалСнтный ΠΏΠΎΠΊΠ°Π·Π°Ρ‚Π΅Π»ΡŒ сопротивляСмости нСслоТно. Достаточно ΠΈΡΠΏΠΎΠ»ΡŒΠ·ΠΎΠ²Π°Ρ‚ΡŒ ΡΠ»Π΅Π΄ΡƒΡŽΡ‰ΡƒΡŽ Ρ„ΠΎΡ€ΠΌΡƒΠ»Ρƒ: (R1 + R2) R3Β / (R1 + R2 + R3) + R4.

Π­Ρ‚ΠΎ соСдинСниС ΠΈΡΠΏΠΎΠ»ΡŒΠ·ΡƒΠ΅Ρ‚ΡΡ, Ρ‡Ρ‚ΠΎΠ±Ρ‹ ΠΈΠ·ΠΌΠ΅Π½ΠΈΡ‚ΡŒ ΡΠΎΠΏΡ€ΠΎΡ‚ΠΈΠ²Π»ΡΠ΅ΠΌΠΎΡΡ‚ΡŒ в пусковых рСостатах, ΠΏΠΈΡ‚Π°ΡŽΡ‰ΠΈΡ…ΡΡ от постоянного Ρ‚ΠΎΠΊΠ°. Для подсчСта ΠΈΡΠΏΠΎΠ»ΡŒΠ·ΡƒΡŽΡ‚ΡΡ ΡΠΏΠ΅Ρ†ΠΈΠ°Π»ΡŒΠ½Ρ‹Π΅ ΠΎΠ½Π»Π°ΠΉΠ½-сСрвисы. Π­Ρ‚ΠΎ ΠΏΠΎΠΌΠΎΠ³Π°Π΅Ρ‚ быстрСС Π²Ρ‹Ρ‡ΠΈΡΠ»ΠΈΡ‚ΡŒ, ΡƒΠΏΡ€ΠΎΡΡ‚ΠΈΡ‚ΡŒ ΠΈΒ ΡƒΡΠΊΠΎΡ€ΠΈΡ‚ΡŒ расчСты элСктротСхничСских ΠΏΠ°Ρ€Π°ΠΌΠ΅Ρ‚Ρ€ΠΎΠ².

Π€ΠΎΡ€ΠΌΡƒΠ»Π° расчСта ΠΏΡ€ΠΈ смСшанном соСдинСнии устройств

Π’Β Ρ€Π΅Π·ΡƒΠ»ΡŒΡ‚Π°Ρ‚Π΅, Ρ‡Ρ‚ΠΎΠ±Ρ‹ Ρ€Π°ΡΡΡ‡ΠΈΡ‚Π°Ρ‚ΡŒ эквивалСнтноС сопротивлСниС Ρ†Π΅ΠΏΠΈ, Π½Π΅ΠΎΠ±Ρ…ΠΎΠ΄ΠΈΠΌΠΎ Π²ΡΠΏΠΎΠΌΠ½ΠΈΡ‚ΡŒ ΠΏΡ€ΠΎ Π·Π°ΠΊΠΎΠ½ Ома ΠΈΒ ΠΎΠ±ΡΠ·Π°Ρ‚Π΅Π»ΡŒΠ½ΠΎ ΠΏΠΎΠ»ΡŒΠ·ΠΎΠ²Π°Ρ‚ΡŒΡΡ ΡƒΠΊΠ°Π·Π°Π½Π½Ρ‹ΠΌΠΈ Ρ„ΠΎΡ€ΠΌΡƒΠ»Π°ΠΌΠΈ Π²Ρ‹ΡˆΠ΅. Волько ΠΏΡ€ΠΈ смСшСнном Ρ‚ΠΈΠΏΠ΅ соСдинСния ΠΆΠ΅Π»Π°Ρ‚Π΅Π»ΡŒΠ½ΠΎ вСсти подсчСты Π²Β ΠΎΠ½Π»Π°ΠΉΠ½-ΠΊΠ°Π»ΡŒΠΊΡƒΠ»ΡΡ‚ΠΎΡ€Π°Ρ…, Ρ‚Π°ΠΊ ΠΊΠ°ΠΊ Π΅ΡΡ‚ΡŒ риск Π΄ΠΎΠΏΡƒΡΡ‚ΠΈΡ‚ΡŒ ΠΎΡˆΠΈΠ±ΠΊΡƒ в расчСтах.

Как Π½Π°ΠΉΡ‚ΠΈ эквивалСнтноС сопротивлСниС Ρ†Π΅ΠΏΠΈ Ρ„ΠΎΡ€ΠΌΡƒΠ»Π°

БопротивлСния Π² элСктричСских цСпях ΠΌΠΎΠ³ΡƒΡ‚ Π±Ρ‹Ρ‚ΡŒ соСдинСны ΠΏΠΎΡΠ»Π΅Π΄ΠΎΠ²Π°Ρ‚Π΅Π»ΡŒΠ½ΠΎ, ΠΏΠ°Ρ€Π°Π»Π»Π΅Π»ΡŒΠ½ΠΎ, ΠΏΠΎ смСшанной схСмС ΠΈ ΠΏΠΎ схСмам Β«Π·Π²Π΅Π·Π΄Π°Β», Β«Ρ‚Ρ€Π΅ΡƒΠ³ΠΎΠ»ΡŒΠ½ΠΈΠΊΒ». РасчСт слоТной схСмы упрощаСтся, Ссли сопротивлСния Π² этой схСмС Π·Π°ΠΌΠ΅Π½ΡΡŽΡ‚ΡΡ ΠΎΠ΄Π½ΠΈΠΌ эквивалСнтным сопротивлСниСм Rэкв, ΠΈ вся схСма прСдставляСтся Π² Π²ΠΈΠ΄Π΅ схСмы Π½Π° рис. 1.3, Π³Π΄Π΅ R=Rэкв, Π° расчСт Ρ‚ΠΎΠΊΠΎΠ² ΠΈ напряТСний производится с ΠΏΠΎΠΌΠΎΡ‰ΡŒΡŽ Π·Π°ΠΊΠΎΠ½ΠΎΠ² Ома ΠΈ ΠšΠΈΡ€Ρ…Π³ΠΎΡ„Π°.

ЭлСктричСская Ρ†Π΅ΠΏΡŒ с ΠΏΠΎΡΠ»Π΅Π΄ΠΎΠ²Π°Ρ‚Π΅Π»ΡŒΠ½Ρ‹ΠΌ соСдинСниСм элСмСнтов

Рис. 1.4

Рис. 1.5

ΠŸΠΎΡΠ»Π΅Π΄ΠΎΠ²Π°Ρ‚Π΅Π»ΡŒΠ½Ρ‹ΠΌ Π½Π°Π·Ρ‹Π²Π°ΡŽΡ‚ Ρ‚Π°ΠΊΠΎΠ΅ соСдинСниС элСмСнтов Ρ†Π΅ΠΏΠΈ, ΠΏΡ€ΠΈ ΠΊΠΎΡ‚ΠΎΡ€ΠΎΠΌ Π²ΠΎ всСх Π²ΠΊΠ»ΡŽΡ‡Π΅Π½Π½Ρ‹Ρ… Π² Ρ†Π΅ΠΏΡŒ элСмСнтах Π²ΠΎΠ·Π½ΠΈΠΊΠ°Π΅Ρ‚ ΠΎΠ΄ΠΈΠ½ ΠΈ Ρ‚ΠΎΡ‚ ΠΆΠ΅ Ρ‚ΠΎΠΊ I (рис. 1.4).

На основании Π²Ρ‚ΠΎΡ€ΠΎΠ³ΠΎ Π·Π°ΠΊΠΎΠ½Π° ΠšΠΈΡ€Ρ…Π³ΠΎΡ„Π° (1.5) ΠΎΠ±Ρ‰Π΅Π΅ напряТСниС U всСй Ρ†Π΅ΠΏΠΈ Ρ€Π°Π²Π½ΠΎ суммС напряТСний Π½Π° ΠΎΡ‚Π΄Π΅Π»ΡŒΠ½Ρ‹Ρ… участках:

Π’Π°ΠΊΠΈΠΌ ΠΎΠ±Ρ€Π°Π·ΠΎΠΌ, ΠΏΡ€ΠΈ ΠΏΠΎΡΠ»Π΅Π΄ΠΎΠ²Π°Ρ‚Π΅Π»ΡŒΠ½ΠΎΠΌ соСдинСнии элСмСнтов Ρ†Π΅ΠΏΠΈ ΠΎΠ±Ρ‰Π΅Π΅ эквивалСнтноС сопротивлСниС Ρ†Π΅ΠΏΠΈ Ρ€Π°Π²Π½ΠΎ арифмСтичСской суммС сопротивлСний ΠΎΡ‚Π΄Π΅Π»ΡŒΠ½Ρ‹Ρ… участков. Π‘Π»Π΅Π΄ΠΎΠ²Π°Ρ‚Π΅Π»ΡŒΠ½ΠΎ, Ρ†Π΅ΠΏΡŒ с Π»ΡŽΠ±Ρ‹ΠΌ числом ΠΏΠΎΡΠ»Π΅Π΄ΠΎΠ²Π°Ρ‚Π΅Π»ΡŒΠ½ΠΎ Π²ΠΊΠ»ΡŽΡ‡Π΅Π½Π½Ρ‹Ρ… сопротивлСний ΠΌΠΎΠΆΠ½ΠΎ Π·Π°ΠΌΠ΅Π½ΠΈΡ‚ΡŒ простой Ρ†Π΅ΠΏΡŒΡŽ с ΠΎΠ΄Π½ΠΈΠΌ эквивалСнтным сопротивлСниСм Rэкв (рис. 1.5). ПослС этого расчСт Ρ†Π΅ΠΏΠΈ сводится ΠΊ ΠΎΠΏΡ€Π΅Π΄Π΅Π»Π΅Π½ΠΈΡŽ Ρ‚ΠΎΠΊΠ° I всСй Ρ†Π΅ΠΏΠΈ ΠΏΠΎ Π·Π°ΠΊΠΎΠ½Ρƒ Ома

,

ΠΈ ΠΏΠΎ Π²Ρ‹ΡˆΠ΅ΠΏΡ€ΠΈΠ²Π΅Π΄Π΅Π½Π½Ρ‹ΠΌ Ρ„ΠΎΡ€ΠΌΡƒΠ»Π°ΠΌ Ρ€Π°ΡΡΡ‡ΠΈΡ‚Ρ‹Π²Π°ΡŽΡ‚ ΠΏΠ°Π΄Π΅Π½ΠΈΠ΅ напряТСний U

1, U2, U3 Π½Π° ΡΠΎΠΎΡ‚Π²Π΅Ρ‚ΡΡ‚Π²ΡƒΡŽΡ‰ΠΈΡ… участках элСктричСской Ρ†Π΅ΠΏΠΈ (рис. 1.4).

НСдостаток ΠΏΠΎΡΠ»Π΅Π΄ΠΎΠ²Π°Ρ‚Π΅Π»ΡŒΠ½ΠΎΠ³ΠΎ Π²ΠΊΠ»ΡŽΡ‡Π΅Π½ΠΈΡ элСмСнтов Π·Π°ΠΊΠ»ΡŽΡ‡Π°Π΅Ρ‚ΡΡ Π² Ρ‚ΠΎΠΌ, Ρ‡Ρ‚ΠΎ ΠΏΡ€ΠΈ Π²Ρ‹Ρ…ΠΎΠ΄Π΅ ΠΈΠ· строя хотя Π±Ρ‹ ΠΎΠ΄Π½ΠΎΠ³ΠΎ элСмСнта, прСкращаСтся Ρ€Π°Π±ΠΎΡ‚Π° всСх ΠΎΡΡ‚Π°Π»ΡŒΠ½Ρ‹Ρ… элСмСнтов Ρ†Π΅ΠΏΠΈ.

ЭлСктричСская Ρ†Π΅ΠΏΡŒ с ΠΏΠ°Ρ€Π°Π»Π»Π΅Π»ΡŒΠ½Ρ‹ΠΌ соСдинСниСм элСмСнтов

ΠŸΠ°Ρ€Π°Π»Π»Π΅Π»ΡŒΠ½Ρ‹ΠΌ Π½Π°Π·Ρ‹Π²Π°ΡŽΡ‚ Ρ‚Π°ΠΊΠΎΠ΅ соСдинСниС, ΠΏΡ€ΠΈ ΠΊΠΎΡ‚ΠΎΡ€ΠΎΠΌ всС Π²ΠΊΠ»ΡŽΡ‡Π΅Π½Π½Ρ‹Π΅ Π² Ρ†Π΅ΠΏΡŒ ΠΏΠΎΡ‚Ρ€Π΅Π±ΠΈΡ‚Π΅Π»ΠΈ элСктричСской энСргии, находятся ΠΏΠΎΠ΄ ΠΎΠ΄Π½ΠΈΠΌ ΠΈ Ρ‚Π΅ΠΌ ΠΆΠ΅ напряТСниСм (рис. 1.6).

Π’ этом случаС ΠΎΠ½ΠΈ присоСдинСны ΠΊ Π΄Π²ΡƒΠΌ ΡƒΠ·Π»Π°ΠΌ Ρ†Π΅ΠΏΠΈ Π° ΠΈ b, ΠΈ Π½Π° основании ΠΏΠ΅Ρ€Π²ΠΎΠ³ΠΎ Π·Π°ΠΊΠΎΠ½Π° ΠšΠΈΡ€Ρ…Π³ΠΎΡ„Π° (1.3) ΠΌΠΎΠΆΠ½ΠΎ Π·Π°ΠΏΠΈΡΠ°Ρ‚ΡŒ, Ρ‡Ρ‚ΠΎ ΠΎΠ±Ρ‰ΠΈΠΉ Ρ‚ΠΎΠΊ I всСй Ρ†Π΅ΠΏΠΈ Ρ€Π°Π²Π΅Π½ алгСбраичСской суммС Ρ‚ΠΎΠΊΠΎΠ² ΠΎΡ‚Π΄Π΅Π»ΡŒΠ½Ρ‹Ρ… Π²Π΅Ρ‚Π²Π΅ΠΉ:

I = I1 + I2 + I3, Ρ‚.Π΅. ,

ΠΎΡ‚ΠΊΡƒΠ΄Π° слСдуСт, Ρ‡Ρ‚ΠΎ

.

Π’ Ρ‚ΠΎΠΌ случаС, ΠΊΠΎΠ³Π΄Π° ΠΏΠ°Ρ€Π°Π»Π»Π΅Π»ΡŒΠ½ΠΎ Π²ΠΊΠ»ΡŽΡ‡Π΅Π½Ρ‹ Π΄Π²Π° сопротивлСния R1 ΠΈ R2, ΠΎΠ½ΠΈ Π·Π°ΠΌΠ΅Π½ΡΡŽΡ‚ΡΡ ΠΎΠ΄Π½ΠΈΠΌ эквивалСнтным сопротивлСниСм

.

Из ΡΠΎΠΎΡ‚Π½ΠΎΡˆΠ΅Π½ΠΈΡ (1.6), слСдуСт, Ρ‡Ρ‚ΠΎ эквивалСнтная ΠΏΡ€ΠΎΠ²ΠΎΠ΄ΠΈΠΌΠΎΡΡ‚ΡŒ Ρ†Π΅ΠΏΠΈ Ρ€Π°Π²Π½Π° арифмСтичСской суммС проводимостСй ΠΎΡ‚Π΄Π΅Π»ΡŒΠ½Ρ‹Ρ… Π²Π΅Ρ‚Π²Π΅ΠΉ:

По ΠΌΠ΅Ρ€Π΅ роста числа ΠΏΠ°Ρ€Π°Π»Π»Π΅Π»ΡŒΠ½ΠΎ Π²ΠΊΠ»ΡŽΡ‡Π΅Π½Π½Ρ‹Ρ… ΠΏΠΎΡ‚Ρ€Π΅Π±ΠΈΡ‚Π΅Π»Π΅ΠΉ ΠΏΡ€ΠΎΠ²ΠΎΠ΄ΠΈΠΌΠΎΡΡ‚ΡŒ Ρ†Π΅ΠΏΠΈ gэкв возрастаСт, ΠΈ Π½Π°ΠΎΠ±ΠΎΡ€ΠΎΡ‚, ΠΎΠ±Ρ‰Π΅Π΅ сопротивлСниС Rэкв ΡƒΠΌΠ΅Π½ΡŒΡˆΠ°Π΅Ρ‚ΡΡ.

НапряТСния Π² элСктричСской Ρ†Π΅ΠΏΠΈ с ΠΏΠ°Ρ€Π°Π»Π»Π΅Π»ΡŒΠ½ΠΎ соСдинСнными сопротивлСниями (рис. 1.6)

ΠžΡ‚ΡΡŽΠ΄Π° слСдуСт, Ρ‡Ρ‚ΠΎ

,

Ρ‚.Π΅. Ρ‚ΠΎΠΊ Π² Ρ†Π΅ΠΏΠΈ распрСдСляСтся ΠΌΠ΅ΠΆΠ΄Ρƒ ΠΏΠ°Ρ€Π°Π»Π»Π΅Π»ΡŒΠ½Ρ‹ΠΌΠΈ вСтвями ΠΎΠ±Ρ€Π°Ρ‚Π½ΠΎ ΠΏΡ€ΠΎΠΏΠΎΡ€Ρ†ΠΈΠΎΠ½Π°Π»ΡŒΠ½ΠΎ ΠΈΡ… сопротивлСниям.

По ΠΏΠ°Ρ€Π°Π»Π»Π΅Π»ΡŒΠ½ΠΎ Π²ΠΊΠ»ΡŽΡ‡Π΅Π½Π½ΠΎΠΉ схСмС Ρ€Π°Π±ΠΎΡ‚Π°ΡŽΡ‚ Π² номинальном Ρ€Π΅ΠΆΠΈΠΌΠ΅ ΠΏΠΎΡ‚Ρ€Π΅Π±ΠΈΡ‚Π΅Π»ΠΈ любой мощности, рассчитанныС Π½Π° ΠΎΠ΄Π½ΠΎ ΠΈ Ρ‚ΠΎ ΠΆΠ΅ напряТСниС. ΠŸΡ€ΠΈΡ‡Π΅ΠΌ Π²ΠΊΠ»ΡŽΡ‡Π΅Π½ΠΈΠ΅ ΠΈΠ»ΠΈ ΠΎΡ‚ΠΊΠ»ΡŽΡ‡Π΅Π½ΠΈΠ΅ ΠΎΠ΄Π½ΠΎΠ³ΠΎ ΠΈΠ»ΠΈ Π½Π΅ΡΠΊΠΎΠ»ΡŒΠΊΠΈΡ… ΠΏΠΎΡ‚Ρ€Π΅Π±ΠΈΡ‚Π΅Π»Π΅ΠΉ Π½Π΅ отраТаСтся Π½Π° Ρ€Π°Π±ΠΎΡ‚Π΅ ΠΎΡΡ‚Π°Π»ΡŒΠ½Ρ‹Ρ…. ΠŸΠΎΡΡ‚ΠΎΠΌΡƒ эта схСма являСтся основной схСмой ΠΏΠΎΠ΄ΠΊΠ»ΡŽΡ‡Π΅Π½ΠΈΡ ΠΏΠΎΡ‚Ρ€Π΅Π±ΠΈΡ‚Π΅Π»Π΅ΠΉ ΠΊ источнику элСктричСской энСргии.

ЭлСктричСская Ρ†Π΅ΠΏΡŒ со ΡΠΌΠ΅ΡˆΠ°Π½Π½Ρ‹ΠΌ соСдинСниСм элСмСнтов

Π‘ΠΌΠ΅ΡˆΠ°Π½Π½Ρ‹ΠΌ называСтся Ρ‚Π°ΠΊΠΎΠ΅ соСдинСниС, ΠΏΡ€ΠΈ ΠΊΠΎΡ‚ΠΎΡ€ΠΎΠΌ Π² Ρ†Π΅ΠΏΠΈ ΠΈΠΌΠ΅ΡŽΡ‚ΡΡ Π³Ρ€ΡƒΠΏΠΏΡ‹ ΠΏΠ°Ρ€Π°Π»Π»Π΅Π»ΡŒΠ½ΠΎ ΠΈ ΠΏΠΎΡΠ»Π΅Π΄ΠΎΠ²Π°Ρ‚Π΅Π»ΡŒΠ½ΠΎ Π²ΠΊΠ»ΡŽΡ‡Π΅Π½Π½Ρ‹Ρ… сопротивлСний.

Для Ρ†Π΅ΠΏΠΈ, прСдставлСнной Π½Π° рис. 1.7, расчСт эквивалСнтного сопротивлСния начинаСтся с ΠΊΠΎΠ½Ρ†Π° схСмы. Для упрощСния расчСтов ΠΏΡ€ΠΈΠΌΠ΅ΠΌ, Ρ‡Ρ‚ΠΎ всС сопротивлСния Π² этой схСмС ΡΠ²Π»ΡΡŽΡ‚ΡΡ ΠΎΠ΄ΠΈΠ½Π°ΠΊΠΎΠ²Ρ‹ΠΌΠΈ: R

1=R2=R3=R4=R5=R. БопротивлСния R4 ΠΈ R5 Π²ΠΊΠ»ΡŽΡ‡Π΅Π½Ρ‹ ΠΏΠ°Ρ€Π°Π»Π»Π΅Π»ΡŒΠ½ΠΎ, Ρ‚ΠΎΠ³Π΄Π° сопротивлСниС участка Ρ†Π΅ΠΏΠΈ cd Ρ€Π°Π²Π½ΠΎ:

.

Π’ этом случаС ΠΈΡΡ…ΠΎΠ΄Π½ΡƒΡŽ схСму (рис. 1.7) ΠΌΠΎΠΆΠ½ΠΎ ΠΏΡ€Π΅Π΄ΡΡ‚Π°Π²ΠΈΡ‚ΡŒ Π² ΡΠ»Π΅Π΄ΡƒΡŽΡ‰Π΅ΠΌ Π²ΠΈΠ΄Π΅ (рис. 1.8):

На схСмС (рис. 1.8) сопротивлСниС R3 ΠΈ Rcd соСдинСны ΠΏΠΎΡΠ»Π΅Π΄ΠΎΠ²Π°Ρ‚Π΅Π»ΡŒΠ½ΠΎ, ΠΈ Ρ‚ΠΎΠ³Π΄Π° сопротивлСниС участка Ρ†Π΅ΠΏΠΈ ad Ρ€Π°Π²Π½ΠΎ:

.

Π’ΠΎΠ³Π΄Π° схСму (рис. 1.8) ΠΌΠΎΠΆΠ½ΠΎ ΠΏΡ€Π΅Π΄ΡΡ‚Π°Π²ΠΈΡ‚ΡŒ Π² сокращСнном Π²Π°Ρ€ΠΈΠ°Π½Ρ‚Π΅ (рис. 1.9):

На схСмС (рис. 1.9) сопротивлСниС R

2 ΠΈ Rad соСдинСны ΠΏΠ°Ρ€Π°Π»Π»Π΅Π»ΡŒΠ½ΠΎ, Ρ‚ΠΎΠ³Π΄Π° сопротивлСниС участка Ρ†Π΅ΠΏΠΈ Π°b Ρ€Π°Π²Π½ΠΎ

.

Π‘Ρ…Π΅ΠΌΡƒ (рис. 1.9) ΠΌΠΎΠΆΠ½ΠΎ ΠΏΡ€Π΅Π΄ΡΡ‚Π°Π²ΠΈΡ‚ΡŒ Π² ΡƒΠΏΡ€ΠΎΡ‰Π΅Π½Π½ΠΎΠΌ Π²Π°Ρ€ΠΈΠ°Π½Ρ‚Π΅ (рис. 1.10), Π³Π΄Π΅ сопротивлСния R1 ΠΈ Rab Π²ΠΊΠ»ΡŽΡ‡Π΅Π½Ρ‹ ΠΏΠΎΡΠ»Π΅Π΄ΠΎΠ²Π°Ρ‚Π΅Π»ΡŒΠ½ΠΎ.

Π’ΠΎΠ³Π΄Π° эквивалСнтноС сопротивлСниС исходной схСмы (рис. 1.7) Π±ΡƒΠ΄Π΅Ρ‚ Ρ€Π°Π²Π½ΠΎ:

.

Рис. 1.10

Рис. 1.11

Π’ Ρ€Π΅Π·ΡƒΠ»ΡŒΡ‚Π°Ρ‚Π΅ ΠΏΡ€Π΅ΠΎΠ±Ρ€Π°Π·ΠΎΠ²Π°Π½ΠΈΠΉ исходная схСма (рис. 1.7) прСдставлСна Π² Π²ΠΈΠ΄Π΅ схСмы (рис. 1.11) с ΠΎΠ΄Π½ΠΈΠΌ сопротивлСниСм Rэкв. РасчСт Ρ‚ΠΎΠΊΠΎΠ² ΠΈ напряТСний для всСх элСмСнтов схСмы ΠΌΠΎΠΆΠ½ΠΎ произвСсти ΠΏΠΎ Π·Π°ΠΊΠΎΠ½Π°ΠΌ Ома ΠΈ ΠšΠΈΡ€Ρ…Π³ΠΎΡ„Π°.

Π‘ΠΎΠ΅Π΄ΠΈΠ½Π΅Π½ΠΈΠ΅ элСмСнтов элСктричСской Ρ†Π΅ΠΏΠΈ ΠΏΠΎ схСмам Β«Π·Π²Π΅Π·Π΄Π°Β» ΠΈ Β«Ρ‚Ρ€Π΅ΡƒΠ³ΠΎΠ»ΡŒΠ½ΠΈΠΊΒ»

Π’ элСктротСхничСских ΠΈ элСктронных устройствах элСмСнты Ρ†Π΅ΠΏΠΈ ΡΠΎΠ΅Π΄ΠΈΠ½ΡΡŽΡ‚ΡΡ ΠΏΠΎ мостовой схСмС (рис. 1.12). БопротивлСния R12, R13, R24, R34 Π²ΠΊΠ»ΡŽΡ‡Π΅Π½Ρ‹ Π² ΠΏΠ»Π΅Ρ‡ΠΈ моста, Π² диагональ 1–4 Π²ΠΊΠ»ΡŽΡ‡Π΅Π½ источник питания с Π­Π”Π‘ Π•, другая диагональ 3–4 называСтся ΠΈΠ·ΠΌΠ΅Ρ€ΠΈΡ‚Π΅Π»ΡŒΠ½ΠΎΠΉ диагональю моста.

Рис. 1.12

Рис. 1.13

Π’ мостовой схСмС сопротивлСния R13, R12, R23 ΠΈ R24, R34, R23 соСдинСны ΠΏΠΎ схСмС Β«Ρ‚Ρ€Π΅ΡƒΠ³ΠΎΠ»ΡŒΠ½ΠΈΠΊΒ». Π­ΠΊΠ²ΠΈΠ²Π°Π»Π΅Π½Ρ‚Π½ΠΎΠ΅ сопротивлСниС этой схСмы ΠΌΠΎΠΆΠ½ΠΎ ΠΎΠΏΡ€Π΅Π΄Π΅Π»ΠΈΡ‚ΡŒ Ρ‚ΠΎΠ»ΡŒΠΊΠΎ послС Π·Π°ΠΌΠ΅Π½Ρ‹ ΠΎΠ΄Π½ΠΎΠ³ΠΎ ΠΈΠ· Ρ‚Ρ€Π΅ΡƒΠ³ΠΎΠ»ΡŒΠ½ΠΈΠΊΠΎΠ², Π½Π°ΠΏΡ€ΠΈΠΌΠ΅Ρ€ Ρ‚Ρ€Π΅ΡƒΠ³ΠΎΠ»ΡŒΠ½ΠΈΠΊΠ° R24 R34 R23 Π·Π²Π΅Π·Π΄ΠΎΠΉ R2 R3 R4 (рис. 1.13). Вакая Π·Π°ΠΌΠ΅Π½Π° Π±ΡƒΠ΄Π΅Ρ‚ эквивалСнтной, Ссли ΠΎΠ½Π° Π½Π΅ Π²Ρ‹Π·ΠΎΠ²Π΅Ρ‚ измСнСния Ρ‚ΠΎΠΊΠΎΠ² всСх ΠΎΡΡ‚Π°Π»ΡŒΠ½Ρ‹Ρ… элСмСнтов Ρ†Π΅ΠΏΠΈ. Для этого Π²Π΅Π»ΠΈΡ‡ΠΈΠ½Ρ‹ сопротивлСний Π·Π²Π΅Π·Π΄Ρ‹ Π΄ΠΎΠ»ΠΆΠ½Ρ‹ Ρ€Π°ΡΡΡ‡ΠΈΡ‚Ρ‹Π²Π°Ρ‚ΡŒΡΡ ΠΏΠΎ ΡΠ»Π΅Π΄ΡƒΡŽΡ‰ΠΈΠΌ ΡΠΎΠΎΡ‚Π½ΠΎΡˆΠ΅Π½ΠΈΡΠΌ:

; ; .

Для Π·Π°ΠΌΠ΅Π½Ρ‹ схСмы Β«Π·Π²Π΅Π·Π΄Π°Β» эквивалСнтным Ρ‚Ρ€Π΅ΡƒΠ³ΠΎΠ»ΡŒΠ½ΠΈΠΊΠΎΠΌ Π½Π΅ΠΎΠ±Ρ…ΠΎΠ΄ΠΈΠΌΠΎ Ρ€Π°ΡΡΡ‡ΠΈΡ‚Π°Ρ‚ΡŒ сопротивлСния Ρ‚Ρ€Π΅ΡƒΠ³ΠΎΠ»ΡŒΠ½ΠΈΠΊΠ°:

; ; .

ПослС ΠΏΡ€ΠΎΠ²Π΅Π΄Π΅Π½Π½Ρ‹Ρ… ΠΏΡ€Π΅ΠΎΠ±Ρ€Π°Π·ΠΎΠ²Π°Π½ΠΈΠΉ (рис. 1.13) ΠΌΠΎΠΆΠ½ΠΎ ΠΎΠΏΡ€Π΅Π΄Π΅Π»ΠΈΡ‚ΡŒ Π²Π΅Π»ΠΈΡ‡ΠΈΠ½Ρƒ эквивалСнтного сопротивлСния мостовой схСмы (рис. 1.12)

.

БопротивлСния Π² элСктричСских цСпях ΠΌΠΎΠ³ΡƒΡ‚ Π±Ρ‹Ρ‚ΡŒ соСдинСны ΠΏΠΎΡΠ»Π΅Π΄ΠΎΠ²Π°Ρ‚Π΅Π»ΡŒΠ½ΠΎ, ΠΏΠ°Ρ€Π°Π»Π»Π΅Π»ΡŒΠ½ΠΎ, ΠΏΠΎ смСшанной схСмС ΠΈ ΠΏΠΎ схСмам Β«Π·Π²Π΅Π·Π΄Π°Β», Β«Ρ‚Ρ€Π΅ΡƒΠ³ΠΎΠ»ΡŒΠ½ΠΈΠΊΒ». РасчСт слоТной схСмы упрощаСтся, Ссли сопротивлСния Π² этой схСмС Π·Π°ΠΌΠ΅Π½ΡΡŽΡ‚ΡΡ ΠΎΠ΄Π½ΠΈΠΌ эквивалСнтным сопротивлСниСм Rэкв, ΠΈ вся схСма прСдставляСтся Π² Π²ΠΈΠ΄Π΅ схСмы Π½Π° рис. 1.3, Π³Π΄Π΅ R=Rэкв, Π° расчСт Ρ‚ΠΎΠΊΠΎΠ² ΠΈ напряТСний производится с ΠΏΠΎΠΌΠΎΡ‰ΡŒΡŽ Π·Π°ΠΊΠΎΠ½ΠΎΠ² Ома ΠΈ ΠšΠΈΡ€Ρ…Π³ΠΎΡ„Π°.

ЭлСктричСская Ρ†Π΅ΠΏΡŒ с ΠΏΠΎΡΠ»Π΅Π΄ΠΎΠ²Π°Ρ‚Π΅Π»ΡŒΠ½Ρ‹ΠΌ соСдинСниСм элСмСнтов

Рис. 1.4

Рис. 1.5

ΠŸΠΎΡΠ»Π΅Π΄ΠΎΠ²Π°Ρ‚Π΅Π»ΡŒΠ½Ρ‹ΠΌ Π½Π°Π·Ρ‹Π²Π°ΡŽΡ‚ Ρ‚Π°ΠΊΠΎΠ΅ соСдинСниС элСмСнтов Ρ†Π΅ΠΏΠΈ, ΠΏΡ€ΠΈ ΠΊΠΎΡ‚ΠΎΡ€ΠΎΠΌ Π²ΠΎ всСх Π²ΠΊΠ»ΡŽΡ‡Π΅Π½Π½Ρ‹Ρ… Π² Ρ†Π΅ΠΏΡŒ элСмСнтах Π²ΠΎΠ·Π½ΠΈΠΊΠ°Π΅Ρ‚ ΠΎΠ΄ΠΈΠ½ ΠΈ Ρ‚ΠΎΡ‚ ΠΆΠ΅ Ρ‚ΠΎΠΊ I (рис. 1.4).

На основании Π²Ρ‚ΠΎΡ€ΠΎΠ³ΠΎ Π·Π°ΠΊΠΎΠ½Π° ΠšΠΈΡ€Ρ…Π³ΠΎΡ„Π° (1.5) ΠΎΠ±Ρ‰Π΅Π΅ напряТСниС U всСй Ρ†Π΅ΠΏΠΈ Ρ€Π°Π²Π½ΠΎ суммС напряТСний Π½Π° ΠΎΡ‚Π΄Π΅Π»ΡŒΠ½Ρ‹Ρ… участках:

Π’Π°ΠΊΠΈΠΌ ΠΎΠ±Ρ€Π°Π·ΠΎΠΌ, ΠΏΡ€ΠΈ ΠΏΠΎΡΠ»Π΅Π΄ΠΎΠ²Π°Ρ‚Π΅Π»ΡŒΠ½ΠΎΠΌ соСдинСнии элСмСнтов Ρ†Π΅ΠΏΠΈ ΠΎΠ±Ρ‰Π΅Π΅ эквивалСнтноС сопротивлСниС Ρ†Π΅ΠΏΠΈ Ρ€Π°Π²Π½ΠΎ арифмСтичСской суммС сопротивлСний ΠΎΡ‚Π΄Π΅Π»ΡŒΠ½Ρ‹Ρ… участков. Π‘Π»Π΅Π΄ΠΎΠ²Π°Ρ‚Π΅Π»ΡŒΠ½ΠΎ, Ρ†Π΅ΠΏΡŒ с Π»ΡŽΠ±Ρ‹ΠΌ числом ΠΏΠΎΡΠ»Π΅Π΄ΠΎΠ²Π°Ρ‚Π΅Π»ΡŒΠ½ΠΎ Π²ΠΊΠ»ΡŽΡ‡Π΅Π½Π½Ρ‹Ρ… сопротивлСний ΠΌΠΎΠΆΠ½ΠΎ Π·Π°ΠΌΠ΅Π½ΠΈΡ‚ΡŒ простой Ρ†Π΅ΠΏΡŒΡŽ с ΠΎΠ΄Π½ΠΈΠΌ эквивалСнтным сопротивлСниСм Rэкв (рис. 1.5). ПослС этого расчСт Ρ†Π΅ΠΏΠΈ сводится ΠΊ ΠΎΠΏΡ€Π΅Π΄Π΅Π»Π΅Π½ΠΈΡŽ Ρ‚ΠΎΠΊΠ° I всСй Ρ†Π΅ΠΏΠΈ ΠΏΠΎ Π·Π°ΠΊΠΎΠ½Ρƒ Ома

,

ΠΈ ΠΏΠΎ Π²Ρ‹ΡˆΠ΅ΠΏΡ€ΠΈΠ²Π΅Π΄Π΅Π½Π½Ρ‹ΠΌ Ρ„ΠΎΡ€ΠΌΡƒΠ»Π°ΠΌ Ρ€Π°ΡΡΡ‡ΠΈΡ‚Ρ‹Π²Π°ΡŽΡ‚ ΠΏΠ°Π΄Π΅Π½ΠΈΠ΅ напряТСний U1, U2, U3 Π½Π° ΡΠΎΠΎΡ‚Π²Π΅Ρ‚ΡΡ‚Π²ΡƒΡŽΡ‰ΠΈΡ… участках элСктричСской Ρ†Π΅ΠΏΠΈ (рис. 1.4).

НСдостаток ΠΏΠΎΡΠ»Π΅Π΄ΠΎΠ²Π°Ρ‚Π΅Π»ΡŒΠ½ΠΎΠ³ΠΎ Π²ΠΊΠ»ΡŽΡ‡Π΅Π½ΠΈΡ элСмСнтов Π·Π°ΠΊΠ»ΡŽΡ‡Π°Π΅Ρ‚ΡΡ Π² Ρ‚ΠΎΠΌ, Ρ‡Ρ‚ΠΎ ΠΏΡ€ΠΈ Π²Ρ‹Ρ…ΠΎΠ΄Π΅ ΠΈΠ· строя хотя Π±Ρ‹ ΠΎΠ΄Π½ΠΎΠ³ΠΎ элСмСнта, прСкращаСтся Ρ€Π°Π±ΠΎΡ‚Π° всСх ΠΎΡΡ‚Π°Π»ΡŒΠ½Ρ‹Ρ… элСмСнтов Ρ†Π΅ΠΏΠΈ.

ЭлСктричСская Ρ†Π΅ΠΏΡŒ с ΠΏΠ°Ρ€Π°Π»Π»Π΅Π»ΡŒΠ½Ρ‹ΠΌ соСдинСниСм элСмСнтов

ΠŸΠ°Ρ€Π°Π»Π»Π΅Π»ΡŒΠ½Ρ‹ΠΌ Π½Π°Π·Ρ‹Π²Π°ΡŽΡ‚ Ρ‚Π°ΠΊΠΎΠ΅ соСдинСниС, ΠΏΡ€ΠΈ ΠΊΠΎΡ‚ΠΎΡ€ΠΎΠΌ всС Π²ΠΊΠ»ΡŽΡ‡Π΅Π½Π½Ρ‹Π΅ Π² Ρ†Π΅ΠΏΡŒ ΠΏΠΎΡ‚Ρ€Π΅Π±ΠΈΡ‚Π΅Π»ΠΈ элСктричСской энСргии, находятся ΠΏΠΎΠ΄ ΠΎΠ΄Π½ΠΈΠΌ ΠΈ Ρ‚Π΅ΠΌ ΠΆΠ΅ напряТСниСм (рис. 1.6).

Π’ этом случаС ΠΎΠ½ΠΈ присоСдинСны ΠΊ Π΄Π²ΡƒΠΌ ΡƒΠ·Π»Π°ΠΌ Ρ†Π΅ΠΏΠΈ Π° ΠΈ b, ΠΈ Π½Π° основании ΠΏΠ΅Ρ€Π²ΠΎΠ³ΠΎ Π·Π°ΠΊΠΎΠ½Π° ΠšΠΈΡ€Ρ…Π³ΠΎΡ„Π° (1.3) ΠΌΠΎΠΆΠ½ΠΎ Π·Π°ΠΏΠΈΡΠ°Ρ‚ΡŒ, Ρ‡Ρ‚ΠΎ ΠΎΠ±Ρ‰ΠΈΠΉ Ρ‚ΠΎΠΊ I всСй Ρ†Π΅ΠΏΠΈ Ρ€Π°Π²Π΅Π½ алгСбраичСской суммС Ρ‚ΠΎΠΊΠΎΠ² ΠΎΡ‚Π΄Π΅Π»ΡŒΠ½Ρ‹Ρ… Π²Π΅Ρ‚Π²Π΅ΠΉ:

I = I1 + I2 + I3, Ρ‚.Π΅. ,

ΠΎΡ‚ΠΊΡƒΠ΄Π° слСдуСт, Ρ‡Ρ‚ΠΎ

.

Π’ Ρ‚ΠΎΠΌ случаС, ΠΊΠΎΠ³Π΄Π° ΠΏΠ°Ρ€Π°Π»Π»Π΅Π»ΡŒΠ½ΠΎ Π²ΠΊΠ»ΡŽΡ‡Π΅Π½Ρ‹ Π΄Π²Π° сопротивлСния R1 ΠΈ R2, ΠΎΠ½ΠΈ Π·Π°ΠΌΠ΅Π½ΡΡŽΡ‚ΡΡ ΠΎΠ΄Π½ΠΈΠΌ эквивалСнтным сопротивлСниСм

.

Из ΡΠΎΠΎΡ‚Π½ΠΎΡˆΠ΅Π½ΠΈΡ (1.6), слСдуСт, Ρ‡Ρ‚ΠΎ эквивалСнтная ΠΏΡ€ΠΎΠ²ΠΎΠ΄ΠΈΠΌΠΎΡΡ‚ΡŒ Ρ†Π΅ΠΏΠΈ Ρ€Π°Π²Π½Π° арифмСтичСской суммС проводимостСй ΠΎΡ‚Π΄Π΅Π»ΡŒΠ½Ρ‹Ρ… Π²Π΅Ρ‚Π²Π΅ΠΉ:

По ΠΌΠ΅Ρ€Π΅ роста числа ΠΏΠ°Ρ€Π°Π»Π»Π΅Π»ΡŒΠ½ΠΎ Π²ΠΊΠ»ΡŽΡ‡Π΅Π½Π½Ρ‹Ρ… ΠΏΠΎΡ‚Ρ€Π΅Π±ΠΈΡ‚Π΅Π»Π΅ΠΉ ΠΏΡ€ΠΎΠ²ΠΎΠ΄ΠΈΠΌΠΎΡΡ‚ΡŒ Ρ†Π΅ΠΏΠΈ gэкв возрастаСт, ΠΈ Π½Π°ΠΎΠ±ΠΎΡ€ΠΎΡ‚, ΠΎΠ±Ρ‰Π΅Π΅ сопротивлСниС Rэкв ΡƒΠΌΠ΅Π½ΡŒΡˆΠ°Π΅Ρ‚ΡΡ.

НапряТСния Π² элСктричСской Ρ†Π΅ΠΏΠΈ с ΠΏΠ°Ρ€Π°Π»Π»Π΅Π»ΡŒΠ½ΠΎ соСдинСнными сопротивлСниями (рис. 1.6)

ΠžΡ‚ΡΡŽΠ΄Π° слСдуСт, Ρ‡Ρ‚ΠΎ

,

Ρ‚.Π΅. Ρ‚ΠΎΠΊ Π² Ρ†Π΅ΠΏΠΈ распрСдСляСтся ΠΌΠ΅ΠΆΠ΄Ρƒ ΠΏΠ°Ρ€Π°Π»Π»Π΅Π»ΡŒΠ½Ρ‹ΠΌΠΈ вСтвями ΠΎΠ±Ρ€Π°Ρ‚Π½ΠΎ ΠΏΡ€ΠΎΠΏΠΎΡ€Ρ†ΠΈΠΎΠ½Π°Π»ΡŒΠ½ΠΎ ΠΈΡ… сопротивлСниям.

По ΠΏΠ°Ρ€Π°Π»Π»Π΅Π»ΡŒΠ½ΠΎ Π²ΠΊΠ»ΡŽΡ‡Π΅Π½Π½ΠΎΠΉ схСмС Ρ€Π°Π±ΠΎΡ‚Π°ΡŽΡ‚ Π² номинальном Ρ€Π΅ΠΆΠΈΠΌΠ΅ ΠΏΠΎΡ‚Ρ€Π΅Π±ΠΈΡ‚Π΅Π»ΠΈ любой мощности, рассчитанныС Π½Π° ΠΎΠ΄Π½ΠΎ ΠΈ Ρ‚ΠΎ ΠΆΠ΅ напряТСниС. ΠŸΡ€ΠΈΡ‡Π΅ΠΌ Π²ΠΊΠ»ΡŽΡ‡Π΅Π½ΠΈΠ΅ ΠΈΠ»ΠΈ ΠΎΡ‚ΠΊΠ»ΡŽΡ‡Π΅Π½ΠΈΠ΅ ΠΎΠ΄Π½ΠΎΠ³ΠΎ ΠΈΠ»ΠΈ Π½Π΅ΡΠΊΠΎΠ»ΡŒΠΊΠΈΡ… ΠΏΠΎΡ‚Ρ€Π΅Π±ΠΈΡ‚Π΅Π»Π΅ΠΉ Π½Π΅ отраТаСтся Π½Π° Ρ€Π°Π±ΠΎΡ‚Π΅ ΠΎΡΡ‚Π°Π»ΡŒΠ½Ρ‹Ρ…. ΠŸΠΎΡΡ‚ΠΎΠΌΡƒ эта схСма являСтся основной схСмой ΠΏΠΎΠ΄ΠΊΠ»ΡŽΡ‡Π΅Π½ΠΈΡ ΠΏΠΎΡ‚Ρ€Π΅Π±ΠΈΡ‚Π΅Π»Π΅ΠΉ ΠΊ источнику элСктричСской энСргии.

ЭлСктричСская Ρ†Π΅ΠΏΡŒ со ΡΠΌΠ΅ΡˆΠ°Π½Π½Ρ‹ΠΌ соСдинСниСм элСмСнтов

Π‘ΠΌΠ΅ΡˆΠ°Π½Π½Ρ‹ΠΌ называСтся Ρ‚Π°ΠΊΠΎΠ΅ соСдинСниС, ΠΏΡ€ΠΈ ΠΊΠΎΡ‚ΠΎΡ€ΠΎΠΌ Π² Ρ†Π΅ΠΏΠΈ ΠΈΠΌΠ΅ΡŽΡ‚ΡΡ Π³Ρ€ΡƒΠΏΠΏΡ‹ ΠΏΠ°Ρ€Π°Π»Π»Π΅Π»ΡŒΠ½ΠΎ ΠΈ ΠΏΠΎΡΠ»Π΅Π΄ΠΎΠ²Π°Ρ‚Π΅Π»ΡŒΠ½ΠΎ Π²ΠΊΠ»ΡŽΡ‡Π΅Π½Π½Ρ‹Ρ… сопротивлСний.

Для Ρ†Π΅ΠΏΠΈ, прСдставлСнной Π½Π° рис. 1.7, расчСт эквивалСнтного сопротивлСния начинаСтся с ΠΊΠΎΠ½Ρ†Π° схСмы. Для упрощСния расчСтов ΠΏΡ€ΠΈΠΌΠ΅ΠΌ, Ρ‡Ρ‚ΠΎ всС сопротивлСния Π² этой схСмС ΡΠ²Π»ΡΡŽΡ‚ΡΡ ΠΎΠ΄ΠΈΠ½Π°ΠΊΠΎΠ²Ρ‹ΠΌΠΈ: R1=R2=R3=R4=R5=R. БопротивлСния R4 ΠΈ R5 Π²ΠΊΠ»ΡŽΡ‡Π΅Π½Ρ‹ ΠΏΠ°Ρ€Π°Π»Π»Π΅Π»ΡŒΠ½ΠΎ, Ρ‚ΠΎΠ³Π΄Π° сопротивлСниС участка Ρ†Π΅ΠΏΠΈ cd Ρ€Π°Π²Π½ΠΎ:

.

Π’ этом случаС ΠΈΡΡ…ΠΎΠ΄Π½ΡƒΡŽ схСму (рис. 1.7) ΠΌΠΎΠΆΠ½ΠΎ ΠΏΡ€Π΅Π΄ΡΡ‚Π°Π²ΠΈΡ‚ΡŒ Π² ΡΠ»Π΅Π΄ΡƒΡŽΡ‰Π΅ΠΌ Π²ΠΈΠ΄Π΅ (рис. 1.8):

На схСмС (рис. 1.8) сопротивлСниС R3 ΠΈ Rcd соСдинСны ΠΏΠΎΡΠ»Π΅Π΄ΠΎΠ²Π°Ρ‚Π΅Π»ΡŒΠ½ΠΎ, ΠΈ Ρ‚ΠΎΠ³Π΄Π° сопротивлСниС участка Ρ†Π΅ΠΏΠΈ ad Ρ€Π°Π²Π½ΠΎ:

.

Π’ΠΎΠ³Π΄Π° схСму (рис. 1.8) ΠΌΠΎΠΆΠ½ΠΎ ΠΏΡ€Π΅Π΄ΡΡ‚Π°Π²ΠΈΡ‚ΡŒ Π² сокращСнном Π²Π°Ρ€ΠΈΠ°Π½Ρ‚Π΅ (рис. 1.9):

На схСмС (рис. 1.9) сопротивлСниС R2 ΠΈ Rad соСдинСны ΠΏΠ°Ρ€Π°Π»Π»Π΅Π»ΡŒΠ½ΠΎ, Ρ‚ΠΎΠ³Π΄Π° сопротивлСниС участка Ρ†Π΅ΠΏΠΈ Π°b Ρ€Π°Π²Π½ΠΎ

.

Π‘Ρ…Π΅ΠΌΡƒ (рис. 1.9) ΠΌΠΎΠΆΠ½ΠΎ ΠΏΡ€Π΅Π΄ΡΡ‚Π°Π²ΠΈΡ‚ΡŒ Π² ΡƒΠΏΡ€ΠΎΡ‰Π΅Π½Π½ΠΎΠΌ Π²Π°Ρ€ΠΈΠ°Π½Ρ‚Π΅ (рис. 1.10), Π³Π΄Π΅ сопротивлСния R1 ΠΈ Rab Π²ΠΊΠ»ΡŽΡ‡Π΅Π½Ρ‹ ΠΏΠΎΡΠ»Π΅Π΄ΠΎΠ²Π°Ρ‚Π΅Π»ΡŒΠ½ΠΎ.

Π’ΠΎΠ³Π΄Π° эквивалСнтноС сопротивлСниС исходной схСмы (рис. 1.7) Π±ΡƒΠ΄Π΅Ρ‚ Ρ€Π°Π²Π½ΠΎ:

.

Рис. 1.10

Рис. 1.11

Π’ Ρ€Π΅Π·ΡƒΠ»ΡŒΡ‚Π°Ρ‚Π΅ ΠΏΡ€Π΅ΠΎΠ±Ρ€Π°Π·ΠΎΠ²Π°Π½ΠΈΠΉ исходная схСма (рис. 1.7) прСдставлСна Π² Π²ΠΈΠ΄Π΅ схСмы (рис. 1.11) с ΠΎΠ΄Π½ΠΈΠΌ сопротивлСниСм Rэкв. РасчСт Ρ‚ΠΎΠΊΠΎΠ² ΠΈ напряТСний для всСх элСмСнтов схСмы ΠΌΠΎΠΆΠ½ΠΎ произвСсти ΠΏΠΎ Π·Π°ΠΊΠΎΠ½Π°ΠΌ Ома ΠΈ ΠšΠΈΡ€Ρ…Π³ΠΎΡ„Π°.

Π‘ΠΎΠ΅Π΄ΠΈΠ½Π΅Π½ΠΈΠ΅ элСмСнтов элСктричСской Ρ†Π΅ΠΏΠΈ ΠΏΠΎ схСмам Β«Π·Π²Π΅Π·Π΄Π°Β» ΠΈ Β«Ρ‚Ρ€Π΅ΡƒΠ³ΠΎΠ»ΡŒΠ½ΠΈΠΊΒ»

Π’ элСктротСхничСских ΠΈ элСктронных устройствах элСмСнты Ρ†Π΅ΠΏΠΈ ΡΠΎΠ΅Π΄ΠΈΠ½ΡΡŽΡ‚ΡΡ ΠΏΠΎ мостовой схСмС (рис. 1.12). БопротивлСния R12, R13, R24, R34 Π²ΠΊΠ»ΡŽΡ‡Π΅Π½Ρ‹ Π² ΠΏΠ»Π΅Ρ‡ΠΈ моста, Π² диагональ 1–4 Π²ΠΊΠ»ΡŽΡ‡Π΅Π½ источник питания с Π­Π”Π‘ Π•, другая диагональ 3–4 называСтся ΠΈΠ·ΠΌΠ΅Ρ€ΠΈΡ‚Π΅Π»ΡŒΠ½ΠΎΠΉ диагональю моста.

Рис. 1.12

Рис. 1.13

Π’ мостовой схСмС сопротивлСния R13, R12, R23 ΠΈ R24, R34, R23 соСдинСны ΠΏΠΎ схСмС Β«Ρ‚Ρ€Π΅ΡƒΠ³ΠΎΠ»ΡŒΠ½ΠΈΠΊΒ». Π­ΠΊΠ²ΠΈΠ²Π°Π»Π΅Π½Ρ‚Π½ΠΎΠ΅ сопротивлСниС этой схСмы ΠΌΠΎΠΆΠ½ΠΎ ΠΎΠΏΡ€Π΅Π΄Π΅Π»ΠΈΡ‚ΡŒ Ρ‚ΠΎΠ»ΡŒΠΊΠΎ послС Π·Π°ΠΌΠ΅Π½Ρ‹ ΠΎΠ΄Π½ΠΎΠ³ΠΎ ΠΈΠ· Ρ‚Ρ€Π΅ΡƒΠ³ΠΎΠ»ΡŒΠ½ΠΈΠΊΠΎΠ², Π½Π°ΠΏΡ€ΠΈΠΌΠ΅Ρ€ Ρ‚Ρ€Π΅ΡƒΠ³ΠΎΠ»ΡŒΠ½ΠΈΠΊΠ° R24 R34 R23 Π·Π²Π΅Π·Π΄ΠΎΠΉ R2 R3 R4 (рис. 1.13). Вакая Π·Π°ΠΌΠ΅Π½Π° Π±ΡƒΠ΄Π΅Ρ‚ эквивалСнтной, Ссли ΠΎΠ½Π° Π½Π΅ Π²Ρ‹Π·ΠΎΠ²Π΅Ρ‚ измСнСния Ρ‚ΠΎΠΊΠΎΠ² всСх ΠΎΡΡ‚Π°Π»ΡŒΠ½Ρ‹Ρ… элСмСнтов Ρ†Π΅ΠΏΠΈ. Для этого Π²Π΅Π»ΠΈΡ‡ΠΈΠ½Ρ‹ сопротивлСний Π·Π²Π΅Π·Π΄Ρ‹ Π΄ΠΎΠ»ΠΆΠ½Ρ‹ Ρ€Π°ΡΡΡ‡ΠΈΡ‚Ρ‹Π²Π°Ρ‚ΡŒΡΡ ΠΏΠΎ ΡΠ»Π΅Π΄ΡƒΡŽΡ‰ΠΈΠΌ ΡΠΎΠΎΡ‚Π½ΠΎΡˆΠ΅Π½ΠΈΡΠΌ:

; ; .

Для Π·Π°ΠΌΠ΅Π½Ρ‹ схСмы Β«Π·Π²Π΅Π·Π΄Π°Β» эквивалСнтным Ρ‚Ρ€Π΅ΡƒΠ³ΠΎΠ»ΡŒΠ½ΠΈΠΊΠΎΠΌ Π½Π΅ΠΎΠ±Ρ…ΠΎΠ΄ΠΈΠΌΠΎ Ρ€Π°ΡΡΡ‡ΠΈΡ‚Π°Ρ‚ΡŒ сопротивлСния Ρ‚Ρ€Π΅ΡƒΠ³ΠΎΠ»ΡŒΠ½ΠΈΠΊΠ°:

; ; .

ПослС ΠΏΡ€ΠΎΠ²Π΅Π΄Π΅Π½Π½Ρ‹Ρ… ΠΏΡ€Π΅ΠΎΠ±Ρ€Π°Π·ΠΎΠ²Π°Π½ΠΈΠΉ (рис. 1.13) ΠΌΠΎΠΆΠ½ΠΎ ΠΎΠΏΡ€Π΅Π΄Π΅Π»ΠΈΡ‚ΡŒ Π²Π΅Π»ΠΈΡ‡ΠΈΠ½Ρƒ эквивалСнтного сопротивлСния мостовой схСмы (рис. 1.12)

.

Расчёт элСктричСских схСм, содСрТащих нСсколько сопротивлСний (рСзисторов), ΠΏΡ€ΠΈ Π½Π°Ρ…ΠΎΠΆΠ΄Π΅Π½ΠΈΠΈ силы Ρ‚ΠΎΠΊΠ° Π² Ρ†Π΅ΠΏΠΈ, напряТСния ΠΈΠ»ΠΈ мощности, производится с использованиСм ΠΌΠ΅Ρ‚ΠΎΠ΄Π° свёртывания. ΠœΠ΅Ρ‚ΠΎΠ΄ Π·Π°ΠΊΠ»ΡŽΡ‡Π°Π΅Ρ‚ΡΡ Π² Ρ‚ΠΎΠΌ, Ρ‡Ρ‚ΠΎΠ±Ρ‹ Π½Π°ΠΉΡ‚ΠΈ эквивалСнтноС сопротивлСниС Π²Ρ‹Π΄Π΅Π»Π΅Π½Π½Ρ‹Ρ… участков Ρ†Π΅ΠΏΠΈ. Основная Π·Π°Π΄Π°Ρ‡Π° – Π·Π°ΠΌΠ΅Π½Π° рСзисторов, ΠΈΠΌΠ΅ΡŽΡ‰ΠΈΡ… Ρ€Π°Π·Π»ΠΈΡ‡Π½ΠΎΠ΅ ΠΏΠΎΠ΄ΠΊΠ»ΡŽΡ‡Π΅Π½ΠΈΠ΅ ΠΎΡ‚Π½ΠΎΡΠΈΡ‚Π΅Π»ΡŒΠ½ΠΎ Π΄Ρ€ΡƒΠ³ Π΄Ρ€ΡƒΠ³Π°, Π½Π° эквивалСнт (Rэкв.).

ΠžΠΏΡ€Π΅Π΄Π΅Π»Π΅Π½ΠΈΠ΅ эквивалСнтного сопротивлСния

ΠŸΡ€ΠΈ рассмотрСнии схСм Π»ΡŽΠ±Ρ‹Ρ… элСктричСских ΠΈΠ»ΠΈ элСктронных устройств ΠΌΠΎΠΆΠ½ΠΎ ΡƒΠ²ΠΈΠ΄Π΅Ρ‚ΡŒ, Ρ‡Ρ‚ΠΎ Ρ‚Π°ΠΊΠΈΠ΅ ΠΊΠΎΠΌΠΏΠΎΠ½Π΅Π½Ρ‚Ρ‹, ΠΊΠ°ΠΊ рСзисторы, ΠΈΠΌΠ΅ΡŽΡ‚ Ρ€Π°Π·Π½Ρ‹Π΅ Ρ‚ΠΈΠΏΡ‹ соСдинСний ΠΌΠ΅ΠΆΠ΄Ρƒ собой. Π§Ρ‚ΠΎΠ±Ρ‹ ΠΎΠΏΡ€Π΅Π΄Π΅Π»ΠΈΡ‚ΡŒ эквивалСнтноС соСдинСниС, Π½Π΅ΠΎΠ±Ρ…ΠΎΠ΄ΠΈΠΌΠΎ Ρ€Π°ΡΡΠΌΠ°Ρ‚Ρ€ΠΈΠ²Π°Ρ‚ΡŒ Π΄Π²Π° элСмСнта, Π²ΠΊΠ»ΡŽΡ‡Π΅Π½Π½Ρ‹Ρ… Π² ΠΎΠΏΡ€Π΅Π΄Π΅Π»Ρ‘Π½Π½ΠΎΠΌ порядкС. НСсмотря Π½Π° Ρ‚ΠΎ, Ρ‡Ρ‚ΠΎ Π½Π° Ρ‡Π΅Ρ€Ρ‚Π΅ΠΆΠ΅ ΠΈΡ… ΠΌΠΎΠΆΠ΅Ρ‚ Π±Ρ‹Ρ‚ΡŒ нСсколько дСсятков, ΠΈ соСдинСны ΠΎΠ½ΠΈ ΠΏΠΎ-Ρ€Π°Π·Π½ΠΎΠΌΡƒ, Π΅ΡΡ‚ΡŒ Ρ‚ΠΎΠ»ΡŒΠΊΠΎ Π΄Π²Π° Ρ‚ΠΈΠΏΠ° Π²ΠΊΠ»ΡŽΡ‡Π΅Π½ΠΈΡ ΠΈΡ… Π΄Ρ€ΡƒΠ³ с Π΄Ρ€ΡƒΠ³ΠΎΠΌ: ΠΏΠΎΡΠ»Π΅Π΄ΠΎΠ²Π°Ρ‚Π΅Π»ΡŒΠ½ΠΎΠ΅ ΠΈ ΠΏΠ°Ρ€Π°Π»Π»Π΅Π»ΡŒΠ½ΠΎΠ΅. ΠžΡΡ‚Π°Π»ΡŒΠ½Ρ‹Π΅ ΠΊΠΎΠ½Ρ„ΠΈΠ³ΡƒΡ€Π°Ρ†ΠΈΠΈ – это лишь ΠΈΡ… Π²Π°Ρ€ΠΈΠ°Ρ†ΠΈΠΈ.

ΠŸΠΎΡΠ»Π΅Π΄ΠΎΠ²Π°Ρ‚Π΅Π»ΡŒΠ½ΠΎΠ΅ соСдинСниС элСмСнтов

ПодобноС Π²ΠΊΠ»ΡŽΡ‡Π΅Π½ΠΈΠ΅ ΠΏΠΎΠ΄Ρ€Π°Π·ΡƒΠΌΠ΅Π²Π°Π΅Ρ‚ ΠΊΠΎΠΌΠ±ΠΈΠ½Π°Ρ†ΠΈΡŽ Π΄Π΅Ρ‚Π°Π»Π΅ΠΉ Π² прямой ΠΏΠΎΡΠ»Π΅Π΄ΠΎΠ²Π°Ρ‚Π΅Π»ΡŒΠ½ΠΎΡΡ‚ΠΈ. Π’Ρ‹Ρ…ΠΎΠ΄ ΠΎΠ΄Π½ΠΎΠ³ΠΎ сопротивлСния ΠΏΠΎΠ΄ΠΊΠ»ΡŽΡ‡Π°Π΅Ρ‚ΡΡ ΠΊ Π²Ρ…ΠΎΠ΄Ρƒ Π΄Ρ€ΡƒΠ³ΠΎΠ³ΠΎ. ΠŸΡ€ΠΈ этом ΠΎΡ‚ΡΡƒΡ‚ΡΡ‚Π²ΡƒΡŽΡ‚ ΠΊΠ°ΠΊΠΈΠ΅-Π»ΠΈΠ±ΠΎ отвСтвлСния Π½Π° участкС. Π’Π΅Π»ΠΈΡ‡ΠΈΠ½Π° Ρ‚ΠΎΠΊΠ°, ΠΊΠΎΡ‚ΠΎΡ€Ρ‹ΠΉ ΠΏΡ€ΠΎΡ…ΠΎΠ΄ΠΈΡ‚ Ρ‡Π΅Ρ€Π΅Π· всС соСдинённыС ΠΏΠΎΡΠ»Π΅Π΄ΠΎΠ²Π°Ρ‚Π΅Π»ΡŒΠ½ΠΎ ΠΊΠΎΠΌΠΏΠΎΠ½Π΅Π½Ρ‚Ρ‹, Π±ΡƒΠ΄Π΅Ρ‚ ΠΎΠ΄Π½Π° ΠΈ Ρ‚Π° ΠΆΠ΅.

Π’Π½ΠΈΠΌΠ°Π½ΠΈΠ΅! Π‘Π½ΠΈΠΆΠ΅Π½ΠΈΠ΅ ΠΏΠΎΡ‚Π΅Π½Ρ†ΠΈΠ°Π»Π° Π½Π° ΠΊΠ°ΠΆΠ΄ΠΎΠΌ рСзистивном элСмСнтС Π² суммС даст ΠΏΠΎΠ»Π½ΠΎΠ΅ напряТСниС, ΠΏΡ€ΠΈΠ»ΠΎΠΆΠ΅Π½Π½ΠΎΠ΅ ΠΊ ΠΏΠΎΡΠ»Π΅Π΄ΠΎΠ²Π°Ρ‚Π΅Π»ΡŒΠ½ΠΎΠΉ Ρ†Π΅ΠΏΠΈ.

Π’ случаС постоянного Ρ‚ΠΎΠΊΠ° Ρ„ΠΎΡ€ΠΌΡƒΠ»Π° Π·Π°ΠΊΠΎΠ½Π° Ома для ΠΎΡ‚Ρ€Π΅Π·ΠΊΠ° Ρ†Π΅ΠΏΠΈ ΠΈΠΌΠ΅Π΅Ρ‚ Π²ΠΈΠ΄:

Π‘ΠΈΠ»Π° Ρ‚ΠΎΠΊΠ° зависит ΠΎΡ‚ ΠΏΡ€ΠΈΠ»ΠΎΠΆΠ΅Π½Π½ΠΎΠ³ΠΎ напряТСния ΠΈ ΠΎΠΊΠ°Π·Π°Π½Π½ΠΎΠ³ΠΎ Π΅ΠΌΡƒ сопротивлСния. Если Π²Ρ‹Ρ€Π°Π·ΠΈΡ‚ΡŒ R, Π΅Π³ΠΎ Ρ„ΠΎΡ€ΠΌΡƒΠ»Π°:

ΠŸΠ°Ρ€Π°ΠΌΠ΅Ρ‚Ρ€Ρ‹ ΠΏΠΎΡΠ»Π΅Π΄ΠΎΠ²Π°Ρ‚Π΅Π»ΡŒΠ½ΠΎΠΉ Ρ†Π΅ΠΏΠΈ, Π²ΠΊΠ»ΡŽΡ‡Π°ΡŽΡ‰Π΅ΠΉ n соСдинённых Π΄Ρ€ΡƒΠ³ с Π΄Ρ€ΡƒΠ³ΠΎΠΌ элСмСнтов, ΠΈΠΌΠ΅ΡŽΡ‚ свои особСнности.

ΠŸΡ€ΠΎΡ…ΠΎΠ΄ΡΡ‰ΠΈΠΉ ΠΏΠΎ Ρ†Π΅ΠΏΠΈ Ρ‚ΠΎΠΊ Π²Π΅Π·Π΄Π΅ ΠΎΠ΄ΠΈΠ½Π°ΠΊΠΎΠ²Ρ‹ΠΉ:

ΠŸΡ€ΠΈΠΊΠ»Π°Π΄Ρ‹Π²Π°Π΅ΠΌΠΎΠ΅ напряТСниС являСтся суммой напряТСний Π½Π° ΠΊΠ°ΠΆΠ΄ΠΎΠΌ рСзисторС:

Π‘Π»Π΅Π΄ΠΎΠ²Π°Ρ‚Π΅Π»ΡŒΠ½ΠΎ, Ρ€Π°ΡΡΡ‡ΠΈΡ‚Π°Ρ‚ΡŒ ΠΌΠΎΠΆΠ½ΠΎ ΠΎΠ±Ρ‰Π΅Π΅:

Rэкв.= U1/I + U2/I + … +Un/I) = R1 + R2 + … +Rn.

Π’Π°ΠΆΠ½ΠΎ! ΠŸΠΎΡΠ»Π΅Π΄ΠΎΠ²Π°Ρ‚Π΅Π»ΡŒΠ½Π°Ρ Ρ†Π΅ΠΏΡŒ, ΠΈΠΌΠ΅ΡŽΡ‰Π°Ρ Π² своём составС N рСзисторов Ρ€Π°Π²Π½ΠΎΠ³ΠΎ Π½ΠΎΠΌΠΈΠ½Π°Π»Π°, ΠΈΠΌΠ΅Π΅Ρ‚ эквивалСнтноС сопротивлСниС Rэкв. = N*R.

ΠŸΠ°Ρ€Π°Π»Π»Π΅Π»ΡŒΠ½ΠΎΠ΅ соСдинСниС

Когда условныС Π²Ρ‹Ρ…ΠΎΠ΄Ρ‹ Π΄Π΅Ρ‚Π°Π»Π΅ΠΉ ΠΈΠΌΠ΅ΡŽΡ‚ ΠΎΠ±Ρ‰ΠΈΠΉ ΠΊΠΎΠ½Ρ‚Π°ΠΊΡ‚ Π² ΠΎΠ΄Π½ΠΎΠΉ Ρ‚ΠΎΡ‡ΠΊΠ΅ (ΡƒΠ·Π»Π΅) схСмы, Π° условныС Π²Ρ…ΠΎΠ΄Ρ‹ Ρ‚Π°ΠΊ ΠΆΠ΅ ΠΎΠ±ΡŠΠ΅Π΄ΠΈΠ½Π΅Π½Ρ‹ Π²ΠΎ Π²Ρ‚ΠΎΡ€ΠΎΠΉ, говорят ΠΎ ΠΏΠ°Ρ€Π°Π»Π»Π΅Π»ΡŒΠ½ΠΎΠΌ соСдинСнии. Π£Π·Π΅Π» Π½Π° Ρ‡Π΅Ρ€Ρ‚Π΅ΠΆΠ΅ обозначаСтся графичСской Ρ‚ΠΎΡ‡ΠΊΠΎΠΉ. Π­Ρ‚ΠΎ мСсто, Π³Π΄Π΅ происходят развСтвлСния Ρ†Π΅ΠΏΠ΅ΠΉ Π² схСмах. Π’Π°ΠΊΠΎΠΉ Π²Π°Ρ€ΠΈΠ°Π½Ρ‚ ΠΏΠΎΠ΄ΠΊΠ»ΡŽΡ‡Π΅Π½ΠΈΡ рСзисторов обСспСчиваСт ΠΎΠ΄ΠΈΠ½Π°ΠΊΠΎΠ²ΠΎΠ΅ ΠΏΠ°Π΄Π΅Π½ΠΈΠ΅ напряТСния U для всСх ΠΏΠ°Ρ€Π°Π»Π»Π΅Π»ΡŒΠ½Ρ‹Ρ… элСмСнтов. Π’ΠΎΠΊ Π² этой ΠΏΠΎΠ·ΠΈΡ†ΠΈΠΈ Π±ΡƒΠ΄Π΅Ρ‚ Ρ€Π°Π²Π΅Π½ суммС Ρ‚ΠΎΠΊΠΎΠ², ΠΈΠ΄ΡƒΡ‰ΠΈΡ… ΠΏΠΎ ΠΊΠ°ΠΆΠ΄ΠΎΠΌΡƒ ΠΊΠΎΠΌΠΏΠΎΠ½Π΅Π½Ρ‚Ρƒ.

Когда Π² ΠΏΠ°Ρ€Π°Π»Π»Π΅Π»ΡŒΠ½ΠΎΠ΅ ΠΏΠΎΠ΄ΠΊΠ»ΡŽΡ‡Π΅Π½ΠΈΠ΅ Π²Ρ…ΠΎΠ΄ΠΈΡ‚ n рСзистивных элСмСнтов, Ρ‚ΠΎ Ρ€Π°Π·Π½ΠΎΡΡ‚ΡŒ ΠΏΠΎΡ‚Π΅Π½Ρ†ΠΈΠ°Π»ΠΎΠ², Ρ‚ΠΎΠΊ ΠΈ ΠΎΠ±Ρ‰Π΅Π΅ сопротивлСниС Π±ΡƒΠ΄ΡƒΡ‚ ΠΈΠΌΠ΅Ρ‚ΡŒ ΡΠ»Π΅Π΄ΡƒΡŽΡ‰ΠΈΠ΅ выраТСния:

  • ΠΎΠ±Ρ‰ΠΈΠΉ Ρ‚ΠΎΠΊ: I = I1 + I2 + … + In;
  • ΠΎΠ±Ρ‰Π΅Π΅ напряТСниС: U = U1 = U2 = … = Un;
  • RΠΎΠ±Ρ‰. = Rэкв. = U/I1 + U/I2 + …+ U/In) = 1/R1 + 1/R2 +…+ 1/Rn.

Π’Π΅Π»ΠΈΡ‡ΠΈΠ½Ρƒ, ΠΎΠ±Ρ€Π°Ρ‚Π½ΠΎ ΠΏΡ€ΠΎΠΏΠΎΡ€Ρ†ΠΈΠΎΠ½Π°Π»ΡŒΠ½ΡƒΡŽ ΡΠΎΠΏΡ€ΠΎΡ‚ΠΈΠ²Π»Π΅Π½ΠΈΡŽ 1/R, Π½Π°Π·Ρ‹Π²Π°ΡŽΡ‚ ΠΏΡ€ΠΎΠ²ΠΎΠ΄ΠΈΠΌΠΎΡΡ‚ΡŒΡŽ.

Если n Ρ€Π°Π²Π½Ρ‹Ρ… ΠΏΠΎ Π½ΠΎΠΌΠΈΠ½Π°Π»Ρƒ сопротивлСний Π²ΠΊΠ»ΡŽΡ‡ΠΈΡ‚ΡŒ ΠΏΠ°Ρ€Π°Π»Π»Π΅Π»ΡŒΠ½ΠΎ, Ρ‚ΠΎ Rэкв. = (R*R)/n*R = R/n. Π€ΠΎΡ€ΠΌΡƒΠ»Π° ΠΏΠΎΠ΄Ρ…ΠΎΠ΄ΠΈΡ‚ ΠΈ для ΠΈΠ½Π΄ΡƒΠΊΡ‚ΠΈΠ²Π½Ρ‹Ρ… сопротивлСний ΠΏΡ€ΠΎΠ²ΠΎΠ»ΠΎΡ‡Π½Ρ‹Ρ… ΠΊΠ°Ρ‚ΡƒΡˆΠ΅ΠΊ ΠΈ ёмкостных сопротивлСний кондСнсаторов.

Расчёт ΠΏΡ€ΠΈ смСшанном соСдинСнии устройств

ΠŸΡ€ΠΎΠΈΠ·Π²Π΅ΡΡ‚ΠΈ расчСт сопротивлСния Ρ†Π΅ΠΏΠΈ, ΠΊΠΎΠ³Π΄Π° ΠΎΠ½Π° Ρ€Π°Π·Π²Π΅Ρ‚Π²Π»Π΅Π½Π° ΠΈ Π½Π°ΠΏΠΎΠ»Π½Π΅Π½Π° Ρ€Π°Π·Π½Ρ‹ΠΌΠΈ Π²ΠΈΠ΄Π°ΠΌΠΈ рСзистивных соСдинСний, просто Π½Π΅ получится. ЗатрудняСт Ρ€Π΅ΡˆΠ΅Π½ΠΈΠ΅ Π·Π°Π΄Π°Ρ‡ΠΈ мноТСство участков, Π³Π΄Π΅ Π΄Π΅Ρ‚Π°Π»ΠΈ ΠΏΠΎΠ΄ΠΊΠ»ΡŽΡ‡Π΅Π½Ρ‹ Π΄Ρ€ΡƒΠ³ Π΄Ρ€ΡƒΠ³Ρƒ Π² Ρ€Π°Π·Π½Ρ‹Ρ… комбинациях. Π’ Ρ‚Π°ΠΊΠΈΡ… ΠΎΠ±ΡΡ‚ΠΎΡΡ‚Π΅Π»ΡŒΡΡ‚Π²Π°Ρ… ΠΆΠ΅Π»Π°Ρ‚Π΅Π»ΡŒΠ½ΠΎ Π²Ρ‹ΠΏΠΎΠ»Π½ΡΡ‚ΡŒ ряд ΠΏΡ€Π΅ΠΎΠ±Ρ€Π°Π·ΠΎΠ²Π°Π½ΠΈΠΉ, добиваясь упрощСния схСмы Π²Π²ΠΎΠ΄ΠΎΠΌ ΠΎΡ‚Π΄Π΅Π»ΡŒΠ½Ρ‹Ρ… эквивалСнтных элСмСнтов. Π’Ρ‹ΡΠ²Π»ΡΡŽΡ‚ ΠΏΡ€ΠΈ этом подходящиС ΠΊΠΎΠ½Ρ‚ΡƒΡ€Ρ‹ ΠΏΠΎΡΠ»Π΅Π΄ΠΎΠ²Π°Ρ‚Π΅Π»ΡŒΠ½Ρ‹Ρ… ΠΈ ΠΏΠ°Ρ€Π°Π»Π»Π΅Π»ΡŒΠ½Ρ‹Ρ… присоСдинСний.

НапримСр, выискав Π½Π΅ΠΊΠΎΡ‚ΠΎΡ€ΠΎΠ΅ количСство ΠΏΠΎΡΠ»Π΅Π΄ΠΎΠ²Π°Ρ‚Π΅Π»ΡŒΠ½Ρ‹Ρ… ΠΏΠΎΠ΄ΠΊΠ»ΡŽΡ‡Π΅Π½ΠΈΠΉ рСзисторов, Π·Π°ΠΌΠ΅Π½ΡΡŽΡ‚ ΠΈΡ… Π½Π° ΠΎΠ΄ΠΈΠ½ эквивалСнтный ΠΊΠΎΠΌΠΏΠΎΠ½Π΅Π½Ρ‚. ΠžΠΏΡ€Π΅Π΄Π΅Π»ΠΈΠ² элСмСнты, соСдинённыС ΠΏΠΎΡΠ»Π΅Π΄ΠΎΠ²Π°Ρ‚Π΅Π»ΡŒΠ½ΠΎ, Ρ‚Π°ΠΊΠΆΠ΅ Ρ€ΠΈΡΡƒΡŽΡ‚ вмСсто Π½Π΅Π³ΠΎ эквивалСнт. Π’Π½ΠΎΠ²ΡŒ Π½Π°Ρ‡ΠΈΠ½Π°ΡŽΡ‚ ΠΈΡΠΊΠ°Ρ‚ΡŒ ΠΏΠΎΠ΄ΠΎΠ±Π½Ρ‹Π΅ простыС соСдинСния.

ΠœΠ΅Ρ‚ΠΎΠ΄ Π½Π°Π·Ρ‹Π²Π°ΡŽΡ‚ Β«ΠΌΠ΅Ρ‚ΠΎΠ΄ΠΎΠΌ свёртывания». Π‘Ρ…Π΅ΠΌΡƒ ΡƒΠΏΡ€ΠΎΡ‰Π°ΡŽΡ‚ Π΄ΠΎ Ρ‚Π΅Ρ… ΠΏΠΎΡ€, ΠΏΠΎΠΊΠ° Π² Π½Π΅ΠΉ Π½Π΅ останСтся ΠΎΠ΄Π½ΠΎ Rэкв.

Π’Π°ΠΆΠ½ΠΎ! ΠœΠ΅Ρ‚ΠΎΠ΄ эквивалСнтных ΠΏΡ€Π΅ΠΎΠ±Ρ€Π°Π·ΠΎΠ²Π°Π½ΠΈΠΉ примСняСтся Ρ‚ΠΎΠ³Π΄Π°, ΠΊΠΎΠ³Π΄Π° ΠΏΠΈΡ‚Π°Π½ΠΈΠ΅ рассматриваСмого участка Ρ†Π΅ΠΏΠΈ осущСствляСтся ΠΎΡ‚ ΠΎΠ΄Π½ΠΎΠ³ΠΎ источника элСктричСского Ρ‚ΠΎΠΊΠ°, Π° Ρ‚Π°ΠΊΠΆΠ΅ ΠΏΡ€ΠΈ ΠΎΠΏΡ€Π΅Π΄Π΅Π»Π΅Π½ΠΈΠΈ Rэкв. Π² Π·Π°ΠΌΠΊΠ½ΡƒΡ‚ΠΎΠΌ ΠΊΠΎΠ½Ρ‚ΡƒΡ€Π΅ с ΠΎΠ΄Π½ΠΎΠΉ Π­Π”Π‘.

Π’Π°ΠΊΠΎΠΉ ΠΎΡ‚Π½ΠΎΡΠΈΡ‚Π΅Π»ΡŒΠ½Ρ‹ΠΉ способ опрСдСлСния Rэкв ΠΈΡΠΏΠΎΠ»ΡŒΠ·ΡƒΡŽΡ‚ ΠΈ для изучСния зависимости Ρ‚ΠΎΠΊΠΎΠ² Π² Π½Π΅ΠΊΠΎΡ‚ΠΎΡ€ΠΎΠΉ Ρ†Π΅ΠΏΠΈ ΠΎΡ‚ значСния R Π½Π°Π³Ρ€ΡƒΠ·ΠΊΠΈ. Π­Ρ‚ΠΎ ΠΌΠ΅Ρ‚ΠΎΠ΄ эквивалСнтного Π³Π΅Π½Π΅Ρ€Π°Ρ‚ΠΎΡ€Π°, ΠΏΡ€ΠΈ ΠΊΠΎΡ‚ΠΎΡ€ΠΎΠΌ слоТный Π΄Π²ΡƒΡ…ΠΏΠΎΠ»ΡŽΡΠ½ΠΈΠΊ, ΡΠ²Π»ΡΡŽΡ‰ΠΈΠΉΡΡ Π°ΠΊΡ‚ΠΈΠ²Π½Ρ‹ΠΌ, ΠΏΡ€Π΅Π΄ΡΡ‚Π°Π²Π»ΡΡŽΡ‚ эквивалСнтным Π³Π΅Π½Π΅Ρ€Π°Ρ‚ΠΎΡ€ΠΎΠΌ. ΠŸΡ€ΠΈ этом ΡΡ‡ΠΈΡ‚Π°ΡŽΡ‚, Ρ‡Ρ‚ΠΎ Π­Π”Π‘ Π΅Π³ΠΎ соотвСтствуСт UΡ….Ρ…. (холостого Ρ…ΠΎΠ΄Π°) Π½Π° Π·Π°ΠΆΠΈΠΌΠ°Ρ…, R Π²Π½ΡƒΡ‚Ρ€Π΅Π½Π½Π΅Π΅ соотвСтствуСт R Π²Ρ…ΠΎΠ΄Π½ΠΎΠΌΡƒ Π΄Π²ΡƒΡ…ΠΏΠΎΠ»ΡŽΡΠ½ΠΈΠΊΠ° пассивного Π½Π° Ρ‚Π΅Ρ… ΠΆΠ΅ Π·Π°ΠΆΠΈΠΌΠ°Ρ…. Для Ρ‚Π°ΠΊΠΎΠ³ΠΎ опрСдСлСния источники Ρ‚ΠΎΠΊΠ° Ρ€Π°Π·ΡŠΠ΅Π΄ΠΈΠ½ΡΡŽΡ‚, Π° ΠΊΠ°Π½Π°Π» Π­Π”Π‘ Π·Π°ΠΊΠΎΡ€Π°Ρ‡ΠΈΠ²Π°ΡŽΡ‚.

ЀизичСскиС Ρ„ΠΎΡ€ΠΌΡƒΠ»Ρ‹ ΠΈ ΠΏΡ€ΠΈΠΌΠ΅Ρ€Ρ‹ вычислСний

Π€ΠΎΡ€ΠΌΡƒΠ»Ρ‹ для эквивалСнтных сопротивлСний Ρ†Π΅ΠΏΠΈ, состоящСй ΠΈΠ· ΠΏΠ°Ρ€Ρ‹ рСзисторов R1 ΠΈ R2, ΠΌΠΎΠΆΠ½ΠΎ Π²Ρ‹Π΄Π΅Π»ΠΈΡ‚ΡŒ Π² ΠΎΠΏΡ€Π΅Π΄Π΅Π»Ρ‘Π½Π½Ρ‹ΠΉ ряд:

  • ΠΏΠ°Ρ€Π°Π»Π»Π΅Π»ΡŒΠ½ΠΎΠ΅ присоСдинСниС ΠΎΠΏΡ€Π΅Π΄Π΅Π»ΡΡŽΡ‚ ΠΏΠΎ Ρ„ΠΎΡ€ΠΌΡƒΠ»Π΅ Rэкв. = (R1*R2)/R1+R2;
  • ΠΏΠΎΡΠ»Π΅Π΄ΠΎΠ²Π°Ρ‚Π΅Π»ΡŒΠ½ΠΎΠ΅ Π²ΠΊΠ»ΡŽΡ‡Π΅Π½ΠΈΠ΅ Π²Ρ‹Ρ‡ΠΈΡΠ»ΡΡŽΡ‚, опрСдСляя Π΅Π³ΠΎ сумму Rэкв. = R1+R2.

Π£ смСшанного соСдинСния рСзистивных элСмСнтов Π½Π΅Ρ‚ ΠΊΠΎΠ½ΠΊΡ€Π΅Ρ‚Π½ΠΎΠΉ Ρ„ΠΎΡ€ΠΌΡƒΠ»Ρ‹. Π§Ρ‚ΠΎΠ±Ρ‹ Π½Π΅ Π·Π°ΠΏΡƒΡ‚Π°Ρ‚ΡŒΡΡ ΠΏΡ€ΠΈ Π΄Π»ΠΈΡ‚Π΅Π»ΡŒΠ½Ρ‹Ρ… прСобразованиях, здСсь допустимо Π²ΠΎΡΠΏΠΎΠ»ΡŒΠ·ΠΎΠ²Π°Ρ‚ΡŒΡΡ ΡΠΏΠ΅Ρ†ΠΈΠ°Π»ΡŒΠ½ΠΎΠΉ ΠΏΡ€ΠΎΠ³Ρ€Π°ΠΌΠΌΠΎΠΉ ΠΈΠ· ΠΈΠ½Ρ‚Π΅Ρ€Π½Π΅Ρ‚Π°. Π­Ρ‚ΠΎ сСрвис Β«ΠΎΠ½Π»Π°ΠΉΠ½-ΠΊΠ°Π»ΡŒΠΊΡƒΠ»ΡΡ‚ΠΎΡ€Β». Он ΠΏΠΎΠΌΠΎΠΆΠ΅Ρ‚ Ρ€Π°Π·ΠΎΠ±Ρ€Π°Ρ‚ΡŒΡΡ со слоТными схСмами соСдинСния, Π±ΡƒΠ΄ΡŒ Ρ‚ΠΎ Ρ‚Ρ€Π΅ΡƒΠ³ΠΎΠ»ΡŒΠ½ΠΈΠΊ, ΠΊΠ²Π°Π΄Ρ€Π°Ρ‚, ΠΏΡΡ‚ΠΈΡƒΠ³ΠΎΠ»ΡŒΠ½ΠΈΠΊ ΠΈΠ»ΠΈ иная схСматичная Ρ„ΠΈΠ³ΡƒΡ€Π°, образованная рСзистивными элСмСнтами.

ΠŸΠΎΠ½ΡΡ‚ΡŒ, ΠΊΠ°ΠΊ Ρ€Π°Π±ΠΎΡ‚Π°ΡŽΡ‚ всС Ρ„ΠΎΡ€ΠΌΡƒΠ»Ρ‹ ΠΈ ΠΌΠ΅Ρ‚ΠΎΠ΄Ρ‹, ΠΌΠΎΠΆΠ½ΠΎ Π½Π° ΠΊΠΎΠ½ΠΊΡ€Π΅Ρ‚Π½ΠΎΠΉ Π·Π°Π΄Π°Ρ‡Π΅. На прСдставлСнном ΠΏΠ΅Ρ€Π²ΠΎΠΌ рисункС – смСшанная элСктричСская схСма. Она Π²ΠΊΠ»ΡŽΡ‡Π°Π΅Ρ‚ Π² сСбя 10 рСзисторов. Π­Π»Π΅ΠΌΠ΅Π½Ρ‚Ρ‹ прСдставлСны Π² ΡΠ»Π΅Π΄ΡƒΡŽΡ‰ΠΈΡ… Π½ΠΎΠΌΠΈΠ½Π°Π»Π°Ρ…:

  • R1 = 1 Ом;
  • R2 = 2 Ом;
  • R3 = 3 Ом;
  • R4 = 6 Ом;
  • R5 = 9 Ом;
  • R6 = 18 Ом;
  • R7 = 2Ом;
  • R8 = 2Ом;
  • R9 = 8 Ом;
  • R10 = 4 Ом.

НапряТСниС, ΠΏΠΎΠ΄Π°Π½Π½ΠΎΠ΅ Π½Π° схСму:

ВрСбуСтся Ρ€Π°ΡΡΡ‡ΠΈΡ‚Π°Ρ‚ΡŒ Ρ‚ΠΎΠΊΠΈ Π½Π° всСх рСзистивных элСмСнтах.

Для расчётов примСняСтся Π·Π°ΠΊΠΎΠ½ Ома:

I = U/R, подставляя вмСсто R эквивалСнтноС сопротивлСниС.

Π’Π½ΠΈΠΌΠ°Π½ΠΈΠ΅! Для Ρ€Π΅ΡˆΠ΅Π½ΠΈΡ этой Π·Π°Π΄Π°Ρ‡ΠΈ сначала Π²Ρ‹Ρ‡ΠΈΡΠ»ΡΡŽΡ‚ ΠΎΠ±Ρ‰Π΅Π΅ (эквивалСнтноС) R, послС Ρ‡Π΅Π³ΠΎ ΡƒΠΆΠ΅ Ρ€Π°ΡΡΡ‡ΠΈΡ‚Ρ‹Π²Π°ΡŽΡ‚ Ρ‚ΠΎΠΊ Π² Ρ†Π΅ΠΏΠΈ ΠΈ напряТСниС Π½Π° ΠΊΠ°ΠΆΠ΄ΠΎΠΌ рСзистивном ΠΊΠΎΠΌΠΏΠΎΠ½Π΅Π½Ρ‚Π΅.

Вычисляя Rэкв., Ρ€Π°Π·Π΄Π΅Π»ΡΡŽΡ‚ Π·Π°Π΄Π°Π½Π½ΡƒΡŽ Ρ†Π΅ΠΏΡŒ Π½Π° звСнья, Π²ΠΌΠ΅Ρ‰Π°ΡŽΡ‰ΠΈΠ΅ Π² сСбя ΠΏΠ°Ρ€Π°Π»Π»Π΅Π»ΡŒΠ½Ρ‹Π΅ ΠΈ ΠΏΠΎΡΠ»Π΅Π΄ΠΎΠ²Π°Ρ‚Π΅Π»ΡŒΠ½Ρ‹Π΅ Π²ΠΊΠ»ΡŽΡ‡Π΅Π½ΠΈΡ. Π”Π΅Π»Π°ΡŽΡ‚ расчёты для ΠΊΠ°ΠΆΠ΄ΠΎΠ³ΠΎ Ρ‚Π°ΠΊΠΎΠ³ΠΎ Π·Π²Π΅Π½Π°, послС – всСй Ρ†Π΅ΠΏΠΈ Ρ†Π΅Π»ΠΈΠΊΠΎΠΌ.

На рисункС Π²Ρ‹ΡˆΠ΅ ΠΈΠ·ΠΎΠ±Ρ€Π°ΠΆΠ΅Π½ΠΎ смСшанноС соСдинСниС сопротивлСний. Π•Π³ΠΎ ΠΌΠΎΠΆΠ½ΠΎ Ρ€Π°Π·Π±ΠΈΡ‚ΡŒ Π½Π° Ρ‚Ρ€ΠΈ участка:

  • АВ – участок, ΠΈΠΌΠ΅ΡŽΡ‰ΠΈΠΉ Π΄Π²Π΅ ΠΏΠ°Ρ€Π°Π»Π»Π΅Π»ΡŒΠ½Ρ‹Ρ… Π²Π΅Ρ‚Π²ΠΈ;
  • Π’Π‘ – ΠΎΡ‚Ρ€Π΅Π·ΠΎΠΊ, Π²ΠΌΠ΅Ρ‰Π°ΡŽΡ‰ΠΈΠΉ Π² сСбя ΠΏΠΎΡΠ»Π΅Π΄ΠΎΠ²Π°Ρ‚Π΅Π»ΡŒΠ½ΠΎΠ΅ сопряТСниС;
  • CD – ΠΎΡ‚Ρ€Π΅Π·ΠΎΠΊ схСмы с располоТСниСм Ρ‚Ρ€Ρ‘Ρ… ΠΏΠ°Ρ€Π°Π»Π»Π΅Π»ΡŒΠ½Ρ‹Ρ… Ρ†Π΅ΠΏΠΎΡ‡Π΅ΠΊ.

БопротивлСния R2 ΠΈ R3, ΠΎΠ±Ρ€Π°Π·ΡƒΡŽΡ‰ΠΈΠ΅ ниТнюю Π²Π΅Ρ‚ΠΊΡƒ ΠΎΡ‚Ρ€Π΅Π·ΠΊΠ° АВ, соСдинСны ΠΏΠΎΡΠ»Π΅Π΄ΠΎΠ²Π°Ρ‚Π΅Π»ΡŒΠ½ΠΎ, Ρ‡Ρ‚ΠΎ учитываСтся ΠΏΡ€ΠΈ расчётС.

Если ΠΏΠΎΡΠΌΠΎΡ‚Ρ€Π΅Ρ‚ΡŒ Π½Π° участок Π‘D, Ρ‚ΠΎ ΠΌΠΎΠΆΠ½ΠΎ ΠΎΡ‚ΠΌΠ΅Ρ‚ΠΈΡ‚ΡŒ смСшанноС Π²ΠΊΠ»ΡŽΡ‡Π΅Π½ΠΈΠ΅ рСзистивных элСмСнтов.

Начало расчётов состоит Π² ΠΎΠΏΡ€Π΅Π΄Π΅Π»Π΅Π½ΠΈΠΈ эквивалСнтных сопротивлСний для этих ΡΠΌΠ΅ΡˆΠ°Π½Π½Ρ‹Ρ… Ρ„Ρ€Π°Π³ΠΌΠ΅Π½Ρ‚ΠΎΠ². Π’Ρ‹ΠΏΠΎΠ»Π½ΡΡŽΡ‚ это Π² ΡΠ»Π΅Π΄ΡƒΡŽΡ‰Π΅ΠΌ порядкС:

  • Rэкв.2,3 = R2+R3=2 + 3 = 5 Ом;
  • Rэкв.7,8 = (R7*R8)/R7 + R8 = (2*2)/2 + 2 = 1 Ом;
  • Rэкв.7,8,9 = Rэкв.7,8 + R9 = 1 + 8 = 9 Ом.

Зная значСния ΠΏΠΎΠ»ΡƒΡ‡Π΅Π½Π½Ρ‹Ρ… эквивалСнтов, ΡƒΠΏΡ€ΠΎΡ‰Π°ΡŽΡ‚ ΠΏΠ΅Ρ€Π²ΠΎΠ½Π°Ρ‡Π°Π»ΡŒΠ½ΡƒΡŽ схСму. Она Π±ΡƒΠ΄Π΅Ρ‚ ΠΈΠΌΠ΅Ρ‚ΡŒ Π²ΠΈΠ΄, прСдставлСнный Π½Π° рисункС Π½ΠΈΠΆΠ΅.

Π”Π°Π»Π΅Π΅ ΠΌΠΎΠΆΠ½ΠΎ ΡƒΠΆΠ΅ ΠΎΠΏΡ€Π΅Π΄Π΅Π»ΠΈΡ‚ΡŒ Rэкв. для участков AB, BC, CD, ΠΏΠΎ Ρ„ΠΎΡ€ΠΌΡƒΠ»Π°ΠΌ:

  • Rэкв.AB = (R1*Rэкв 2,3)/R1 + Rэкв 2,3 = (1*5)/1 + 5 = 0,83 Ом;
  • Rэкв.BC = R4 + R5 = 6 + 9 = 15 Ом;
  • 1/Rэкв.CD = 1/R6 + 1/Rэкв.7,8,9 + 1/R10 = 1/18 + 1/9 + 1/4 = 0,05 + 0,11 + 0,25 = 0,41 Ом.

Π’ Ρ€Π΅Π·ΡƒΠ»ΡŒΡ‚Π°Ρ‚Π΅ Π²Ρ‹ΠΏΠΎΠ»Π½Π΅Π½Π½Ρ‹Ρ… вычислСний получаСтся эквивалСнтная схСма, Π² ΠΊΠΎΡ‚ΠΎΡ€ΡƒΡŽ входят Ρ‚Ρ€ΠΈ Rэкв. сопротивлСния. Она ΠΈΠΌΠ΅Π΅Ρ‚ Π²ΠΈΠ΄, ΠΏΠΎΠΊΠ°Π·Π°Π½Π½Ρ‹ΠΉ Π½Π° рисункС Π½ΠΈΠΆΠ΅.

Π’Π΅ΠΏΠ΅Ρ€ΡŒ ΠΌΠΎΠΆΠ½ΠΎ ΠΎΠΏΡ€Π΅Π΄Π΅Π»ΠΈΡ‚ΡŒ эквивалСнтноС сопротивлСниС всСй ΠΏΠ΅Ρ€Π²ΠΎΠ½Π°Ρ‡Π°Π»ΡŒΠ½ΠΎΠΉ схСмы, слоТив эквивалСнтныС значСния всСх Ρ‚Ρ€Ρ‘Ρ… участков:

Rэкв. = Rэкв.AB + Rэкв.BC + Rэкв.CD = 0,83 + 15 + 0,41 = 56,83 Ом.

Π”Π°Π»Π΅Π΅, ΠΈΡΠΏΠΎΠ»ΡŒΠ·ΡƒΡ Π·Π°ΠΊΠΎΠ½ Ома, находят Ρ‚ΠΎΠΊ Π² послСднСм ΠΏΠΎΡΠ»Π΅Π΄ΠΎΠ²Π°Ρ‚Π΅Π»ΡŒΠ½ΠΎΠΌ участкС:

I = U/ Rэкв. = 24/56,83 = 0,42 А.

Зная силу Ρ‚ΠΎΠΊΠ°, ΠΌΠΎΠΆΠ½ΠΎ Π½Π°ΠΉΡ‚ΠΈ, ΠΊΠ°ΠΊΠΎΠ΅ ΠΏΠ°Π΄Π΅Π½ΠΈΠ΅ напряТСния Π½Π° рассмотрСнных участках AB, BC, CD. Π­Ρ‚ΠΎ выполняСтся ΡΠ»Π΅Π΄ΡƒΡŽΡ‰ΠΈΠΌ ΠΎΠ±Ρ€Π°Π·ΠΎΠΌ:

  • UAB = I* Rэкв.AB= 0,42*0,83 = 0,35 Π’;
  • UBC = I* Rэкв.BC= 0,42*15 = 6,3Π’;
  • UCD = I* Rэкв.CD = 0,42*0,41 = 0,17 Π’.

Π‘Π»Π΅Π΄ΡƒΡŽΡ‰ΠΈΠΌ шагом станСт ΠΎΠΏΡ€Π΅Π΄Π΅Π»Π΅Π½ΠΈΠ΅ Ρ‚ΠΎΠΊΠΎΠ² Π½Π° ΠΏΠ°Ρ€Π°Π»Π»Π΅Π»ΡŒΠ½Ρ‹Ρ… ΠΎΡ‚Ρ€Π΅Π·ΠΊΠ°Ρ… AB ΠΈ CD:

  • I1 = UAB/R1 = 0,35/1 = 0,35 А;
  • I2 = UAB/Rэкв.2,3 = 0,35/5 = 0,07 А;
  • I3 = UCD/R6 = 0,17/18 = 0,009 А;
  • I6 = UCD/Rэкв.7,8,9= 0,17/9 = 0,02 А;
  • I7 = UCD/R10 = 0,17/4 = 0,04 А.

Π”Π°Π»Π΅Π΅, Ρ‡Ρ‚ΠΎΠ±Ρ‹ Π½Π°ΠΉΡ‚ΠΈ значСния Ρ‚ΠΎΠΊΠΎΠ², проходящих Ρ‡Π΅Ρ€Π΅Π· R7 ΠΈ R8, Π½ΡƒΠΆΠ½ΠΎ Ρ€Π°ΡΡΡ‡ΠΈΡ‚Π°Ρ‚ΡŒ напряТСниС Π½Π° этих Π΄Π²ΡƒΡ… рСзисторах. ΠŸΡ€Π΅Π΄Π²Π°Ρ€ΠΈΡ‚Π΅Π»ΡŒΠ½ΠΎ находят ΠΏΠ°Π΄Π΅Π½ΠΈΠ΅ напряТСния Π½Π° R9.

U9 = R9*I6 = 8*0,02 = 0,16 Π’.

Π’Π΅ΠΏΠ΅Ρ€ΡŒ напряТСниС, ΠΏΠ°Π΄Π°ΡŽΡ‰Π΅Π΅ Π½Π° Rэкв.7,8, Π±ΡƒΠ΄Π΅Ρ‚ Ρ€Π°Π·Π½ΠΎΡΡ‚ΡŒΡŽ ΠΌΠ΅ΠΆΠ΄Ρƒ U CD ΠΈ U9.

U7,8 = UCD – U9= 0,17 – 0,16 = 1 Π’.

ПослС этого ΠΌΠΎΠΆΠ½ΠΎ ΡƒΠΆΠ΅ ΡƒΠ·Π½Π°Ρ‚ΡŒ Π·Π½Π°Ρ‡Π΅Π½ΠΈΠ΅ Ρ‚ΠΎΠΊΠΎΠ², двиТущихся ΠΏΠΎ рСзисторам R7 ΠΈ R8, ΠΈΡΠΏΠΎΠ»ΡŒΠ·ΡƒΡ Ρ„ΠΎΡ€ΠΌΡƒΠ»Ρ‹:

  • I4 = U7,8/R7 = 1/2 = 0,5 A;
  • I5 = U7,8/R8 = 1/2 = 0,5 A.

Π‘Ρ‚ΠΎΠΈΡ‚ Π·Π°ΠΌΠ΅Ρ‚ΠΈΡ‚ΡŒ! Π’ΠΎΠΊ, ΠΏΡ€ΠΎΡ‚Π΅ΠΊΠ°ΡŽΡ‰ΠΈΠΉ Ρ‡Π΅Ρ€Π΅Π· R4 ΠΈ R5, ΠΏΠΎ своСму Π·Π½Π°Ρ‡Π΅Π½ΠΈΡŽ Ρ€Π°Π²Π΅Π½ Ρ‚ΠΎΠΊΡƒ Π½Π° ΠΎΡ‚Ρ€Π΅Π·ΠΊΠ΅, Π½Π΅ ΠΈΠΌΠ΅ΡŽΡ‰Π΅ΠΌ развСтвлСния.

Рассчитывая схСмы ΠΈ Ρ€Π΅ΡˆΠ°Ρ Π·Π°Π΄Π°Ρ‡ΠΈ ΠΏΠΎ Π½Π°Ρ…ΠΎΠΆΠ΄Π΅Π½ΠΈΡŽ Π·Π½Π°Ρ‡Π΅Π½ΠΈΠΉ элСктричСских ΠΏΠ°Ρ€Π°ΠΌΠ΅Ρ‚Ρ€ΠΎΠ², Π½Π΅ΠΎΠ±Ρ…ΠΎΠ΄ΠΈΠΌΠΎ ΠΈΡΠΏΠΎΠ»ΡŒΠ·ΠΎΠ²Π°Ρ‚ΡŒ эквивалСнтныС сопротивлСния. Π‘ ΠΏΠΎΠΌΠΎΡ‰ΡŒΡŽ Ρ‚Π°ΠΊΠΎΠΉ Π·Π°ΠΌΠ΅Π½Ρ‹ слоТныС построСния ΠΏΡ€Π΅Π²Ρ€Π°Ρ‰Π°ΡŽΡ‚ΡΡ Π² элСмСнтарныС Ρ†Π΅ΠΏΠΈ, ΠΊΠΎΡ‚ΠΎΡ€Ρ‹Π΅ сводятся ΠΊ ΠΏΠ°Ρ€Π°Π»Π»Π΅Π»ΡŒΠ½Ρ‹ΠΌ ΠΈ ΠΏΠΎΡΠ»Π΅Π΄ΠΎΠ²Π°Ρ‚Π΅Π»ΡŒΠ½Ρ‹ΠΌ соСдинСниям рСзистивных элСмСнтов.

Π’ΠΈΠ΄Π΅ΠΎ

Π­ΠΊΠ²ΠΈΠ²Π°Π»Π΅Π½Ρ‚Π½ΠΎΠ΅ сопротивлСниС рСзисторов ΠΎΠΏΡ€Π΅Π΄Π΅Π»ΠΈΡ‚ΡŒ эквивалСнтноС

РасчСт Ρ€Π΅Π°Π»ΡŒΠ½ΠΎΠΉ элСктричСской Ρ†Π΅ΠΏΠΈ Π² идСальном Π²ΠΈΠ΄Π΅ Π½Π΅Π²ΠΎΠ·ΠΌΠΎΠΆΠ΅Π½ ΠΏΠΎ ΠΏΡ€ΠΈΡ‡ΠΈΠ½Π΅ отсутствия матСматичСских ΠΌΠ΅Ρ‚ΠΎΠ΄ΠΈΠΊ ΡƒΡ‡Π΅Ρ‚Π° ΠΈΠ½Π΄ΠΈΠ²ΠΈΠ΄ΡƒΠ°Π»ΡŒΠ½Ρ‹Ρ… ΠΏΠ°Ρ€Π°ΠΌΠ΅Ρ‚Ρ€ΠΎΠ² ΠΊΠ°ΠΆΠ΄ΠΎΠ³ΠΎ ΡΠΎΡΡ‚Π°Π²Π»ΡΡŽΡ‰Π΅Π³ΠΎ элСмСнта. Π­Ρ‚ΠΎ СстСствСнно, Ρ‚Π°ΠΊ ΠΊΠ°ΠΊ любая Π΄Π΅Ρ‚Π°Π»ΡŒ ΠΈΠΌΠ΅Π΅Ρ‚ свои ΠΏΠ°Ρ€Π°Π·ΠΈΡ‚Π½Ρ‹Π΅ характСристики, ΠΊΠΎΡ‚ΠΎΡ€Ρ‹Π΅ Π½Π΅Ρ€Π΅Π°Π»ΡŒΠ½ΠΎ ΡƒΡ‡Π΅ΡΡ‚ΡŒ ΠΏΡ€ΠΈ расчСтах. Для устранСния этой ΠΏΡ€ΠΎΠ±Π»Π΅ΠΌΡ‹ Π±Ρ‹Π»ΠΎ Π²Π²Π΅Π΄Π΅Π½ΠΎ понятиС эквивалСнтной Π·Π°ΠΌΠ΅Π½Ρ‹. ΠŸΡ€ΠΈ этом Π² расчСт принимаСтся Ρ‚ΠΎΠ»ΡŒΠΊΠΎ ΠΎΠ΄Π½Π° ΠΎΠΏΡ€Π΅Π΄Π΅Π»ΡΡŽΡ‰Π°Ρ характСристика элСмСнта. Π’Π°ΠΊ, Π½Π°ΠΏΡ€ΠΈΠΌΠ΅Ρ€, эквивалСнтноС сопротивлСниС рСзисторов Π² элСктричСской схСмС, ΠΎΡ‚ΠΎΠ±Ρ€Π°ΠΆΠ°Π΅Ρ‚ Ρ‚ΠΎΠ»ΡŒΠΊΠΎ Π²Π΅Π»ΠΈΡ‡ΠΈΠ½Ρƒ сопротивлСния Π±Π΅Π· влияния Π½Π° Π½Π΅Π³ΠΎ сторонних Ρ„Π°ΠΊΡ‚ΠΎΡ€ΠΎΠ².

Π’ элСктротСхникС сущСствуСт Π΄Π²Π° основных Π²Π°Ρ€ΠΈΠ°Π½Ρ‚Π° Π²ΠΊΠ»ΡŽΡ‡Π΅Π½ΠΈΡ Π΄Π΅Ρ‚Π°Π»Π΅ΠΉ Π² элСктричСской Ρ†Π΅ΠΏΠΈ – это ΠΏΠΎΡΠ»Π΅Π΄ΠΎΠ²Π°Ρ‚Π΅Π»ΡŒΠ½ΠΎΠ΅ ΠΈ ΠΏΠ°Ρ€Π°Π»Π»Π΅Π»ΡŒΠ½ΠΎΠ΅ соСдинСниС. ΠžΠ±ΡŠΠ΅Π΄ΠΈΠ½ΡΡŽΡ‰Π΅ΠΉ для Π½ΠΈΡ… являСтся смСшанная схСма, которая ΠΏΠΎ сути ΠΌΠΎΠΆΠ΅Ρ‚ Π±Ρ‹Ρ‚ΡŒ Ρ€Π°Π·Π±ΠΈΡ‚Π° Π½Π° участки с Π²Ρ‹ΡˆΠ΅ΠΏΡ€ΠΈΠ²Π΅Π΄Π΅Π½Π½Ρ‹ΠΌΠΈ характСристиками.

Рассмотрим эквивалСнтноС соСдинСниС рСзисторов Π² ΠΊΠ°ΠΆΠ΄ΠΎΠΌ ΠΎΡ‚Π΄Π΅Π»ΡŒΠ½ΠΎΠΌ случаС.

Π­ΠΊΠ²ΠΈΠ²Π°Π»Π΅Π½Ρ‚Π½ΠΎΠ΅ сопротивлСниС ΠΏΡ€ΠΈ ΠΏΠΎΡΠ»Π΅Π΄ΠΎΠ²Π°Ρ‚Π΅Π»ΡŒΠ½ΠΎ соСдинСнных рСзисторов

ΠŸΡ€ΠΈ Π΄Π°Π½Π½ΠΎΠΌ Ρ‚ΠΈΠΏΠ΅ размСщСния рСзисторов Π² Ρ†Π΅ΠΏΠΈ условная схСма Π±ΡƒΠ΄Π΅Ρ‚ ΡΠΎΠΎΡ‚Π²Π΅Ρ‚ΡΡ‚Π²ΠΎΠ²Π°Ρ‚ΡŒ рис. 1.

Рисунок 1

Для Ρ‚ΠΎΠ³ΠΎ Ρ‡Ρ‚ΠΎΠ±Ρ‹ ΠΎΠΏΡ€Π΅Π΄Π΅Π»ΠΈΡ‚ΡŒ эквивалСнтноС сопротивлСниС рСзисторов Π½Π΅ΠΎΠ±Ρ…ΠΎΠ΄ΠΈΠΌΠΎ Π²ΡΠΏΠΎΠΌΠ½ΠΈΡ‚ΡŒ Π·Π°ΠΊΠΎΠ½ Ома. Для ΠΏΠΎΡΠ»Π΅Π΄ΠΎΠ²Π°Ρ‚Π΅Π»ΡŒΠ½ΠΎΠ³ΠΎ соСдинСния ΠΎΠ½ гласит Ρ‡Ρ‚ΠΎ ΠΎΠ±Ρ‰Π΅Π΅, Π° Π² нашСм случаС эквивалСнтноС сопротивлСниС, соотвСтствуСт ΡΠ»Π΅Π΄ΡƒΡŽΡ‰Π΅ΠΌΡƒ ΡƒΡ€Π°Π²Π½Π΅Π½ΠΈΡŽ:

Rэкв= R1+R2+R3+RN-1+RN

Рассмотрим ΠΏΡ€ΠΈΠΌΠ΅Ρ€ ΠΏΠΎΡΠ»Π΅Π΄ΠΎΠ²Π°Ρ‚Π΅Π»ΡŒΠ½ΠΎΠ³ΠΎ соСдинСния Ρ‚Ρ€Π΅Ρ… рСзисторов, сопротивлСниС ΠΊΠΎΡ‚ΠΎΡ€Ρ‹Ρ… Ρ€Π°Π²Π½ΠΎ 10, 20 ΠΈ 30 Ом, соотвСтствСнно. Богласно Π²Ρ‹ΡˆΠ΅ ΠΏΡ€ΠΈΠ²Π΅Π΄Π΅Π½Π½ΠΎΠΉ Ρ„ΠΎΡ€ΠΌΡƒΠ»Π΅ ΠΎΠ±Ρ‰Π΅Π΅ сопротивлСниС всСх этих рСзисторов Π½Π° Π΄Π°Π½Π½ΠΎΠΌ участкС Ρ†Π΅ΠΏΠΈ Π±ΡƒΠ΄Π΅Ρ‚ Ρ€Π°Π²Π½ΠΎ 60 Ом. Π’Π°ΠΊΠΈΠΌ ΠΎΠ±Ρ€Π°Π·ΠΎΠΌ, ΠΏΡ€ΠΈ расчСтах ΠΏΠ°Ρ€Π°ΠΌΠ΅Ρ‚Ρ€ΠΎΠ² элСктричСской схСмы Π½Π΅Ρ‚ надобности ΠΈΡΠΏΠΎΠ»ΡŒΠ·ΠΎΠ²Π°Ρ‚ΡŒ ΠΈΠ½Π΄ΠΈΠ²ΠΈΠ΄ΡƒΠ°Π»ΡŒΠ½Ρ‹Π΅ характСристики ΠΎΡ‚Π΄Π΅Π»ΡŒΠ½Ρ‹Ρ… элСмСнтов. Π˜Ρ… ΠΌΠΎΠΆΠ½ΠΎ просто Π·Π°ΠΌΠ΅Π½ΠΈΡ‚ΡŒ ΠΎΠ΄Π½ΠΈΠΌ Π·Π½Π°Ρ‡Π΅Π½ΠΈΠ΅ΠΌ эквивалСнтным ΠΈΡ… суммС.

ΠšΡ€ΠΎΠΌΠ΅ Ρ‚Π΅ΠΎΡ€ΠΈΠΈ, Π΄Π°Π½Π½ΠΎΠ΅ суммированиС Π·Π½Π°Ρ‡Π΅Π½ΠΈΠΉ сопротивлСний элСмСнтов, ΠΈΠΌΠ΅Π΅Ρ‚ ΠΈ практичСскоС ΠΏΡ€ΠΈΠΌΠ΅Π½Π΅Π½ΠΈΠ΅ – Π² случаС нСобходимости всСгда ΠΌΠΎΠΆΠ½ΠΎ Π·Π°ΠΌΠ΅Π½ΠΈΡ‚ΡŒ нСсколько рСзисторов ΠΎΠ΄Π½ΠΈΠΌ. Π’Π°ΠΊΠΆΠ΅ ΠΈΠΌΠ΅Π΅Ρ‚ мСсто ΠΈ ΠΎΠ±Ρ€Π°Ρ‚Π½ΠΎΠ΅ ΡƒΡ‚Π²Π΅Ρ€ΠΆΠ΄Π΅Π½ΠΈΠ΅ – ΠΏΡ€ΠΈ отсутствии Π΄Π΅Ρ‚Π°Π»Π΅ΠΉ с Ρ‚Ρ€Π΅Π±ΡƒΠ΅ΠΌΠΎΠΉ характСристикой Π΅Π΅ ΠΌΠΎΠΆΠ½ΠΎ Π·Π°ΠΌΠ΅Π½ΠΈΡ‚ΡŒ Π½Π° нСсколько Π΄Ρ€ΡƒΠ³ΠΈΡ…, эквивалСнтноС сопротивлСниС ΠΊΠΎΡ‚ΠΎΡ€Ρ‹Ρ… Π±ΡƒΠ΄Π΅Ρ‚ ΡΠΎΠΎΡ‚Π²Π΅Ρ‚ΡΡ‚Π²ΠΎΠ²Π°Ρ‚ΡŒ Ρ‚Ρ€Π΅Π±ΡƒΠ΅ΠΌΠΎΠΌΡƒ Π·Π½Π°Ρ‡Π΅Π½ΠΈΡŽ.  ВсС это справСдливо ΠΈ для ΠΏΠ°Ρ€Π°Π»Π»Π΅Π»ΡŒΠ½ΠΎΠ³ΠΎ соСдинСния рСзисторов, Ρ‚ΠΎΠ»ΡŒΠΊΠΎ с Π½Π΅ΠΊΠΎΡ‚ΠΎΡ€Ρ‹ΠΌΠΈ особСнности.

Π­ΠΊΠ²ΠΈΠ²Π°Π»Π΅Π½Ρ‚Π½ΠΎΠ΅ сопротивлСниС ΠΏΡ€ΠΈ ΠΏΠ°Ρ€Π°Π»Π»Π΅Π»ΡŒΠ½ΠΎΠΌ соСдинСнии рСзисторов

ΠžΠ±Ρ‰Π°Ρ схСма ΠΏΡ€ΠΈ Π΄Π°Π½Π½ΠΎΠΌ Π²ΠΊΠ»ΡŽΡ‡Π΅Π½ΠΈΠΈ рСзисторов Π² Ρ†Π΅ΠΏΡŒ соотвСтствуСт рис. 2.


Рисунок 2

ΠžΠΏΡ€Π΅Π΄Π΅Π»ΠΈΡ‚ΡŒ эквивалСнтноС сопротивлСниС ΠΏΠ°Ρ€Π°Π»Π»Π΅Π»ΡŒΠ½ΠΎ соСдинСнных рСзисторов позволяСт Π·Π°ΠΊΠΎΠ½ Ома согласно ΠΊΠΎΡ‚ΠΎΡ€ΠΎΠΌΡƒ, Π² Π΄Π°Π½Π½ΠΎΠΌ Π²Π°Ρ€ΠΈΠ°Π½Ρ‚Π΅, справСдливо равСнство:

1/R экв =1/R1+1/R2+1/R3+1/R N-1+1/RN

Π’ΠΎΠ·Π²Ρ€Π°Ρ‰Π°ΡΡΡŒ ΠΊ Π½Π°ΡˆΠ΅ΠΌΡƒ ΠΏΡ€ΠΈΠΌΠ΅Ρ€Ρƒ с рСзисторами 10, 20 ΠΈ 30 Ом. МоТно ΠΎΠΏΡ€Π΅Π΄Π΅Π»ΠΈΡ‚ΡŒ эквивалСнтноС сопротивлСниС для Π΄Π°Π½Π½ΠΎΠ³ΠΎ случая, прСобразуя ΡƒΡ€Π°Π²Π½Π΅Π½ΠΈΠ΅ ΠΈ ΠΏΠΎΠ»ΡƒΡ‡Π°Π΅ΠΌ ΡΠ»Π΅Π΄ΡƒΡŽΡ‰ΡƒΡŽ Ρ„ΠΎΡ€ΠΌΡƒΠ»Ρƒ:

R экв = R1 Ρ… R2 Ρ… R3 / (R1 x R2) + (R1 x R3) + (R2 x R3) = 5,45 Ом

Π’Π°ΠΆΠ½Ρ‹ΠΉ ΠΌΠΎΠΌΠ΅Π½Ρ‚: ΠŸΡ€ΠΈ ΠΏΠ°Ρ€Π°Π»Π»Π΅Π»ΡŒΠ½ΠΎΠΌ Π²ΠΊΠ»ΡŽΡ‡Π΅Π½ΠΈΠΈ рСзисторов Π² Ρ†Π΅ΠΏΡŒ эквивалСнтноС сопротивлСниС Π±ΡƒΠ΄Π΅Ρ‚ всСгда мСньшС наимСньшСго значСния ΠΎΡ‚Π΄Π΅Π»ΡŒΠ½ΠΎΠ³ΠΎ элСмСнта. ΠŸΡ€ΠΈ ΠΏΠΎΡΠ»Π΅Π΄ΠΎΠ²Π°Ρ‚Π΅Π»ΡŒΠ½ΠΎΠΌ соСдинСнии R экв ΠΎΠ±ΡΠ·Π°Ρ‚Π΅Π»ΡŒΠ½ΠΎ большС самого большого ΠΏΠ°Ρ€Π°ΠΌΠ΅Ρ‚Ρ€Π°.

Π­ΠΊΠ²ΠΈΠ²Π°Π»Π΅Π½Ρ‚Π½ΠΎΠ΅ сопротивлСниС ΠΏΡ€ΠΈ смСшанном соСдинСнии рСзисторов

ΠžΠΏΡ€Π΅Π΄Π΅Π»Π΅Π½ΠΈΠ΅ эквивалСнтного сопротивлСния ΠΏΡ€ΠΈ смСшанном соСдинСнии рСзисторов Π½Π΅ прСдставляСт особых слоТностСй. Для этого достаточно Ρ€Π°Π·Π±ΠΈΡ‚ΡŒ ΡΡƒΡ‰Π΅ΡΡ‚Π²ΡƒΡŽΡ‰ΡƒΡŽ Ρ†Π΅ΠΏΠΎΡ‡ΠΊΡƒ Π½Π° логичСскиС ΡΠΎΡΡ‚Π°Π²Π»ΡΡŽΡ‰ΠΈΠ΅ – Π±Π»ΠΎΠΊΠΈ. Π’.Π΅. максимально ΡƒΠΏΡ€ΠΎΡΡ‚ΠΈΡ‚ΡŒ схСму, привСдя Π΅Π΅ Π² соотвСтствиС с характСристиками свойствСнных Ρ‚ΠΎΠΌΡƒ ΠΈΠ»ΠΈ ΠΈΠ½ΠΎΠΌΡƒ Ρ‚ΠΈΠΏΡƒ соСдинСния. На рис. 3 ΠΏΡ€ΠΈΠ²Π΅Π΄Π΅Π½Π° типичная схСма упрощСния, которая ΠΏΠΎΠ»ΡƒΡ‡ΠΈΠ»Π° Π½Π°Π·Π²Π°Π½ΠΈΠ΅ ΠΌΠ΅Ρ‚ΠΎΠ΄ свСртывания Ρ†Π΅ΠΏΠΈ.

Рисунок 3

Данная схСма позволяСт наглядно ΠΏΠΎΠ½ΡΡ‚ΡŒ, ΠΊΠ°ΠΊ ΠΌΠΎΠΆΠ½ΠΎ ΠΎΠΏΡ€Π΅Π΄Π΅Π»ΠΈΡ‚ΡŒ эквивалСнтноС сопротивлСниС рСзисторов ΠΏΡ€ΠΈ смСшанном соСдинСнии. ΠžΠ±Ρ€Π°Ρ‰Π°Π΅ΠΌ Π²Π½ΠΈΠΌΠ°Π½ΠΈΠ΅, Ρ‡Ρ‚ΠΎ Π½Π°Ρ‡ΠΈΠ½Π°Ρ‚ΡŒ процСсс упрощСния ΠΌΠΎΠΆΠ½ΠΎ Π² ΠΏΡ€ΠΎΠΈΠ·Π²ΠΎΠ»ΡŒΠ½ΠΎΠΌ порядкС. Π’Π°ΠΊ, Π½Π°ΠΏΡ€ΠΈΠΌΠ΅Ρ€, объСдинСниС рСзисторов R1 ΠΈ R2 Π½Π΅ ΠΎΠ±ΡΠ·Π°Ρ‚Π΅Π»ΡŒΠ½ΠΎ Π΄ΠΎΠ»ΠΆΠ½ΠΎ Π±Ρ‹Ρ‚ΡŒ ΠΏΠ΅Ρ€Π²Ρ‹ΠΌ шагом. МоТно ΡΠΎΠ²Π΅Ρ€ΡˆΠ΅Π½Π½ΠΎ смСло Π½Π° ΠΏΠ΅Ρ€Π²ΠΎΠΌ этапС Π½Π°ΠΉΡ‚ΠΈ R экв суммС сопротивлСний ΠΏΠΎΡΠ»Π΅Π΄ΠΎΠ²Π°Ρ‚Π΅Π»ΡŒΠ½ΠΎ Π²ΠΊΠ»ΡŽΡ‡Π΅Π½Π½Ρ‹Ρ… Π² Ρ†Π΅ΠΏΡŒ рСзисторов R4 ΠΈ R5. Β ΠžΠΏΡ€Π΅Π΄Π΅Π»Π΅Π½ΠΈΠ΅ эквивалСнтного сопротивлСния для рСзисторов Π½Π΅ΠΎΠ±Ρ…ΠΎΠ΄ΠΈΠΌΠΎ ΠΎΡΡƒΡ‰Π΅ΡΡ‚Π²Π»ΡΡ‚ΡŒ Π² зависимости ΠΎΡ‚ Ρ‚ΠΈΠΏΠ° соСдинСния.

Π’ Π·Π°ΠΊΠ»ΡŽΡ‡Π΅Π½ΠΈΠ΅ вСрнСмся ΠΊ самому ΠΏΠΎΠ½ΡΡ‚ΠΈΡŽ эквивалСнтной Π·Π°ΠΌΠ΅Π½Ρ‹ рСзисторов. Π’ рассмотрСнных Π½Π°ΠΌΠΈ случаях Ρ€Π΅Ρ‡ΡŒ шла ΠΎΠ± идСальном Π²Π°Ρ€ΠΈΠ°Π½Ρ‚Π΅. Π’ΠΎ Π΅ΡΡ‚ΡŒ Π² расчСт принимаСтся Ρ‚ΠΎΠ»ΡŒΠΊΠΎ Π²Π΅Π»ΠΈΡ‡ΠΈΠ½Π° сопротивлСния ΠΏΡ€ΠΈ Π½ΡƒΠ»Π΅Π²Ρ‹Ρ… значСниях ΠΎΡΡ‚Π°Π»ΡŒΠ½Ρ‹Ρ… характСристик. Π’Π°ΠΊΠΆΠ΅ ΠΎΠ±Ρ€Π°Ρ‰Π°Π΅ΠΌ Π²Π½ΠΈΠΌΠ°Π½ΠΈΠ΅, Ρ‡Ρ‚ΠΎ ΠΏΡ€ΠΈ составлСнии эквивалСнтной схСмы Π»ΡŽΠ±Ρ‹Ρ… элСмСнтов элСктричСской Ρ†Π΅ΠΏΠΈ, Π½Π΅ Ρ‚ΠΎΠ»ΡŒΠΊΠΎ рСзисторов, ΠΌΠΎΠΆΠ½ΠΎ Π²Π²ΠΎΠ΄ΠΈΡ‚ΡŒ Π΄ΠΎΠΏΠΎΠ»Π½ΠΈΡ‚Π΅Π»ΡŒΠ½Ρ‹Π΅ ΠΏΠ΅Ρ€Π΅ΠΌΠ΅Π½Π½Ρ‹Π΅, ΠΊΠΎΡ‚ΠΎΡ€Ρ‹Π΅ Π±ΡƒΠ΄ΡƒΡ‚ Π²Π»ΠΈΡΡ‚ΡŒ Π½Π° ΠΊΠΎΠ½Π΅Ρ‡Π½Ρ‹Π΅ ΠΈΡ‚ΠΎΠ³ΠΈ.

Β   На рис. 2.1 ΠΈΠ·ΠΎΠ±Ρ€Π°ΠΆΠ΅Π½Π° элСктричСская Ρ†Π΅ΠΏΡŒ с ΠΏΠΎΡΠ»Π΅Π΄ΠΎΠ²Π°Ρ‚Π΅Π»ΡŒΠ½ΠΎ соСдинСнными сопротивлСниями.


Рис. 2.1

НапряТСниС Π½Π° Π·Π°ΠΆΠΈΠΌΠ°Ρ… источника Π­Π”Π‘ Ρ€Π°Π²Π½ΠΎ Π²Π΅Π»ΠΈΡ‡ΠΈΠ½Π΅ элСктродвиТущСй силы. ΠŸΠΎΡΡ‚ΠΎΠΌΡƒ часто источник Π½Π° схСмС Π½Π΅ ΠΈΠ·ΠΎΠ±Ρ€Π°ΠΆΠ°ΡŽΡ‚.
ПадСния напряТСний Π½Π° сопротивлСниях ΠΎΠΏΡ€Π΅Π΄Π΅Π»ΡΡŽΡ‚ΡΡ ΠΏΠΎ Ρ„ΠΎΡ€ΠΌΡƒΠ»Π°ΠΌ

Π’ соотвСтствии со Π²Ρ‚ΠΎΡ€Ρ‹ΠΌ Π·Π°ΠΊΠΎΠ½ΠΎΠΌ ΠšΠΈΡ€Ρ…Π³ΠΎΡ„Π°, напряТСниС Π½Π° Π²Ρ…ΠΎΠ΄Π΅ элСктричСской Ρ†Π΅ΠΏΠΈ Ρ€Π°Π²Π½ΠΎ суммС ΠΏΠ°Π΄Π΅Π½ΠΈΠΉ напряТСний Π½Π° сопротивлСниях Ρ†Π΅ΠΏΠΈ.

Β  Β  Β  Β  Π³Π΄Π΅ Β  — эквивалСнтноС сопротивлСниС.

Β  Β  Π­ΠΊΠ²ΠΈΠ²Π°Π»Π΅Π½Ρ‚Π½ΠΎΠ΅ сопротивлСниС элСктричСской Ρ†Π΅ΠΏΠΈ, состоящСй ΠΈΠ· n ΠΏΠΎΡΠ»Π΅Π΄ΠΎΠ²Π°Ρ‚Π΅Π»ΡŒΠ½ΠΎ Π²ΠΊΠ»ΡŽΡ‡Π΅Π½Π½Ρ‹Ρ… элСмСнтов, Ρ€Π°Π²Π½ΠΎ суммС сопротивлСний этих элСмСнтов.

2.2. ΠŸΠ°Ρ€Π°Π»Π»Π΅Π»ΡŒΠ½ΠΎΠ΅ соСдинСниС элСмСнтов


элСктричСских Ρ†Π΅ΠΏΠ΅ΠΉ

На рис. 2.2 ΠΏΠΎΠΊΠ°Π·Π°Π½Π° элСктричСская Ρ†Π΅ΠΏΡŒ с ΠΏΠ°Ρ€Π°Π»Π»Π΅Π»ΡŒΠ½Ρ‹ΠΌ соСдинСниСм сопротивлСний.


Рис. 2.2

Π’ΠΎΠΊΠΈ Π² ΠΏΠ°Ρ€Π°Π»Π»Π΅Π»ΡŒΠ½Ρ‹Ρ… вСтвях ΠΎΠΏΡ€Π΅Π΄Π΅Π»ΡΡŽΡ‚ΡΡ ΠΏΠΎ Ρ„ΠΎΡ€ΠΌΡƒΠ»Π°ΠΌ:

Β  Β  Β  Β  Π³Π΄Π΅ — проводимости 1-ΠΉ, 2-ΠΉ ΠΈ n-ΠΉ Π²Π΅Ρ‚Π²Π΅ΠΉ.

Β  Β  Β  Π’ соотвСтствии с ΠΏΠ΅Ρ€Π²Ρ‹ΠΌ Π·Π°ΠΊΠΎΠ½ΠΎΠΌ ΠšΠΈΡ€Ρ…Π³ΠΎΡ„Π°, Ρ‚ΠΎΠΊ Π² Π½Π΅Ρ€Π°Π·Π²Π΅Ρ‚Π²Π»Π΅Π½Π½ΠΎΠΉ части схСмы Ρ€Π°Π²Π΅Π½ суммС Ρ‚ΠΎΠΊΠΎΠ² Π² ΠΏΠ°Ρ€Π°Π»Π»Π΅Π»ΡŒΠ½Ρ‹Ρ… вСтвях.

Β  Β  Β  Β  Π³Π΄Π΅ Β  Β   ЭквивалСнтная ΠΏΡ€ΠΎΠ²ΠΎΠ΄ΠΈΠΌΠΎΡΡ‚ΡŒ элСктричСской Ρ†Π΅ΠΏΠΈ, состоящСй ΠΈΠ· n ΠΏΠ°Ρ€Π°Π»Π»Π΅Π»ΡŒΠ½ΠΎ Π²ΠΊΠ»ΡŽΡ‡Π΅Π½Π½Ρ‹Ρ… элСмСнтов, Ρ€Π°Π²Π½Π° суммС проводимостСй ΠΏΠ°Ρ€Π°Π»Π»Π΅Π»ΡŒΠ½ΠΎ Π²ΠΊΠ»ΡŽΡ‡Π΅Π½Π½Ρ‹Ρ… элСмСнтов.
Π­ΠΊΠ²ΠΈΠ²Π°Π»Π΅Π½Ρ‚Π½Ρ‹ΠΌ сопротивлСниСм Ρ†Π΅ΠΏΠΈ называСтся Π²Π΅Π»ΠΈΡ‡ΠΈΠ½Π°, обратная эквивалСнтной проводимости

Β  ΠŸΡƒΡΡ‚ΡŒ элСктричСская схСма содСрТит Ρ‚Ρ€ΠΈ ΠΏΠ°Ρ€Π°Π»Π»Π΅Π»ΡŒΠ½ΠΎ Π²ΠΊΠ»ΡŽΡ‡Π΅Π½Π½Ρ‹Ρ… сопротивлСния.
ЭквивалСнтная ΠΏΡ€ΠΎΠ²ΠΎΠ΄ΠΈΠΌΠΎΡΡ‚ΡŒ

Β  Π­ΠΊΠ²ΠΈΠ²Π°Π»Π΅Π½Ρ‚Π½ΠΎΠ΅ сопротивлСниС схСмы, состоящСй ΠΈΠ· n ΠΎΠ΄ΠΈΠ½Π°ΠΊΠΎΠ²Ρ‹Ρ… элСмСнтов, Π² n Ρ€Π°Π· мСньшС сопротивлСний R ΠΎΠ΄Π½ΠΎΠ³ΠΎ элСмСнта

Π’ΠΎΠ·ΡŒΠΌΠ΅ΠΌ схСму, ΡΠΎΡΡ‚ΠΎΡΡ‰ΡƒΡŽ ΠΈΠ· Π΄Π²ΡƒΡ… ΠΏΠ°Ρ€Π°Π»Π»Π΅Π»ΡŒΠ½ΠΎ Π²ΠΊΠ»ΡŽΡ‡Π΅Π½Π½Ρ‹Ρ… сопротивлСний (рис. 2.3). Π˜Π·Π²Π΅ΡΡ‚Π½Ρ‹ Π²Π΅Π»ΠΈΡ‡ΠΈΠ½Ρ‹ сопротивлСний ΠΈ Ρ‚ΠΎΠΊ Π² Π½Π΅Ρ€Π°Π·Π²Π΅Ρ‚Π²Π»Π΅Π½Π½ΠΎΠΉ части схСмы. НСобходимо ΠΎΠΏΡ€Π΅Π΄Π΅Π»ΠΈΡ‚ΡŒ Ρ‚ΠΎΠΊΠΈ Π² ΠΏΠ°Ρ€Π°Π»Π»Π΅Π»ΡŒΠ½Ρ‹Ρ… вСтвях.


Рис. 2.3 ЭквивалСнтная ΠΏΡ€ΠΎΠ²ΠΎΠ΄ΠΈΠΌΠΎΡΡ‚ΡŒ схСмы

,

Β  Β  Π° эквивалСнтноС сопротивлСниС

Β  Β  Β  НапряТСниС Π½Π° Π²Ρ…ΠΎΠ΄Π΅ схСмы

Β  Β  Β  Β Π’ΠΎΠΊΠΈ Π² ΠΏΠ°Ρ€Π°Π»Π»Π΅Π»ΡŒΠ½Ρ‹Ρ… вСтвях

Β  Β  Β   Аналогично

Β  Β  Β  Π’ΠΎΠΊ Π² ΠΏΠ°Ρ€Π°Π»Π»Π΅Π»ΡŒΠ½ΠΎΠΉ Π²Π΅Ρ‚Π²ΠΈ Ρ€Π°Π²Π΅Π½ Ρ‚ΠΎΠΊΡƒ Π² Π½Π΅Ρ€Π°Π·Π²Π΅Ρ‚Π²Π»Π΅Π½Π½ΠΎΠΉ части схСмы, ΡƒΠΌΠ½ΠΎΠΆΠ΅Π½Π½ΠΎΠΌΡƒ Π½Π° сопротивлСниС ΠΏΡ€ΠΎΡ‚ΠΈΠ²ΠΎΠ»Π΅ΠΆΠ°Ρ‰Π΅ΠΉ, Ρ‡ΡƒΠΆΠΎΠΉ ΠΏΠ°Ρ€Π°Π»Π»Π΅Π»ΡŒΠ½ΠΎΠΉ Π²Π΅Ρ‚Π²ΠΈ ΠΈ Π΄Π΅Π»Π΅Π½Π½ΠΎΠΌΡƒ Π½Π° сумму сопротивлСний Ρ‡ΡƒΠΆΠΎΠΉ ΠΈ своСй ΠΏΠ°Ρ€Π°Π»Π»Π΅Π»ΡŒΠ½ΠΎ Π²ΠΊΠ»ΡŽΡ‡Π΅Π½Π½Ρ‹Ρ… Π²Π΅Ρ‚Π²Π΅ΠΉ.

2.3.ΠŸΡ€Π΅ΠΎΠ±Ρ€Π°Π·ΠΎΠ²Π°Π½ΠΈΠ΅ Ρ‚Ρ€Π΅ΡƒΠ³ΠΎΠ»ΡŒΠ½ΠΈΠΊΠ° сопротивлСний


Π² ΡΠΊΠ²ΠΈΠ²Π°Π»Π΅Π½Ρ‚Π½ΡƒΡŽ Π·Π²Π΅Π·Π΄Ρƒ

Π’ΡΡ‚Ρ€Π΅Ρ‡Π°ΡŽΡ‚ΡΡ схСмы, Π² ΠΊΠΎΡ‚ΠΎΡ€Ρ‹Ρ… ΠΎΡ‚ΡΡƒΡ‚ΡΡ‚Π²ΡƒΡŽΡ‚ сопротивлСния, Π²ΠΊΠ»ΡŽΡ‡Π΅Π½Π½Ρ‹Π΅ ΠΏΠΎΡΠ»Π΅Π΄ΠΎΠ²Π°Ρ‚Π΅Π»ΡŒΠ½ΠΎ ΠΈΠ»ΠΈ ΠΏΠ°Ρ€Π°Π»Π»Π΅Π»ΡŒΠ½ΠΎ, Π½Π°ΠΏΡ€ΠΈΠΌΠ΅Ρ€, мостовая схСма, изобраТСнная Π½Π° рис. 2.4. ΠžΠΏΡ€Π΅Π΄Π΅Π»ΠΈΡ‚ΡŒ эквивалСнтноС сопротивлСниС этой схСмы ΠΎΡ‚Π½ΠΎΡΠΈΡ‚Π΅Π»ΡŒΠ½ΠΎ Π²Π΅Ρ‚Π²ΠΈ с источником Π­Π”Π‘ описанными Π²Ρ‹ΡˆΠ΅ ΠΌΠ΅Ρ‚ΠΎΠ΄Π°ΠΌΠΈ нСльзя. Если ΠΆΠ΅ Π·Π°ΠΌΠ΅Π½ΠΈΡ‚ΡŒ Ρ‚Ρ€Π΅ΡƒΠ³ΠΎΠ»ΡŒΠ½ΠΈΠΊ сопротивлСний
R1-R2-R3, Π²ΠΊΠ»ΡŽΡ‡Π΅Π½Π½Ρ‹Ρ… ΠΌΠ΅ΠΆΠ΄Ρƒ ΡƒΠ·Π»Π°ΠΌΠΈ 1-2-3, Ρ‚Ρ€Π΅Ρ…Π»ΡƒΡ‡Π΅Π²ΠΎΠΉ Π·Π²Π΅Π·Π΄ΠΎΠΉ сопротивлСний, Π»ΡƒΡ‡ΠΈ ΠΊΠΎΡ‚ΠΎΡ€ΠΎΠΉ расходятся ΠΈΠ· Ρ‚ΠΎΡ‡ΠΊΠΈ 0 Π² Ρ‚Π΅ ΠΆΠ΅ ΡƒΠ·Π»Ρ‹ 1-2-3, эквивалСнтноС сопротивлСниС ΠΏΠΎΠ»ΡƒΡ‡Π΅Π½Π½ΠΎΠΉ схСмы Π»Π΅Π³ΠΊΠΎ опрСдСляСтся.


Рис. 2.4 Π‘ΠΎΠΏΡ€ΠΎΡ‚ΠΈΠ²Π»Π΅Π½ΠΈΠ΅ Π»ΡƒΡ‡Π° эквивалСнтной Π·Π²Π΅Π·Π΄Ρ‹ сопротивлСний Ρ€Π°Π²Π½ΠΎ ΠΏΡ€ΠΎΠΈΠ·Π²Π΅Π΄Π΅Π½ΠΈΡŽ сопротивлСний ΠΏΡ€ΠΈΠ»Π΅Π³Π°ΡŽΡ‰ΠΈΡ… сторон Ρ‚Ρ€Π΅ΡƒΠ³ΠΎΠ»ΡŒΠ½ΠΈΠΊΠ°, Π΄Π΅Π»Π΅Π½Π½ΠΎΠΌΡƒ Π½Π° сумму сопротивлСний всСх сторон Ρ‚Ρ€Π΅ΡƒΠ³ΠΎΠ»ΡŒΠ½ΠΈΠΊΠ°.
Π’ соотвСтствии с ΡƒΠΊΠ°Π·Π°Π½Π½Ρ‹ΠΌ ΠΏΡ€Π°Π²ΠΈΠ»ΠΎΠΌ, сопротивлСния Π»ΡƒΡ‡Π΅ΠΉ Π·Π²Π΅Π·Π΄Ρ‹ ΠΎΠΏΡ€Π΅Π΄Π΅Π»ΡΡŽΡ‚ΡΡ ΠΏΠΎ Ρ„ΠΎΡ€ΠΌΡƒΠ»Π°ΠΌ:

Β  Β  Π­ΠΊΠ²ΠΈΠ²Π°Π»Π΅Π½Ρ‚Π½ΠΎΠ΅ соСдинСниС ΠΏΠΎΠ»ΡƒΡ‡Π΅Π½Π½ΠΎΠΉ схСмы опрСдСляСтся ΠΏΠΎ Ρ„ΠΎΡ€ΠΌΡƒΠ»Π΅

Β  Β  Β   БопротивлСния R0 ΠΈ R?1 Π²ΠΊΠ»ΡŽΡ‡Π΅Π½Ρ‹ ΠΏΠΎΡΠ»Π΅Π΄ΠΎΠ²Π°Ρ‚Π΅Π»ΡŒΠ½ΠΎ, Π° Π²Π΅Ρ‚Π²ΠΈ с сопротивлСниями R?1 + R4 ΠΈ R?3 + R5 соСдинСны ΠΏΠ°Ρ€Π°Π»Π»Π΅Π»ΡŒΠ½ΠΎ.

2.4.ΠŸΡ€Π΅ΠΎΠ±Ρ€Π°Π·ΠΎΠ²Π°Π½ΠΈΠ΅ Π·Π²Π΅Π·Π΄Ρ‹ сопротивлСний


Π² эквивалСнтный Ρ‚Ρ€Π΅ΡƒΠ³ΠΎΠ»ΡŒΠ½ΠΈΠΊ

Иногда для упрощСния схСмы ΠΏΠΎΠ»Π΅Π·Π½ΠΎ ΠΏΡ€Π΅ΠΎΠ±Ρ€Π°Π·ΠΎΠ²Π°Ρ‚ΡŒ Π·Π²Π΅Π·Π΄Ρƒ сопротивлСний Π² эквивалСнтный Ρ‚Ρ€Π΅ΡƒΠ³ΠΎΠ»ΡŒΠ½ΠΈΠΊ.
Рассмотрим схСму Π½Π° рис. 2.5. Π—Π°ΠΌΠ΅Π½ΠΈΠΌ Π·Π²Π΅Π·Π΄Ρƒ сопротивлСний R1-R2-R3 эквивалСнтным Ρ‚Ρ€Π΅ΡƒΠ³ΠΎΠ»ΡŒΠ½ΠΈΠΊΠΎΠΌ сопротивлСний R?1-R?2-R?3, Π²ΠΊΠ»ΡŽΡ‡Π΅Π½Π½Ρ‹Ρ… ΠΌΠ΅ΠΆΠ΄Ρƒ ΡƒΠ·Π»Π°ΠΌΠΈ 1-2-3.


2.5. ΠŸΡ€Π΅ΠΎΠ±Ρ€Π°Π·ΠΎΠ²Π°Π½ΠΈΠ΅ Π·Π²Π΅Π·Π΄Ρ‹ сопротивлСний
Π² эквивалСнтный Ρ‚Ρ€Π΅ΡƒΠ³ΠΎΠ»ΡŒΠ½ΠΈΠΊ

Π‘ΠΎΠΏΡ€ΠΎΡ‚ΠΈΠ²Π»Π΅Π½ΠΈΠ΅ стороны эквивалСнтного Ρ‚Ρ€Π΅ΡƒΠ³ΠΎΠ»ΡŒΠ½ΠΈΠΊΠ° сопротивлСний Ρ€Π°Π²Π½ΠΎ суммС сопротивлСний Π΄Π²ΡƒΡ… ΠΏΡ€ΠΈΠ»Π΅Π³Π°ΡŽΡ‰ΠΈΡ… Π»ΡƒΡ‡Π΅ΠΉ Π·Π²Π΅Π·Π΄Ρ‹ плюс ΠΏΡ€ΠΎΠΈΠ·Π²Π΅Π΄Π΅Π½ΠΈΠ΅ этих ΠΆΠ΅ сопротивлСний, Π΄Π΅Π»Π΅Π½Π½ΠΎΠ΅ Π½Π° сопротивлСниС ΠΎΡΡ‚Π°Π²ΡˆΠ΅Π³ΠΎΡΡ (ΠΏΡ€ΠΎΡ‚ΠΈΠ²ΠΎΠ»Π΅ΠΆΠ°Ρ‰Π΅Π³ΠΎ) Π»ΡƒΡ‡Π°. БопротивлСния сторон Ρ‚Ρ€Π΅ΡƒΠ³ΠΎΠ»ΡŒΠ½ΠΈΠΊΠ° ΠΎΠΏΡ€Π΅Π΄Π΅Π»ΡΡŽΡ‚ΡΡ ΠΏΠΎ Ρ„ΠΎΡ€ΠΌΡƒΠ»Π°ΠΌ:

Π’Ρ…ΠΎΠ΄Π½ΠΎΠ΅ ΠΈ Π²Ρ‹Ρ…ΠΎΠ΄Π½ΠΎΠ΅ сопротивлСниС | ΠŸΡ€Π°ΠΊΡ‚ΠΈΡ‡Π΅ΡΠΊΠ°Ρ элСктроника

Π’Ρ…ΠΎΠ΄Π½ΠΎΠ΅ ΠΈ Π²Ρ‹Ρ…ΠΎΠ΄Π½ΠΎΠ΅ сопротивлСниС являСтся ΠΎΡ‡Π΅Π½ΡŒ Π²Π°ΠΆΠ½Ρ‹ΠΌ Π² элСктроникС.

ΠŸΡ€Π΅Π΄ΠΈΡΠ»ΠΎΠ²ΠΈΠ΅

Π›Π°Π΄Π½ΠΎ, Π½Π°Ρ‡Π½Π΅ΠΌ издалСка… Как Π²Ρ‹ Π·Π½Π°Π΅Ρ‚Π΅, всС элСктронныС устройства состоят ΠΈΠ· Π±Π»ΠΎΠΊΠΎΠ². Π˜Ρ… Π΅Ρ‰Π΅ часто Π½Π°Π·Ρ‹Π²Π°ΡŽΡ‚ каскады, ΠΌΠΎΠ΄ΡƒΠ»ΠΈ, ΡƒΠ·Π»Ρ‹ ΠΈ Ρ‚Π΄. Π’ нашСй ΡΡ‚Π°Ρ‚ΡŒΠ΅ Π±ΡƒΠ΄Π΅ΠΌ ΠΈΡΠΏΠΎΠ»ΡŒΠ·ΠΎΠ²Π°Ρ‚ΡŒ понятиС β€œΠ±Π»ΠΎΠΊβ€. НапримСр, источник питания, собранный ΠΏΠΎ этой схСмС:

состоит ΠΈΠ· Π΄Π²ΡƒΡ… Π±Π»ΠΎΠΊΠΎΠ². Π― ΠΈΡ… ΠΏΠΎΠΌΠ΅Ρ‚ΠΈΠ» Π² красном ΠΈ Π·Π΅Π»Π΅Π½ΠΎΠΌ ΠΏΡ€ΡΠΌΠΎΡƒΠ³ΠΎΠ»ΡŒΠ½ΠΈΠΊΠ°Ρ….

Π’ красном Π±Π»ΠΎΠΊΠ΅ ΠΌΡ‹ ΠΏΠΎΠ»ΡƒΡ‡Π°Π΅ΠΌ постоянноС напряТСниС, Π° Π² Π·Π΅Π»Π΅Π½ΠΎΠΌ Π±Π»ΠΎΠΊΠ΅ ΠΌΡ‹ Π΅Π³ΠΎ стабилизируСм. Π’ΠΎ Π΅ΡΡ‚ΡŒ блочная схСма Π±ΡƒΠ΄Π΅Ρ‚ Ρ‚Π°ΠΊΠΎΠΉ:

Блочная схСма – это условноС Π΄Π΅Π»Π΅Π½ΠΈΠ΅. Π’ этом ΠΏΡ€ΠΈΠΌΠ΅Ρ€Π΅ ΠΌΡ‹ ΠΌΠΎΠ³Π»ΠΈ Π±Ρ‹ Π΄Π°ΠΆΠ΅ Π²Π·ΡΡ‚ΡŒ трансформатор, ΠΊΠ°ΠΊ ΠΎΡ‚Π΄Π΅Π»ΡŒΠ½Ρ‹ΠΉ Π±Π»ΠΎΠΊ, ΠΊΠΎΡ‚ΠΎΡ€Ρ‹ΠΉ ΠΏΠΎΠ½ΠΈΠΆΠ°Π΅Ρ‚ ΠΏΠ΅Ρ€Π΅ΠΌΠ΅Π½Π½ΠΎΠ΅ напряТСниС ΠΎΠ΄Π½ΠΎΠ³ΠΎ Π½ΠΎΠΌΠΈΠ½Π°Π»Π° ΠΊ Π΄Ρ€ΡƒΠ³ΠΎΠΌΡƒ. Как Π½Π°ΠΌ ΡƒΠ΄ΠΎΠ±Π½Π΅Π΅, Ρ‚Π°ΠΊ ΠΈ Π΄Π΅Π»ΠΈΠΌ Π½Π° Π±Π»ΠΎΠΊΠΈ Π½Π°ΡˆΡƒ ΡΠ»Π΅ΠΊΡ‚Ρ€ΠΎΠ½Π½ΡƒΡŽ Π±Π΅Π·Π΄Π΅Π»ΡƒΡˆΠΊΡƒ. ΠœΠ΅Ρ‚ΠΎΠ΄ β€œΠΎΡ‚ простого ΠΊ слоТному” ΠΏΠΎΠ»Π½ΠΎΡΡ‚ΡŒΡŽ Ρ€Π°Π±ΠΎΡ‚Π°Π΅Ρ‚ Π² нашСм ΠΌΠΈΡ€Π΅. На низшСм ΡƒΡ€ΠΎΠ²Π½Π΅ находятся радиоэлСмСнты, Π½Π° Π²Ρ‹ΡΡˆΠ΅ΠΌ – Π³ΠΎΡ‚ΠΎΠ²ΠΎΠ΅ устройство, Π½Π°ΠΏΡ€ΠΈΠΌΠ΅Ρ€, Ρ‚Π΅Π»Π΅Π²ΠΈΠ·ΠΎΡ€.

Π›Π°Π΄Π½ΠΎ, Ρ‡Ρ‚ΠΎ-Ρ‚ΠΎ ΠΎΡ‚Π²Π»Π΅ΠΊΠ»ΠΈΡΡŒ. Как Π²Ρ‹ поняли, любоС устройство состоит ΠΈΠ· Π±Π»ΠΎΠΊΠΎΠ², ΠΊΠΎΡ‚ΠΎΡ€Ρ‹Π΅ Π²Ρ‹ΠΏΠΎΠ»Π½ΡΡŽΡ‚ ΠΎΠΏΡ€Π΅Π΄Π΅Π»Π΅Π½Π½ΡƒΡŽ Ρ„ΡƒΠ½ΠΊΡ†ΠΈΡŽ.

– Ага! Π’Π°ΠΊ Ρ‡Ρ‚ΠΎ ΠΆΠ΅ получаСтся? Π― ΠΌΠΎΠ³Ρƒ просто Ρ‚ΡƒΠΏΠΎ Π²Π·ΡΡ‚ΡŒ Π³ΠΎΡ‚ΠΎΠ²Ρ‹Π΅ Π±Π»ΠΎΠΊΠΈ ΠΈ изобрСсти любоС элСктронноС устройство, ΠΊΠΎΡ‚ΠΎΡ€ΠΎΠ΅ ΠΌΠ½Π΅ ΠΏΡ€ΠΈΠ΄Π΅Ρ‚ Π² Π³ΠΎΠ»ΠΎΠ²Ρƒ?

Π”Π°! ИмСнно Π½Π° это Π½Π°Ρ†Π΅Π»Π΅Π½Π° сСйчас соврСмСнная элСктроника πŸ˜‰ ΠœΠΈΠΊΡ€ΠΎΠΊΠΎΠ½Ρ‚Ρ€ΠΎΠ»Π»Π΅Ρ€Ρ‹Β  ΠΈ конструкторы, Ρ‚ΠΈΠΏΠ° Arduino, Π΄ΠΎΠ±Π°Π²Π»ΡΡŽΡ‚ Π΅Ρ‰Π΅ большС гибкости Π² творчСскиС начинания ΠΌΠΎΠ»ΠΎΠ΄Ρ‹Ρ… ΠΈΠ·ΠΎΠ±Ρ€Π΅Ρ‚Π°Ρ‚Π΅Π»Π΅ΠΉ.

На словах всС Π²Ρ‹Ρ…ΠΎΠ΄ΠΈΡ‚ прСкрасно, Π½ΠΎ всСгда Π΅ΡΡ‚ΡŒ ΠΏΠΎΠ΄Π²ΠΎΠ΄Π½Ρ‹Π΅ ΠΊΠ°ΠΌΠ½ΠΈ, ΠΊΠΎΡ‚ΠΎΡ€Ρ‹Π΅ слСдуСт ΠΈΠ·ΡƒΡ‡ΠΈΡ‚ΡŒ, Ρ‡Ρ‚ΠΎΠ±Ρ‹ Π½Π°Ρ‡Π°Ρ‚ΡŒ ΠΏΡ€ΠΎΠ΅ΠΊΡ‚ΠΈΡ€ΠΎΠ²Π°Ρ‚ΡŒ элСктронныС устройства. НСкоторыС ΠΈΠ· этих ΠΊΠ°ΠΌΡƒΡˆΠΊΠΎΠ² Π½Π°Π·Ρ‹Π²Π°ΡŽΡ‚ΡΡ Π²Ρ…ΠΎΠ΄Π½Ρ‹ΠΌ ΠΈ Π²Ρ‹Ρ…ΠΎΠ΄Π½Ρ‹ΠΌ сопротивлСниСм.

Π”ΡƒΠΌΠ°ΡŽ, всС помнят, Ρ‡Ρ‚ΠΎ Ρ‚Π°ΠΊΠΎΠ΅ сопротивлСниС ΠΈ Ρ‡Ρ‚ΠΎ Ρ‚Π°ΠΊΠΎΠ΅ рСзистор. РСзистор Ρ…ΠΎΡ‚ΡŒ ΠΈ ΠΎΠ±Π»Π°Π΄Π°Π΅Ρ‚ сопротивлСниСм, Π½ΠΎ это Π°ΠΊΡ‚ΠΈΠ²Π½ΠΎΠ΅ сопротивлСниС. ΠšΠ°Ρ‚ΡƒΡˆΠΊΠ° индуктивности ΠΈ кондСнсатор Π±ΡƒΠ΄ΡƒΡ‚ ΡƒΠΆΠ΅ ΠΎΠ±Π»Π°Π΄Π°Ρ‚ΡŒ, Ρ‚Π°ΠΊ Π½Π°Π·Ρ‹Π²Π°Π΅ΠΌΡ‹ΠΌ, Ρ€Π΅Π°ΠΊΡ‚ΠΈΠ²Π½Ρ‹ΠΌ сопротивлСниСм. Но Ρ‡Ρ‚ΠΎ Ρ‚Π°ΠΊΠΎΠ΅ Π²Ρ…ΠΎΠ΄Π½ΠΎΠ΅ ΠΈ Π²Ρ‹Ρ…ΠΎΠ΄Π½ΠΎΠ΅ сопротивлСниС? Π­Ρ‚ΠΎ ΡƒΠΆΠ΅ Ρ‡Ρ‚ΠΎ-Ρ‚ΠΎ новСнькоС. Если ΠΏΡ€ΠΈΡΠ»ΡƒΡˆΠ°Ρ‚ΡŒΡΡ ΠΊ этим Ρ„Ρ€Π°Π·Π°ΠΌ, Ρ‚ΠΎ Π²Ρ…ΠΎΠ΄Π½ΠΎΠ΅ сопротивлСниС – это сопротивлСниС ΠΊΠ°ΠΊΠΎΠ³ΠΎ-Ρ‚ΠΎ Π²Ρ…ΠΎΠ΄Π°, Π° Π²Ρ‹Ρ…ΠΎΠ΄Π½ΠΎΠ΅ – сопротивлСниС ΠΊΠ°ΠΊΠΎΠ³ΠΎ-Π»ΠΈΠ±ΠΎ Π²Ρ‹Ρ…ΠΎΠ΄Π°. Ну Π΄Π°, всС ΠΏΠΎΡ‡Ρ‚ΠΈ Ρ‚Π°ΠΊ ΠΈ Π΅ΡΡ‚ΡŒ. И Π³Π΄Π΅ ΠΆΠ΅ Π½Π°ΠΌ Π½Π°ΠΉΡ‚ΠΈ Π² схСмС эти Π²Ρ…ΠΎΠ΄Π½Ρ‹Π΅ ΠΈ Π²Ρ‹Ρ…ΠΎΠ΄Π½Ρ‹Π΅ сопротивлСния?Β  А Π²ΠΎΡ‚ β€œΠΏΡ€ΡΡ‡ΡƒΡ‚ΡΡβ€ ΠΎΠ½ΠΈ Π² самих Π±Π»ΠΎΠΊΠ°Ρ… радиоэлСктронных устройств.

Π’Ρ…ΠΎΠ΄Π½ΠΎΠ΅ сопротивлСниС

Π˜Ρ‚Π°ΠΊ, ΠΈΠΌΠ΅Π΅ΠΌ ΠΊΠ°ΠΊΠΎΠΉ-Π»ΠΈΠ±ΠΎ Π±Π»ΠΎΠΊ. Как принято Π²ΠΎ всСм ΠΌΠΈΡ€Π΅, слСва – это Π²Ρ…ΠΎΠ΄ Π±Π»ΠΎΠΊΠ°, справа – Π²Ρ‹Ρ…ΠΎΠ΄.

Как ΠΈ полагаСтся, этот Π±Π»ΠΎΠΊ ΠΈΡΠΏΠΎΠ»ΡŒΠ·ΡƒΠ΅Ρ‚ΡΡ Π² ΠΊΠ°ΠΊΠΎΠΌ-Π½ΠΈΠ±ΡƒΠ΄ΡŒ радиоэлСктронном устройствС ΠΈ выполняСт ΠΊΠ°ΠΊΡƒΡŽ-Π»ΠΈΠ±ΠΎ Ρ„ΡƒΠ½ΠΊΡ†ΠΈΡŽ. Π—Π½Π°Ρ‡ΠΈΡ‚, Π½Π° Π΅Π³ΠΎ Π²Ρ…ΠΎΠ΄ Π±ΡƒΠ΄Π΅Ρ‚ ΠΏΠΎΠ΄Π°Π²Π°Ρ‚ΡŒΡΡ ΠΊΠ°ΠΊΠΎΠ΅-Ρ‚ΠΎ Π²Ρ…ΠΎΠ΄Π½ΠΎΠ΅ напряТСниС UΠ²Ρ… ΠΎΡ‚ Π΄Ρ€ΡƒΠ³ΠΎΠ³ΠΎ Π±Π»ΠΎΠΊΠ° ΠΈΠ»ΠΈ ΠΎΡ‚ источника питания, Π° Π½Π° Π΅Π³ΠΎ Π²Ρ‹Ρ…ΠΎΠ΄Π΅ появится напряТСниС UΠ²Ρ‹Ρ… (ΠΈΠ»ΠΈ Π½Π΅ появится, Ссли Π±Π»ΠΎΠΊ являСтся ΠΊΠΎΠ½Π΅Ρ‡Π½Ρ‹ΠΌ).

Но Ρ€Π°Π· ΡƒΠΆ ΠΌΡ‹ ΠΏΠΎΠ΄Π°Π΅ΠΌ напряТСниС Π½Π° Π²Ρ…ΠΎΠ΄ (Π²Ρ…ΠΎΠ΄Π½ΠΎΠ΅ напряТСниС UΠ²Ρ…), ΡΠ»Π΅Π΄ΠΎΠ²Π°Ρ‚Π΅Π»ΡŒΠ½ΠΎ, Ρƒ нас этот Π±Π»ΠΎΠΊ Π±ΡƒΠ΄Π΅Ρ‚ ΠΊΡƒΡˆΠ°Ρ‚ΡŒ ΠΊΠ°ΠΊΡƒΡŽ-Ρ‚ΠΎ силу Ρ‚ΠΎΠΊΠ° IΠ²Ρ….

Π’Π΅ΠΏΠ΅Ρ€ΡŒ самоС интСрСсноС… ΠžΡ‚ Ρ‡Π΅Π³ΠΎ зависит IΠ²Ρ… ? Π’ΠΎΠΎΠ±Ρ‰Π΅, ΠΎΡ‚ Ρ‡Π΅Π³ΠΎ зависит сила Ρ‚ΠΎΠΊΠ° Π² Ρ†Π΅ΠΏΠΈ? ВспоминаСм Π·Π°ΠΊΠΎΠ½ Ома для участка Ρ†Π΅ΠΏΠΈ :

Π—Π½Π°Ρ‡ΠΈΡ‚, сила Ρ‚ΠΎΠΊΠ° Ρƒ нас зависит ΠΎΡ‚ напряТСния ΠΈ ΠΎΡ‚ сопротивлСния. ΠŸΡ€Π΅Π΄ΠΏΠΎΠ»ΠΎΠΆΠΈΠΌ, Ρ‡Ρ‚ΠΎ напряТСниС Ρƒ нас Π½Π΅ мСняСтся, ΡΠ»Π΅Π΄ΠΎΠ²Π°Ρ‚Π΅Π»ΡŒΠ½ΠΎ, сила Ρ‚ΠΎΠΊΠ° Π² Ρ†Π΅ΠΏΠΈ Π±ΡƒΠ΄Π΅Ρ‚ Π·Π°Π²ΠΈΡΠ΅Ρ‚ΡŒ от… Π‘ΠžΠŸΠ ΠžΠ’Π˜Π’Π›Π•ΠΠ˜Π―. Но Π³Π΄Π΅ Π½Π°ΠΌ Π΅Π³ΠΎ Π½Π°ΠΉΡ‚ΠΈ?Β  А прячСтся ΠΎΠ½ΠΎ Π² самом каскадС ΠΈ называСтся Π²Ρ…ΠΎΠ΄Π½Ρ‹ΠΌ сопротивлСниСм.

Π’ΠΎ Π΅ΡΡ‚ΡŒ, Ρ€Π°Π·ΠΎΠ±Ρ€Π°Π² Ρ‚Π°ΠΊΠΎΠΉ Π±Π»ΠΎΠΊ, Π²Π½ΡƒΡ‚Ρ€ΠΈ Π½Π΅Π³ΠΎ ΠΌΡ‹ ΠΌΠΎΠΆΠ΅ΠΌ Π½Π°ΠΉΡ‚ΠΈ этот рСзистор? ΠšΠΎΠ½Π΅Ρ‡Π½ΠΎ ΠΆΠ΅ Π½Π΅Ρ‚). Он являСтся своСго Ρ€ΠΎΠ΄Π° сопротивлСниСм радиоэлСмСнтов, соСдинСнных ΠΏΠΎ схСмС этого Π±Π»ΠΎΠΊΠ°. Π‘ΠΊΠ°ΠΆΠ΅ΠΌ Ρ‚Π°ΠΊ, совокупноС сопротивлСниС.

Как ΠΈΠ·ΠΌΠ΅Ρ€ΠΈΡ‚ΡŒ Π²Ρ…ΠΎΠ΄Π½ΠΎΠ΅ сопротивлСниС

Как ΠΌΡ‹ Π·Π½Π°Π΅ΠΌ, Π½Π° ΠΊΠ°ΠΆΠ΄Ρ‹ΠΉ Π±Π»ΠΎΠΊ подаСтся ΠΊΠ°ΠΊΠΎΠΉ-Π»ΠΈΠ±ΠΎ сигнал ΠΎΡ‚ ΠΏΡ€Π΅Π΄Ρ‹Π΄ΡƒΡ‰Π΅Π³ΠΎ Π±Π»ΠΎΠΊΠ° ΠΈΠ»ΠΈ это ΠΌΠΎΠΆΠ΅Ρ‚ Π±Ρ‹Ρ‚ΡŒ Π΄Π°ΠΆΠ΅ ΠΏΠΈΡ‚Π°Π½ΠΈΠ΅ ΠΎΡ‚ сСти ΠΈΠ»ΠΈ Π±Π°Ρ‚Π°Ρ€Π΅ΠΈ. Π§Ρ‚ΠΎ Π½Π°ΠΌ остаСтся ΡΠ΄Π΅Π»Π°Ρ‚ΡŒ?

1)Π—Π°ΠΌΠ΅Ρ€ΠΈΡ‚ΡŒ напряТСниС UΠ²Ρ…, ΠΏΠΎΠ΄Π°Π²Π°Π΅ΠΌΠΎΠ΅ Π½Π° этот Π±Π»ΠΎΠΊ

2)Π—Π°ΠΌΠ΅Ρ€ΠΈΡ‚ΡŒ силу Ρ‚ΠΎΠΊΠ° IΠ²Ρ…, ΠΊΠΎΡ‚ΠΎΡ€ΡƒΡŽ потрСбляСт наш Π±Π»ΠΎΠΊ

3) По Π·Π°ΠΊΠΎΠ½Ρƒ Ома Π½Π°ΠΉΡ‚ΠΈ Π²Ρ…ΠΎΠ΄Π½ΠΎΠ΅ сопротивлСниС RΠ²Ρ….

Если Ρƒ вас Π²Ρ…ΠΎΠ΄Π½ΠΎΠ΅ сопротивлСниС получаСтся ΠΎΡ‡Π΅Π½ΡŒ большоС, Ρ‡Ρ‚ΠΎΠ±Ρ‹ Π·Π°ΠΌΠ΅Ρ€ΠΈΡ‚ΡŒ Π΅Π³ΠΎ ΠΊΠ°ΠΊ ΠΌΠΎΠΆΠ½ΠΎ Ρ‚ΠΎΡ‡Π½Π΅Π΅, ΠΈΡΠΏΠΎΠ»ΡŒΠ·ΡƒΡŽΡ‚ Π²ΠΎΡ‚ Ρ‚Π°ΠΊΡƒΡŽ схСму.

ΠœΡ‹Β  с Π²Π°ΠΌΠΈ Π·Π½Π°Π΅ΠΌ, Ρ‡Ρ‚ΠΎ Ссли Π²Ρ…ΠΎΠ΄Π½ΠΎΠ΅ сопротивлСниС Ρƒ нас большоС, Ρ‚ΠΎ входная сила Ρ‚ΠΎΠΊΠ° Π² Ρ†Π΅ΠΏΠΈ Ρƒ нас Π±ΡƒΠ΄Π΅Ρ‚ ΠΎΡ‡Π΅Π½ΡŒ малСнькая (ΠΈΠ· Π·Π°ΠΊΠΎΠ½Π° Ома).

ПадСниС напряТСния Π½Π° рСзисторС R ΠΎΠ±ΠΎΠ·Π½Π°Ρ‡ΠΈΠΌ, ΠΊΠ°ΠΊ UR

Из всСго этого получаСм…

Когда ΠΌΡ‹ ΠΏΡ€ΠΎΠ²ΠΎΠ΄ΠΈΠΌ эти измСрСния, ΠΈΠΌΠ΅ΠΉΡ‚Π΅ Π²Π²ΠΈΠ΄Ρƒ, Ρ‡Ρ‚ΠΎ напряТСниС Π½Π° Π²Ρ‹Ρ…ΠΎΠ΄Π΅ Π³Π΅Π½Π΅Ρ€Π°Ρ‚ΠΎΡ€Π° Π½Π΅ Π΄ΠΎΠ»ΠΆΠ½ΠΎ ΠΌΠ΅Π½ΡΡ‚ΡŒΡΡ!

[quads id=1]

Π˜Ρ‚Π°ΠΊ, Π΄Π°Π²Π°ΠΉΡ‚Π΅ посчитаСм, ΠΊΠ°ΠΊΠΎΠΉ ΠΆΠ΅ рСзистор Π½Π°ΠΌ Π½Π΅ΠΎΠ±Ρ…ΠΎΠ΄ΠΈΠΌΠΎ ΠΏΠΎΠ΄ΠΎΠ±Ρ€Π°Ρ‚ΡŒ, Ρ‡Ρ‚ΠΎΠ±Ρ‹ ΠΊΠ°ΠΊ ΠΌΠΎΠΆΠ½ΠΎ Ρ‚ΠΎΡ‡Π½Π΅Π΅ Π·Π°ΠΌΠ΅Ρ€ΡΡ‚ΡŒ это Π²Ρ…ΠΎΠ΄Π½ΠΎΠ΅ сопротивлСниС. Допустим, Ρ‡Ρ‚ΠΎ Ρƒ нас Π²Ρ…ΠΎΠ΄Π½ΠΎΠ΅ сопротивлСниС RΠ²Ρ…=1 МСгаОм, Π° рСзистор взяли  R=1 КилоОм. ΠŸΡƒΡΡ‚ΡŒ Π³Π΅Π½Π΅Ρ€Π°Ρ‚ΠΎΡ€ Π²Ρ‹Π΄Π°Π΅Ρ‚ постоянноС напряТСниС U=10 Π’ΠΎΠ»ΡŒΡ‚. Π’ Ρ€Π΅Π·ΡƒΠ»ΡŒΡ‚Π°Ρ‚Π΅, Ρƒ нас получаСтся Ρ†Π΅ΠΏΡŒ с двумя сопротивлСниями. ΠŸΡ€Π°Π²ΠΈΠ»ΠΎ дСлитСля напряТСния гласит: сумма ΠΏΠ°Π΄Π΅Π½ΠΈΠΉ напряТСний Π½Π° всСх сопротивлСниях Π² Ρ†Π΅ΠΏΠΈ равняСтся Π­Π”Π‘ Π³Π΅Π½Π΅Ρ€Π°Ρ‚ΠΎΡ€Π°.

Π’ Ρ€Π΅Π·ΡƒΠ»ΡŒΡ‚Π°Ρ‚Π΅ получаСтся Ρ†Π΅ΠΏΡŒ:

 ВысчитываСм силу Ρ‚ΠΎΠΊΠ° Π² Ρ†Π΅ΠΏΠΈ Π² АмпСрах

ΠŸΠΎΠ»ΡƒΡ‡Π°Π΅Ρ‚ΡΡ, Ρ‡Ρ‚ΠΎ ΠΏΠ°Π΄Π΅Π½ΠΈΠ΅ напряТСния Π½Π° сопротивлСнии R Π² Π’ΠΎΠ»ΡŒΡ‚Π°Ρ… Π±ΡƒΠ΄Π΅Ρ‚:

Π“Ρ€ΡƒΠ±ΠΎ говоря 0,01 Π’ΠΎΠ»ΡŒΡ‚. Вряд Π»ΠΈ Π²Ρ‹ смоТСтС Ρ‚ΠΎΡ‡Π½ΠΎ Π·Π°ΠΌΠ΅Ρ€ΠΈΡ‚ΡŒ Ρ‚Π°ΠΊΠΎΠ΅ малСнькоС напряТСниС Π½Π° своСм китайском ΠΌΡƒΠ»ΡŒΡ‚ΠΈΠΌΠ΅Ρ‚Ρ€Π΅.

Какой ΠΎΡ‚ΡΡŽΠ΄Π° Π²Ρ‹Π²ΠΎΠ΄? Для Π±ΠΎΠ»Π΅Π΅ Ρ‚ΠΎΡ‡Π½ΠΎΠ³ΠΎ измСрСния высокого Π²Ρ…ΠΎΠ΄Π½ΠΎΠ³ΠΎ сопротивлСния Π½Π°Π΄ΠΎ Π±Ρ€Π°Ρ‚ΡŒ Π΄ΠΎΠ±Π°Π²ΠΎΡ‡Π½ΠΎΠ΅ сопротивлСниС Ρ‚Π°ΠΊΠΆΠ΅Β  ΠΎΡ‡Π΅Π½ΡŒ большого Π½ΠΎΠΌΠΈΠ½Π°Π»Π°.Β  Π’ этом случаС Ρ€Π°Π±ΠΎΡ‚Π°Π΅Ρ‚ ΠΏΡ€Π°Π²ΠΈΠ»ΠΎ ΡˆΡƒΠ½Ρ‚Π°: Π½Π° бОльшСм сопротивлСнии ΠΏΠ°Π΄Π°Π΅Ρ‚ бОльшСС напряТСниС, ΠΈ Π½Π°ΠΎΠ±ΠΎΡ€ΠΎΡ‚, Π½Π° мСньшСм сопротивлСнии ΠΏΠ°Π΄Π°Π΅Ρ‚ мСньшСС напряТСниС.

Π˜Π·ΠΌΠ΅Ρ€Π΅Π½ΠΈΠ΅ Π²Ρ…ΠΎΠ΄Π½ΠΎΠ³ΠΎ сопротивлСния Π½Π° ΠΏΡ€Π°ΠΊΡ‚ΠΈΠΊΠ΅

Ну всС, Π·Π°ΠΏΠ°Ρ€ΠΊΠ° ΠΏΡ€ΠΎΡˆΠ»Π° ;-). Π”Π°Π²Π°ΠΉΡ‚Π΅ Ρ‚Π΅ΠΏΠ΅Ρ€ΡŒ Π½Π° ΠΏΡ€Π°ΠΊΡ‚ΠΈΠΊΠ΅ ΠΏΠΎΠΏΡ€ΠΎΠ±ΡƒΠ΅ΠΌ Π·Π°ΠΌΠ΅Ρ€ΠΈΡ‚ΡŒ Π²Ρ…ΠΎΠ΄Π½ΠΎΠ΅ сопротивлСниС ΠΊΠ°ΠΊΠΎΠ³ΠΎ-Π»ΠΈΠ±ΠΎ устройства. Мой взгляд сразу ΡƒΠΏΠ°Π» Π½Π° Вранзистор-ΠΌΠ΅Ρ‚Ρ€. Π˜Ρ‚Π°ΠΊ, выставляСм Π½Π° Π±Π»ΠΎΠΊΠ΅ питания Ρ€Π°Π±ΠΎΡ‡Π΅Π΅ напряТСниС этого транзистор-ΠΌΠ΅Ρ‚Ρ€Π°, Ρ‚ΠΎ Π΅ΡΡ‚ΡŒ 9 Π’ΠΎΠ»ΡŒΡ‚, ΠΈ Π²ΠΎ Π²ΠΊΠ»ΡŽΡ‡Π΅Π½Π½ΠΎΠΌ состоянии замСряСм ΠΏΠΎΡ‚Ρ€Π΅Π±Π»ΡΠ΅ΠΌΡƒΡŽ силу Ρ‚ΠΎΠΊΠ°. Как Π·Π°ΠΌΠ΅Ρ€ΠΈΡ‚ΡŒ силу Ρ‚ΠΎΠΊΠ° Π² Ρ†Π΅ΠΏΠΈ, Ρ‡ΠΈΡ‚Π°Π΅ΠΌ Π² этой ΡΡ‚Π°Ρ‚ΡŒΠ΅. По схСмС всС это Π±ΡƒΠ΄Π΅Ρ‚ Π²Ρ‹Π³Π»ΡΠ΄Π΅Ρ‚ΡŒ Π²ΠΎΡ‚ Ρ‚Π°ΠΊ:

А Π½Π° Π΄Π΅Π»Π΅ Π²ΠΎΡ‚ Ρ‚Π°ΠΊ:

Π˜Ρ‚Π°ΠΊ, Ρƒ нас ΠΏΠΎΠ»ΡƒΡ‡ΠΈΠ»ΠΎΡΡŒ 22,5 миллиАмпСр.

Π’Π΅ΠΏΠ΅Ρ€ΡŒ, зная Π·Π½Π°Ρ‡Π΅Π½ΠΈΠ΅ потрСбляСмого Ρ‚ΠΎΠΊΠ°, ΠΌΠΎΠΆΠ½ΠΎ Π½Π°ΠΉΡ‚ΠΈ ΠΏΠΎ этой Ρ„ΠΎΡ€ΠΌΡƒΠ»Π΅ Π²Ρ…ΠΎΠ΄Π½ΠΎΠ΅ сопротивлСниС:

ΠŸΠΎΠ»ΡƒΡ‡Π°Π΅ΠΌ:

Π’Ρ‹Ρ…ΠΎΠ΄Π½ΠΎΠ΅ сопротивлСниС

Π―Ρ€ΠΊΠΈΠΉ ΠΏΡ€ΠΈΠΌΠ΅Ρ€ Π²Ρ‹Ρ…ΠΎΠ΄Π½ΠΎΠ³ΠΎ сопротивлСния – это Π·Π°ΠΊΠΎΠ½ Ома для ΠΏΠΎΠ»Π½ΠΎΠΉ Ρ†Π΅ΠΏΠΈ, Π² ΠΊΠΎΡ‚ΠΎΡ€ΠΎΠΌ Π΅ΡΡ‚ΡŒ Ρ‚Π°ΠΊ Π½Π°Π·Ρ‹Π²Π°Π΅ΠΌΠΎΠ΅ β€œΠ²Π½ΡƒΡ‚Ρ€Π΅Π½Π½Π΅Π΅ сопротивлСниС”. ΠšΠΎΠΌΡƒ лСнь Ρ‡ΠΈΡ‚Π°Ρ‚ΡŒ ΠΏΡ€ΠΎ этот Π·Π°ΠΊΠΎΠ½, Π²ΠΊΡ€Π°Ρ‚Ρ†Π΅ рассмотрим Π΅Π³ΠΎ здСсь.

Π§Ρ‚ΠΎ ΠΌΡ‹ ΠΈΠΌΠ΅Π»ΠΈ? Π£ нас Π±Ρ‹Π» Π°Π²Ρ‚ΠΎΠΌΠΎΠ±ΠΈΠ»ΡŒΠ½Ρ‹ΠΉ аккумулятор, с ΠΏΠΎΠΌΠΎΡ‰ΡŒΡŽ ΠΊΠΎΡ‚ΠΎΡ€ΠΎΠ³ΠΎ ΠΌΡ‹ ΠΏΠΎΠ΄ΠΆΠΈΠ³Π°Π»ΠΈ Π³Π°Π»ΠΎΠ³Π΅Π½Π½ΡƒΡŽ Π»Π°ΠΌΠΏΠΎΡ‡ΠΊΡƒ. ΠŸΠ΅Ρ€Π΅Π΄ Ρ‚Π΅ΠΌ, ΠΊΠ°ΠΊ Ρ†Π΅ΠΏΠ»ΡΡ‚ΡŒ Π»Π°ΠΌΠΏΠΎΡ‡ΠΊΡƒ, ΠΌΡ‹ замСряли напряТСниС Π½Π° ΠΊΠ»Π΅ΠΌΠΌΠ°Ρ… аккумулятора:

И ΠΊΠ°ΠΊ Ρ‚ΠΎΠ»ΡŒΠΊΠΎΒ  подсоСдиняли Π»Π°ΠΌΠΏΠΎΡ‡ΠΊΡƒ, Ρƒ нас напряТСниС Π½Π° аккумуляторС ΡΡ‚Π°Π½ΠΎΠ²ΠΈΠ»ΠΎΡΡŒ мСньшС.

Π Π°Π·Π½ΠΈΡ†Π° напряТСния,Β  Ρ‚ΠΎ Π΅ΡΡ‚ΡŒ 0,3 Π’ΠΎΠ»ΡŒΡ‚Π° (12,09-11,79) Ρƒ нас ΠΏΠ°Π΄Π°Π»ΠΎ Π½Π° Ρ‚Π°ΠΊ Π½Π°Π·Ρ‹Π²Π°Π΅ΠΌΠΎΠΌ Π²Π½ΡƒΡ‚Ρ€Π΅Π½Π½Π΅ΠΌ сопротивлСнии r πŸ˜‰ Оно ΠΆΠ΅ ΠΈ Π΅ΡΡ‚ΡŒ Π’Π«Π₯ΠžΠ”ΠΠžΠ• Π‘ΠžΠŸΠ ΠžΠ’Π˜Π’Π›Π•ΠΠ˜Π•. Π•Π³ΠΎ Ρ‚Π°ΠΊΠΆΠ΅ Π½Π°Π·Ρ‹Π²Π°ΡŽΡ‚ Π΅Ρ‰Π΅ сопротивлСниСм источника ΠΈΠ»ΠΈ эквивалСнтным сопротивлСниСм.

Π£ всСх аккумуляторов Π΅ΡΡ‚ΡŒ это Π²Π½ΡƒΡ‚Ρ€Π΅Π½Π½Π΅Π΅ сопротивлСниС r, ΠΈ β€œΡ†Π΅ΠΏΠ»ΡΠ΅Ρ‚ΡΡβ€ ΠΎΠ½ΠΎ ΠΏΠΎΡΠ»Π΅Π΄ΠΎΠ²Π°Ρ‚Π΅Π»ΡŒΠ½ΠΎ с источником Π­Π”Π‘ (Π•).

Но Ρ‚ΠΎΠ»ΡŒΠΊΠΎ Π»ΠΈ аккумуляторы ΠΈ Ρ€Π°Π·Π»ΠΈΡ‡Π½Ρ‹Π΅ Π±Π°Ρ‚Π°Ρ€Π΅ΠΉΠΊΠΈ ΠΎΠ±Π»Π°Π΄Π°ΡŽΡ‚ Π²Ρ‹Ρ…ΠΎΠ΄Π½Ρ‹ΠΌ сопротивлСниСм? НС Ρ‚ΠΎΠ»ΡŒΠΊΠΎ. Π’Ρ‹Ρ…ΠΎΠ΄Π½Ρ‹ΠΌ сопротивлСниСм ΠΎΠ±Π»Π°Π΄Π°ΡŽΡ‚ всС источники питания. Π­Ρ‚ΠΎ ΠΌΠΎΠΆΠ΅Ρ‚ Π±Ρ‹Ρ‚ΡŒ Π±Π»ΠΎΠΊ питания, Π³Π΅Π½Π΅Ρ€Π°Ρ‚ΠΎΡ€ частоты, Π»ΠΈΠ±ΠΎ Π²ΠΎΠΎΠ±Ρ‰Π΅ ΠΊΠ°ΠΊΠΎΠΉ-Π½ΠΈΠ±ΡƒΠ΄ΡŒ ΡƒΡΠΈΠ»ΠΈΡ‚Π΅Π»ΡŒ.

Π’ Ρ‚Π΅ΠΎΡ€Π΅ΠΌΠ΅ Π’Π΅Π²Π΅Π½ΠΈΠ½Π° (ΠΊΠΎΡ€ΠΎΡ‡Π΅, ΡƒΠΌΠ½Ρ‹ΠΉ ΠΌΡƒΠΆΠΈΠΊ Ρ‚Π°ΠΊΠΎΠΉ Π±Ρ‹Π»)Β  Π³ΠΎΠ²ΠΎΡ€ΠΈΠ»ΠΎΡΡŒ, Ρ‡Ρ‚ΠΎ Π»ΡŽΠ±ΡƒΡŽ Ρ†Π΅ΠΏΡŒ, которая ΠΈΠΌΠ΅Π΅Ρ‚ Π΄Π²Π΅ ΠΊΠ»Π΅ΠΌΠΌΡ‹ ΠΈ содСрТит Π² сСбС Ρ‚ΡƒΠ΅Π²Ρƒ ΠΊΡƒΡ‡Ρƒ Ρ€Π°Π·Π»ΠΈΡ‡Π½Ρ‹Ρ… источников Π­Π”Π‘ ΠΈ рСзисторов Ρ€Π°Π·Π½ΠΎΠ³ΠΎ Π½ΠΎΠΌΠΈΠ½Π°Π»Π° ΠΌΠΎΠΆΠ½ΠΎ привСсти Ρ‚ΡƒΠΏΠΎ ΠΊ источнику Π­Π”Π‘ с ΠΊΠ°ΠΊΠΈΠΌ-Ρ‚ΠΎ Π·Π½Π°Ρ‡Π΅Π½ΠΈΠ΅ΠΌ напряТСния (EэквивалСнтноС) ΠΈ с ΠΊΠ°ΠΊΠΈΠΌ-Ρ‚ΠΎ Π²Π½ΡƒΡ‚Ρ€Π΅Π½Π½ΠΈΠΌ сопротивлСниСм (RэквивалСнтноС).

Eэкв  – эквивалСнтный источник Π­Π”Π‘

Rэкв  – эквивалСнтноС сопротивлСниС

Π’ΠΎ Π΅ΡΡ‚ΡŒ получаСтся, Ссли ΠΊΠ°ΠΊΠΎΠΉ-Π»ΠΈΠ±ΠΎ источник напряТСния ΠΏΠΈΡ‚Π°Π΅Ρ‚ Π½Π°Π³Ρ€ΡƒΠ·ΠΊΡƒ, Π·Π½Π°Ρ‡ΠΈΡ‚, Π² источникС напряТСния Π΅ΡΡ‚ΡŒ Π­Π”Π‘ ΠΈ эквивалСнтноС сопротивлСниС, ΠΎΠ½ΠΎ ΠΆΠ΅ Π²Ρ‹Ρ…ΠΎΠ΄Π½ΠΎΠ΅ сопротивлСниС.

Π’ Ρ€Π΅ΠΆΠΈΠΌΠ΅ холостого Ρ…ΠΎΠ΄Π° (Ρ‚ΠΎ Π΅ΡΡ‚ΡŒ, ΠΊΠΎΠ³Π΄Π° ΠΊ Π²Ρ‹Ρ…ΠΎΠ΄Π½Ρ‹ΠΌ ΠΊΠ»Π΅ΠΌΠΌΠ°ΠΌ Π½Π΅ ΠΏΠΎΠ΄Ρ†Π΅ΠΏΠ»Π΅Π½Π° Π½Π°Π³Ρ€ΡƒΠ·ΠΊΠ°) с ΠΏΠΎΠΌΠΎΡ‰ΡŒΡŽ ΠΌΡƒΠ»ΡŒΡ‚ΠΈΠΌΠ΅Ρ‚Ρ€Π° ΠΌΡ‹ ΠΌΠΎΠΆΠ΅ΠΌ Π·Π°ΠΌΠ΅Ρ€ΠΈΡ‚ΡŒ Π­Π”Π‘ (E). Π‘ Π·Π°ΠΌΠ΅Ρ€ΠΎΠΌ Π­Π”Π‘ Π²Ρ€ΠΎΠ΄Π΅ Π±Ρ‹ понятно, Π½ΠΎ Π²ΠΎΡ‚ ΠΊΠ°ΠΊ Π·Π°ΠΌΠ΅Ρ€ΠΈΡ‚ΡŒ RΠ²Ρ‹Ρ… ?

Π’ ΠΏΡ€ΠΈΠ½Ρ†ΠΈΠΏΠ΅, ΠΌΠΎΠΆΠ½ΠΎ ΡƒΡΡ‚Ρ€ΠΎΠΈΡ‚ΡŒ ΠΊΠΎΡ€ΠΎΡ‚ΠΊΠΎΠ΅ Π·Π°ΠΌΡ‹ΠΊΠ°Π½ΠΈΠ΅. Π’ΠΎ Π΅ΡΡ‚ΡŒ Π·Π°ΠΌΠΊΠ½ΡƒΡ‚ΡŒ Π²Ρ‹Ρ…ΠΎΠ΄Π½Ρ‹Π΅ ΠΊΠ»Π΅ΠΌΠΌΡ‹ толстым ΠΌΠ΅Π΄Π½Ρ‹ΠΌ ΠΏΡ€ΠΎΠ²ΠΎΠ΄ΠΎΠΌ, ΠΏΠΎ ΠΊΠΎΡ‚ΠΎΡ€ΠΎΠΌΡƒ Ρƒ нас Π±ΡƒΠ΄Π΅Ρ‚ Ρ‚Π΅Ρ‡ΡŒ Ρ‚ΠΎΠΊ ΠΊΠΎΡ€ΠΎΡ‚ΠΊΠΎΠ³ΠΎ замыкания IΠΊΠ·.

Π’ Ρ€Π΅Π·ΡƒΠ»ΡŒΡ‚Π°Ρ‚Π΅ Ρƒ нас получаСтся замкнутая Ρ†Π΅ΠΏΡŒ с ΠΎΠ΄Π½ΠΈΠΌ рСзистором. Из Π·Π°ΠΊΠΎΠ½Π° Ома ΠΏΠΎΠ»ΡƒΡ‡Π°Π΅ΠΌ, Ρ‡Ρ‚ΠΎ

Но Π΅ΡΡ‚ΡŒ нСбольшая Π·Π°Π³Π²ΠΎΠ·Π΄ΠΊΠ°. ВСорСтичСски  – Ρ„ΠΎΡ€ΠΌΡƒΠ»Π° Π²Π΅Ρ€Π½Π°. Но Π½Π° ΠΏΡ€Π°ΠΊΡ‚ΠΈΠΊΠ΅ я Π±Ρ‹ Π½Π΅ Ρ€Π΅ΠΊΠΎΠΌΠ΅Π½Π΄ΠΎΠ²Π°Π» ΠΈΡΠΏΠΎΠ»ΡŒΠ·ΠΎΠ²Π°Ρ‚ΡŒ этот способ. Π’ этом случаС сила Ρ‚ΠΎΠΊΠ° достигаСт бСшСного значСния, Π΄Π° Π²ΠΎΠΎΠ±Ρ‰Π΅, вся схСма Π²Π΅Π΄Π΅Ρ‚ сСбя Π½Π΅Π°Π΄Π΅ΠΊΠ²Π°Ρ‚Π½ΠΎ.

Π˜Π·ΠΌΠ΅Ρ€Π΅Π½ΠΈΠ΅ Π²Ρ‹Ρ…ΠΎΠ΄Π½ΠΎΠ³ΠΎ сопротивлСния Π½Π° ΠΏΡ€Π°ΠΊΡ‚ΠΈΠΊΠ΅

Π•ΡΡ‚ΡŒ Π΄Ρ€ΡƒΠ³ΠΎΠΉ, Π±ΠΎΠ»Π΅Π΅ бСзопасный способ. НС Π±ΡƒΠ΄Ρƒ ΠΏΠΎΠ²Ρ‚ΠΎΡ€ΡΡ‚ΡŒΡΡ, просто ΡΠΊΠΎΠΏΠΈΡ€ΡƒΡŽ со ΡΡ‚Π°Ρ‚ΡŒΠΈ Π·Π°ΠΊΠΎΠ½ Ома для ΠΏΠΎΠ»Π½ΠΎΠΉ Ρ†Π΅ΠΏΠΈ, Π³Π΄Π΅ ΠΌΡ‹ Π½Π°Ρ…ΠΎΠ΄ΠΈΠ»ΠΈ Π²Π½ΡƒΡ‚Ρ€Π΅Π½Π½Π΅Π΅ сопротивлСниС аккумулятора. Π’ Ρ‚ΠΎΠΉ ΡΡ‚Π°Ρ‚ΡŒΠ΅, ΠΌΡ‹ ΠΊ Π°ΠΊΡƒΠΌΡƒ цСпляли Π³Π°Π»ΠΎΠ³Π΅Π½Π½ΡƒΡŽ Π»Π°ΠΌΠΏΠΎΡ‡ΠΊΡƒ, которая Π±Ρ‹Π»Π° Π½Π°Π³Ρ€ΡƒΠ·ΠΊΠΎΠΉ R. Π’ Ρ€Π΅Π·ΡƒΠ»ΡŒΡ‚Π°Ρ‚Π΅ ΠΏΠΎ Ρ†Π΅ΠΏΠΈ шСл элСктричСский Ρ‚ΠΎΠΊ. На Π»Π°ΠΌΠΏΠΎΡ‡ΠΊΠ΅ ΠΈ Π½Π° Π²Π½ΡƒΡ‚Ρ€Π΅Π½Π½Π΅ΠΌ сопротивлСнии Ρƒ нас ΠΏΠ°Π΄Π°Π»ΠΎ напряТСниС, сумма ΠΊΠΎΡ‚ΠΎΡ€Ρ‹Ρ… Ρ€Π°Π²Π½ΡΠ»Π°ΡΡŒ Π­Π”Π‘.

Π˜Ρ‚Π°ΠΊ, для Π½Π°Ρ‡Π°Π»Π° замСряСм напряТСниС Π½Π° аккумуляторС Π±Π΅Π· Π»Π°ΠΌΠΏΠΎΡ‡ΠΊΠΈ.

Π’Π°ΠΊ ΠΊΠ°ΠΊ Ρƒ нас Π² этом случаС Ρ†Π΅ΠΏΡŒ Ρ€Π°Π·ΠΎΠΌΠΊΠ½ΡƒΡ‚Π° (Π½Π΅Ρ‚ внСшнСй Π½Π°Π³Ρ€ΡƒΠ·ΠΊΠΈ), ΡΠ»Π΅Π΄ΠΎΠ²Π°Ρ‚Π΅Π»ΡŒΠ½ΠΎ сила Ρ‚ΠΎΠΊΠ° Π² Ρ†Π΅ΠΏΠΈ I равняСтся Π½ΡƒΠ»ΡŽ. Π—Π½Π°Ρ‡ΠΈΡ‚, ΠΈ ΠΏΠ°Π΄Π΅Π½ΠΈΠ΅ напряТСниС Π½Π° Π²Π½ΡƒΡ‚Ρ€Π΅Π½Π½Π΅ΠΌ рСзисторС Ur Ρ‚ΠΎΠΆΠ΅ Π±ΡƒΠ΄Π΅Ρ‚ Ρ€Π°Π²Π½ΡΡ‚ΡŒΡΡ Π½ΡƒΠ»ΡŽ. Π’ ΠΈΡ‚ΠΎΠ³Π΅, Ρƒ нас остаСтся Ρ‚ΠΎΠ»ΡŒΠΊΠΎ источник Π­Π”Π‘, Ρƒ ΠΊΠΎΡ‚ΠΎΡ€ΠΎΠ³ΠΎ ΠΌΡ‹ ΠΈ замСряСм напряТСниС. Π’ нашСм случаС E=12,09 Π’ΠΎΠ»ΡŒΡ‚.

Как Ρ‚ΠΎΠ»ΡŒΠΊΠΎ ΠΌΡ‹ подсоСдинили Π½Π°Π³Ρ€ΡƒΠ·ΠΊΡƒ, Ρ‚ΠΎ Ρƒ нас сразу ΠΆΠ΅ ΡƒΠΏΠ°Π»ΠΎ напряТСниС Π½Π° Π²Π½ΡƒΡ‚Ρ€Π΅Π½Π½Π΅ΠΌ рСзисторС ΠΈ Π½Π° Π½Π°Π³Ρ€ΡƒΠ·ΠΊΠ΅, Π² Π΄Π°Π½Π½ΠΎΠΌ случаС Π½Π° Π»Π°ΠΌΠΏΠΎΡ‡ΠΊΠ΅:

БСйчас Π½Π° Π½Π°Π³Ρ€ΡƒΠ·ΠΊΠ΅ (Π½Π° Π³Π°Π»ΠΎΠ³Π΅Π½ΠΊΠ΅) Ρƒ нас ΡƒΠΏΠ°Π»ΠΎ напряТСниС UR=11,79 Π’ΠΎΠ»ΡŒΡ‚, ΡΠ»Π΅Π΄ΠΎΠ²Π°Ρ‚Π΅Π»ΡŒΠ½ΠΎ, Π½Π° Π²Π½ΡƒΡ‚Ρ€Π΅Π½Π½Π΅ΠΌ рСзисторС ΠΏΠ°Π΄Π΅Π½ΠΈΠ΅ напряТСния составило Ur=E-UR=12,09-11,79=0,3 Π’ΠΎΠ»ΡŒΡ‚Π°. Π‘ΠΈΠ»Π° Ρ‚ΠΎΠΊΠ° Π² Ρ†Π΅ΠΏΠΈ равняСтся I=4,35 АмпСр. Как я ΡƒΠΆΠ΅ сказал, Π­Π”Π‘ Ρƒ нас равняСтся E=12,09 Π’ΠΎΠ»ΡŒΡ‚. Π‘Π»Π΅Π΄ΠΎΠ²Π°Ρ‚Π΅Π»ΡŒΠ½ΠΎ, ΠΈΠ· Π·Π°ΠΊΠΎΠ½Π° Ома для ΠΏΠΎΠ»Π½ΠΎΠΉ Ρ†Π΅ΠΏΠΈ высчитываСм, Ρ‡Π΅ΠΌΡƒ Ρƒ нас Π±ΡƒΠ΄Π΅Ρ‚ Ρ€Π°Π²Π½ΡΡ‚ΡŒΡΡ Π²Π½ΡƒΡ‚Ρ€Π΅Π½Π½Π΅Π΅ сопротивлСниС r:

Π—Π°ΠΊΠ»ΡŽΡ‡Π΅Π½ΠΈΠ΅

Π’Ρ…ΠΎΠ΄Π½ΠΎΠ΅ ΠΈ Π²Ρ‹Ρ…ΠΎΠ΄Π½ΠΎΠ΅ сопротивлСниС каскадов (Π±Π»ΠΎΠΊΠΎΠ²) Π² элСктроникС ΠΈΠ³Ρ€Π°ΡŽΡ‚ ΠΎΡ‡Π΅Π½ΡŒ Π²Π°ΠΆΠ½ΡƒΡŽ Ρ€ΠΎΠ»ΡŒ. Π’ этом ΠΌΡ‹ убСдимся, ΠΊΠΎΠ³Π΄Π° Π½Π°Ρ‡Π½Π΅ΠΌ Ρ€Π°ΡΡΠΌΠ°Ρ‚Ρ€ΠΈΠ²Π°Ρ‚ΡŒ ΡΡ‚Π°Ρ‚ΡŒΡŽ ΠΏΠΎ согласованию ΡƒΠ·Π»ΠΎΠ² радиоэлСктронных схСм. ВсС качСствСнныС Π²ΠΎΠ»ΡŒΡ‚ΠΌΠ΅Ρ‚Ρ€Ρ‹ ΠΈ осциллографы Ρ‚Π°ΠΊΠΆΠ΅ ΡΡ‚Π°Ρ€Π°ΡŽΡ‚ΡΡ Π΄Π΅Π»Π°Ρ‚ΡŒ с ΠΎΡ‡Π΅Π½ΡŒ высоким Π²Ρ…ΠΎΠ΄Π½Ρ‹ΠΌ сопротивлСниСм, Ρ‡Ρ‚ΠΎΠ±Ρ‹ ΠΎΠ½ΠΎ мСньшС ΡΠΊΠ°Π·Ρ‹Π²Π°Π»ΠΎΡΡŒ Π½Π° замСряСмый сигнал ΠΈ Π½Π΅ гасило Π΅Π³ΠΎ Π°ΠΌΠΏΠ»ΠΈΡ‚ΡƒΠ΄Ρƒ.

Π‘ Π²Ρ‹Ρ…ΠΎΠ΄Π½Ρ‹ΠΌ сопротивлСниСм всС Π½Π°ΠΌΠ½ΠΎΠ³ΠΎ интСрСснСС. Когда ΠΌΡ‹ ΠΏΠΎΠ΄ΠΊΠ»ΡŽΡ‡Π°Π΅ΠΌ Π½ΠΈΠ·ΠΊΠΎΠΎΠΌΠ½ΡƒΡŽ Π½Π°Π³Ρ€ΡƒΠ·ΠΊΡƒ, Ρ‚ΠΎ Ρ‡Π΅ΠΌ большС Π²Π½ΡƒΡ‚Ρ€Π΅Π½Π½Π΅Π΅ сопротивлСниС, Ρ‚Π΅ΠΌ большС напряТСниС ΠΏΠ°Π΄Π°Π΅Ρ‚ Π½Π° Π²Π½ΡƒΡ‚Ρ€Π΅Π½Π½Π΅ΠΌ сопротивлСнии. Π’ΠΎ Π΅ΡΡ‚ΡŒ Π² Π½Π°Π³Ρ€ΡƒΠ·ΠΊΡƒ Π±ΡƒΠ΄Π΅Ρ‚ ΠΎΡ‚Π΄Π°Π²Π°Ρ‚ΡŒΡΡ мСньшСС напряТСниС, Ρ‚Π°ΠΊ ΠΊΠ°ΠΊ Ρ€Π°Π·Π½ΠΈΡ†Π° осядСт Π½Π° Π²Π½ΡƒΡ‚Ρ€Π΅Π½Π½Π΅ΠΌ рСзисторС. ΠŸΠΎΡΡ‚ΠΎΠΌΡƒ, качСствСнныС источники питания, Ρ‚ΠΈΠΏΠ° Π±Π»ΠΎΠΊΠ° питания Π»ΠΈΠ±ΠΎ Π³Π΅Π½Π΅Ρ€Π°Ρ‚ΠΎΡ€Π° частоты, ΠΏΡ‹Ρ‚Π°ΡŽΡ‚ΡΡ Π΄Π΅Π»Π°Ρ‚ΡŒ ΠΊΠ°ΠΊ ΠΌΠΎΠΆΠ½ΠΎ с мСньшим Π²Ρ‹Ρ…ΠΎΠ΄Π½Ρ‹ΠΌ сопротивлСниСм, Ρ‡Ρ‚ΠΎΠ±Ρ‹ напряТСниС Π½Π° Π²Ρ‹Ρ…ΠΎΠ΄Π΅ β€œΠ½Π΅ просСдало” ΠΏΡ€ΠΈ ΠΏΠΎΠ΄ΠΊΠ»ΡŽΡ‡Π΅Π½ΠΈΠΈ Π½ΠΈΠ·ΠΊΠΎΠΎΠΌΠ½ΠΎΠΉ Π½Π°Π³Ρ€ΡƒΠ·ΠΊΠΈ. Π”Π°ΠΆΠ΅ Ссли сильно просядСт, Ρ‚ΠΎ ΠΌΡ‹ ΠΌΠΎΠΆΠ΅ΠΌ Π²Ρ€ΡƒΡ‡Π½ΡƒΡŽ ΠΏΠΎΠ΄ΠΊΠΎΡ€Ρ€Π΅ΠΊΡ‚ΠΈΡ€ΠΎΠ²Π°Ρ‚ΡŒ с ΠΏΠΎΠΌΠΎΡ‰ΡŒΡŽ Ρ€Π΅Π³ΡƒΠ»ΠΈΡ€ΠΎΠ²ΠΊΠΈ Π²Ρ‹Ρ…ΠΎΠ΄Π½ΠΎΠ³ΠΎ напряТСния, ΠΊΠΎΡ‚ΠΎΡ€Ρ‹Π΅ Π΅ΡΡ‚ΡŒ Π² ΠΊΠ°ΠΆΠ΄ΠΎΠΌ Π½ΠΎΡ€ΠΌΠ°Π»ΡŒΠ½ΠΎΠΌ источникС питания. Π’ Π½Π΅ΠΊΠΎΡ‚ΠΎΡ€Ρ‹Ρ… источниках это дСлаСтся автоматичСски.

НапряТСниС ΠΏΡ€ΠΈ ΠΏΠ°Ρ€Π°Π»Π»Π΅Π»ΡŒΠ½ΠΎΠΌ соСдинСнии рСзисторов. Π‘ΠΎΠΏΡ€ΠΎΡ‚ΠΈΠ²Π»Π΅Π½ΠΈΠ΅ ΠΏΡ€ΠΈ ΠΏΠ°Ρ€Π°Π»Π»Π΅Π»ΡŒΠ½ΠΎΠΌ соСдинСнии: Ρ„ΠΎΡ€ΠΌΡƒΠ»Π° расчСта

ΠŸΡ€ΠΈ Ρ€Π΅ΡˆΠ΅Π½ΠΈΠΈ Π·Π°Π΄Π°Ρ‡ принято ΠΏΡ€Π΅ΠΎΠ±Ρ€Π°Π·ΠΎΠ²Ρ‹Π²Π°Ρ‚ΡŒ схСму, Ρ‚Π°ΠΊ, Ρ‡Ρ‚ΠΎΠ±Ρ‹ ΠΎΠ½Π° Π±Ρ‹Π»Π° ΠΊΠ°ΠΊ ΠΌΠΎΠΆΠ½ΠΎ ΠΏΡ€ΠΎΡ‰Π΅. Для этого ΠΏΡ€ΠΈΠΌΠ΅Π½ΡΡŽΡ‚ эквивалСнтныС прСобразования. Π­ΠΊΠ²ΠΈΠ²Π°Π»Π΅Π½Ρ‚Π½Ρ‹ΠΌΠΈ Π½Π°Π·Ρ‹Π²Π°ΡŽΡ‚ Ρ‚Π°ΠΊΠΈΠ΅ прСобразования части схСмы элСктричСской Ρ†Π΅ΠΏΠΈ, ΠΏΡ€ΠΈ ΠΊΠΎΡ‚ΠΎΡ€Ρ‹Ρ… Ρ‚ΠΎΠΊΠΈ ΠΈ напряТСния Π² Π½Π΅ ΠΏΡ€Π΅ΠΎΠ±Ρ€Π°Π·ΠΎΠ²Π°Π½Π½ΠΎΠΉ Π΅Ρ‘ части ΠΎΡΡ‚Π°ΡŽΡ‚ΡΡ Π½Π΅ΠΈΠ·ΠΌΠ΅Π½Π½Ρ‹ΠΌΠΈ.

БущСствуСт Ρ‡Π΅Ρ‚Ρ‹Ρ€Π΅ основных Π²ΠΈΠ΄Π° соСдинСния ΠΏΡ€ΠΎΠ²ΠΎΠ΄Π½ΠΈΠΊΠΎΠ²: ΠΏΠΎΡΠ»Π΅Π΄ΠΎΠ²Π°Ρ‚Π΅Π»ΡŒΠ½ΠΎΠ΅, ΠΏΠ°Ρ€Π°Π»Π»Π΅Π»ΡŒΠ½ΠΎΠ΅, смСшанноС ΠΈ мостовоС.

ΠŸΠΎΡΠ»Π΅Π΄ΠΎΠ²Π°Ρ‚Π΅Π»ΡŒΠ½ΠΎΠ΅ соСдинСниС

ΠŸΠΎΡΠ»Π΅Π΄ΠΎΠ²Π°Ρ‚Π΅Π»ΡŒΠ½ΠΎΠ΅ соСдинСниС – это Ρ‚Π°ΠΊΠΎΠ΅ соСдинСниС, ΠΏΡ€ΠΈ ΠΊΠΎΡ‚ΠΎΡ€ΠΎΠΌ сила Ρ‚ΠΎΠΊΠ° Π½Π° всСм участкС Ρ†Π΅ΠΏΠΈ ΠΎΠ΄ΠΈΠ½Π°ΠΊΠΎΠ²Π°. Π―Ρ€ΠΊΠΈΠΌ ΠΏΡ€ΠΈΠΌΠ΅Ρ€ΠΎΠΌ ΠΏΠΎΡΠ»Π΅Π΄ΠΎΠ²Π°Ρ‚Π΅Π»ΡŒΠ½ΠΎΠ³ΠΎ соСдинСния являСтся старая Слочная гирлянда. Π’Π°ΠΌ Π»Π°ΠΌΠΏΠΎΡ‡ΠΊΠΈ ΠΏΠΎΠ΄ΠΊΠ»ΡŽΡ‡Π΅Π½Ρ‹ ΠΏΠΎΡΠ»Π΅Π΄ΠΎΠ²Π°Ρ‚Π΅Π»ΡŒΠ½ΠΎ, Π΄Ρ€ΡƒΠ³ Π·Π° Π΄Ρ€ΡƒΠ³ΠΎΠΌ. Π’Π΅ΠΏΠ΅Ρ€ΡŒ ΠΏΡ€Π΅Π΄ΡΡ‚Π°Π²ΡŒΡ‚Π΅, ΠΎΠ΄Π½Π° Π»Π°ΠΌΠΏΠΎΡ‡ΠΊΠ° ΠΏΠ΅Ρ€Π΅Π³ΠΎΡ€Π°Π΅Ρ‚, Ρ†Π΅ΠΏΡŒ Π½Π°Ρ€ΡƒΡˆΠ΅Π½Π° ΠΈ ΠΎΡΡ‚Π°Π»ΡŒΠ½Ρ‹Π΅ Π»Π°ΠΌΠΏΠΎΡ‡ΠΊΠΈ гаснут. Π’Ρ‹Ρ…ΠΎΠ΄ ΠΈΠ· строя ΠΎΠ΄Π½ΠΎΠ³ΠΎ элСмСнта, Π²Π΅Π΄Π΅Ρ‚ Π·Π° собой ΠΎΡ‚ΠΊΠ»ΡŽΡ‡Π΅Π½ΠΈΠ΅ всСх ΠΎΡΡ‚Π°Π»ΡŒΠ½Ρ‹Ρ…, это являСтся сущСствСнным нСдостатком ΠΏΠΎΡΠ»Π΅Π΄ΠΎΠ²Π°Ρ‚Π΅Π»ΡŒΠ½ΠΎΠ³ΠΎ соСдинСния.

ΠŸΡ€ΠΈ ΠΏΠΎΡΠ»Π΅Π΄ΠΎΠ²Π°Ρ‚Π΅Π»ΡŒΠ½ΠΎΠΌ соСдинСнии сопротивлСния элСмСнтов ΡΡƒΠΌΠΌΠΈΡ€ΡƒΡŽΡ‚ΡΡ.

ΠŸΠ°Ρ€Π°Π»Π»Π΅Π»ΡŒΠ½ΠΎΠ΅ соСдинСниС

ΠŸΠ°Ρ€Π°Π»Π»Π΅Π»ΡŒΠ½ΠΎΠ΅ соСдинСниС – это соСдинСниС, ΠΏΡ€ΠΈ ΠΊΠΎΡ‚ΠΎΡ€ΠΎΠΌ напряТСниС Π½Π° ΠΊΠΎΠ½Ρ†Π°Ρ… участка Ρ†Π΅ΠΏΠΈ ΠΎΠ΄ΠΈΠ½Π°ΠΊΠΎΠ²ΠΎ. ΠŸΠ°Ρ€Π°Π»Π»Π΅Π»ΡŒΠ½ΠΎΠ΅ соСдинСниС Π½Π°ΠΈΠ±ΠΎΠ»Π΅Π΅ распространСно, Π² основном ΠΏΠΎΡ‚ΠΎΠΌΡƒ, Ρ‡Ρ‚ΠΎ всС элСмСнты находятся ΠΏΠΎΠ΄ ΠΎΠ΄Π½ΠΈΠΌ напряТСниСм, сила Ρ‚ΠΎΠΊΠ° распрСдСлСна ΠΏΠΎ-Ρ€Π°Π·Π½ΠΎΠΌΡƒ ΠΈ ΠΏΡ€ΠΈ Π²Ρ‹Ρ…ΠΎΠ΄Π΅ ΠΎΠ΄Π½ΠΎΠ³ΠΎ ΠΈΠ· элСмСнтов всС ΠΎΡΡ‚Π°Π»ΡŒΠ½Ρ‹Π΅ ΠΏΡ€ΠΎΠ΄ΠΎΠ»ΠΆΠ°ΡŽΡ‚ свою Ρ€Π°Π±ΠΎΡ‚Ρƒ.

ΠŸΡ€ΠΈ ΠΏΠ°Ρ€Π°Π»Π»Π΅Π»ΡŒΠ½ΠΎΠΌ соСдинСнии эквивалСнтноС сопротивлСниС находится ΠΊΠ°ΠΊ:

Π’ случаС Π΄Π²ΡƒΡ… ΠΏΠ°Ρ€Π°Π»Π»Π΅Π»ΡŒΠ½ΠΎ соСдинСнных рСзисторов

Π’ случаС Ρ‚Ρ€Π΅Ρ… ΠΏΠ°Ρ€Π°Π»Π»Π΅Π»ΡŒΠ½ΠΎ ΠΏΠΎΠ΄ΠΊΠ»ΡŽΡ‡Π΅Π½Π½Ρ‹Ρ… рСзисторов:

БмСшанноС соСдинСниС

БмСшанноС соСдинСниС – соСдинСниС, ΠΊΠΎΡ‚ΠΎΡ€ΠΎΠ΅ являСтся ΡΠΎΠ²ΠΎΠΊΡƒΠΏΠ½ΠΎΡΡ‚ΡŒΡŽ ΠΏΠΎΡΠ»Π΅Π΄ΠΎΠ²Π°Ρ‚Π΅Π»ΡŒΠ½Ρ‹Ρ… ΠΈ ΠΏΠ°Ρ€Π°Π»Π»Π΅Π»ΡŒΠ½Ρ‹Ρ… соСдинСний. Для нахоТдСния эквивалСнтного сопротивлСния Π½ΡƒΠΆΠ½ΠΎ, β€œΡΠ²Π΅Ρ€Π½ΡƒΡ‚ΡŒβ€ схСму ΠΏΠΎΠΎΡ‡Π΅Ρ€Π΅Π΄Π½Ρ‹ΠΌ ΠΏΡ€Π΅ΠΎΠ±Ρ€Π°Π·ΠΎΠ²Π°Π½ΠΈΠ΅ΠΌ ΠΏΠ°Ρ€Π°Π»Π»Π΅Π»ΡŒΠ½Ρ‹Ρ… ΠΈ ΠΏΠΎΡΠ»Π΅Π΄ΠΎΠ²Π°Ρ‚Π΅Π»ΡŒΠ½Ρ‹Ρ… участков Ρ†Π΅ΠΏΠΈ.


Π‘Π½Π°Ρ‡Π°Π»Π° Π½Π°ΠΉΠ΄Π΅ΠΌ эквивалСнтноС сопротивлСниС для ΠΏΠ°Ρ€Π°Π»Π»Π΅Π»ΡŒΠ½ΠΎΠ³ΠΎ участка Ρ†Π΅ΠΏΠΈ, Π° Π·Π°Ρ‚Π΅ΠΌ ΠΏΡ€ΠΈΠ±Π°Π²ΠΈΠΌ ΠΊ Π½Π΅ΠΌΡƒ ΠΎΡΡ‚Π°Π²ΡˆΠ΅Π΅ΡΡ сопротивлСниС R 3 . Π‘Π»Π΅Π΄ΡƒΠ΅Ρ‚ ΠΏΠΎΠ½ΠΈΠΌΠ°Ρ‚ΡŒ, Ρ‡Ρ‚ΠΎ послС прСобразования эквивалСнтноС сопротивлСниС R 1 R 2 ΠΈ рСзистор R 3 , соСдинСны ΠΏΠΎΡΠ»Π΅Π΄ΠΎΠ²Π°Ρ‚Π΅Π»ΡŒΠ½ΠΎ.

Π˜Ρ‚Π°ΠΊ, остаСтся самоС интСрСсноС ΠΈ самоС слоТноС соСдинСниС ΠΏΡ€ΠΎΠ²ΠΎΠ΄Π½ΠΈΠΊΠΎΠ².

ΠœΠΎΡΡ‚ΠΎΠ²Π°Ρ схСма

ΠœΠΎΡΡ‚ΠΎΠ²Π°Ρ схСма соСдинСния прСдставлСна Π½Π° рисункС Π½ΠΈΠΆΠ΅.



Для Ρ‚ΠΎΠ³ΠΎ Ρ‡Ρ‚ΠΎΠ±Ρ‹ ΡΠ²Π΅Ρ€Π½ΡƒΡ‚ΡŒ ΠΌΠΎΡΡ‚ΠΎΠ²ΡƒΡŽ схСму, ΠΎΠ΄ΠΈΠ½ ΠΈΠ· Ρ‚Ρ€Π΅ΡƒΠ³ΠΎΠ»ΡŒΠ½ΠΈΠΊΠΎΠ² моста, Π·Π°ΠΌΠ΅Π½ΡΡŽΡ‚ эквивалСнтной Π·Π²Π΅Π·Π΄ΠΎΠΉ.

И находят сопротивлСния R 1 , R 2 ΠΈ R 3 .

ΠŸΠΎΡΠ»Π΅Π΄ΠΎΠ²Π°Ρ‚Π΅Π»ΡŒΠ½ΠΎΠ΅ соСдинСниС – это соСдинСниС Π΄Π²ΡƒΡ… ΠΈΠ»ΠΈ Π±ΠΎΠ»Π΅Π΅ рСзисторов Π² Ρ„ΠΎΡ€ΠΌΠ΅ Ρ†Π΅ΠΏΠΈ, Π² ΠΊΠΎΡ‚ΠΎΡ€ΠΎΠΉ ΠΊΠ°ΠΆΠ΄Ρ‹ΠΉ ΠΎΡ‚Π΄Π΅Π»ΡŒΠ½Ρ‹ΠΉ рСзистор соСдиняСтся с Π΄Ρ€ΡƒΠ³ΠΈΠΌ ΠΎΡ‚Π΄Π΅Π»ΡŒΠ½Ρ‹ΠΌ рСзистором Ρ‚ΠΎΠ»ΡŒΠΊΠΎ Π² ΠΎΠ΄Π½ΠΎΠΉ Ρ‚ΠΎΡ‡ΠΊΠ΅.

ΠŸΠ°Ρ€Π°Π»Π»Π΅Π»ΡŒΠ½ΠΎΠ΅ соСдинСниС – это соСдинСниС, ΠΏΡ€ΠΈ ΠΊΠΎΡ‚ΠΎΡ€ΠΎΠΌ рСзисторы ΡΠΎΠ΅Π΄ΠΈΠ½ΡΡŽΡ‚ΡΡ ΠΌΠ΅ΠΆΠ΄Ρƒ собой ΠΎΠ±ΠΎΠΈΠΌΠΈ ΠΊΠΎΠ½Ρ‚Π°ΠΊΡ‚Π°ΠΌΠΈ. Π’ Ρ€Π΅Π·ΡƒΠ»ΡŒΡ‚Π°Ρ‚Π΅ ΠΊ ΠΎΠ΄Π½ΠΎΠΉ Ρ‚ΠΎΡ‡ΠΊΠ΅ (элСктричСскому ΡƒΠ·Π»Ρƒ) ΠΌΠΎΠΆΠ΅Ρ‚ Π±Ρ‹Ρ‚ΡŒ присоСдинСно нСсколько рСзисторов.

2) ΠžΠ±Ρ‰Π΅Π΅ сопротивлСниС R ΠΎΠ±Ρ‰

ΠŸΡ€ΠΈ Ρ‚Π°ΠΊΠΎΠΌ соСдинСнии, Ρ‡Π΅Ρ€Π΅Π· всС рСзисторы ΠΏΡ€ΠΎΡ…ΠΎΠ΄ΠΈΡ‚ ΠΎΠ΄ΠΈΠ½ ΠΈ Ρ‚ΠΎΡ‚ ΠΆΠ΅ элСктричСский Ρ‚ΠΎΠΊ. Π§Π΅ΠΌ большС элСмСнтов Π½Π° Π΄Π°Π½Π½ΠΎΠΌ участкС элСктричСской Ρ†Π΅ΠΏΠΈ, Ρ‚Π΅ΠΌ Β«Ρ‚Ρ€ΡƒΠ΄Π½Π΅Π΅Β» Ρ‚ΠΎΠΊΡƒ ΠΏΡ€ΠΎΡ‚Π΅ΠΊΠ°Ρ‚ΡŒ Ρ‡Π΅Ρ€Π΅Π· Π½Π΅Π³ΠΎ. Π‘Π»Π΅Π΄ΠΎΠ²Π°Ρ‚Π΅Π»ΡŒΠ½ΠΎ, ΠΏΡ€ΠΈ ΠΏΠΎΡΠ»Π΅Π΄ΠΎΠ²Π°Ρ‚Π΅Π»ΡŒΠ½ΠΎΠΌ соСдинСнии рСзисторов ΠΈΡ… ΠΎΠ±Ρ‰Π΅Π΅ сопротивлСниС увСличиваСтся, ΠΈ ΠΎΠ½ΠΎ Ρ€Π°Π²Π½ΠΎ суммС всСх сопротивлСний.

ΠžΠ±Ρ‰Π΅Π΅ сопротивлСниС R ΠΎΠ±Ρ‰

ΠŸΡ€ΠΈ Ρ‚Π°ΠΊΠΎΠΌ соСдинСнии, Ρ‡Π΅Ρ€Π΅Π· ΠΊΠ°ΠΆΠ΄Ρ‹ΠΉ рСзистор ΠΏΠΎΡ‚Π΅Ρ‡Π΅Ρ‚ ΠΎΡ‚Π΄Π΅Π»ΡŒΠ½Ρ‹ΠΉ Ρ‚ΠΎΠΊ. Π‘ΠΈΠ»Π° Π΄Π°Π½Π½ΠΎΠ³ΠΎ Ρ‚ΠΎΠΊΠ° Π±ΡƒΠ΄Π΅Ρ‚ ΠΎΠ±Ρ€Π°Ρ‚Π½ΠΎ ΠΏΡ€ΠΎΠΏΠΎΡ€Ρ†ΠΈΠΎΠ½Π°Π»ΡŒΠ½Π° ΡΠΎΠΏΡ€ΠΎΡ‚ΠΈΠ²Π»Π΅Π½ΠΈΡŽ рСзистора. Π’ Ρ€Π΅Π·ΡƒΠ»ΡŒΡ‚Π°Ρ‚Π΅ общая ΠΏΡ€ΠΎΠ²ΠΎΠ΄ΠΈΠΌΠΎΡΡ‚ΡŒ Ρ‚Π°ΠΊΠΎΠ³ΠΎ участка элСктричСской Ρ†Π΅ΠΏΠΈ увСличиваСтся, Π° ΠΎΠ±Ρ‰Π΅Π΅ сопротивлСниС Π² свою ΠΎΡ‡Π΅Ρ€Π΅Π΄ΡŒ ΡƒΠΌΠ΅Π½ΡŒΡˆΠ°Π΅Ρ‚ΡΡ.

Π’Π°ΠΊΠΈΠΌ ΠΎΠ±Ρ€Π°Π·ΠΎΠΌ, ΠΏΡ€ΠΈ ΠΏΠ°Ρ€Π°Π»Π»Π΅Π»ΡŒΠ½ΠΎΠΌ подсоСдинСнии рСзисторов с Ρ€Π°Π·Π½Ρ‹ΠΌ сопротивлСниСм, ΠΎΠ±Ρ‰Π΅Π΅ сопротивлСниС Π±ΡƒΠ΄Π΅Ρ‚ всСгда мСньшС значСния самого малСнького ΠΎΡ‚Π΄Π΅Π»ΡŒΠ½ΠΎΠ³ΠΎ рСзистора.

Π€ΠΎΡ€ΠΌΡƒΠ»Π° эквивалСнтного ΠΎΠ±Ρ‰Π΅Π³ΠΎ сопротивлСния ΠΏΡ€ΠΈ ΠΏΠ°Ρ€Π°Π»Π»Π΅Π»ΡŒΠ½ΠΎΠΌ соСдинСнии рСзисторов:

Для Π΄Π²ΡƒΡ… ΠΎΠ΄ΠΈΠ½Π°ΠΊΠΎΠ²Ρ‹Ρ… рСзисторов ΠΎΠ±Ρ‰Π΅Π΅ сопротивлСниС Π±ΡƒΠ΄Π΅Ρ‚ Ρ€Π°Π²Π½ΠΎ ΠΏΠΎΠ»ΠΎΠ²ΠΈΠ½Π΅ ΠΎΠ΄Π½ΠΎΠ³ΠΎ ΠΎΡ‚Π΄Π΅Π»ΡŒΠ½ΠΎΠ³ΠΎ рСзистора:

БоотвСтствСнно, для n ΠΎΠ΄ΠΈΠ½Π°ΠΊΠΎΠ²Ρ‹Ρ… рСзисторов ΠΎΠ±Ρ‰Π΅Π΅ сопротивлСниС Π±ΡƒΠ΄Π΅Ρ‚ Ρ€Π°Π²Π½ΠΎ Π·Π½Π°Ρ‡Π΅Π½ΠΈΡŽ ΠΎΠ΄Π½ΠΎΠ³ΠΎ рСзистора, Ρ€Π°Π·Π΄Π΅Π»Π΅Π½Π½ΠΎΠ³ΠΎ Π½Π° n.

3)Π­Π»Π΅ΠΊΡ‚Ρ€ΠΎΠΏΡ€ΠΎΠ²ΠΎΠ΄Π½ΠΎΡΡ‚ΡŒ, элСктричСская ΠΏΡ€ΠΎΠ²ΠΎΠ΄ΠΈΠΌΠΎΡΡ‚ΡŒ, ΠΏΡ€ΠΎΠ²ΠΎΠ΄ΠΈΠΌΠΎΡΡ‚ΡŒ, ΡΠΏΠΎΡΠΎΠ±Π½ΠΎΡΡ‚ΡŒ Ρ‚Π΅Π»Π° ΠΏΡ€ΠΎΠΏΡƒΡΠΊΠ°Ρ‚ΡŒ элСктричСский Ρ‚ΠΎΠΊ ΠΏΠΎΠ΄ воздСйствиСм элСктричСского поля, Π° Ρ‚Π°ΠΊΠΆΠ΅ физичСская Π²Π΅Π»ΠΈΡ‡ΠΈΠ½Π°, количСствСнно Ρ…Π°Ρ€Π°ΠΊΡ‚Π΅Ρ€ΠΈΠ·ΡƒΡŽΡ‰Π°Ρ эту ΡΠΏΠΎΡΠΎΠ±Π½ΠΎΡΡ‚ΡŒ. Π’Π΅Π»Π°, проводящиС элСктричСский Ρ‚ΠΎΠΊ, Π½Π°Π·Ρ‹Π²Π°ΡŽΡ‚ΡΡ ΠΏΡ€ΠΎΠ²ΠΎΠ΄Π½ΠΈΠΊΠ°ΠΌΠΈ, Π² ΠΎΡ‚Π»ΠΈΡ‡ΠΈΠ΅ ΠΎΡ‚ изоляторов.. .
Основная Π΅Π΄ΠΈΠ½ΠΈΡ†Π° измСрСния сопротивлСния — Ом. УдСльная ΠΏΡ€ΠΎΠ²ΠΎΠ΄ΠΈΠΌΠΎΡΡ‚ΡŒ — Π²Π΅Π»ΠΈΡ‡ΠΈΠ½Π° обратная ΡΠΎΠΏΡ€ΠΎΡ‚ΠΈΠ²Π»Π΅Π½ΠΈΡŽ, ΠΎΠ½Π° измСряСтся Π² БимСнсах, Ρ€Π°Π½Π΅Π΅ Π½Π°Π·Ρ‹Π²ΡˆΠΈΡ…ΡΡ mho. ΠŸΡ€ΠΈΠΌΠ΅Π½ΠΈΡ‚Π΅Π»ΡŒΠ½ΠΎ ΠΊ сыпучим вСщСствам ΡƒΠ΄ΠΎΠ±Π½Π΅Π΅ Π³ΠΎΠ²ΠΎΡ€ΠΈΡ‚ΡŒ ΠΎΠ± особой проводимости, ΠΎΠ±Ρ‹Ρ‡Π½ΠΎ Π½Π°Π·Ρ‹Π²Π°Π΅ΠΌΠΎΠΉ ΡƒΠ΄Π΅Π»ΡŒΠ½ΠΎΠΉ ΠΏΡ€ΠΎΠ²ΠΎΠ΄ΠΈΠΌΠΎΡΡ‚ΡŒΡŽ.
УдСльная ΠΏΡ€ΠΎΠ²ΠΎΠ΄ΠΈΠΌΠΎΡΡ‚ΡŒ — это ΠΏΡ€ΠΎΠ²ΠΎΠ΄ΠΈΠΌΠΎΡΡ‚ΡŒ, измСрСнная ΠΌΠ΅ΠΆΠ΄Ρƒ ΠΏΡ€ΠΎΡ‚ΠΈΠ²ΠΎΠΏΠΎΠ»ΠΎΠΆΠ½Ρ‹ΠΌΠΈ сторонами ΠΊΡƒΠ±Π° вСщСства со стороной 1 см. Π•Π΄ΠΈΠ½ΠΈΡ†Π΅ΠΉ Π΄Π°Π½Π½ΠΎΠ³ΠΎ Ρ‚ΠΈΠΏΠ° ΠΈΠ·ΠΌΠ΅Ρ€Π΅Π½ΠΈΠΉ являСтся БимСнс/см. ΠŸΡ€ΠΈ ΠΈΠ·ΠΌΠ΅Ρ€Π΅Π½ΠΈΠΈ проводимости Π²ΠΎΠ΄Ρ‹ Ρ‡Π°Ρ‰Π΅ ΠΈΡΠΏΠΎΠ»ΡŒΠ·ΡƒΡŽΡ‚ΡΡ Π±ΠΎΠ»Π΅Π΅ Ρ‚ΠΎΡ‡Π½Ρ‹Π΅ ΠΌΠΊΠ‘/см (микросимСнс) ΠΈ ΠΌΠ‘/см (миллисимСнс) .
Π‘ΠΎΠΎΡ‚Π²Π΅Ρ‚ΡΡ‚Π²ΡƒΡŽΡ‰ΠΈΠ΅ Π΅Π΄ΠΈΠ½ΠΈΡ†Ρ‹ измСрСния сопротивлСния (ΠΈΠ»ΠΈ ΡƒΠ΄Π΅Π»ΡŒΠ½ΠΎΠ³ΠΎ сопротивлСния) — Ом/см, МСгаОм/см ΠΈ килоОм/см. ΠŸΡ€ΠΈ ΠΈΠ·ΠΌΠ΅Ρ€Π΅Π½ΠΈΠΈ свСрхчистой Π²ΠΎΠ΄Ρ‹ Ρ‡Π°Ρ‰Π΅ ΠΈΡΠΏΠΎΠ»ΡŒΠ·ΡƒΡŽΡ‚ МСгаОм/см, Ρ‚Π°ΠΊ ΠΊΠ°ΠΊ это Π΄Π°Π΅Ρ‚ Π±ΠΎΠ»Π΅Π΅ Ρ‚ΠΎΡ‡Π½Ρ‹Π΅ Ρ€Π΅Π·ΡƒΠ»ΡŒΡ‚Π°Ρ‚Ρ‹. Π‘ΠΎΠΏΡ€ΠΎΡ‚ΠΈΠ²Π»Π΅Π½ΠΈΠ΅ ΠΌΠ΅Π½Π΅Π΅ чистой Π²ΠΎΠ΄Ρ‹, ΠΊΠ°ΠΊ Π½Π°ΠΏΡ€ΠΈΠΌΠ΅Ρ€, Π²ΠΎΠ΄ΠΎΠΏΡ€ΠΎΠ²ΠΎΠ΄Π½ΠΎΠΉ, ΠΈΠ·ΠΌΠ΅Ρ€ΡΡŽΡ‚ Π² килоОм/см.

4) ΠžΠ±Ρ‰Π΅Π΅ сопротивлСниС ΠΏΡ€ΠΈ ΠΏΠΎΡΠ»Π΅Π΄ΠΎΠ²Π°Ρ‚Π΅Π»ΡŒΠ½ΠΎΠΌ соСдинСнии Ρ€Π°Π²Π½ΠΎ суммС сопротивлСний Rсумм=R1+R2+R3…
Π’ΠΎΠΊ Ρ‡Π΅Ρ€Π΅Π· всС сопротивлСния ΠΏΡ€ΠΎΡ‚Π΅ΠΊΠ°Π΅Ρ‚ ΠΎΠ΄ΠΈΠ½ (I). ΠŸΠΎΡΡ‚ΠΎΠΌΡƒ Ρ‚ΠΎΠΊ Π²Ρ‹Ρ‡ΠΈΡΠ»ΡΠ΅ΡˆΡŒ ΠΊΠ°ΠΊ ΠžΡ‚Π½ΠΎΡˆΠ΅Π½ΠΈΠ΅ напряТСния источника U ΠΊ Rсумм.

ΠœΠΎΡ‰Π½ΠΎΡΡ‚ΡŒ

P=U*I ΠΈΠ»ΠΈ P=I*I*R (Ρ‚Π°ΠΊ ΠΊΠ°ΠΊ U=I*R).

P1=I*I*R1
P2=I*I*R2
P3=I*I*R3

5) ΠΌΠΎΡ‰Π½ΠΎΡΡ‚ΡŒ элСктричСского Ρ‚ΠΎΠΊΠ° Π² Ρ†Π΅ΠΏΠΈ, состоящСй ΠΈΠ· ΠΏΠ°Ρ€Π°Π»Π»Π΅Π»ΡŒΠ½ΠΎ соСдинСнных участков,
Ρ€Π°Π²Π½Π° суммС мощностСй Π½Π° ΠΎΡ‚Π΄Π΅Π»ΡŒΠ½Ρ‹Ρ… участках:

ΠŸΡ€ΠΈ ΠΏΠ°Ρ€Π°Π»Π»Π΅Π»ΡŒΠ½ΠΎΠΌ соСдинСнии каТдая Π»Π°ΠΌΠΏΠ° подсоСдиняСтся Π½Π° своё номинальноС напряТСниС 220 Π’. ΠΏΡ€ΠΈ этом Π² ΠΊΠ°ΠΆΠ΄ΠΎΠΉ Π»Π°ΠΌΠΏΠ΅ появляСтся свой Π½ΠΎΠΌΠΈΠ½Π°Π»ΡŒΠ½Ρ‹ΠΉ Ρ‚ΠΎΠΊ, ΠΎΠ±Π΅ΡΠΏΠ΅Ρ‡ΠΈΠ²Π°ΡŽΡ‰ΠΈΠΉ Π·Π°Π΄Π°Π½Π½ΠΎΠ΅ свСчСниС Π² соотвСтствии с номинальной ΠΌΠΎΡ‰Π½ΠΎΡΡ‚ΡŒΡŽ. ΠΌΠΎΡ‰Π½ΠΎΡΡ‚ΡŒ зависит ΠΎΡ‚ сопротивлСния Π½ΠΈΡ‚ΠΈ накаливания. Ρ‡Π΅ΠΌ большС сопротивлСниС Π½ΠΈΡ‚ΠΈ, Ρ‚Π΅ΠΌ мСньшС Ρ‚ΠΎΠΊ ΠΈ соотвСтствСнно мСньшС номинальная ΠΌΠΎΡ‰Π½ΠΎΡΡ‚ΡŒ.
ΠΏΡ€ΠΈ ΠΏΠΎΡΠ»Π΅Π΄ΠΎΠ²Π°Ρ‚Π΅Π»ΡŒΠ½ΠΎΠΌ соСдинСнии Ρ‚ΠΎΠΊ ΠΈΠ΄Ρ‘Ρ‚ ΠΎΠ΄ΠΈΠ½ ΠΈ Ρ‚ΠΎΡ‚ ΠΆΠ΅ Π² ΠΊΠ°ΠΆΠ΄ΠΎΠΉ Π»Π°ΠΌΠΏΠ΅. Π° напряТСниС распрСдСляСтся Π² зависимости ΠΎΡ‚ Π΄ΠΎΠ»ΠΈ сопротивлСния ΠΊΠ°ΠΆΠ΄ΠΎΠΉ Π»Π°ΠΌΠΏΡ‹ ΠΏΠΎ ΠΎΡ‚Π½ΠΎΡˆΠ΅Π½ΠΈΡŽ ΠΊ ΡΠΎΠΏΡ€ΠΎΡ‚ΠΈΠ²Π»Π΅Π½ΠΈΡŽ всСй Ρ†Π΅ΠΏΠΈ.
для Ρ†Π΅ΠΏΠΈ ΠΈΠ· Π΄Π²ΡƒΡ… Π»Π°ΠΌΠΏ ΠΎΠ±Ρ‰Π΅Π΅ напряТСниС дСлится.
напряТСниС Π½Π° Π»Π°ΠΌΠΏΠ΅ 40 Π’Ρ‚ Π±ΡƒΠ΄Π΅Ρ‚ 220Π₯60:(40+60)=132; Π’.
напряТСниС Π½Π° Π»Π°ΠΌΠΏΠ΅ 60 Π’Ρ‚ Π±ΡƒΠ΄Π΅Ρ‚ 220Π₯40:(40+60)=80; Π’.

ΠŸΠ°Ρ€Π°Π»Π»Π΅Π»ΡŒΠ½Ρ‹ΠΌ соСдинСниСм сопротивлСний называСтся Ρ‚Π°ΠΊΠΎΠ΅ соСдинСниС, ΠΊΠΎΠ³Π΄Π° Π½Π°Ρ‡Π°Π»Π° сопротивлСний соСдинСны Π² ΠΎΠ΄Π½Ρƒ ΠΎΠ±Ρ‰ΡƒΡŽ Ρ‚ΠΎΡ‡ΠΊΡƒ, Π° ΠΊΠΎΠ½Ρ†Ρ‹ — Π² Π΄Ρ€ΡƒΠ³ΡƒΡŽ.

Для ΠΏΠ°Ρ€Π°Π»Π»Π΅Π»ΡŒΠ½ΠΎΠ³ΠΎ соСдинСния сопротивлСний Ρ…Π°Ρ€Π°ΠΊΡ‚Π΅Ρ€Π½Ρ‹ ΡΠ»Π΅Π΄ΡƒΡŽΡ‰ΠΈΠ΅ свойства:

НапряТСния Π½Π° Π·Π°ΠΆΠΈΠΌΠ°Ρ… всСх сопротивлСний ΠΎΠ΄ΠΈΠ½Π°ΠΊΠΎΠ²Ρ‹:
U 1 = U 2 = U 3 = U;
— ΠΏΡ€ΠΎΠ²ΠΎΠ΄ΠΈΠΌΠΎΡΡ‚ΡŒ всСх ΠΏΠ°Ρ€Π°Π»Π»Π΅Π»ΡŒΠ½ΠΎ соСдинённых сопротивлСний Ρ€Π°Π²Π½Π° суммС проводимостСй ΠΎΡ‚Π΄Π΅Π»ΡŒΠ½Ρ‹Ρ… сопротивлСний:
1/R = 1/R 1 + 1/R 2 + 1/R 3 = R 1 R 2 + R 1 R 3 + R 2 R 3 /R 1 R 2 R 3 ,
Π³Π΄Π΅ R — эквивалСнтноС (Ρ€Π°Π²Π½ΠΎΠ΄Π΅ΠΉΡΡ‚Π²ΡƒΡŽΡ‰Π΅Π΅) сопротивлСниС Ρ‚Ρ€Ρ‘Ρ… сопротивлСний (Π² Π΄Π°Π½Π½ΠΎΠΌ случаС R 1 , R 2 ΠΈ R 3).

Π§Ρ‚ΠΎΠ±Ρ‹ ΠΏΠΎΠ»ΡƒΡ‡ΠΈΡ‚ΡŒ сопротивлСниС Ρ‚Π°ΠΊΠΎΠΉ Ρ†Π΅ΠΏΠΈ, Π½Π°Π΄ΠΎ ΠΏΠ΅Ρ€Π΅Π²Π΅Ρ€Π½ΡƒΡ‚ΡŒ Π΄Ρ€ΠΎΠ±ΡŒ, ΠΎΠΏΡ€Π΅Π΄Π΅Π»ΡΡŽΡ‰ΡƒΡŽ Π²Π΅Π»ΠΈΡ‡ΠΈΠ½Ρƒ Π΅Ρ‘ проводимости. Π‘Π»Π΅Π΄ΠΎΠ²Π°Ρ‚Π΅Π»ΡŒΠ½ΠΎ, сопротивлСниС ΠΏΠ°Ρ€Π°Π»Π»Π΅Π»ΡŒΠ½ΠΎΠ³ΠΎ развСтвлСния ΠΈΠ· Ρ‚Ρ€Ρ‘Ρ… рСзисторов:
R = R 1 R 2 R 3 /R 1 R 2 + R 2 R 3 + R 1 R 3 .

Π­ΠΊΠ²ΠΈΠ²Π°Π»Π΅Π½Ρ‚Π½Ρ‹ΠΌ сопротивлСниСм называСтся Ρ‚Π°ΠΊΠΎΠ΅ сопротивлСниС, ΠΊΠΎΡ‚ΠΎΡ€Ρ‹ΠΌ ΠΌΠΎΠΆΠ½ΠΎ Π·Π°ΠΌΠ΅Π½ΠΈΡ‚ΡŒ нСсколько сопротивлСний (Π²ΠΊΠ»ΡŽΡ‡Π΅Π½Π½Ρ‹Ρ… ΠΏΠ°Ρ€Π°Π»Π»Π΅Π»ΡŒΠ½ΠΎ ΠΈΠ»ΠΈ ΠΏΠΎΡΠ»Π΅Π΄ΠΎΠ²Π°Ρ‚Π΅Π»ΡŒΠ½ΠΎ), Π½Π΅ измСняя Π²Π΅Π»ΠΈΡ‡ΠΈΠ½Ρ‹ Ρ‚ΠΎΠΊΠ° Π² Ρ†Π΅ΠΏΠΈ.

Π§Ρ‚ΠΎΠ±Ρ‹ Π½Π°ΠΉΡ‚ΠΈ эквивалСнтноС сопротивлСниС ΠΏΡ€ΠΈ ΠΏΠ°Ρ€Π°Π»Π»Π΅Π»ΡŒΠ½ΠΎΠΌ соСдинСнии, Π½Π΅ΠΎΠ±Ρ…ΠΎΠ΄ΠΈΠΌΠΎ ΡΠ»ΠΎΠΆΠΈΡ‚ΡŒ проводимости всСх ΠΎΡ‚Π΄Π΅Π»ΡŒΠ½Ρ‹Ρ… участков, Ρ‚.Π΅. Π½Π°ΠΉΡ‚ΠΈ ΠΎΠ±Ρ‰ΡƒΡŽ ΠΏΡ€ΠΎΠ²ΠΎΠ΄ΠΈΠΌΠΎΡΡ‚ΡŒ. Π’Π΅Π»ΠΈΡ‡ΠΈΠ½Π°, обратная ΠΎΠ±Ρ‰Π΅ΠΉ проводимости, ΠΈ являСтся ΠΎΠ±Ρ‰ΠΈΠΌ сопротивлСниСм.

ΠŸΡ€ΠΈ ΠΏΠ°Ρ€Π°Π»Π»Π΅Π»ΡŒΠ½ΠΎΠΌ соСдинСнии эквивалСнтная ΠΏΡ€ΠΎΠ²ΠΎΠ΄ΠΈΠΌΠΎΡΡ‚ΡŒ Ρ€Π°Π²Π½Π° суммС проводимостСй ΠΎΡ‚Π΄Π΅Π»ΡŒΠ½Ρ‹Ρ… Π²Π΅Ρ‚Π²Π΅ΠΉ, ΡΠ»Π΅Π΄ΠΎΠ²Π°Ρ‚Π΅Π»ΡŒΠ½ΠΎ, эквивалСнтноС сопротивлСниС Π² этом случаС всСгда мСньшС наимСньшСго ΠΈΠ· ΠΏΠ°Ρ€Π°Π»Π»Π΅Π»ΡŒΠ½ΠΎ Π²ΠΊΠ»ΡŽΡ‡Π΅Π½Π½Ρ‹Ρ… сопротивлСний.

На ΠΏΡ€Π°ΠΊΡ‚ΠΈΠΊΠ΅ ΠΌΠΎΠ³ΡƒΡ‚ Π±Ρ‹Ρ‚ΡŒ случаи, ΠΊΠΎΠ³Π΄Π° Ρ†Π΅ΠΏΡŒ состоит ΠΈΠ· Π±ΠΎΠ»Π΅Π΅, Ρ‡Π΅ΠΌ Ρ‚Ρ€Ρ‘Ρ… ΠΏΠ°Ρ€Π°Π»Π»Π΅Π»ΡŒΠ½Ρ‹Ρ… Π²Π΅Ρ‚Π²Π΅ΠΉ. ВсС ΠΏΠΎΠ»ΡƒΡ‡Π΅Π½Π½Ρ‹Π΅ ΡΠΎΠΎΡ‚Π½ΠΎΡˆΠ΅Π½ΠΈΡ ΠΎΡΡ‚Π°ΡŽΡ‚ΡΡ справСдливыми ΠΈ для Ρ†Π΅ΠΏΠ΅ΠΉ, состоящих ΠΈΠ· любого числа ΠΏΠ°Ρ€Π°Π»Π»Π΅Π»ΡŒΠ½ΠΎ соСдинённых рСзисторов.

Найдём эквивалСнтноС сопротивлСниС Π΄Π²ΡƒΡ… ΠΏΠ°Ρ€Π°Π»Π»Π΅Π»ΡŒΠ½ΠΎ Π²ΠΊΠ»ΡŽΡ‡Π΅Π½Π½Ρ‹Ρ… сопротивлСний R 1 ΠΈ R 2 (см. рис.). ΠŸΡ€ΠΎΠ²ΠΎΠ΄ΠΈΠΌΠΎΡΡ‚ΡŒ ΠΏΠ΅Ρ€Π²ΠΎΠΉ Π²Π΅Ρ‚Π²ΠΈ Ρ€Π°Π²Π½Π° 1/R 1 , ΠΏΡ€ΠΎΠ²ΠΎΠ΄ΠΈΠΌΠΎΡΡ‚ΡŒ Π²Ρ‚ΠΎΡ€ΠΎΠΉ Π²Π΅Ρ‚Π²ΠΈ — 1/R 2 . ΠžΠ±Ρ‰Π°Ρ ΠΏΡ€ΠΎΠ²ΠΎΠ΄ΠΈΠΌΠΎΡΡ‚ΡŒ:
1/R = 1/R 1 + 1/R 2 .

ΠŸΡ€ΠΈΠ²Π΅Π΄Ρ‘ΠΌ ΠΊ ΠΎΠ±Ρ‰Π΅ΠΌΡƒ Π·Π½Π°ΠΌΠ΅Π½Π°Ρ‚Π΅Π»ΡŽ:
1/R = R 2 + R 1 /R 1 R 2 ,
ΠΎΡ‚ΡΡŽΠ΄Π° эквивалСнтноС сопротивлСниС
R = R 1 R 2 /R 1 + R 2 .

Π­Ρ‚Π° Ρ„ΠΎΡ€ΠΌΡƒΠ»Π° ΠΈ слуТит для расчётов ΠΎΠ±Ρ‰Π΅Π³ΠΎ сопротивлСния Ρ†Π΅ΠΏΠΈ, состоящСй ΠΈΠ· Π΄Π²ΡƒΡ… ΠΏΠ°Ρ€Π°Π»Π»Π΅Π»ΡŒΠ½ΠΎ Π²ΠΊΠ»ΡŽΡ‡Π΅Π½Π½Ρ‹Ρ… сопротивлСний.

Π’Π°ΠΊΠΈΠΌ ΠΎΠ±Ρ€Π°Π·ΠΎΠΌ, эквивалСнтноС сопротивлСниС Π΄Π²ΡƒΡ… ΠΏΠ°Ρ€Π°Π»Π»Π΅Π»ΡŒΠ½ΠΎ Π²ΠΊΠ»ΡŽΡ‡Π΅Π½Π½Ρ‹Ρ… сопротивлСний Ρ€Π°Π²Π½ΠΎ ΠΏΡ€ΠΎΠΈΠ·Π²Π΅Π΄Π΅Π½ΠΈΡŽ этих сопротивлСний, Π΄Π΅Π»Ρ‘Π½Π½ΠΎΠΌΡƒ Π½Π° ΠΈΡ… сумму.

ΠŸΡ€ΠΈ ΠΏΠ°Ρ€Π°Π»Π»Π΅Π»ΡŒΠ½ΠΎΠΌ соСдинСнии n Ρ€Π°Π²Π½Ρ‹Ρ… сопротивлСний R1 эквивалСнтноС сопротивлСниС ΠΈΡ… Π±ΡƒΠ΄Π΅Ρ‚ Π² n Ρ€Π°Π· мСньшС, Ρ‚.Π΅.
R = R 1 /n.

На схСмС, ΠΈΠ·ΠΎΠ±Ρ€Π°ΠΆΡ‘Π½Π½ΠΎΠΉ Π½Π° послСднСм рисункС, Π²ΠΊΠ»ΡŽΡ‡Π΅Π½ΠΎ ΠΏΡΡ‚ΡŒ сопротивлСний R 1 ΠΏΠΎ 30 Ом ΠΊΠ°ΠΆΠ΄ΠΎΠ΅. Π‘Π»Π΅Π΄ΠΎΠ²Π°Ρ‚Π΅Π»ΡŒΠ½ΠΎ, ΠΎΠ±Ρ‰Π΅Π΅ сопротивлСниС R Π±ΡƒΠ΄Π΅Ρ‚
R = R 1 /5 = 30/5 = 6 Ом.

МоТно ΡΠΊΠ°Π·Π°Ρ‚ΡŒ, Ρ‡Ρ‚ΠΎ сумма Ρ‚ΠΎΠΊΠΎΠ², подходящих ΠΊ ΡƒΠ·Π»ΠΎΠ²ΠΎΠΉ Ρ‚ΠΎΡ‡ΠΊΠ΅ А (Π½Π° ΠΏΠ΅Ρ€Π²ΠΎΠΌ рисункС), Ρ€Π°Π²Π½Π° суммС Ρ‚ΠΎΠΊΠΎΠ², ΠΎΡ‚ Π½Π΅Ρ‘ отходящих:
I = I 1 + I 2 + I 3 .

Рассмотрим, ΠΊΠ°ΠΊ происходит Ρ€Π°Π·Π²Π΅Ρ‚Π²Π»Π΅Π½ΠΈΠ΅ Ρ‚ΠΎΠΊΠ° Π² цСпях с сопротивлСниями R 1 ΠΈ R 2 (Π²Ρ‚ΠΎΡ€ΠΎΠΉ рисунок). Π’Π°ΠΊ ΠΊΠ°ΠΊ напряТСниС Π½Π° Π·Π°ΠΆΠΈΠΌΠ°Ρ… этих сопротивлСний ΠΎΠ΄ΠΈΠ½Π°ΠΊΠΎΠ²ΠΎ, Ρ‚ΠΎ
U = I 1 R 1 ΠΈ U = I 2 R 2 .

Π›Π΅Π²Ρ‹Π΅ части этих равСнств ΠΎΠ΄ΠΈΠ½Π°ΠΊΠΎΠ²Ρ‹, ΡΠ»Π΅Π΄ΠΎΠ²Π°Ρ‚Π΅Π»ΡŒΠ½ΠΎ, Ρ€Π°Π²Π½Ρ‹ ΠΈ ΠΏΡ€Π°Π²Ρ‹Π΅ части:
I 1 R 1 = I 2 R 2 ,
ΠΈΠ»ΠΈ
I 1 /I 2 = R 2 /R 1 ,
Ρ‚.Π΅. Ρ‚ΠΎΠΊ ΠΏΡ€ΠΈ ΠΏΠ°Ρ€Π°Π»Π»Π΅Π»ΡŒΠ½ΠΎΠΌ соСдинСнии сопротивлСний развСтвляСтся ΠΎΠ±Ρ€Π°Ρ‚Π½ΠΎ ΠΏΡ€ΠΎΠΏΠΎΡ€Ρ†ΠΈΠΎΠ½Π°Π»ΡŒΠ½ΠΎ сопротивлСниям Π²Π΅Ρ‚Π²Π΅ΠΉ (ΠΈΠ»ΠΈ прямо ΠΏΡ€ΠΎΠΏΠΎΡ€Ρ†ΠΈΠΎΠ½Π°Π»ΡŒΠ½ΠΎ ΠΈΡ… проводимостям). Π§Π΅ΠΌ большС сопротивлСниС Π²Π΅Ρ‚Π²ΠΈ, Ρ‚Π΅ΠΌ мСньшС Ρ‚ΠΎΠΊ Π² Π½Π΅ΠΉ, ΠΈ Π½Π°ΠΎΠ±ΠΎΡ€ΠΎΡ‚.

Π’Π°ΠΊΠΈΠΌ ΠΎΠ±Ρ€Π°Π·ΠΎΠΌ, ΠΈΠ· Π½Π΅ΡΠΊΠΎΠ»ΡŒΠΊΠΈΡ… ΠΎΠ΄ΠΈΠ½Π°ΠΊΠΎΠ²Ρ‹Ρ… рСзисторов ΠΌΠΎΠΆΠ½ΠΎ ΠΏΠΎΠ»ΡƒΡ‡ΠΈΡ‚ΡŒ ΠΎΠ±Ρ‰ΠΈΠΉ рСзистор с бОльшСй ΠΌΠΎΡ‰Π½ΠΎΡΡ‚ΡŒΡŽ рассСивания.

ΠŸΡ€ΠΈ ΠΏΠ°Ρ€Π°Π»Π»Π΅Π»ΡŒΠ½ΠΎΠΌ соСдинСнии Π½Π΅ΠΎΠ΄ΠΈΠ½Π°ΠΊΠΎΠ²Ρ‹Ρ… рСзисторов Π² Π½Π°ΠΈΠ±ΠΎΠ»Π΅Π΅ высокоомном рСзисторС выдСляСтся наибольшая ΠΌΠΎΡ‰Π½ΠΎΡΡ‚ΡŒ.

ΠŸΡ€ΠΈΠΌΠ΅Ρ€ 1. Π˜ΠΌΠ΅ΡŽΡ‚ΡΡ Π΄Π²Π° сопротивлСния, Π²ΠΊΠ»ΡŽΡ‡Π΅Π½Π½Ρ‹Ρ… ΠΏΠ°Ρ€Π°Π»Π»Π΅Π»ΡŒΠ½ΠΎ. Π‘ΠΎΠΏΡ€ΠΎΡ‚ΠΈΠ²Π»Π΅Π½ΠΈΠ΅ R 1 = 25 Ом, Π° R 2 = 50 Ом. ΠžΠΏΡ€Π΅Π΄Π΅Π»ΠΈΡ‚ΡŒ ΠΎΠ±Ρ‰Π΅Π΅ сопротивлСниС Ρ†Π΅ΠΏΠΈ R ΠΎΠ±Ρ‰.

РСшСниС. RΠΎΠ±Ρ‰ = R 1 R 2 /R 1 + R 2 = 25 x 50 / 25 + 50 β‰ˆ 16, 6 Ом.

ΠŸΡ€ΠΈΠΌΠ΅Ρ€ 2. Π’ Π»Π°ΠΌΠΏΠΎΠ²ΠΎΠΌ усилитСлС ΠΈΠΌΠ΅ΡŽΡ‚ΡΡ Ρ‚Ρ€ΠΈ Π»Π°ΠΌΠΏΡ‹, Π½ΠΈΡ‚ΠΈ Π½Π°ΠΊΠ°Π»Π° ΠΊΠΎΡ‚ΠΎΡ€Ρ‹Ρ… Π²ΠΊΠ»ΡŽΡ‡Π΅Π½Ρ‹ ΠΏΠ°Ρ€Π°Π»Π»Π΅Π»ΡŒΠ½ΠΎ. Π’ΠΎΠΊ Π½Π°ΠΊΠ°Π»Π° ΠΏΠ΅Ρ€Π²ΠΎΠΉ Π»Π°ΠΌΠΏΡ‹ I 1 = 1 Π°ΠΌΠΏΠ΅Ρ€, Π²Ρ‚ΠΎΡ€ΠΎΠΉ I 2 = 1, 5 Π°ΠΌΠΏΠ΅Ρ€Π° ΠΈ Ρ‚Ρ€Π΅Ρ‚ΡŒΠ΅ΠΉ I 3 = 2, 5 Π°ΠΌΠΏΠ΅Ρ€Π°. ΠžΠΏΡ€Π΅Π΄Π΅Π»ΠΈΡ‚ΡŒ ΠΎΠ±Ρ‰ΠΈΠΉ Ρ‚ΠΎΠΊ Ρ†Π΅ΠΏΠΈ Π½Π°ΠΊΠ°Π»Π° Π»Π°ΠΌΠΏ усилитСля I ΠΎΠ±Ρ‰.

РСшСниС. I ΠΎΠ±Ρ‰ = I 1 + I 2 + I 3 = 1 + 1, 5 + 2, 5 = 5 Π°ΠΌΠΏΠ΅Ρ€.

ΠŸΠ°Ρ€Π°Π»Π»Π΅Π»ΡŒΠ½ΠΎΠ΅ соСдинСниС рСзисторов часто встрСчаСтся Π² радиотСхничСской Π°ΠΏΠΏΠ°Ρ€Π°Ρ‚ΡƒΡ€Π΅. Π”Π²Π° ΠΈΠ»ΠΈ Π±ΠΎΠ»Π΅Π΅ рСзисторов Π²ΠΊΠ»ΡŽΡ‡Π°Π΅Ρ‚ΡΡ ΠΏΠ°Ρ€Π°Π»Π»Π΅Π»ΡŒΠ½ΠΎ Π² Ρ‚Π΅Ρ… случаях, ΠΊΠΎΠ³Π΄Π° Ρ‚ΠΎΠΊ Π² Ρ†Π΅ΠΏΠΈ слишком большой ΠΈ ΠΌΠΎΠΆΠ΅Ρ‚ Π²Ρ‹Π·Π²Π°Ρ‚ΡŒ Ρ‡Ρ€Π΅Π·ΠΌΠ΅Ρ€Π½Ρ‹ΠΉ Π½Π°Π³Ρ€Π΅Π² рСзистора.

ΠŸΡ€ΠΈΠΌΠ΅Ρ€ΠΎΠΌ ΠΏΠ°Ρ€Π°Π»Π»Π΅Π»ΡŒΠ½ΠΎΠ³ΠΎ соСдинСния ΠΏΠΎΡ‚Ρ€Π΅Π±ΠΈΡ‚Π΅Π»Π΅ΠΉ элСктричСской энСргии ΠΌΠΎΠΆΠ΅Ρ‚ ΡΠ»ΡƒΠΆΠΈΡ‚ΡŒ Π²ΠΊΠ»ΡŽΡ‡Π΅Π½ΠΈΠ΅ элСктричСских Π»Π°ΠΌΠΏ ΠΎΠ±Ρ‹Ρ‡Π½ΠΎΠΉ ΠΎΡΠ²Π΅Ρ‚ΠΈΡ‚Π΅Π»ΡŒΠ½ΠΎΠΉ сСти, ΠΊΠΎΡ‚ΠΎΡ€Ρ‹Π΅ ΡΠΎΠ΅Π΄ΠΈΠ½ΡΡŽΡ‚ΡΡ ΠΏΠ°Ρ€Π°Π»Π»Π΅Π»ΡŒΠ½ΠΎ. Достоинство ΠΏΠ°Ρ€Π°Π»Π»Π΅Π»ΡŒΠ½ΠΎΠ³ΠΎ соСдинСния ΠΏΠΎΡ‚Ρ€Π΅Π±ΠΈΡ‚Π΅Π»Π΅ΠΉ Π·Π°ΠΊΠ»ΡŽΡ‡Π°Π΅Ρ‚ΡΡ Π² Ρ‚ΠΎΠΌ, Ρ‡Ρ‚ΠΎ Π²Ρ‹ΠΊΠ»ΡŽΡ‡Π΅Π½ΠΈΠ΅ ΠΎΠ΄Π½ΠΎΠ³ΠΎ ΠΈΠ· Π½ΠΈΡ… Π½Π΅ влияСт Π½Π° Ρ€Π°Π±ΠΎΡ‚Ρƒ Π΄Ρ€ΡƒΠ³ΠΈΡ….

ΠŸΠ°Ρ€Π°Π»Π»Π΅Π»ΡŒΠ½ΠΎΠ΅ соСдинСниС рСзисторов. ΠŸΡ€ΠΈ ΠΏΠ°Ρ€Π°Π»Π»Π΅Π»ΡŒΠ½ΠΎΠΌ соСдинСнии рСзисторов Π½Π΅ΡΠΊΠΎΠ»ΡŒΠΊΠΈΡ… ΠΏΡ€ΠΈΠ΅ΠΌΠ½ΠΈΠΊΠΎΠ² ΠΎΠ½ΠΈ Π²ΠΊΠ»ΡŽΡ‡Π°ΡŽΡ‚ΡΡ ΠΌΠ΅ΠΆΠ΄Ρƒ двумя Ρ‚ΠΎΡ‡ΠΊΠ°ΠΌΠΈ элСктричСской Ρ†Π΅ΠΏΠΈ, образуя ΠΏΠ°Ρ€Π°Π»Π»Π΅Π»ΡŒΠ½Ρ‹Π΅ Π²Π΅Ρ‚Π²ΠΈ (рис. 26, Π°). ЗамСняя

Π»Π°ΠΌΠΏΡ‹ рСзисторами с сопротивлСниями R1, R2, R3, ΠΏΠΎΠ»ΡƒΡ‡ΠΈΠΌ схСму, ΠΏΠΎΠΊΠ°Π·Π°Π½Π½ΡƒΡŽ Π½Π° рис. 26, Π±.
ΠŸΡ€ΠΈ ΠΏΠ°Ρ€Π°Π»Π»Π΅Π»ΡŒΠ½ΠΎΠΌ соСдинСнии ΠΊΠΎ всСм рСзисторам ΠΏΡ€ΠΈΠ»ΠΎΠΆΠ΅Π½ΠΎ ΠΎΠ΄ΠΈΠ½Π°ΠΊΠΎΠ²ΠΎΠ΅ напряТСниС U. ΠŸΠΎΡΡ‚ΠΎΠΌΡƒ согласно Π·Π°ΠΊΠΎΠ½Ρƒ Ома:

I 1 =U/R 1 ; I 2 =U/R 2 ; I 3 =U/R 3 .

Π’ΠΎΠΊ Π² Π½Π΅Ρ€Π°Π·Π²Π΅Ρ‚Π²Π»Π΅Π½Π½ΠΎΠΉ части Ρ†Π΅ΠΏΠΈ согласно ΠΏΠ΅Ρ€Π²ΠΎΠΌΡƒ Π·Π°ΠΊΠΎΠ½Ρƒ ΠšΠΈΡ€Ρ…Π³ΠΎΡ„Π° I = I 1 +I 2 +I 3 , ΠΈΠ»ΠΈ

I = U / R 1 + U / R 2 + U / R 3 = U (1/R 1 + 1/R 2 + 1/R 3) = U / R эк (23)

Π‘Π»Π΅Π΄ΠΎΠ²Π°Ρ‚Π΅Π»ΡŒΠ½ΠΎ, эквивалСнтноС сопротивлСниС рассматриваСмой Ρ†Π΅ΠΏΠΈ ΠΏΡ€ΠΈ ΠΏΠ°Ρ€Π°Π»Π»Π΅Π»ΡŒΠ½ΠΎΠΌ соСдинСнии Ρ‚Ρ€Π΅Ρ… рСзисторов опрСдСляСтся Ρ„ΠΎΡ€ΠΌΡƒΠ»ΠΎΠΉ

1/R эк = 1/R 1 + 1/R 2 + 1/R 3 (24)

Вводя Π² Ρ„ΠΎΡ€ΠΌΡƒΠ»Ρƒ (24) вмСсто Π·Π½Π°Ρ‡Π΅Π½ΠΈΠΉ 1/R эк, 1/R 1 , 1/R 2 ΠΈ 1/R 3 ΡΠΎΠΎΡ‚Π²Π΅Ρ‚ΡΡ‚Π²ΡƒΡŽΡ‰ΠΈΠ΅ проводимости G эк, G 1 , G 2 ΠΈ G 3 , ΠΏΠΎΠ»ΡƒΡ‡ΠΈΠΌ: эквивалСнтная ΠΏΡ€ΠΎΠ²ΠΎΠ΄ΠΈΠΌΠΎΡΡ‚ΡŒ ΠΏΠ°Ρ€Π°Π»Π»Π΅Π»ΡŒΠ½ΠΎΠΉ Ρ†Π΅ΠΏΠΈ Ρ€Π°Π²Π½Π° суммС проводимостСй ΠΏΠ°Ρ€Π°Π»Π»Π΅Π»ΡŒΠ½ΠΎ соСдинСнных рСзисторов :

G эк = G 1 + G 2 +G 3 (25)

Π’Π°ΠΊΠΈΠΌ ΠΎΠ±Ρ€Π°Π·ΠΎΠΌ, ΠΏΡ€ΠΈ ΡƒΠ²Π΅Π»ΠΈΡ‡Π΅Π½ΠΈΠΈ числа ΠΏΠ°Ρ€Π°Π»Π»Π΅Π»ΡŒΠ½ΠΎ Π²ΠΊΠ»ΡŽΡ‡Π°Π΅ΠΌΡ‹Ρ… рСзисторов Ρ€Π΅Π·ΡƒΠ»ΡŒΡ‚ΠΈΡ€ΡƒΡŽΡ‰Π°Ρ ΠΏΡ€ΠΎΠ²ΠΎΠ΄ΠΈΠΌΠΎΡΡ‚ΡŒ элСктричСской Ρ†Π΅ΠΏΠΈ увСличиваСтся, Π° Ρ€Π΅Π·ΡƒΠ»ΡŒΡ‚ΠΈΡ€ΡƒΡŽΡ‰Π΅Π΅ сопротивлСниС ΡƒΠΌΠ΅Π½ΡŒΡˆΠ°Π΅Ρ‚ΡΡ.
Из ΠΏΡ€ΠΈΠ²Π΅Π΄Π΅Π½Π½Ρ‹Ρ… Ρ„ΠΎΡ€ΠΌΡƒΠ» слСдуСт, Ρ‡Ρ‚ΠΎ Ρ‚ΠΎΠΊΠΈ Ρ€Π°ΡΠΏΡ€Π΅Π΄Π΅Π»ΡΡŽΡ‚ΡΡ ΠΌΠ΅ΠΆΠ΄Ρƒ ΠΏΠ°Ρ€Π°Π»Π»Π΅Π»ΡŒΠ½Ρ‹ΠΌΠΈ вСтвями ΠΎΠ±Ρ€Π°Ρ‚Π½ΠΎ ΠΏΡ€ΠΎΠΏΠΎΡ€Ρ†ΠΈΠΎΠ½Π°Π»ΡŒΠ½ΠΎ ΠΈΡ… элСктричСским сопротивлСниям ΠΈΠ»ΠΈ прямо ΠΏΡ€ΠΎΠΏΠΎΡ€Ρ†ΠΈΠΎΠ½Π°Π»ΡŒΠ½ΠΎ ΠΈΡ… проводимостям. НапримСр, ΠΏΡ€ΠΈ Ρ‚Ρ€Π΅Ρ… вСтвях

I 1: I 2: I 3 = 1/R 1: 1/R 2: 1/R 3 = G 1 + G 2 + G 3 (26)

Π’ этом ΠΎΡ‚Π½ΠΎΡˆΠ΅Π½ΠΈΠΈ ΠΈΠΌΠ΅Π΅Ρ‚ мСсто полная аналогия ΠΌΠ΅ΠΆΠ΄Ρƒ распрСдСлСниСм Ρ‚ΠΎΠΊΠΎΠ² ΠΏΠΎ ΠΎΡ‚Π΄Π΅Π»ΡŒΠ½Ρ‹ΠΌ вСтвям ΠΈ распрСдСлСниСм ΠΏΠΎΡ‚ΠΎΠΊΠΎΠ² Π²ΠΎΠ΄Ρ‹ ΠΏΠΎ Ρ‚Ρ€ΡƒΠ±Π°ΠΌ.
ΠŸΡ€ΠΈΠ²Π΅Π΄Π΅Π½Π½Ρ‹Π΅ Ρ„ΠΎΡ€ΠΌΡƒΠ»Ρ‹ Π΄Π°ΡŽΡ‚ Π²ΠΎΠ·ΠΌΠΎΠΆΠ½ΠΎΡΡ‚ΡŒ ΠΎΠΏΡ€Π΅Π΄Π΅Π»ΠΈΡ‚ΡŒ эквивалСнтноС сопротивлСниС Ρ†Π΅ΠΏΠΈ для Ρ€Π°Π·Π»ΠΈΡ‡Π½Ρ‹Ρ… ΠΊΠΎΠ½ΠΊΡ€Π΅Ρ‚Π½Ρ‹Ρ… случаСв. НапримСр, ΠΏΡ€ΠΈ Π΄Π²ΡƒΡ… ΠΏΠ°Ρ€Π°Π»Π»Π΅Π»ΡŒΠ½ΠΎ Π²ΠΊΠ»ΡŽΡ‡Π΅Π½Π½Ρ‹Ρ… рСзисторах Ρ€Π΅Π·ΡƒΠ»ΡŒΡ‚ΠΈΡ€ΡƒΡŽΡ‰Π΅Π΅ сопротивлСниС Ρ†Π΅ΠΏΠΈ

R эк =R 1 R 2 /(R 1 +R 2)

ΠΏΡ€ΠΈ Ρ‚Ρ€Π΅Ρ… ΠΏΠ°Ρ€Π°Π»Π»Π΅Π»ΡŒΠ½ΠΎ Π²ΠΊΠ»ΡŽΡ‡Π΅Π½Π½Ρ‹Ρ… рСзисторах

R эк =R 1 R 2 R 3 /(R 1 R 2 +R 2 R 3 +R 1 R 3)

ΠŸΡ€ΠΈ ΠΏΠ°Ρ€Π°Π»Π»Π΅Π»ΡŒΠ½ΠΎΠΌ соСдинСнии Π½Π΅ΡΠΊΠΎΠ»ΡŒΠΊΠΈΡ…, Π½Π°ΠΏΡ€ΠΈΠΌΠ΅Ρ€ n, рСзисторов с ΠΎΠ΄ΠΈΠ½Π°ΠΊΠΎΠ²Ρ‹ΠΌ сопротивлСниСм R1 Ρ€Π΅Π·ΡƒΠ»ΡŒΡ‚ΠΈΡ€ΡƒΡŽΡ‰Π΅Π΅ сопротивлСниС Ρ†Π΅ΠΏΠΈ Rэк Π±ΡƒΠ΄Π΅Ρ‚ Π² n Ρ€Π°Π· мСньшС сопротивлСния R1, Ρ‚.Π΅.

R эк = R1 / n (27)

ΠŸΡ€ΠΎΡ…ΠΎΠ΄ΡΡ‰ΠΈΠΉ ΠΏΠΎ ΠΊΠ°ΠΆΠ΄ΠΎΠΉ Π²Π΅Ρ‚Π²ΠΈ Ρ‚ΠΎΠΊ I1, Π² этом случаС Π±ΡƒΠ΄Π΅Ρ‚ Π² ΠΏ Ρ€Π°Π· мСньшС ΠΎΠ±Ρ‰Π΅Π³ΠΎ Ρ‚ΠΎΠΊΠ°:

I1 = I / n (28)

ΠŸΡ€ΠΈ ΠΏΠ°Ρ€Π°Π»Π»Π΅Π»ΡŒΠ½ΠΎΠΌ соСдинСнии ΠΏΡ€ΠΈΠ΅ΠΌΠ½ΠΈΠΊΠΎΠ², всС ΠΎΠ½ΠΈ находятся ΠΏΠΎΠ΄ ΠΎΠ΄Π½ΠΈΠΌ ΠΈ Ρ‚Π΅ΠΌ ΠΆΠ΅ напряТСниСм, ΠΈ Ρ€Π΅ΠΆΠΈΠΌ Ρ€Π°Π±ΠΎΡ‚Ρ‹ ΠΊΠ°ΠΆΠ΄ΠΎΠ³ΠΎ ΠΈΠ· Π½ΠΈΡ… Π½Π΅ зависит ΠΎΡ‚ ΠΎΡΡ‚Π°Π»ΡŒΠ½Ρ‹Ρ…. Π­Ρ‚ΠΎ ΠΎΠ·Π½Π°Ρ‡Π°Π΅Ρ‚, Ρ‡Ρ‚ΠΎ Ρ‚ΠΎΠΊ, проходящий ΠΏΠΎ ΠΊΠ°ΠΊΠΎΠΌΡƒ-Π»ΠΈΠ±ΠΎ ΠΈΠ· ΠΏΡ€ΠΈΠ΅ΠΌΠ½ΠΈΠΊΠΎΠ², Π½Π΅ Π±ΡƒΠ΄Π΅Ρ‚ ΠΎΠΊΠ°Π·Ρ‹Π²Π°Ρ‚ΡŒ сущСствСнного влияния Π½Π° Π΄Ρ€ΡƒΠ³ΠΈΠ΅ ΠΏΡ€ΠΈΠ΅ΠΌΠ½ΠΈΠΊΠΈ. ΠŸΡ€ΠΈ всяком Π²Ρ‹ΠΊΠ»ΡŽΡ‡Π΅Π½ΠΈΠΈ ΠΈΠ»ΠΈ Π²Ρ‹Ρ…ΠΎΠ΄Π΅ ΠΈΠ· строя любого ΠΏΡ€ΠΈΠ΅ΠΌΠ½ΠΈΠΊΠ° ΠΎΡΡ‚Π°Π»ΡŒΠ½Ρ‹Π΅ ΠΏΡ€ΠΈΠ΅ΠΌΠ½ΠΈΠΊΠΈ ΠΎΡΡ‚Π°ΡŽΡ‚ΡΡ Π²ΠΊΠ»ΡŽΡ‡Π΅Π½Π½Ρ‹ΠΌΠΈ. ΠŸΠΎΡΡ‚ΠΎΠΌΡƒ ΠΏΠ°Ρ€Π°Π»Π»Π΅Π»ΡŒΠ½ΠΎΠ΅ соСдинСниС ΠΈΠΌΠ΅Π΅Ρ‚ сущСствСнныС прСимущСства ΠΏΠ΅Ρ€Π΅Π΄ ΠΏΠΎΡΠ»Π΅Π΄ΠΎΠ²Π°Ρ‚Π΅Π»ΡŒΠ½Ρ‹ΠΌ, вслСдствиС Ρ‡Π΅Π³ΠΎ ΠΎΠ½ΠΎ ΠΏΠΎΠ»ΡƒΡ‡ΠΈΠ»ΠΎ Π½Π°ΠΈΠ±ΠΎΠ»Π΅Π΅ ΡˆΠΈΡ€ΠΎΠΊΠΎΠ΅ распространСниС. Π’ частности, элСктричСскиС Π»Π°ΠΌΠΏΡ‹ ΠΈ Π΄Π²ΠΈΠ³Π°Ρ‚Π΅Π»ΠΈ, ΠΏΡ€Π΅Π΄Π½Π°Π·Π½Π°Ρ‡Π΅Π½Π½Ρ‹Π΅ для Ρ€Π°Π±ΠΎΡ‚Ρ‹ ΠΏΡ€ΠΈ ΠΎΠΏΡ€Π΅Π΄Π΅Π»Π΅Π½Π½ΠΎΠΌ (номинальном) напряТСнии, всСгда Π²ΠΊΠ»ΡŽΡ‡Π°ΡŽΡ‚ ΠΏΠ°Ρ€Π°Π»Π»Π΅Π»ΡŒΠ½ΠΎ.
На элСктровозах постоянного Ρ‚ΠΎΠΊΠ° ΠΈ Π½Π΅ΠΊΠΎΡ‚ΠΎΡ€Ρ‹Ρ… Ρ‚Π΅ΠΏΠ»ΠΎΠ²ΠΎΠ·Π°Ρ… тяговыС Π΄Π²ΠΈΠ³Π°Ρ‚Π΅Π»ΠΈ Π² процСссС рСгулирования скорости двиТСния Π½ΡƒΠΆΠ½ΠΎ Π²ΠΊΠ»ΡŽΡ‡Π°Ρ‚ΡŒ ΠΏΠΎΠ΄ Ρ€Π°Π·Π»ΠΈΡ‡Π½Ρ‹Π΅ напряТСния, поэтому ΠΎΠ½ΠΈ Π² процСссС Ρ€Π°Π·Π³ΠΎΠ½Π° ΠΏΠ΅Ρ€Π΅ΠΊΠ»ΡŽΡ‡Π°ΡŽΡ‚ΡΡ с ΠΏΠΎΡΠ»Π΅Π΄ΠΎΠ²Π°Ρ‚Π΅Π»ΡŒΠ½ΠΎΠ³ΠΎ соСдинСния Π½Π° ΠΏΠ°Ρ€Π°Π»Π»Π΅Π»ΡŒΠ½ΠΎΠ΅.

Π‘ΠΎΠ΅Π΄ΠΈΠ½Π΅Π½ΠΈΠ΅ Ρ€Π°Π·Π½Ρ‹ΠΌΠΈ способами позволяСт ΠΏΠΎΠ»ΡƒΡ‡ΠΈΡ‚ΡŒ Π½Π΅ΠΎΠ±Ρ…ΠΎΠ΄ΠΈΠΌΡƒΡŽ Π²Π΅Π»ΠΈΡ‡ΠΈΠ½Ρƒ сопротивлСния ΠΈ ΠΎΠ΄Π½ΠΎΠ³ΠΎ эквивалСнтного рСзистора. ВсСго сущСствуСт Ρ‚Ρ€ΠΈ способы соСдинСния рСзисторов – ΠΏΠΎΡΠ»Π΅Π΄ΠΎΠ²Π°Ρ‚Π΅Π»ΡŒΠ½ΠΎΠ΅, ΠΏΠ°Ρ€Π°Π»Π»Π΅Π»ΡŒΠ½ΠΎΠ΅ ΠΈ смСшанноС.

ΠŸΠΎΡΠ»Π΅Π΄ΠΎΠ²Π°Ρ‚Π΅Π»ΡŒΠ½ΠΎΠ΅ соСдинСниС рСзисторов

ΠŸΠΎΡΠ»Π΅Π΄ΠΎΠ²Π°Ρ‚Π΅Π»ΡŒΠ½ΠΎΠ΅ соСдинСниС рСзисторов ΠΏΡ€Π΅Π΄ΠΏΠΎΠ»Π°Π³Π°Π΅Ρ‚ использованиС Π΄Π²ΡƒΡ… ΠΈ Π±ΠΎΠ»Π΅Π΅ радиоэлСктронных элСмСнта. ΠšΠΎΠ½Π΅Ρ† ΠΏΡ€Π΅Π΄Ρ‹Π΄ΡƒΡ‰Π΅Π³ΠΎ элСмСнта соСдиняСтся с Π½Π°Ρ‡Π°Π»ΠΎΠΌ ΠΏΠΎΡΠ»Π΅Π΄ΡƒΡŽΡ‰Π΅Π³ΠΎ ΠΈ Ρ‚Π°ΠΊ Π΄Π°Π»Π΅Π΅. ΠŸΡ€ΠΈ ΠΏΠΎΡΠ»Π΅Π΄ΠΎΠ²Π°Ρ‚Π΅Π»ΡŒΠ½ΠΎΠΌ соСдинСнии сопротивлСния ΠΈ мощности рассСивания всСх рСзисторов ΡΠΊΠ»Π°Π΄Ρ‹Π²Π°ΡŽΡ‚ΡΡ.
Рассмотрим ΡΠ»Π΅Π΄ΡƒΡŽΡ‰ΠΈΠΉ ΠΏΡ€ΠΈΠΌΠ΅Ρ€. Π‘ΠΎΠ΅Π΄ΠΈΠ½ΠΈΠΌ ΠΏΠΎΡΠ»Π΅Π΄ΠΎΠ²Π°Ρ‚Π΅Π»ΡŒΠ½ΠΎ Ρ‡Π΅Ρ‚Ρ‹Ρ€Π΅ рСзистора, ΠΊΠ°ΠΆΠ΄Ρ‹ΠΉ ΠΈΠΌΠ΅Π΅Ρ‚ R = 1 кОм ΠΈ ΠΌΠΎΡ‰Π½ΠΎΡΡ‚ΡŒ рассСивания P = 0,25 Π’Ρ‚ .

RΠΎΠ±Ρ‰ = R1 + R2 + R3 + R4 = 1кОм + 1кОм + 1кОм + 1кОм = 4 кОм.

PΠΎΠ±Ρ‰ = P1 + P2 + P3 + P4 = 0,25 Π’Ρ‚ + 0,25 Π’Ρ‚ + 0,25 Π’Ρ‚ + 0,25 Π’Ρ‚ = 1 Π’Ρ‚.

Π’Π°ΠΊΠΈΠΌ ΠΎΠ±Ρ€Π°Π·ΠΎΠΌ, получаСтся ΠΎΠ΄ΠΈΠ½ эквивалСнтный ΠΈΠ»ΠΈ ΠΎΠ±Ρ‰ΠΈΠΉ рСзистор, ΠΈΠΌΠ΅ΡŽΡ‰ΠΈΠΉ ΡΠ»Π΅Π΄ΡƒΡŽΡ‰ΠΈΠ΅ ΠΏΠ°Ρ€Π°ΠΌΠ΅Ρ‚Ρ€Ρ‹:
RΠΎΠ±Ρ‰ = 4 кОм; PΠΎΠ±Ρ‰ΠΈ = 1 Π’Ρ‚ .

Π’ ΠΏΠΎΡΠ»Π΅Π΄ΠΎΠ²Π°Ρ‚Π΅Π»ΡŒΠ½ΠΎΠΉ Ρ†Π΅ΠΏΠΈ элСктричСской Ρ‚ΠΎΠΊ ΠΏΡ€ΠΎΡ‚Π΅ΠΊΠ°Π΅Ρ‚ ΠΎΠ΄Π½ΠΎΠΉ ΠΈ Ρ‚ΠΎΠΉ ΠΆΠ΅ Π²Π΅Π»ΠΈΡ‡ΠΈΠ½Ρ‹, поэтому элСктроны Π½Π° протяТСнии всСго ΠΏΡƒΡ‚ΠΈ Π½Π΅ΠΈΠ·Π±Π΅ΠΆΠ½ΠΎ Π½Π°Ρ‚Π°Π»ΠΊΠΈΠ²Π°ΡŽΡ‚ΡΡ Π½Π° всС прСпятствия Π² Π²ΠΈΠ΄Π΅ сопротивлСний. Π‘ ΠΊΠ°ΠΆΠ΄Ρ‹ΠΌ прСпятствиСм ΡƒΠΌΠ΅Π½ΡŒΡˆΠ°Π΅Ρ‚ΡΡ число свободных зарядов, Ρ‡Ρ‚ΠΎ ΠΏΡ€ΠΈΠ²ΠΎΠ΄ΠΈΡ‚ ΠΊ сниТСнию силы элСктричСского Ρ‚ΠΎΠΊΠ°.

ΠŸΡ€ΠΈ ΠΏΠ°Ρ€Π°Π»Π»Π΅Π»ΡŒΠ½ΠΎΠΌ соСдинСнии рСзисторов увСличиваСтся количСство ΠΏΡƒΡ‚Π΅ΠΉ для пСрСмСщСния свободных зарядов, Ρ‚ΠΎ Π΅ΡΡ‚ΡŒ элСктронов, ΠΈΠ· ΠΎΠ΄Π½ΠΎΠ³ΠΎ участка ΠΏΡƒΡ‚ΠΈ ΠΊ Π΄Ρ€ΡƒΠ³ΠΎΠΌΡƒ. ΠŸΠΎΡΡ‚ΠΎΠΌΡƒ ΠΏΡ€ΠΈ ΠΏΠ°Ρ€Π°Π»Π»Π΅Π»ΡŒΠ½ΠΎΠΌ соСдинСнии рСзисторов ΠΈΡ… суммарноС (ΠΎΠ±Ρ‰Π΅Π΅, эквивалСнтноС) сопротивлСниС всСгда Π½ΠΈΠΆΠ΅ наимСньшСго сопротивлСния ΠΈΠ· всСх рСзисторов.

Π’Π΅Π»ΠΈΡ‡ΠΈΠ½Π°, обратная ΡΠΎΠΏΡ€ΠΎΡ‚ΠΈΠ²Π»Π΅Π½ΠΈΡŽ называСтся ΠΏΡ€ΠΎΠ²ΠΎΠ΄ΠΈΠΌΠΎΡΡ‚ΡŒΡŽ. ΠŸΡ€ΠΎΠ²ΠΎΠ΄ΠΈΠΌΠΎΡΡ‚ΡŒ измСряСтся Π² симСнсах [Π‘ΠΌ] ΠΈ обозначаСтся большСй латинской Π±ΡƒΠΊΠ²ΠΎΠΉ G .

G = 1/R = 1/Ом = Бм

ΠŸΠΎΡΡ‚ΠΎΠΌΡƒ ΠΏΡ€ΠΈ Π²Ρ‹ΠΏΠΎΠ»Π½Π΅Π½ΠΈΠΈ Ρ€Π°Π·Π»ΠΈΡ‡Π½Ρ‹Ρ… подсчСтов Π² элСктричСских цСпях, ΠΈΠΌΠ΅ΡŽΡ‰ΠΈΡ… ΠΏΠ°Ρ€Π°Π»Π»Π΅Π»ΡŒΠ½ΠΎΠ΅ соСдинСниС, ΠΏΠΎΠ»ΡŒΠ·ΡƒΡŽΡ‚ΡΡ ΠΏΡ€ΠΎΠ²ΠΎΠ΄ΠΈΠΌΠΎΡΡ‚ΡŒΡŽ.

Если сопротивлСния всСх ΠΏΠ°Ρ€Π°Π»Π»Π΅Π»ΡŒΠ½ΠΎ соСдинСнных рСзисторов Ρ€Π°Π²Π½Ρ‹, Ρ‚ΠΎ для опрСдСлСния ΠΎΠ±Ρ‰Π΅Π³ΠΎ RΠΎΠ±Ρ‰ достаточно R ΠΎΠ΄Π½ΠΎΠ³ΠΎ ΠΈΠ· Π½ΠΈΡ… Ρ€Π°Π·Π΄Π΅Π»ΠΈΡ‚ΡŒ Π½Π° ΠΈΡ… ΠΎΠ±Ρ‰Π΅Π΅ количСство:

Если R1 = R2 = R3 = R4 = R , Ρ‚ΠΎ

RΠΎΠ±Ρ‰ = R/4.

НапримСр, ΠΊΠ°ΠΆΠ΄Ρ‹ΠΉ ΠΈΠ· Ρ‡Π΅Ρ‚Ρ‹Ρ€Π΅Ρ… рСзисторов ΠΈΠΌΠ΅Π΅Ρ‚ R = 10 кОм , Ρ‚ΠΎΠ³Π΄Π°

RΠΎΠ±Ρ‰ = 10 кОм/4 = 2,5 кОм .

ΠœΠΎΡ‰Π½ΠΎΡΡ‚ΠΈ рассСивания ΡΡƒΠΌΠΌΠΈΡ€ΡƒΡŽΡ‚ΡΡ Ρ‚Π°ΠΊΠΆΠ΅, ΠΊΠ°ΠΊ ΠΈ ΠΏΡ€ΠΈ ΠΏΠΎΡΠ»Π΅Π΄ΠΎΠ²Π°Ρ‚Π΅Π»ΡŒΠ½ΠΎΠΌ соСдинСнии.

БмСшанноС соСдинСниС рСзисторов

БмСшанноС соСдинСниС рСзисторов прСдставляСт собой ΠΊΠΎΠΌΠ±ΠΈΠ½Π°Ρ†ΠΈΠΈ ΠΏΠΎΡΠ»Π΅Π΄ΠΎΠ²Π°Ρ‚Π΅Π»ΡŒΠ½Ρ‹Ρ… ΠΈ ΠΏΠ°Ρ€Π°Π»Π»Π΅Π»ΡŒΠ½Ρ‹Ρ… соСдинСний. Π’ ΠΏΡ€ΠΈΠ½Ρ†ΠΈΠΏΠ΅ Π»ΡŽΠ±ΡƒΡŽ Π΄Π°ΠΆΠ΅ ΡΠ°ΠΌΡƒΡŽ ΡΠ»ΠΎΠΆΠ½ΡƒΡŽ ΡΠ»Π΅ΠΊΡ‚Ρ€ΠΈΡ‡Π΅ΡΠΊΡƒΡŽ Ρ†Π΅ΠΏΡŒ, ΡΠΎΡΡ‚ΠΎΡΡ‰ΡƒΡŽ ΠΈΠ· источников питания, Π΄ΠΈΠΎΠ΄ΠΎΠ², ΠΈ Π΄Ρ€ΡƒΠ³ΠΈΡ… радиоэлСктронных элСмСнтов Π² ΠΊΠΎΠ½ΠΊΡ€Π΅Ρ‚Π½Ρ‹ΠΉ ΠΌΠΎΠΌΠ΅Π½Ρ‚ Π²Ρ€Π΅ΠΌΠ΅Π½ΠΈ ΠΌΠΎΠΆΠ½ΠΎ Π·Π°ΠΌΠ΅Π½ΠΈΡ‚ΡŒ рСзисторами ΠΈ источниками напряТСния, ΠΏΠ°Ρ€Π°ΠΌΠ΅Ρ‚Ρ€Ρ‹ ΠΊΠΎΡ‚ΠΎΡ€Ρ‹Ρ… ΠΈΠ·ΠΌΠ΅Π½ΡΡŽΡ‚ΡΡ с ΠΊΠ°ΠΆΠ΄Ρ‹ΠΌ ΠΏΠΎΡΠ»Π΅Π΄ΡƒΡŽΡ‰ΠΈΠΌ ΠΌΠΎΠΌΠ΅Π½Ρ‚ΠΎΠΌ Π²Ρ€Π΅ΠΌΠ΅Π½ΠΈ. Для ΠΏΡ€ΠΈΠΌΠ΅Ρ€Π° ΠΈΠ·ΠΎΠ±Ρ€Π°Π·ΠΈΠΌ схСму, ΠΈΠΌΠ΅ΡŽΡ‰ΡƒΡŽ нСсколько соСдинСний.

ΠŸΠΎΡΠ»Π΅Π΄ΠΎΠ²Π°Ρ‚Π΅Π»ΡŒΠ½ΠΎΠ΅ соСдинСниС схСма Ρ„ΠΎΡ€ΠΌΡƒΠ»Π°. ΠŸΠΎΡΠ»Π΅Π΄ΠΎΠ²Π°Ρ‚Π΅Π»ΡŒΠ½ΠΎΠ΅ ΠΈ ΠΏΠ°Ρ€Π°Π»Π»Π΅Π»ΡŒΠ½ΠΎΠ΅ соСдинСниС

Π‘ΠΎΠ΄Π΅Ρ€ΠΆΠ°Π½ΠΈΠ΅:

Π’ΠΎ всСх элСктричСских схСмах ΠΈΡΠΏΠΎΠ»ΡŒΠ·ΡƒΡŽΡ‚ΡΡ рСзисторы, ΠΏΡ€Π΅Π΄ΡΡ‚Π°Π²Π»ΡΡŽΡ‰ΠΈΠ΅ собой элСмСнты, с Ρ‚ΠΎΡ‡Π½ΠΎ установлСнным Π·Π½Π°Ρ‡Π΅Π½ΠΈΠ΅ΠΌ сопротивлСния. Благодаря спСцифичСским качСствам этих устройств, становится Π²ΠΎΠ·ΠΌΠΎΠΆΠ½ΠΎΠΉ Ρ€Π΅Π³ΡƒΠ»ΠΈΡ€ΠΎΠ²ΠΊΠ° напряТСния ΠΈ силы Ρ‚ΠΎΠΊΠ° Π½Π° Π»ΡŽΠ±Ρ‹Ρ… участках схСмы. Π”Π°Π½Π½Ρ‹Π΅ свойства Π»Π΅ΠΆΠ°Ρ‚ Π² основС Ρ€Π°Π±ΠΎΡ‚Ρ‹ практичСски всСх элСктронных ΠΏΡ€ΠΈΠ±ΠΎΡ€ΠΎΠ² ΠΈ оборудования. Π’Π°ΠΊ, напряТСниС ΠΏΡ€ΠΈ ΠΏΠ°Ρ€Π°Π»Π»Π΅Π»ΡŒΠ½ΠΎΠΌ ΠΈ ΠΏΠΎΡΠ»Π΅Π΄ΠΎΠ²Π°Ρ‚Π΅Π»ΡŒΠ½ΠΎΠΌ соСдинСнии рСзисторов Π±ΡƒΠ΄Π΅Ρ‚ ΠΎΡ‚Π»ΠΈΡ‡Π°Ρ‚ΡŒΡΡ. ΠŸΠΎΡΡ‚ΠΎΠΌΡƒ ΠΊΠ°ΠΆΠ΄Ρ‹ΠΉ Π²ΠΈΠ΄ соСдинСния ΠΌΠΎΠΆΠ΅Ρ‚ ΠΏΡ€ΠΈΠΌΠ΅Π½ΡΡ‚ΡŒΡΡ Ρ‚ΠΎΠ»ΡŒΠΊΠΎ Π² ΠΎΠΏΡ€Π΅Π΄Π΅Π»Π΅Π½Π½Ρ‹Ρ… условиях, Ρ‡Ρ‚ΠΎΠ±Ρ‹ Ρ‚Π° ΠΈΠ»ΠΈ иная элСктричСская схСма ΠΌΠΎΠ³Π»Π° Π² ΠΏΠΎΠ»Π½ΠΎΠΌ объСмС Π²Ρ‹ΠΏΠΎΠ»Π½ΡΡ‚ΡŒ свои Ρ„ΡƒΠ½ΠΊΡ†ΠΈΠΈ.

НапряТСниС ΠΏΡ€ΠΈ ΠΏΠΎΡΠ»Π΅Π΄ΠΎΠ²Π°Ρ‚Π΅Π»ΡŒΠ½ΠΎΠΌ соСдинСнии

ΠŸΡ€ΠΈ ΠΏΠΎΡΠ»Π΅Π΄ΠΎΠ²Π°Ρ‚Π΅Π»ΡŒΠ½ΠΎΠΌ соСдинСнии Π΄Π²Π° рСзистора ΠΈ Π±ΠΎΠ»Π΅Π΅ ΡΠΎΠ΅Π΄ΠΈΠ½ΡΡŽΡ‚ΡΡ Π² ΠΎΠ±Ρ‰ΡƒΡŽ Ρ†Π΅ΠΏΡŒ Ρ‚Π°ΠΊΠΈΠΌ ΠΎΠ±Ρ€Π°Π·ΠΎΠΌ, Ρ‡Ρ‚ΠΎ ΠΊΠ°ΠΆΠ΄Ρ‹ΠΉ ΠΈΠ· Π½ΠΈΡ… ΠΈΠΌΠ΅Π΅Ρ‚ ΠΊΠΎΠ½Ρ‚Π°ΠΊΡ‚ с Π΄Ρ€ΡƒΠ³ΠΈΠΌ устройством Ρ‚ΠΎΠ»ΡŒΠΊΠΎ Π² ΠΎΠ΄Π½ΠΎΠΉ Ρ‚ΠΎΡ‡ΠΊΠ΅. Π˜Π½Π°Ρ‡Π΅ говоря, ΠΊΠΎΠ½Π΅Ρ† ΠΏΠ΅Ρ€Π²ΠΎΠ³ΠΎ рСзистора соСдиняСтся с Π½Π°Ρ‡Π°Π»ΠΎΠΌ Π²Ρ‚ΠΎΡ€ΠΎΠ³ΠΎ, Π° ΠΊΠΎΠ½Π΅Ρ† Π²Ρ‚ΠΎΡ€ΠΎΠ³ΠΎ — с Π½Π°Ρ‡Π°Π»ΠΎΠΌ Ρ‚Ρ€Π΅Ρ‚ΡŒΠ΅Π³ΠΎ ΠΈ Ρ‚.Π΄.

ΠžΡΠΎΠ±Π΅Π½Π½ΠΎΡΡ‚ΡŒΡŽ Π΄Π°Π½Π½ΠΎΠΉ схСмы являСтся ΠΏΡ€ΠΎΡ…ΠΎΠΆΠ΄Π΅Π½ΠΈΠ΅ Ρ‡Π΅Ρ€Π΅Π· всС ΠΏΠΎΠ΄ΠΊΠ»ΡŽΡ‡Π΅Π½Π½Ρ‹Π΅ рСзисторы ΠΎΠ΄Π½ΠΎΠ³ΠΎ ΠΈ Ρ‚ΠΎΠ³ΠΎ ΠΆΠ΅ значСния элСктричСского Ρ‚ΠΎΠΊΠ°. Π‘ возрастаниСм количСства элСмСнтов Π½Π° рассматриваСмом участкС Ρ†Π΅ΠΏΠΈ, Ρ‚Π΅Ρ‡Π΅Π½ΠΈΠ΅ элСктричСского Ρ‚ΠΎΠΊΠ° становится всС Π±ΠΎΠ»Π΅Π΅ Π·Π°Ρ‚Ρ€ΡƒΠ΄Π½Π΅Π½Π½Ρ‹ΠΌ. Π­Ρ‚ΠΎ происходит ΠΈΠ·-Π·Π° увСличСния ΠΎΠ±Ρ‰Π΅Π³ΠΎ сопротивлСния рСзисторов ΠΏΡ€ΠΈ ΠΈΡ… ΠΏΠΎΡΠ»Π΅Π΄ΠΎΠ²Π°Ρ‚Π΅Π»ΡŒΠ½ΠΎΠΌ соСдинСнии. Π”Π°Π½Π½ΠΎΠ΅ свойство отраТаСтся Ρ„ΠΎΡ€ΠΌΡƒΠ»ΠΎΠΉ: R ΠΎΠ±Ρ‰ = R 1 + R 2 .

РаспрСдСлСниС напряТСния, Π² соотвСтствии с Π·Π°ΠΊΠΎΠ½ΠΎΠΌ Ома, осущСствляСтся Π½Π° ΠΊΠ°ΠΆΠ΄Ρ‹ΠΉ рСзистор ΠΏΠΎ Ρ„ΠΎΡ€ΠΌΡƒΠ»Π΅: V Rn = I Rn x R n . Π’Π°ΠΊΠΈΠΌ ΠΎΠ±Ρ€Π°Π·ΠΎΠΌ, ΠΏΡ€ΠΈ ΡƒΠ²Π΅Π»ΠΈΡ‡Π΅Π½ΠΈΠΈ сопротивлСния рСзистора, возрастаСт ΠΈ ΠΏΠ°Π΄Π°ΡŽΡ‰Π΅Π΅ Π½Π° Π½Π΅Π³ΠΎ напряТСниС.

НапряТСниС ΠΏΡ€ΠΈ ΠΏΠ°Ρ€Π°Π»Π»Π΅Π»ΡŒΠ½ΠΎΠΌ соСдинСнии

ΠŸΡ€ΠΈ ΠΏΠ°Ρ€Π°Π»Π»Π΅Π»ΡŒΠ½ΠΎΠΌ соСдинСнии, Π²ΠΊΠ»ΡŽΡ‡Π΅Π½ΠΈΠ΅ рСзисторов Π² ΡΠ»Π΅ΠΊΡ‚Ρ€ΠΈΡ‡Π΅ΡΠΊΡƒΡŽ Ρ†Π΅ΠΏΡŒ выполняСтся Ρ‚Π°ΠΊΠΈΠΌ ΠΎΠ±Ρ€Π°Π·ΠΎΠΌ, Ρ‡Ρ‚ΠΎ всС элСмСнты сопротивлСний ΠΏΠΎΠ΄ΠΊΠ»ΡŽΡ‡Π°ΡŽΡ‚ΡΡ Π΄Ρ€ΡƒΠ³ ΠΊ Π΄Ρ€ΡƒΠ³Ρƒ сразу ΠΎΠ±ΠΎΠΈΠΌΠΈ ΠΊΠΎΠ½Ρ‚Π°ΠΊΡ‚Π°ΠΌΠΈ. Одна Ρ‚ΠΎΡ‡ΠΊΠ°, ΠΏΡ€Π΅Π΄ΡΡ‚Π°Π²Π»ΡΡŽΡ‰Π°Ρ собой элСктричСский ΡƒΠ·Π΅Π», ΠΌΠΎΠΆΠ΅Ρ‚ ΡΠΎΠ΅Π΄ΠΈΠ½ΡΡ‚ΡŒ ΠΎΠ΄Π½ΠΎΠ²Ρ€Π΅ΠΌΠ΅Π½Π½ΠΎ нСсколько рСзисторов.

Π’Π°ΠΊΠΎΠ΅ соСдинСниС ΠΏΡ€Π΅Π΄ΠΏΠΎΠ»Π°Π³Π°Π΅Ρ‚ Ρ‚Π΅Ρ‡Π΅Π½ΠΈΠ΅ ΠΎΡ‚Π΄Π΅Π»ΡŒΠ½ΠΎΠ³ΠΎ Ρ‚ΠΎΠΊΠ° Π² ΠΊΠ°ΠΆΠ΄ΠΎΠΌ рСзисторС. Π‘ΠΈΠ»Π° этого Ρ‚ΠΎΠΊΠ° находится Π² ΠΎΠ±Ρ€Π°Ρ‚Π½ΠΎ ΠΏΡ€ΠΎΠΏΠΎΡ€Ρ†ΠΈΠΎΠ½Π°Π»ΡŒΠ½ΠΎΠΉ . Π’ Ρ€Π΅Π·ΡƒΠ»ΡŒΡ‚Π°Ρ‚Π΅, происходит ΡƒΠ²Π΅Π»ΠΈΡ‡Π΅Π½ΠΈΠ΅ ΠΎΠ±Ρ‰Π΅ΠΉ проводимости Π΄Π°Π½Π½ΠΎΠ³ΠΎ участка Ρ†Π΅ΠΏΠΈ, ΠΏΡ€ΠΈ ΠΎΠ±Ρ‰Π΅ΠΌ ΡƒΠΌΠ΅Π½ΡŒΡˆΠ΅Π½ΠΈΠΈ сопротивлСния. Π’ случаС ΠΏΠ°Ρ€Π°Π»Π»Π΅Π»ΡŒΠ½ΠΎΠ³ΠΎ соСдинСния рСзисторов с Ρ€Π°Π·Π»ΠΈΡ‡Π½Ρ‹ΠΌ сопротивлСниСм, Π·Π½Π°Ρ‡Π΅Π½ΠΈΠ΅ ΠΎΠ±Ρ‰Π΅Π³ΠΎ сопротивлСния Π½Π° этом участкС всСгда Π±ΡƒΠ΄Π΅Ρ‚ Π½ΠΈΠΆΠ΅ самого малСнького сопротивлСния ΠΎΡ‚Π΄Π΅Π»ΡŒΠ½ΠΎ взятого рСзистора.

На прСдставлСнной схСмС, напряТСниС ΠΌΠ΅ΠΆΠ΄Ρƒ Ρ‚ΠΎΡ‡ΠΊΠ°ΠΌΠΈ А ΠΈ Π’ прСдставляСт собой Π½Π΅ Ρ‚ΠΎΠ»ΡŒΠΊΠΎ ΠΎΠ±Ρ‰Π΅Π΅ напряТСниС для всСго участка, Π½ΠΎ ΠΈ напряТСниС, ΠΏΠΎΡΡ‚ΡƒΠΏΠ°ΡŽΡ‰Π΅Π΅ ΠΊ ΠΊΠ°ΠΆΠ΄ΠΎΠΌΡƒ ΠΎΡ‚Π΄Π΅Π»ΡŒΠ½ΠΎ взятому рСзистору. Π’Π°ΠΊΠΈΠΌ ΠΎΠ±Ρ€Π°Π·ΠΎΠΌ, Π² случаС ΠΏΠ°Ρ€Π°Π»Π»Π΅Π»ΡŒΠ½ΠΎΠ³ΠΎ соСдинСния, напряТСниС, ΠΏΠΎΠ΄Π°Π²Π°Π΅ΠΌΠΎΠ΅ ΠΊΠΎ всСм рСзисторам, Π±ΡƒΠ΄Π΅Ρ‚ ΠΎΠ΄ΠΈΠ½Π°ΠΊΠΎΠ²Ρ‹ΠΌ.

Π’ Ρ€Π΅Π·ΡƒΠ»ΡŒΡ‚Π°Ρ‚Π΅, напряТСниС ΠΏΡ€ΠΈ ΠΏΠ°Ρ€Π°Π»Π»Π΅Π»ΡŒΠ½ΠΎΠΌ ΠΈ ΠΏΠΎΡΠ»Π΅Π΄ΠΎΠ²Π°Ρ‚Π΅Π»ΡŒΠ½ΠΎΠΌ соСдинСнии Π±ΡƒΠ΄Π΅Ρ‚ ΠΎΡ‚Π»ΠΈΡ‡Π°Ρ‚ΡŒΡΡ Π² ΠΊΠ°ΠΆΠ΄ΠΎΠΌ случаС. Благодаря этому свойству, имССтся Ρ€Π΅Π°Π»ΡŒΠ½Π°Ρ Π²ΠΎΠ·ΠΌΠΎΠΆΠ½ΠΎΡΡ‚ΡŒ ΠΎΡ‚Ρ€Π΅Π³ΡƒΠ»ΠΈΡ€ΠΎΠ²Π°Ρ‚ΡŒ Π΄Π°Π½Π½ΡƒΡŽ Π²Π΅Π»ΠΈΡ‡ΠΈΠ½Ρƒ Π½Π° любом участкС Ρ†Π΅ΠΏΠΈ.

ΠŸΠ°Ρ€Π°Π»Π»Π΅Π»ΡŒΠ½ΠΎΠ΅ соСдинСниС элСктричСских элСмСнтов (ΠΏΡ€ΠΎΠ²ΠΎΠ΄Π½ΠΈΠΊΠΎΠ², сопротивлСний, СмкостСй, индуктивностСй) — это Ρ‚Π°ΠΊΠΎΠ΅ соСдинСниС, ΠΏΡ€ΠΈ ΠΊΠΎΡ‚ΠΎΡ€ΠΎΠΌ ΠΏΠΎΠ΄ΠΊΠ»ΡŽΡ‡Π΅Π½Π½Ρ‹Π΅ элСмСнты Ρ†Π΅ΠΏΠΈ ΠΈΠΌΠ΅ΡŽΡ‚ Π΄Π²Π° ΠΎΠ±Ρ‰ΠΈΡ… ΡƒΠ·Π»Π° ΠΏΠΎΠ΄ΠΊΠ»ΡŽΡ‡Π΅Π½ΠΈΡ.

Π”Ρ€ΡƒΠ³ΠΎΠ΅ ΠΎΠΏΡ€Π΅Π΄Π΅Π»Π΅Π½ΠΈΠ΅: сопротивлСния ΠΏΠΎΠ΄ΠΊΠ»ΡŽΡ‡Π΅Π½Ρ‹ ΠΏΠ°Ρ€Π°Π»Π»Π΅Π»ΡŒΠ½ΠΎ, Ссли ΠΎΠ½ΠΈ ΠΏΠΎΠ΄ΠΊΠ»ΡŽΡ‡Π΅Π½Ρ‹ ΠΎΠ΄Π½ΠΎ ΠΈ Ρ‚ΠΎΠΉ ΠΆΠ΅ ΠΏΠ°Ρ€Π΅ ΡƒΠ·Π»ΠΎΠ².

ГрафичСскоС ΠΎΠ±ΠΎΠ·Π½Π°Ρ‡Π΅Π½ΠΈΠ΅ схСмы ΠΏΠ°Ρ€Π°Π»Π»Π΅Π»ΡŒΠ½ΠΎΠ³ΠΎ соСднинСния

На ΠΏΡ€ΠΈΠ²Π΅Π΄Π΅Π½Π½ΠΎΠΌ рисункС ΠΏΠΎΠΊΠ°Π·Π°Π½Π° схСма ΠΏΠ°Ρ€Π°Π»Π»Π΅Π»ΡŒΠ½ΠΎΠ΅ ΠΏΠΎΠ΄ΠΊΠ»ΡŽΡ‡Π΅Π½ΠΈΡ сопротивлСний R1, R2, R3, R4. Из схСмы Π²ΠΈΠ΄Π½ΠΎ, Ρ‡Ρ‚ΠΎ всС эти Ρ‡Π΅Ρ‚Ρ‹Ρ€Π΅ сопротивлСния ΠΈΠΌΠ΅ΡŽΡ‚ Π΄Π²Π΅ ΠΎΠ±Ρ‰ΠΈΠ΅ Ρ‚ΠΎΡ‡ΠΊΠΈ (ΡƒΠ·Π»Π° ΠΏΠΎΠ΄ΠΊΠ»ΡŽΡ‡Π΅Π½ΠΈΡ).

Π’ элСктротСхникС принято, Π½ΠΎ Π½Π΅ строго трСбуСтся, Ρ€ΠΈΡΠΎΠ²Π°Ρ‚ΡŒ ΠΏΡ€ΠΎΠ²ΠΎΠ΄Π° Π³ΠΎΡ€ΠΈΠ·ΠΎΠ½Ρ‚Π°Π»ΡŒΠ½ΠΎ ΠΈ Π²Π΅Ρ€Ρ‚ΠΈΠΊΠ°Π»ΡŒΠ½ΠΎ. ΠŸΠΎΡΡ‚ΠΎΠΌΡƒ эту ΠΆΠ΅ схСму ΠΌΠΎΠΆΠ½ΠΎ ΠΈΠ·ΠΎΠ±Ρ€Π°Π·ΠΈΡ‚ΡŒ, ΠΊΠ°ΠΊ Π½Π° рисункС Π½ΠΈΠΆΠ΅. Π­Ρ‚ΠΎ Ρ‚ΠΎΠΆΠ΅ ΠΏΠ°Ρ€Π°Π»Π»Π΅Π»ΡŒΠ½ΠΎΠ΅ соСдинСниС Ρ‚Π΅Ρ… ΠΆΠ΅ самых сопротивлСний.

Π€ΠΎΡ€ΠΌΡƒΠ»Π° для расчСта ΠΏΠ°Ρ€Π°Π»Π»Π΅Π»ΡŒΠ½ΠΎΠ³ΠΎ соСдинСния сопротивлСний

ΠŸΡ€ΠΈ ΠΏΠ°Ρ€Π°Π»Π»Π΅Π»ΡŒΠ½ΠΎΠΌ соСдинСнии обратная Π²Π΅Π»ΠΈΡ‡ΠΈΠ½Π° ΠΎΡ‚ эквивалСнтного сопротивлСния Ρ€Π°Π²Π½Π° суммС ΠΎΠ±Ρ€Π°Ρ‚Π½Ρ‹Ρ… Π²Π΅Π»ΠΈΡ‡ΠΈΠ½ всСх ΠΏΠ°Ρ€Π°Π»Π»Π΅Π»ΡŒΠ½ΠΎ ΠΏΠΎΠ΄ΠΊΠ»ΡŽΡ‡Π΅Π½Π½Ρ‹Ρ… сопротивлСний. ЭквивалСнтная ΠΏΡ€ΠΎΠ²ΠΎΠ΄ΠΈΠΌΠΎΡΡ‚ΡŒ Ρ€Π°Π²Π½Π° суммС всСх ΠΏΠ°Ρ€Π°Π»Π»Π΅Π»ΡŒΠ½ΠΎ ΠΏΠΎΠ΄ΠΊΠ»ΡŽΡ‡Π΅Π½Π½Ρ‹Ρ… проводимостСй элСктричСской схСмы.

Для ΠΏΡ€ΠΈΠ²Π΅Π΄Π΅Π½Π½ΠΎΠΉ Π²Ρ‹ΡˆΠ΅ схСмы эквивалСнтноС сопротивлСниС ΠΌΠΎΠΆΠ½ΠΎ Ρ€Π°ΡΡΡ‡ΠΈΡ‚Π°Ρ‚ΡŒ ΠΏΠΎ Ρ„ΠΎΡ€ΠΌΡƒΠ»Π΅:

Π’ частном случаС ΠΏΡ€ΠΈ ΠΏΠΎΠ΄ΠΊΠ»ΡŽΡ‡Π΅Π½ΠΈΠΈ ΠΏΠ°Ρ€Π°Π»Π»Π΅Π»ΡŒΠ½ΠΎ Π΄Π²ΡƒΡ… сопротивлСний:

Π­ΠΊΠ²ΠΈΠ²Π°Π»Π΅Π½Ρ‚Π½ΠΎΠ΅ сопротивлСниС Ρ†Π΅ΠΏΠΈ опрСдСляСтся ΠΏΠΎ Ρ„ΠΎΡ€ΠΌΡƒΠ»Π΅:

Π’ случаС ΠΏΠΎΠ΄ΠΊΠ»ΡŽΡ‡Π΅Π½ΠΈΡ «n» ΠΎΠ΄ΠΈΠ½Π°ΠΊΠΎΠ²Ρ‹Ρ… сопротивлСний, эквивалСнтноС сопротивлСниС ΠΌΠΎΠΆΠ½ΠΎ Ρ€Π°ΡΡΡ‡ΠΈΡ‚Π°Ρ‚ΡŒ ΠΏΠΎ частной Ρ„ΠΎΡ€ΠΌΡƒΠ»Π΅:

Π€ΠΎΡ€ΠΌΡƒΠ»Ρ‹ для частного рассчСта Π²Ρ‹Ρ‚Π΅ΠΊΠ°ΡŽΡ‚ ΠΈΠ· основной Ρ„ΠΎΡ€ΠΌΡƒΠ»Ρ‹.

Π€ΠΎΡ€ΠΌΡƒΠ»Π° для расчСта ΠΏΠ°Ρ€Π°Π»Π»Π΅Π»ΡŒΠ½ΠΎΠ³ΠΎ соСдинСния СмкостСй (кондСнсаторов)

ΠŸΡ€ΠΈ ΠΏΠ°Ρ€Π°Π»Π»Π΅Π»ΡŒΠ½ΠΎΠΌ ΠΏΠΎΠ΄ΠΊΠ»ΡŽΡ‡Π΅Π½ΠΈΠΈ СмкостСй (кондСнсаторов) эквивалСнтная Π΅ΠΌΠΊΠΎΡΡ‚ΡŒ Ρ€Π°Π²Π½Π° суммС ΠΏΠ°Ρ€Π°Π»Π»Π΅Π»ΡŒΠ½ΠΎ ΠΏΠΎΠ΄ΠΊΠ»ΡŽΡ‡Π΅Π½Π½Ρ‹Ρ… СмкостСй:

Π€ΠΎΡ€ΠΌΡƒΠ»Π° для расчСта ΠΏΠ°Ρ€Π°Π»Π»Π΅Π»ΡŒΠ½ΠΎΠ³ΠΎ соСдинСния индуктивностСй

ΠŸΡ€ΠΈ ΠΏΠ°Ρ€Π°Π»Π»Π΅Π»ΡŒΠ½ΠΎΠΌ ΠΏΠΎΠ΄ΠΊΠ»ΡŽΡ‡Π΅Π½ΠΈΠΈ индуктивностСй, эквивалСнтная ΠΈΠ½Π΄ΡƒΠΊΡ‚ΠΈΠ²Π½ΠΎΡΡ‚ΡŒ рассчитываСтся Ρ‚Π°ΠΊ ΠΆΠ΅, ΠΊΠ°ΠΊ ΠΈ эквивалСнтноС сопротивлСниС ΠΏΡ€ΠΈ ΠΏΠ°Ρ€Π°Π»Π»Π΅Π»ΡŒΠ½ΠΎΠΌ соСдинСнии:

НСобходимо ΠΎΠ±Ρ€Π°Ρ‚ΠΈΡ‚ΡŒ Π²Π½ΠΈΠΌΠ°Π½ΠΈΠ΅, Ρ‡Ρ‚ΠΎ Π² Ρ„ΠΎΡ€ΠΌΡƒΠ»Π΅ Π½Π΅ ΡƒΡ‡Ρ‚Π΅Π½Ρ‹ Π²Π·Π°ΠΈΠΌΠ½Ρ‹Π΅ индуктивности.

ΠŸΡ€ΠΈΠΌΠ΅Ρ€ свСртывания ΠΏΠ°Ρ€Π°Π»Π»Π΅Π»ΡŒΠ½ΠΎΠ³ΠΎ сопротивлСния

Для участка элСктричСской Ρ†Π΅ΠΏΠΈ Π½Π΅ΠΎΠ±Ρ…ΠΎΠ΄ΠΈΠΌΠΎ Π½Π°ΠΉΡ‚ΠΈ ΠΏΠ°Ρ€Π°Π»Π»Π΅Π»ΡŒΠ½ΠΎΠ΅ соСдинСниС сопротивлСний Π²Ρ‹ΠΏΠΎΠ»Π½ΠΈΡ‚ΡŒ ΠΈΡ… ΠΏΡ€Π΅ΠΎΠ±Ρ€Π°Π·ΠΎΠ²Π°Π½ΠΈΠ΅ Π΄ΠΎ ΠΎΠ΄Π½ΠΎΠ³ΠΎ.

Из схСмы Π²ΠΈΠ΄Π½ΠΎ, Ρ‡Ρ‚ΠΎ ΠΏΠ°Ρ€Π°Π»Π»Π΅Π»ΡŒΠ½ΠΎ ΠΏΠΎΠ΄ΠΊΠ»ΡŽΡ‡Π΅Π½Ρ‹ Ρ‚ΠΎΠ»ΡŒΠΊΠΎ R2 ΠΈ R4. R3 Π½Π΅ ΠΏΠ°Ρ€Π°Π»Π»Π΅Π»ΡŒΠ½ΠΎ, Ρ‚.ΠΊ. ΠΎΠ΄Π½ΠΈΠΌ ΠΊΠΎΠ½Ρ†ΠΎΠΌ ΠΎΠ½ΠΎ ΠΏΠΎΠ΄ΠΊΠ»ΡŽΡ‡Π΅Π½ΠΎ ΠΊ E1. R1 — ΠΎΠ΄Π½ΠΈΠΌ ΠΊΠΎΠ½Ρ†ΠΎΠΌ ΠΏΠΎΠ΄ΠΊΠ»ΡŽΡ‡Π΅Π½ΠΎ ΠΊ R5, Π° Π½Π΅ ΠΊ ΡƒΠ·Π»Ρƒ. R5 — ΠΎΠ΄Π½ΠΈΠΌ ΠΊΠΎΠ½Ρ†ΠΎΠΌ ΠΏΠΎΠ΄ΠΊΠ»ΡŽΡ‡Π΅Π½ΠΎ ΠΊ R1, Π° Π½Π΅ ΠΊ ΡƒΠ·Π»Ρƒ. МоТно Ρ‚Π°ΠΊ ΠΆΠ΅ Π³ΠΎΠ²ΠΎΡ€ΠΈΡ‚ΡŒ, Ρ‡Ρ‚ΠΎ ΠΏΠΎΡΠ»Π΅Π΄ΠΎΠ²Π°Ρ‚Π΅Π»ΡŒΠ½ΠΎΠ΅ соСдинСниС сопротивлСний R1 ΠΈ R5 ΠΏΠΎΠ΄ΠΊΠ»ΡŽΡ‡Π΅Π½ΠΎ ΠΏΠ°Ρ€Π°Π»Π»Π΅Π»ΡŒΠ½ΠΎ с R2 ΠΈ R4.

Π’ΠΎΠΊ ΠΏΡ€ΠΈ ΠΏΠ°Ρ€Π°Π»Π»Π΅Π»ΡŒΠ½ΠΎΠΌ соСдинСнии

ΠŸΡ€ΠΈ ΠΏΠ°Ρ€Π°Π»Π»Π΅Π»ΡŒΠ½ΠΎΠΌ соСдинСнии сопротивлСний Ρ‚ΠΎΠΊ Ρ‡Π΅Ρ€Π΅Π· ΠΊΠ°ΠΆΠ΄ΠΎΠ΅ сопротивлСниС Π² ΠΎΠ±Ρ‰Π΅ΠΌ случаС Ρ€Π°Π·Π½Ρ‹ΠΉ. Π’Π΅Π»ΠΈΡ‡ΠΈΠ½Π° Ρ‚ΠΎΠΊΠ° ΠΎΠ±Ρ€Π°Ρ‚Π½ΠΎ ΠΏΡ€ΠΎΠΏΠΎΡ€Ρ†ΠΈΠΎΠ½Π°Π»ΡŒΠ½Π° Π²Π΅Π»ΠΈΡ‡ΠΈΠ½Π΅ сопротивлСния.

НапряТСниС ΠΏΡ€ΠΈ ΠΏΠ°Ρ€Π°Π»Π»Π΅Π»ΡŒΠ½ΠΎΠΌ соСдинСнии

ΠŸΡ€ΠΈ ΠΏΠ°Ρ€Π°Π»Π»Π΅Π»ΡŒΠ½ΠΎΠΌ соСдинСнии Ρ€Π°Π·Π½ΠΎΡΡ‚ΡŒ ΠΏΠΎΡ‚Π΅Π½Ρ†ΠΈΠ°Π»ΠΎΠ² ΠΌΠ΅ΠΆΠ΄Ρƒ ΡƒΠ·Π»Π°ΠΌΠΈ, ΠΎΠ±ΡŠΠ΅Π΄ΠΈΠ½ΡΡŽΡ‰ΠΈΠΌΠΈ элСмСнты Ρ†Π΅ΠΏΠΈ, ΠΎΠ΄ΠΈΠ½Π°ΠΊΠΎΠ²Π° для всСх элСмСнтов.

ΠŸΡ€ΠΈΠΌΠ΅Π½Π΅Π½ΠΈΠ΅ ΠΏΠ°Ρ€Π°Π»Π»Π΅Π»ΡŒΠ½ΠΎΠ³ΠΎ соСдинСния

1. Π’ ΠΏΡ€ΠΎΠΌΡ‹ΡˆΠ»Π΅Π½Π½ΠΎΡΡ‚ΠΈ ΠΈΠ·Π³ΠΎΡ‚Π°Π²Π»ΠΈΠ²Π°ΡŽΡ‚ΡΡ сопротивлСния ΠΎΠΏΡ€Π΅Π΄Π΅Π»Π΅Π½Π½Ρ‹Ρ… Π²Π΅Π»ΠΈΡ‡ΠΈΠ½. Иногда Π½Π΅ΠΎΠ±Ρ…ΠΎΠ΄ΠΈΠΌΠΎ ΠΏΠΎΠ»ΡƒΡ‡ΠΈΡ‚ΡŒ Π·Π½Π°Ρ‡Π΅Π½ΠΈΠ΅ сопротивлСния Π²Π½Π΅ Π΄Π°Π½Π½Ρ‹Ρ… рядов. Для этого ΠΌΠΎΠΆΠ½ΠΎ ΠΏΠΎΠ΄ΠΊΠ»ΡŽΡ‡ΠΈΡ‚ΡŒ нСсколько сопротивлСний ΠΏΠ°Ρ€Π°Π»Π»Π΅Π»ΡŒΠ½ΠΎ. Π­ΠΊΠ²ΠΈΠ²Π°Π»Π΅Π½Ρ‚Π½ΠΎΠ΅ сопротивлСниС всСгда Π±ΡƒΠ΄Π΅Ρ‚ мСньшС самого большого Π½ΠΎΠΌΠΈΠ½Π°Π»Π° сопротивлСния.

2. Π”Π΅Π»ΠΈΡ‚Π΅Π»ΡŒ Ρ‚ΠΎΠΊΠΎΠ².

Π‘ΠΎΠ΄Π΅Ρ€ΠΆΠ°Π½ΠΈΠ΅:

Π’Π΅Ρ‡Π΅Π½ΠΈΠ΅ Ρ‚ΠΎΠΊΠ° Π² элСктричСской Ρ†Π΅ΠΏΠΈ осущСствляСтся ΠΏΠΎ ΠΏΡ€ΠΎΠ²ΠΎΠ΄Π½ΠΈΠΊΠ°ΠΌ, Π² Π½Π°ΠΏΡ€Π°Π²Π»Π΅Π½ΠΈΠΈ ΠΎΡ‚ источника ΠΊ потрСбитСлям. Π’ Π±ΠΎΠ»ΡŒΡˆΠΈΠ½ΡΡ‚Π²Π΅ ΠΏΠΎΠ΄ΠΎΠ±Π½Ρ‹Ρ… схСм ΠΈΡΠΏΠΎΠ»ΡŒΠ·ΡƒΡŽΡ‚ΡΡ ΠΌΠ΅Π΄Π½Ρ‹Π΅ ΠΏΡ€ΠΎΠ²ΠΎΠ΄Π° ΠΈ элСктричСскиС ΠΏΡ€ΠΈΠ΅ΠΌΠ½ΠΈΠΊΠΈ Π² Π·Π°Π΄Π°Π½Π½ΠΎΠΌ количСствС, ΠΎΠ±Π»Π°Π΄Π°ΡŽΡ‰ΠΈΠ΅ Ρ€Π°Π·Π»ΠΈΡ‡Π½Ρ‹ΠΌ сопротивлСниСм. Π’ зависимости выполняСмых Π·Π°Π΄Π°Ρ‡, Π² элСктричСских цСпях ΠΈΡΠΏΠΎΠ»ΡŒΠ·ΡƒΠ΅Ρ‚ΡΡ ΠΏΠΎΡΠ»Π΅Π΄ΠΎΠ²Π°Ρ‚Π΅Π»ΡŒΠ½ΠΎΠ΅ ΠΈ ΠΏΠ°Ρ€Π°Π»Π»Π΅Π»ΡŒΠ½ΠΎΠ΅ соСдинСниС ΠΏΡ€ΠΎΠ²ΠΎΠ΄Π½ΠΈΠΊΠΎΠ². Π’ Π½Π΅ΠΊΠΎΡ‚ΠΎΡ€Ρ‹Ρ… случаях ΠΌΠΎΠ³ΡƒΡ‚ Π±Ρ‹Ρ‚ΡŒ ΠΏΡ€ΠΈΠΌΠ΅Π½Π΅Π½Ρ‹ ΠΎΠ±Π° Ρ‚ΠΈΠΏΠ° соСдинСний, Ρ‚ΠΎΠ³Π΄Π° этот Π²Π°Ρ€ΠΈΠ°Π½Ρ‚ Π±ΡƒΠ΄Π΅Ρ‚ Π½Π°Π·Ρ‹Π²Π°Ρ‚ΡŒΡΡ ΡΠΌΠ΅ΡˆΠ°Π½Π½Ρ‹ΠΌ. КаТдая схСма ΠΈΠΌΠ΅Π΅Ρ‚ свои особСнности ΠΈ отличия, поэтому ΠΈΡ… Π½ΡƒΠΆΠ½ΠΎ ΠΎΠ±ΡΠ·Π°Ρ‚Π΅Π»ΡŒΠ½ΠΎ Π·Π°Ρ€Π°Π½Π΅Π΅ ΡƒΡ‡ΠΈΡ‚Ρ‹Π²Π°Ρ‚ΡŒ ΠΏΡ€ΠΈ ΠΏΡ€ΠΎΠ΅ΠΊΡ‚ΠΈΡ€ΠΎΠ²Π°Π½ΠΈΠΈ Ρ†Π΅ΠΏΠ΅ΠΉ, Ρ€Π΅ΠΌΠΎΠ½Ρ‚Π΅ ΠΈ обслуТивании элСктрооборудования.

ΠŸΠΎΡΠ»Π΅Π΄ΠΎΠ²Π°Ρ‚Π΅Π»ΡŒΠ½ΠΎΠ΅ соСдинСниС ΠΏΡ€ΠΎΠ²ΠΎΠ΄Π½ΠΈΠΊΠΎΠ²

Π’ элСктротСхникС большоС Π·Π½Π°Ρ‡Π΅Π½ΠΈΠ΅ ΠΈΠΌΠ΅Π΅Ρ‚ ΠΏΠΎΡΠ»Π΅Π΄ΠΎΠ²Π°Ρ‚Π΅Π»ΡŒΠ½ΠΎΠ΅ ΠΈ ΠΏΠ°Ρ€Π°Π»Π»Π΅Π»ΡŒΠ½ΠΎΠ΅ соСдинСниС ΠΏΡ€ΠΎΠ²ΠΎΠ΄Π½ΠΈΠΊΠΎΠ² Π² элСктричСской Ρ†Π΅ΠΏΠΈ. Π‘Ρ€Π΅Π΄ΠΈ Π½ΠΈΡ… часто ΠΈΡΠΏΠΎΠ»ΡŒΠ·ΡƒΠ΅Ρ‚ΡΡ схСма ΠΏΠΎΡΠ»Π΅Π΄ΠΎΠ²Π°Ρ‚Π΅Π»ΡŒΠ½ΠΎΠ³ΠΎ соСдинСния ΠΏΡ€ΠΎΠ²ΠΎΠ΄Π½ΠΈΠΊΠΎΠ² ΠΏΡ€Π΅Π΄ΠΏΠΎΠ»Π°Π³Π°ΡŽΡ‰Π°Ρ Ρ‚Π°ΠΊΠΎΠ΅ ΠΆΠ΅ соСдинСниС ΠΏΠΎΡ‚Ρ€Π΅Π±ΠΈΡ‚Π΅Π»Π΅ΠΉ. Π’ этом случаС Π²ΠΊΠ»ΡŽΡ‡Π΅Π½ΠΈΠ΅ Π² Ρ†Π΅ΠΏΡŒ выполняСтся Π΄Ρ€ΡƒΠ³ Π·Π° Π΄Ρ€ΡƒΠ³ΠΎΠΌ Π² порядкС очСрСдности. Π’ΠΎ Π΅ΡΡ‚ΡŒ, Π½Π°Ρ‡Π°Π»ΠΎ ΠΎΠ΄Π½ΠΎΠ³ΠΎ потрСбитСля соСдиняСтся с ΠΊΠΎΠ½Ρ†ΠΎΠΌ Π΄Ρ€ΡƒΠ³ΠΎΠ³ΠΎ ΠΏΡ€ΠΈ ΠΏΠΎΠΌΠΎΡ‰ΠΈ ΠΏΡ€ΠΎΠ²ΠΎΠ΄ΠΎΠ², Π±Π΅Π· ΠΊΠ°ΠΊΠΈΡ…-Π»ΠΈΠ±ΠΎ ΠΎΡ‚Π²Π΅Ρ‚Π²Π»Π΅Π½ΠΈΠΉ.

Бвойства Ρ‚Π°ΠΊΠΎΠΉ элСктричСской Ρ†Π΅ΠΏΠΈ ΠΌΠΎΠΆΠ½ΠΎ Ρ€Π°ΡΡΠΌΠΎΡ‚Ρ€Π΅Ρ‚ΡŒ Π½Π° ΠΏΡ€ΠΈΠΌΠ΅Ρ€Π΅ участков Ρ†Π΅ΠΏΠΈ с двумя Π½Π°Π³Ρ€ΡƒΠ·ΠΊΠ°ΠΌΠΈ. Π‘ΠΈΠ»Ρƒ Ρ‚ΠΎΠΊΠ°, напряТСниС ΠΈ сопротивлСниС Π½Π° ΠΊΠ°ΠΆΠ΄ΠΎΠΌ ΠΈΠ· Π½ΠΈΡ… слСдуСт ΠΎΠ±ΠΎΠ·Π½Π°Ρ‡ΠΈΡ‚ΡŒ соотвСтствСнно, ΠΊΠ°ΠΊ I1, U1, R1 ΠΈ I2, U2, R2. Π’ Ρ€Π΅Π·ΡƒΠ»ΡŒΡ‚Π°Ρ‚Π΅, ΠΏΠΎΠ»ΡƒΡ‡ΠΈΠ»ΠΈΡΡŒ ΡΠΎΠΎΡ‚Π½ΠΎΡˆΠ΅Π½ΠΈΡ, Π²Ρ‹Ρ€Π°ΠΆΠ°ΡŽΡ‰ΠΈΠ΅ Π·Π°Π²ΠΈΡΠΈΠΌΠΎΡΡ‚ΡŒ ΠΌΠ΅ΠΆΠ΄Ρƒ Π²Π΅Π»ΠΈΡ‡ΠΈΠ½Π°ΠΌΠΈ ΡΠ»Π΅Π΄ΡƒΡŽΡ‰ΠΈΠΌ ΠΎΠ±Ρ€Π°Π·ΠΎΠΌ: I = I1 = I2, U = U1 + U2, R = R1 + R2. ΠŸΠΎΠ»ΡƒΡ‡Π΅Π½Π½Ρ‹Π΅ Π΄Π°Π½Π½Ρ‹Π΅ ΠΏΠΎΠ΄Ρ‚Π²Π΅Ρ€ΠΆΠ΄Π°ΡŽΡ‚ΡΡ практичСским ΠΏΡƒΡ‚Π΅ΠΌ с ΠΏΠΎΠΌΠΎΡ‰ΡŒΡŽ провСдСния ΠΈΠ·ΠΌΠ΅Ρ€Π΅Π½ΠΈΠΉ Π°ΠΌΠΏΠ΅Ρ€ΠΌΠ΅Ρ‚Ρ€ΠΎΠΌ ΠΈ Π²ΠΎΠ»ΡŒΡ‚ΠΌΠ΅Ρ‚Ρ€ΠΎΠΌ ΡΠΎΠΎΡ‚Π²Π΅Ρ‚ΡΡ‚Π²ΡƒΡŽΡ‰ΠΈΡ… участков.

Π’Π°ΠΊΠΈΠΌ ΠΎΠ±Ρ€Π°Π·ΠΎΠΌ, ΠΏΠΎΡΠ»Π΅Π΄ΠΎΠ²Π°Ρ‚Π΅Π»ΡŒΠ½ΠΎΠ΅ соСдинСниС ΠΏΡ€ΠΎΠ²ΠΎΠ΄Π½ΠΈΠΊΠΎΠ² отличаСтся ΡΠ»Π΅Π΄ΡƒΡŽΡ‰ΠΈΠΌΠΈ ΠΈΠ½Π΄ΠΈΠ²ΠΈΠ΄ΡƒΠ°Π»ΡŒΠ½Ρ‹ΠΌΠΈ особСнностями:

  • Π‘ΠΈΠ»Π° Ρ‚ΠΎΠΊΠ° Π½Π° всСх участках Ρ†Π΅ΠΏΠΈ Π±ΡƒΠ΄Π΅Ρ‚ ΠΎΠ΄ΠΈΠ½Π°ΠΊΠΎΠ²ΠΎΠΉ.
  • ΠžΠ±Ρ‰Π΅Π΅ напряТСниС Ρ†Π΅ΠΏΠΈ составляСт сумму напряТСний Π½Π° ΠΊΠ°ΠΆΠ΄ΠΎΠΌ участкС.
  • ΠžΠ±Ρ‰Π΅Π΅ сопротивлСниС Π²ΠΊΠ»ΡŽΡ‡Π°Π΅Ρ‚ Π² сСбя сопротивлСния ΠΊΠ°ΠΆΠ΄ΠΎΠ³ΠΎ ΠΎΡ‚Π΄Π΅Π»ΡŒΠ½ΠΎΠ³ΠΎ ΠΏΡ€ΠΎΠ²ΠΎΠ΄Π½ΠΈΠΊΠ°.

Π”Π°Π½Π½Ρ‹Π΅ ΡΠΎΠΎΡ‚Π½ΠΎΡˆΠ΅Π½ΠΈΡ подходят для любого количСства ΠΏΡ€ΠΎΠ²ΠΎΠ΄Π½ΠΈΠΊΠΎΠ², соСдинСнных ΠΏΠΎΡΠ»Π΅Π΄ΠΎΠ²Π°Ρ‚Π΅Π»ΡŒΠ½ΠΎ. Π—Π½Π°Ρ‡Π΅Π½ΠΈΠ΅ ΠΎΠ±Ρ‰Π΅Π³ΠΎ сопротивлСния всСгда Π²Ρ‹ΡˆΠ΅, Ρ‡Π΅ΠΌ сопротивлСниС любого ΠΎΡ‚Π΄Π΅Π»ΡŒΠ½ΠΎ взятого ΠΏΡ€ΠΎΠ²ΠΎΠ΄Π½ΠΈΠΊΠ°. Π­Ρ‚ΠΎ связано с ΡƒΠ²Π΅Π»ΠΈΡ‡Π΅Π½ΠΈΠ΅ΠΌ ΠΈΡ… ΠΎΠ±Ρ‰Π΅ΠΉ Π΄Π»ΠΈΠ½Ρ‹ ΠΏΡ€ΠΈ ΠΏΠΎΡΠ»Π΅Π΄ΠΎΠ²Π°Ρ‚Π΅Π»ΡŒΠ½ΠΎΠΌ соСдинСнии, Ρ‡Ρ‚ΠΎ ΠΏΡ€ΠΈΠ²ΠΎΠ΄ΠΈΡ‚ ΠΈ ΠΊ росту сопротивлСния.

Если ΡΠΎΠ΅Π΄ΠΈΠ½ΠΈΡ‚ΡŒ ΠΏΠΎΡΠ»Π΅Π΄ΠΎΠ²Π°Ρ‚Π΅Π»ΡŒΠ½ΠΎ ΠΎΠ΄ΠΈΠ½Π°ΠΊΠΎΠ²Ρ‹Π΅ элСмСнты Π² количСствС n, Ρ‚ΠΎ получится R = n Ρ… R1, Π³Π΄Π΅ R — ΠΎΠ±Ρ‰Π΅Π΅ сопротивлСниС, R1 — сопротивлСниС ΠΎΠ΄Π½ΠΎΠ³ΠΎ элСмСнта, Π° n — количСство элСмСнтов. НапряТСниС U, Π½Π°ΠΎΠ±ΠΎΡ€ΠΎΡ‚, дСлится Π½Π° Ρ€Π°Π²Π½Ρ‹Π΅ части, каТдая ΠΈΠ· ΠΊΠΎΡ‚ΠΎΡ€Ρ‹Ρ… Π² n Ρ€Π°Π· мСньшС ΠΎΠ±Ρ‰Π΅Π³ΠΎ значСния. НапримСр, Ссли Π² ΡΠ΅Ρ‚ΡŒ с напряТСниСм 220 Π²ΠΎΠ»ΡŒΡ‚ ΠΏΠΎΡΠ»Π΅Π΄ΠΎΠ²Π°Ρ‚Π΅Π»ΡŒΠ½ΠΎ Π²ΠΊΠ»ΡŽΡ‡Π°ΡŽΡ‚ΡΡ 10 Π»Π°ΠΌΠΏ ΠΎΠ΄ΠΈΠ½Π°ΠΊΠΎΠ²ΠΎΠΉ мощности, Ρ‚ΠΎ напряТСниС Π² любой ΠΈΠ· Π½ΠΈΡ… составит: U1 = U/10 = 22 Π²ΠΎΠ»ΡŒΡ‚Π°.

ΠŸΡ€ΠΎΠ²ΠΎΠ΄Π½ΠΈΠΊΠΈ, соСдинСнныС ΠΏΠΎΡΠ»Π΅Π΄ΠΎΠ²Π°Ρ‚Π΅Π»ΡŒΠ½ΠΎ, ΠΈΠΌΠ΅ΡŽΡ‚ Ρ…Π°Ρ€Π°ΠΊΡ‚Π΅Ρ€Π½ΡƒΡŽ ΠΎΡ‚Π»ΠΈΡ‡ΠΈΡ‚Π΅Π»ΡŒΠ½ΡƒΡŽ ΠΎΡΠΎΠ±Π΅Π½Π½ΠΎΡΡ‚ΡŒ. Если Π²ΠΎ врСмя Ρ€Π°Π±ΠΎΡ‚Ρ‹ ΠΎΡ‚ΠΊΠ°Π·Π°Π» хотя-Π±Ρ‹ ΠΎΠ΄ΠΈΠ½ ΠΈΠ· Π½ΠΈΡ…, Ρ‚ΠΎ Ρ‚Π΅Ρ‡Π΅Π½ΠΈΠ΅ Ρ‚ΠΎΠΊΠ° прСкращаСтся Π²ΠΎ всСй Ρ†Π΅ΠΏΠΈ. НаиболСС ярким ΠΏΡ€ΠΈΠΌΠ΅Ρ€ΠΎΠΌ являСтся , ΠΊΠΎΠ³Π΄Π° ΠΎΠ΄Π½Π° ΠΏΠ΅Ρ€Π΅Π³ΠΎΡ€Π΅Π²ΡˆΠ°Ρ Π»Π°ΠΌΠΏΠΎΡ‡ΠΊΠ° Π² ΠΏΠΎΡΠ»Π΅Π΄ΠΎΠ²Π°Ρ‚Π΅Π»ΡŒΠ½ΠΎΠΉ Ρ†Π΅ΠΏΠΈ, ΠΏΡ€ΠΈΠ²ΠΎΠ΄ΠΈΡ‚ ΠΊ Π²Ρ‹Ρ…ΠΎΠ΄Ρƒ ΠΈΠ· строя всСй систСмы. Для установлСния ΠΏΠ΅Ρ€Π΅Π³ΠΎΡ€Π΅Π²ΡˆΠ΅ΠΉ Π»Π°ΠΌΠΏΠΎΡ‡ΠΊΠΈ понадобится ΠΏΡ€ΠΎΠ²Π΅Ρ€ΠΊΠ° всСй гирлянды.

ΠŸΠ°Ρ€Π°Π»Π»Π΅Π»ΡŒΠ½ΠΎΠ΅ соСдинСниС ΠΏΡ€ΠΎΠ²ΠΎΠ΄Π½ΠΈΠΊΠΎΠ²

Π’ элСктричСских сСтях ΠΏΡ€ΠΎΠ²ΠΎΠ΄Π½ΠΈΠΊΠΈ ΠΌΠΎΠ³ΡƒΡ‚ ΡΠΎΠ΅Π΄ΠΈΠ½ΡΡ‚ΡŒΡΡ Ρ€Π°Π·Π»ΠΈΡ‡Π½Ρ‹ΠΌΠΈ способами: ΠΏΠΎΡΠ»Π΅Π΄ΠΎΠ²Π°Ρ‚Π΅Π»ΡŒΠ½ΠΎ, ΠΏΠ°Ρ€Π°Π»Π»Π΅Π»ΡŒΠ½ΠΎ ΠΈ ΠΊΠΎΠΌΠ±ΠΈΠ½ΠΈΡ€ΠΎΠ²Π°Π½Π½ΠΎ. Π‘Ρ€Π΅Π΄ΠΈ Π½ΠΈΡ… ΠΏΠ°Ρ€Π°Π»Π»Π΅Π»ΡŒΠ½ΠΎΠ΅ соСдинСниС это Ρ‚Π°ΠΊΠΎΠΉ Π²Π°Ρ€ΠΈΠ°Π½Ρ‚, ΠΊΠΎΠ³Π΄Π° ΠΏΡ€ΠΎΠ²ΠΎΠ΄Π½ΠΈΠΊΠΈ Π² Π½Π°Ρ‡Π°Π»ΡŒΠ½Ρ‹Ρ… ΠΈ ΠΊΠΎΠ½Π΅Ρ‡Π½Ρ‹Ρ… Ρ‚ΠΎΡ‡ΠΊΠ°Ρ… ΡΠΎΠ΅Π΄ΠΈΠ½ΡΡŽΡ‚ΡΡ ΠΌΠ΅ΠΆΠ΄Ρƒ собой. Π’Π°ΠΊΠΈΠΌ ΠΎΠ±Ρ€Π°Π·ΠΎΠΌ, Π½Π°Ρ‡Π°Π»Π° ΠΈ ΠΊΠΎΠ½Ρ†Ρ‹ Π½Π°Π³Ρ€ΡƒΠ·ΠΎΠΊ ΡΠΎΠ΅Π΄ΠΈΠ½ΡΡŽΡ‚ΡΡ вмСстС, Π° сами Π½Π°Π³Ρ€ΡƒΠ·ΠΊΠΈ Ρ€Π°ΡΠΏΠΎΠ»Π°Π³Π°ΡŽΡ‚ΡΡ ΠΏΠ°Ρ€Π°Π»Π»Π΅Π»ΡŒΠ½ΠΎ ΠΎΡ‚Π½ΠΎΡΠΈΡ‚Π΅Π»ΡŒΠ½ΠΎ Π΄Ρ€ΡƒΠ³ Π΄Ρ€ΡƒΠ³Π°. Π’ элСктричСской Ρ†Π΅ΠΏΠΈ ΠΌΠΎΠ³ΡƒΡ‚ ΡΠΎΠ΄Π΅Ρ€ΠΆΠ°Ρ‚ΡŒΡΡ Π΄Π²Π°, Ρ‚Ρ€ΠΈ ΠΈ Π±ΠΎΠ»Π΅Π΅ ΠΏΡ€ΠΎΠ²ΠΎΠ΄Π½ΠΈΠΊΠΎΠ², соСдинСнных ΠΏΠ°Ρ€Π°Π»Π»Π΅Π»ΡŒΠ½ΠΎ.

Если Ρ€Π°ΡΡΠΌΠ°Ρ‚Ρ€ΠΈΠ²Π°Ρ‚ΡŒ ΠΏΠΎΡΠ»Π΅Π΄ΠΎΠ²Π°Ρ‚Π΅Π»ΡŒΠ½ΠΎΠ΅ ΠΈ ΠΏΠ°Ρ€Π°Π»Π»Π΅Π»ΡŒΠ½ΠΎΠ΅ соСдинСниС, сила Ρ‚ΠΎΠΊΠ° Π² послСднСм Π²Π°Ρ€ΠΈΠ°Π½Ρ‚Π΅ ΠΌΠΎΠΆΠ΅Ρ‚ Π±Ρ‹Ρ‚ΡŒ исслСдована с ΠΏΠΎΠΌΠΎΡ‰ΡŒΡŽ ΡΠ»Π΅Π΄ΡƒΡŽΡ‰Π΅ΠΉ схСмы. БСрутся Π΄Π²Π΅ Π»Π°ΠΌΠΏΡ‹ накаливания, ΠΎΠ±Π»Π°Π΄Π°ΡŽΡ‰ΠΈΠ΅ ΠΎΠ΄ΠΈΠ½Π°ΠΊΠΎΠ²Ρ‹ΠΌ сопротивлСниСм ΠΈ соСдинСнныС ΠΏΠ°Ρ€Π°Π»Π»Π΅Π»ΡŒΠ½ΠΎ. Для контроля ΠΊ ΠΊΠ°ΠΆΠ΄ΠΎΠΉ Π»Π°ΠΌΠΏΠΎΡ‡ΠΊΠ΅ ΠΏΠΎΠ΄ΠΊΠ»ΡŽΡ‡Π°Π΅Ρ‚ΡΡ собствСнный . ΠšΡ€ΠΎΠΌΠ΅ Ρ‚ΠΎΠ³ΠΎ, ΠΈΡΠΏΠΎΠ»ΡŒΠ·ΡƒΠ΅Ρ‚ΡΡ Π΅Ρ‰Π΅ ΠΎΠ΄ΠΈΠ½ Π°ΠΌΠΏΠ΅Ρ€ΠΌΠ΅Ρ‚Ρ€, ΠΊΠΎΠ½Ρ‚Ρ€ΠΎΠ»ΠΈΡ€ΡƒΡŽΡ‰ΠΈΠΉ ΠΎΠ±Ρ‰ΡƒΡŽ силу Ρ‚ΠΎΠΊΠ° Π² Ρ†Π΅ΠΏΠΈ. ΠŸΡ€ΠΎΠ²Π΅Ρ€ΠΎΡ‡Π½Π°Ρ схСма дополняСтся источником питания ΠΈ ΠΊΠ»ΡŽΡ‡ΠΎΠΌ.

ПослС замыкания ΠΊΠ»ΡŽΡ‡Π° Π½ΡƒΠΆΠ½ΠΎ ΠΊΠΎΠ½Ρ‚Ρ€ΠΎΠ»ΠΈΡ€ΠΎΠ²Π°Ρ‚ΡŒ показания ΠΈΠ·ΠΌΠ΅Ρ€ΠΈΡ‚Π΅Π»ΡŒΠ½Ρ‹Ρ… ΠΏΡ€ΠΈΠ±ΠΎΡ€ΠΎΠ². АмпСрмСтр Π½Π° Π»Π°ΠΌΠΏΠ΅ β„– 1 ΠΏΠΎΠΊΠ°ΠΆΠ΅Ρ‚ силу Ρ‚ΠΎΠΊΠ° I1, Π° Π½Π° Π»Π°ΠΌΠΏΠ΅ β„– 2 — силу Ρ‚ΠΎΠΊΠ° I2. ΠžΠ±Ρ‰ΠΈΠΉ Π°ΠΌΠΏΠ΅Ρ€ΠΌΠ΅Ρ‚Ρ€ ΠΏΠΎΠΊΠ°Π·Ρ‹Π²Π°Π΅Ρ‚ Π·Π½Π°Ρ‡Π΅Π½ΠΈΠ΅ силы Ρ‚ΠΎΠΊΠ°, Ρ€Π°Π²Π½ΠΎΠ΅ суммС Ρ‚ΠΎΠΊΠΎΠ² ΠΎΡ‚Π΄Π΅Π»ΡŒΠ½ΠΎ взятых, ΠΏΠ°Ρ€Π°Π»Π»Π΅Π»ΡŒΠ½ΠΎ соСдинСнных Ρ†Π΅ΠΏΠ΅ΠΉ: I = I1 + I2. Π’ ΠΎΡ‚Π»ΠΈΡ‡ΠΈΠ΅ ΠΎΡ‚ ΠΏΠΎΡΠ»Π΅Π΄ΠΎΠ²Π°Ρ‚Π΅Π»ΡŒΠ½ΠΎΠ³ΠΎ соСдинСния, ΠΏΡ€ΠΈ ΠΏΠ΅Ρ€Π΅Π³ΠΎΡ€Π°Π½ΠΈΠΈ ΠΎΠ΄Π½ΠΎΠΉ ΠΈΠ· Π»Π°ΠΌΠΏΠΎΡ‡Π΅ΠΊ, другая Π±ΡƒΠ΄Π΅Ρ‚ Π½ΠΎΡ€ΠΌΠ°Π»ΡŒΠ½ΠΎ Ρ„ΡƒΠ½ΠΊΡ†ΠΈΠΎΠ½ΠΈΡ€ΠΎΠ²Π°Ρ‚ΡŒ. ΠŸΠΎΡΡ‚ΠΎΠΌΡƒ Π² Π΄ΠΎΠΌΠ°ΡˆΠ½ΠΈΡ… элСктричСских сСтях ΠΈΡΠΏΠΎΠ»ΡŒΠ·ΡƒΠ΅Ρ‚ΡΡ ΠΏΠ°Ρ€Π°Π»Π»Π΅Π»ΡŒΠ½ΠΎΠ΅ ΠΏΠΎΠ΄ΠΊΠ»ΡŽΡ‡Π΅Π½ΠΈΠ΅ ΠΏΡ€ΠΈΠ±ΠΎΡ€ΠΎΠ².

Π‘ ΠΏΠΎΠΌΠΎΡ‰ΡŒΡŽ Ρ‚Π°ΠΊΠΎΠΉ ΠΆΠ΅ схСмы ΠΌΠΎΠΆΠ½ΠΎ ΡƒΡΡ‚Π°Π½ΠΎΠ²ΠΈΡ‚ΡŒ Π·Π½Π°Ρ‡Π΅Π½ΠΈΠ΅ эквивалСнтного сопротивлСния. Π‘ этой Ρ†Π΅Π»ΡŒΡŽ Π² ΡΠ»Π΅ΠΊΡ‚Ρ€ΠΈΡ‡Π΅ΡΠΊΡƒΡŽ Ρ†Π΅ΠΏΡŒ добавляСтся Π²ΠΎΠ»ΡŒΡ‚ΠΌΠ΅Ρ‚Ρ€. Π­Ρ‚ΠΎ позволяСт ΠΈΠ·ΠΌΠ΅Ρ€ΠΈΡ‚ΡŒ напряТСниС ΠΏΡ€ΠΈ ΠΏΠ°Ρ€Π°Π»Π»Π΅Π»ΡŒΠ½ΠΎΠΌ соСдинСнии, сила Ρ‚ΠΎΠΊΠ° ΠΏΡ€ΠΈ этом остаСтся Ρ‚Π°ΠΊΠΎΠΉ ΠΆΠ΅. Π—Π΄Π΅ΡΡŒ Ρ‚Π°ΠΊΠΆΠ΅ ΠΈΠΌΠ΅ΡŽΡ‚ΡΡ Ρ‚ΠΎΡ‡ΠΊΠΈ пСрСсСчСния ΠΏΡ€ΠΎΠ²ΠΎΠ΄Π½ΠΈΠΊΠΎΠ², ΡΠΎΠ΅Π΄ΠΈΠ½ΡΡŽΡ‰ΠΈΡ… ΠΎΠ±Π΅ Π»Π°ΠΌΠΏΡ‹.

Π’ Ρ€Π΅Π·ΡƒΠ»ΡŒΡ‚Π°Ρ‚Π΅ ΠΈΠ·ΠΌΠ΅Ρ€Π΅Π½ΠΈΠΉ ΠΎΠ±Ρ‰Π΅Π΅ напряТСниС ΠΏΡ€ΠΈ ΠΏΠ°Ρ€Π°Π»Π»Π΅Π»ΡŒΠ½ΠΎΠΌ соСдинСнии составит: U = U1 = U2. ПослС этого ΠΌΠΎΠΆΠ½ΠΎ Ρ€Π°ΡΡΡ‡ΠΈΡ‚Π°Ρ‚ΡŒ эквивалСнтноС сопротивлСниС, условно Π·Π°ΠΌΠ΅Π½ΡΡŽΡ‰Π΅Π΅ всС элСмСнты, находящиСся Π² Π΄Π°Π½Π½ΠΎΠΉ Ρ†Π΅ΠΏΠΈ. ΠŸΡ€ΠΈ ΠΏΠ°Ρ€Π°Π»Π»Π΅Π»ΡŒΠ½ΠΎΠΌ соСдинСнии, Π² соотвСтствии с Π·Π°ΠΊΠΎΠ½ΠΎΠΌ Ома I = U/R, получаСтся ΡΠ»Π΅Π΄ΡƒΡŽΡ‰Π°Ρ Ρ„ΠΎΡ€ΠΌΡƒΠ»Π°: U/R = U1/R1 + U2/R2, Π² ΠΊΠΎΡ‚ΠΎΡ€ΠΎΠΉ R являСтся эквивалСнтным сопротивлСниСм, R1 ΠΈ R2 — сопротивлСния ΠΎΠ±Π΅ΠΈΡ… Π»Π°ΠΌΠΏΠΎΡ‡Π΅ΠΊ, U = U1 = U2 — Π·Π½Π°Ρ‡Π΅Π½ΠΈΠ΅ напряТСния, ΠΏΠΎΠΊΠ°Π·Ρ‹Π²Π°Π΅ΠΌΠΎΠ΅ Π²ΠΎΠ»ΡŒΡ‚ΠΌΠ΅Ρ‚Ρ€ΠΎΠΌ.

Π‘Π»Π΅Π΄ΡƒΠ΅Ρ‚ ΡƒΡ‡ΠΈΡ‚Ρ‹Π²Π°Ρ‚ΡŒ ΠΈ Ρ‚ΠΎΡ‚ Ρ„Π°ΠΊΡ‚ΠΎΡ€, Ρ‡Ρ‚ΠΎ Ρ‚ΠΎΠΊΠΈ Π² ΠΊΠ°ΠΆΠ΄ΠΎΠΉ Ρ†Π΅ΠΏΠΈ, Π² суммС ΡΠΎΡΡ‚Π°Π²Π»ΡΡŽΡ‚ ΠΎΠ±Ρ‰ΡƒΡŽ силу Ρ‚ΠΎΠΊΠ° всСй Ρ†Π΅ΠΏΠΈ. Π’ ΠΎΠΊΠΎΠ½Ρ‡Π°Ρ‚Π΅Π»ΡŒΠ½ΠΎΠΌ Π²ΠΈΠ΄Π΅ Ρ„ΠΎΡ€ΠΌΡƒΠ»Π°, ΠΎΡ‚Ρ€Π°ΠΆΠ°ΡŽΡ‰Π°Ρ эквивалСнтноС сопротивлСниС Π±ΡƒΠ΄Π΅Ρ‚ Π²Ρ‹Π³Π»ΡΠ΄Π΅Ρ‚ΡŒ ΡΠ»Π΅Π΄ΡƒΡŽΡ‰ΠΈΠΌ ΠΎΠ±Ρ€Π°Π·ΠΎΠΌ: 1/R = 1/R1 + 1/R2. ΠŸΡ€ΠΈ ΡƒΠ²Π΅Π»ΠΈΡ‡Π΅Π½ΠΈΠΈ количСства элСмСнтов Π² Ρ‚Π°ΠΊΠΈΡ… цСпях — увСличиваСтся ΠΈ число слагаСмых Π² Ρ„ΠΎΡ€ΠΌΡƒΠ»Π΅. Π Π°Π·Π»ΠΈΡ‡ΠΈΠ΅ Π² основных ΠΏΠ°Ρ€Π°ΠΌΠ΅Ρ‚Ρ€Π°Ρ… ΠΎΡ‚Π»ΠΈΡ‡Π°ΡŽΡ‚ Π΄Ρ€ΡƒΠ³ ΠΎΡ‚ Π΄Ρ€ΡƒΠ³Π° ΠΈ источников Ρ‚ΠΎΠΊΠ°, позволяя ΠΈΡΠΏΠΎΠ»ΡŒΠ·ΠΎΠ²Π°Ρ‚ΡŒ ΠΈΡ… Π² Ρ€Π°Π·Π»ΠΈΡ‡Π½Ρ‹Ρ… элСктричСских схСмах.

ΠŸΠ°Ρ€Π°Π»Π»Π΅Π»ΡŒΠ½ΠΎΠ΅ соСдинСниС ΠΏΡ€ΠΎΠ²ΠΎΠ΄Π½ΠΈΠΊΠΎΠ² характСризуСтся достаточно ΠΌΠ°Π»Ρ‹ΠΌ Π·Π½Π°Ρ‡Π΅Π½ΠΈΠ΅ΠΌ эквивалСнтного сопротивлСния, поэтому сила Ρ‚ΠΎΠΊΠ° Π±ΡƒΠ΄Π΅Ρ‚ ΡΡ€Π°Π²Π½ΠΈΡ‚Π΅Π»ΡŒΠ½ΠΎ высокой. Π”Π°Π½Π½Ρ‹ΠΉ Ρ„Π°ΠΊΡ‚ΠΎΡ€ слСдуСт ΡƒΡ‡ΠΈΡ‚Ρ‹Π²Π°Ρ‚ΡŒ, ΠΊΠΎΠ³Π΄Π° Π² Ρ€ΠΎΠ·Π΅Ρ‚ΠΊΠΈ Π²ΠΊΠ»ΡŽΡ‡Π°Π΅Ρ‚ΡΡ большоС количСство элСктроприборов. Π’ этом случаС сила Ρ‚ΠΎΠΊΠ° Π·Π½Π°Ρ‡ΠΈΡ‚Π΅Π»ΡŒΠ½ΠΎ возрастаСт, приводя ΠΊ ΠΏΠ΅Ρ€Π΅Π³Ρ€Π΅Π²Ρƒ ΠΊΠ°Π±Π΅Π»ΡŒΠ½Ρ‹Ρ… Π»ΠΈΠ½ΠΈΠΉ ΠΈ ΠΏΠΎΡΠ»Π΅Π΄ΡƒΡŽΡ‰ΠΈΠΌ возгораниям.

Π—Π°ΠΊΠΎΠ½Ρ‹ ΠΏΠΎΡΠ»Π΅Π΄ΠΎΠ²Π°Ρ‚Π΅Π»ΡŒΠ½ΠΎΠ³ΠΎ ΠΈ ΠΏΠ°Ρ€Π°Π»Π»Π΅Π»ΡŒΠ½ΠΎΠ³ΠΎ соСдинСния ΠΏΡ€ΠΎΠ²ΠΎΠ΄Π½ΠΈΠΊΠΎΠ²

Π”Π°Π½Π½Ρ‹Π΅ Π·Π°ΠΊΠΎΠ½Ρ‹, ΠΊΠ°ΡΠ°ΡŽΡ‰ΠΈΠ΅ΡΡ ΠΎΠ±ΠΎΠΈΡ… Π²ΠΈΠ΄ΠΎΠ² соСдинСний ΠΏΡ€ΠΎΠ²ΠΎΠ΄Π½ΠΈΠΊΠΎΠ², частично ΡƒΠΆΠ΅ Π±Ρ‹Π»ΠΈ рассмотрСны Ρ€Π°Π½Π΅Π΅.

Для Π±ΠΎΠ»Π΅Π΅ Ρ‡Π΅Ρ‚ΠΊΠΎΠ³ΠΎ ΠΈΡ… понимания ΠΈ восприятия Π² практичСской плоскости, ΠΏΠΎΡΠ»Π΅Π΄ΠΎΠ²Π°Ρ‚Π΅Π»ΡŒΠ½ΠΎΠ΅ ΠΈ ΠΏΠ°Ρ€Π°Π»Π»Π΅Π»ΡŒΠ½ΠΎΠ΅ соСдинСниС ΠΏΡ€ΠΎΠ²ΠΎΠ΄Π½ΠΈΠΊΠΎΠ², Ρ„ΠΎΡ€ΠΌΡƒΠ»Ρ‹ слСдуСт Ρ€Π°ΡΡΠΌΠ°Ρ‚Ρ€ΠΈΠ²Π°Ρ‚ΡŒ Π² ΠΎΠΏΡ€Π΅Π΄Π΅Π»Π΅Π½Π½ΠΎΠΉ ΠΏΠΎΡΠ»Π΅Π΄ΠΎΠ²Π°Ρ‚Π΅Π»ΡŒΠ½ΠΎΡΡ‚ΠΈ:

  • ΠŸΠΎΡΠ»Π΅Π΄ΠΎΠ²Π°Ρ‚Π΅Π»ΡŒΠ½ΠΎΠ΅ соСдинСниС ΠΏΡ€Π΅Π΄ΠΏΠΎΠ»Π°Π³Π°Π΅Ρ‚ ΠΎΠ΄ΠΈΠ½Π°ΠΊΠΎΠ²ΡƒΡŽ силу Ρ‚ΠΎΠΊΠ° Π² ΠΊΠ°ΠΆΠ΄ΠΎΠΌ ΠΏΡ€ΠΎΠ²ΠΎΠ΄Π½ΠΈΠΊΠ΅: I = I1 = I2.
  • ΠΏΠ°Ρ€Π°Π»Π»Π΅Π»ΡŒΠ½ΠΎΠ΅ ΠΈ ΠΏΠΎΡΠ»Π΅Π΄ΠΎΠ²Π°Ρ‚Π΅Π»ΡŒΠ½ΠΎΠ΅ соСдинСниС ΠΏΡ€ΠΎΠ²ΠΎΠ΄Π½ΠΈΠΊΠΎΠ² ΠΎΠ±ΡŠΡΡΠ½ΡΠ΅Ρ‚ Π² ΠΊΠ°ΠΆΠ΄ΠΎΠΌ случаС ΠΏΠΎ-своСму. НапримСр, ΠΏΡ€ΠΈ ΠΏΠΎΡΠ»Π΅Π΄ΠΎΠ²Π°Ρ‚Π΅Π»ΡŒΠ½ΠΎΠΌ соСдинСнии, напряТСния Π½Π° всСх ΠΏΡ€ΠΎΠ²ΠΎΠ΄Π½ΠΈΠΊΠ°Ρ… Π±ΡƒΠ΄ΡƒΡ‚ Ρ€Π°Π²Π½Ρ‹ ΠΌΠ΅ΠΆΠ΄Ρƒ собой: U1 = IR1, U2 = IR2. ΠšΡ€ΠΎΠΌΠ΅ Ρ‚ΠΎΠ³ΠΎ, ΠΏΡ€ΠΈ ΠΏΠΎΡΠ»Π΅Π΄ΠΎΠ²Π°Ρ‚Π΅Π»ΡŒΠ½ΠΎΠΌ соСдинСнии напряТСниС составляСт сумму напряТСний ΠΊΠ°ΠΆΠ΄ΠΎΠ³ΠΎ ΠΏΡ€ΠΎΠ²ΠΎΠ΄Π½ΠΈΠΊΠ°: U = U1 + U2 = I(R1 + R2) = IR.
  • ПолноС сопротивлСниС Ρ†Π΅ΠΏΠΈ ΠΏΡ€ΠΈ ΠΏΠΎΡΠ»Π΅Π΄ΠΎΠ²Π°Ρ‚Π΅Π»ΡŒΠ½ΠΎΠΌ соСдинСнии состоит ΠΈΠ· суммы сопротивлСний всСх ΠΎΡ‚Π΄Π΅Π»ΡŒΠ½ΠΎ взятых ΠΏΡ€ΠΎΠ²ΠΎΠ΄Π½ΠΈΠΊΠΎΠ², нСзависимо ΠΎΡ‚ ΠΈΡ… количСства.
  • ΠŸΡ€ΠΈ ΠΏΠ°Ρ€Π°Π»Π»Π΅Π»ΡŒΠ½ΠΎΠΌ соСдинСнии напряТСниС всСй Ρ†Π΅ΠΏΠΈ Ρ€Π°Π²Π½ΠΎ Π½Π°ΠΏΡ€ΡΠΆΠ΅Π½ΠΈΡŽ Π½Π° ΠΊΠ°ΠΆΠ΄ΠΎΠΌ ΠΈΠ· ΠΏΡ€ΠΎΠ²ΠΎΠ΄Π½ΠΈΠΊΠΎΠ²: U1 = U2 = U.
  • ΠžΠ±Ρ‰Π°Ρ сила Ρ‚ΠΎΠΊΠ°, измСрСнная Π²ΠΎ всСй Ρ†Π΅ΠΏΠΈ, Ρ€Π°Π²Π½Π° суммС Ρ‚ΠΎΠΊΠΎΠ², ΠΏΡ€ΠΎΡ‚Π΅ΠΊΠ°ΡŽΡ‰ΠΈΡ… ΠΏΠΎ всСм ΠΏΡ€ΠΎΠ²ΠΎΠ΄Π½ΠΈΠΊΠ°ΠΌ, соСдинСнных ΠΏΠ°Ρ€Π°Π»Π»Π΅Π»ΡŒΠ½ΠΎ ΠΌΠ΅ΠΆΠ΄Ρƒ собой: I = I1 + I2.

Для Ρ‚ΠΎΠ³ΠΎ Ρ‡Ρ‚ΠΎΠ±Ρ‹ Π±ΠΎΠ»Π΅Π΅ эффСктивно ΠΏΡ€ΠΎΠ΅ΠΊΡ‚ΠΈΡ€ΠΎΠ²Π°Ρ‚ΡŒ элСктричСскиС сСти, Π½ΡƒΠΆΠ½ΠΎ Ρ…ΠΎΡ€ΠΎΡˆΠΎ Π·Π½Π°Ρ‚ΡŒ ΠΏΠΎΡΠ»Π΅Π΄ΠΎΠ²Π°Ρ‚Π΅Π»ΡŒΠ½ΠΎΠ΅ ΠΈ ΠΏΠ°Ρ€Π°Π»Π»Π΅Π»ΡŒΠ½ΠΎΠ΅ соСдинСниС ΠΏΡ€ΠΎΠ²ΠΎΠ΄Π½ΠΈΠΊΠΎΠ² ΠΈ Π΅Π³ΠΎ Π·Π°ΠΊΠΎΠ½Ρ‹, находя ΠΈΠΌ Π½Π°ΠΈΠ±ΠΎΠ»Π΅Π΅ Ρ€Π°Ρ†ΠΈΠΎΠ½Π°Π»ΡŒΠ½ΠΎΠ΅ практичСскоС ΠΏΡ€ΠΈΠΌΠ΅Π½Π΅Π½ΠΈΠ΅.

БмСшанноС соСдинСниС ΠΏΡ€ΠΎΠ²ΠΎΠ΄Π½ΠΈΠΊΠΎΠ²

Π’ элСктричСских сСтях ΠΊΠ°ΠΊ ΠΏΡ€Π°Π²ΠΈΠ»ΠΎ ΠΈΡΠΏΠΎΠ»ΡŒΠ·ΡƒΠ΅Ρ‚ΡΡ ΠΏΠΎΡΠ»Π΅Π΄ΠΎΠ²Π°Ρ‚Π΅Π»ΡŒΠ½ΠΎΠ΅ ΠΏΠ°Ρ€Π°Π»Π»Π΅Π»ΡŒΠ½ΠΎΠ΅ ΠΈ смСшанноС соСдинСниС ΠΏΡ€ΠΎΠ²ΠΎΠ΄Π½ΠΈΠΊΠΎΠ², ΠΏΡ€Π΅Π΄Π½Π°Π·Π½Π°Ρ‡Π΅Π½Π½ΠΎΠ΅ для ΠΊΠΎΠ½ΠΊΡ€Π΅Ρ‚Π½Ρ‹Ρ… условий эксплуатации. Однако Ρ‡Π°Ρ‰Π΅ всСго ΠΏΡ€Π΅Π΄ΠΏΠΎΡ‡Ρ‚Π΅Π½ΠΈΠ΅ отдаСтся Ρ‚Ρ€Π΅Ρ‚ΡŒΠ΅ΠΌΡƒ Π²Π°Ρ€ΠΈΠ°Π½Ρ‚Ρƒ, ΠΏΡ€Π΅Π΄ΡΡ‚Π°Π²Π»ΡΡŽΡ‰Π΅ΠΌΡƒ собой ΡΠΎΠ²ΠΎΠΊΡƒΠΏΠ½ΠΎΡΡ‚ΡŒ ΠΊΠΎΠΌΠ±ΠΈΠ½Π°Ρ†ΠΈΠΉ, состоящих ΠΈΠ· Ρ€Π°Π·Π»ΠΈΡ‡Π½Ρ‹Ρ… Ρ‚ΠΈΠΏΠΎΠ² соСдинСний.

Π’ Ρ‚Π°ΠΊΠΈΡ… ΡΠΌΠ΅ΡˆΠ°Π½Π½Ρ‹Ρ… схСмах Π°ΠΊΡ‚ΠΈΠ²Π½ΠΎ примСняСтся ΠΏΠΎΡΠ»Π΅Π΄ΠΎΠ²Π°Ρ‚Π΅Π»ΡŒΠ½ΠΎΠ΅ ΠΈ ΠΏΠ°Ρ€Π°Π»Π»Π΅Π»ΡŒΠ½ΠΎΠ΅ соСдинСниС ΠΏΡ€ΠΎΠ²ΠΎΠ΄Π½ΠΈΠΊΠΎΠ², ΠΏΠ»ΡŽΡΡ‹ ΠΈ минусы ΠΊΠΎΡ‚ΠΎΡ€Ρ‹Ρ… ΠΎΠ±ΡΠ·Π°Ρ‚Π΅Π»ΡŒΠ½ΠΎ ΡƒΡ‡ΠΈΡ‚Ρ‹Π²Π°ΡŽΡ‚ΡΡ ΠΏΡ€ΠΈ ΠΏΡ€ΠΎΠ΅ΠΊΡ‚ΠΈΡ€ΠΎΠ²Π°Π½ΠΈΠΈ элСктричСских сСтСй. Π­Ρ‚ΠΈ соСдинСния состоят Π½Π΅ Ρ‚ΠΎΠ»ΡŒΠΊΠΎ ΠΈΠ· ΠΎΡ‚Π΄Π΅Π»ΡŒΠ½ΠΎ взятых рСзисторов, Π½ΠΎ ΠΈ довольно слоТных участков, Π²ΠΊΠ»ΡŽΡ‡Π°ΡŽΡ‰ΠΈΡ… Π² сСбя мноТСство элСмСнтов.

БмСшанноС соСдинСниС рассчитываСтся Π² соотвСтствии с извСстными свойствами ΠΏΠΎΡΠ»Π΅Π΄ΠΎΠ²Π°Ρ‚Π΅Π»ΡŒΠ½ΠΎΠ³ΠΎ ΠΈ ΠΏΠ°Ρ€Π°Π»Π»Π΅Π»ΡŒΠ½ΠΎΠ³ΠΎ соСдинСния. ΠœΠ΅Ρ‚ΠΎΠ΄ расчСта Π·Π°ΠΊΠ»ΡŽΡ‡Π°Π΅Ρ‚ΡΡ Π² Ρ€Π°Π·Π±ΠΈΠ²ΠΊΠ΅ схСмы Π½Π° Π±ΠΎΠ»Π΅Π΅ простыС составныС части, ΠΊΠΎΡ‚ΠΎΡ€Ρ‹Π΅ ΡΡ‡ΠΈΡ‚Π°ΡŽΡ‚ΡΡ ΠΎΡ‚Π΄Π΅Π»ΡŒΠ½ΠΎ, Π° ΠΏΠΎΡ‚ΠΎΠΌ ΡΡƒΠΌΠΌΠΈΡ€ΡƒΡŽΡ‚ΡΡ Π΄Ρ€ΡƒΠ³ с Π΄Ρ€ΡƒΠ³ΠΎΠΌ.

Π’ ΠΏΡ€Π΅Π΄Ρ‹Π΄ΡƒΡ‰Π΅ΠΌ конспСктС Π±Ρ‹Π» установлСно, Ρ‡Ρ‚ΠΎ сила Ρ‚ΠΎΠΊΠ° Π² ΠΏΡ€ΠΎΠ²ΠΎΠ΄Π½ΠΈΠΊΠ΅ зависит ΠΎΡ‚ напряТСния Π½Π° Π΅Π³ΠΎ ΠΊΠΎΠ½Ρ†Π°Ρ…. Если Π² ΠΎΠΏΡ‹Ρ‚Π΅ ΠΌΠ΅Π½ΡΡ‚ΡŒ ΠΏΡ€ΠΎΠ²ΠΎΠ΄Π½ΠΈΠΊΠΈ, оставляя напряТСниС Π½Π° Π½ΠΈΡ… Π½Π΅ΠΈΠ·ΠΌΠ΅Π½Π½Ρ‹ΠΌ, Ρ‚ΠΎ ΠΌΠΎΠΆΠ½ΠΎ ΠΏΠΎΠΊΠ°Π·Π°Ρ‚ΡŒ, Ρ‡Ρ‚ΠΎ ΠΏΡ€ΠΈ постоянном напряТСнии Π½Π° ΠΊΠΎΠ½Ρ†Π°Ρ… ΠΏΡ€ΠΎΠ²ΠΎΠ΄Π½ΠΈΠΊΠ° сила Ρ‚ΠΎΠΊΠ° ΠΎΠ±Ρ€Π°Ρ‚Π½ΠΎ ΠΏΡ€ΠΎΠΏΠΎΡ€Ρ†ΠΈΠΎΠ½Π°Π»ΡŒΠ½Π° Π΅Π³ΠΎ ΡΠΎΠΏΡ€ΠΎΡ‚ΠΈΠ²Π»Π΅Π½ΠΈΡŽ. ОбъСдинив Π·Π°Π²ΠΈΡΠΈΠΌΠΎΡΡ‚ΡŒ силы Ρ‚ΠΎΠΊΠ° ΠΎΡ‚ напряТСния ΠΈ Π΅Π³ΠΎ Π·Π°Π²ΠΈΡΠΈΠΌΠΎΡΡ‚ΡŒ ΠΎΡ‚ сопротивлСния ΠΏΡ€ΠΎΠ²ΠΎΠ΄Π½ΠΈΠΊΠ°, ΠΌΠΎΠΆΠ½ΠΎ Π·Π°ΠΏΠΈΡΠ°Ρ‚ΡŒ: I = U/R . Π­Ρ‚ΠΎΡ‚ Π·Π°ΠΊΠΎΠ½, установлСнный ΡΠΊΡΠΏΠ΅Ρ€ΠΈΠΌΠ΅Π½Ρ‚Π°Π»ΡŒΠ½ΠΎ, называСтся Π·Π°ΠΊΠΎΠ½ Ома (для участка Ρ†Π΅ΠΏΠΈ).

Π—Π°ΠΊΠΎΠ½ Ома для участка Ρ†Π΅ΠΏΠΈ : сила Ρ‚ΠΎΠΊΠ° Π² ΠΏΡ€ΠΎΠ²ΠΎΠ΄Π½ΠΈΠΊΠ΅ прямо ΠΏΡ€ΠΎΠΏΠΎΡ€Ρ†ΠΈΠΎΠ½Π°Π»ΡŒΠ½Π° ΠΏΡ€ΠΈΠ»ΠΎΠΆΠ΅Π½Π½ΠΎΠΌΡƒ ΠΊ Π΅Π³ΠΎ ΠΊΠΎΠ½Ρ†Π°ΠΌ Π½Π°ΠΏΡ€ΡΠΆΠ΅Π½ΠΈΡŽ ΠΈ ΠΎΠ±Ρ€Π°Ρ‚Π½ΠΎ ΠΏΡ€ΠΎΠΏΠΎΡ€Ρ†ΠΈΠΎΠ½Π°Π»ΡŒΠ½Π° ΡΠΎΠΏΡ€ΠΎΡ‚ΠΈΠ²Π»Π΅Π½ΠΈΡŽ ΠΏΡ€ΠΎΠ²ΠΎΠ΄Π½ΠΈΠΊΠ°. ΠŸΡ€Π΅ΠΆΠ΄Π΅ всСго Π·Π°ΠΊΠΎΠ½ всСгда Π²Π΅Ρ€Π΅Π½ для Ρ‚Π²Ρ‘Ρ€Π΄Ρ‹Ρ… ΠΈ ΠΆΠΈΠ΄ΠΊΠΈΡ… мСталличСских ΠΏΡ€ΠΎΠ²ΠΎΠ΄Π½ΠΈΠΊΠΎΠ². А Ρ‚Π°ΠΊΠΆΠ΅ для Π½Π΅ΠΊΠΎΡ‚ΠΎΡ€Ρ‹Ρ… Π΄Ρ€ΡƒΠ³ΠΈΡ… вСщСств (ΠΊΠ°ΠΊ ΠΏΡ€Π°Π²ΠΈΠ»ΠΎ, Ρ‚Π²Ρ‘Ρ€Π΄Ρ‹Ρ… ΠΈΠ»ΠΈ ΠΆΠΈΠ΄ΠΊΠΈΡ…).

ΠŸΠΎΡ‚Ρ€Π΅Π±ΠΈΡ‚Π΅Π»ΠΈ элСктричСской энСргии (Π»Π°ΠΌΠΏΠΎΡ‡ΠΊΠΈ, рСзисторы ΠΈ ΠΏΡ€.) ΠΌΠΎΠ³ΡƒΡ‚ ΠΏΠΎ-Ρ€Π°Π·Π½ΠΎΠΌΡƒ ΡΠΎΠ΅Π΄ΠΈΠ½ΡΡ‚ΡŒΡΡ Π΄Ρ€ΡƒΠ³ с Π΄Ρ€ΡƒΠ³ΠΎΠΌ Π² элСктричСской Ρ†Π΅ΠΏΠΈ. Π” Π²Π° основных Ρ‚ΠΈΠΏΠ° соСдинСния ΠΏΡ€ΠΎΠ²ΠΎΠ΄Π½ΠΈΠΊΠΎΠ² : ΠΏΠΎΡΠ»Π΅Π΄ΠΎΠ²Π°Ρ‚Π΅Π»ΡŒΠ½ΠΎΠ΅ ΠΈ ΠΏΠ°Ρ€Π°Π»Π»Π΅Π»ΡŒΠ½ΠΎΠ΅. А Ρ‚Π°ΠΊΠΆΠ΅ Π΅ΡΡ‚ΡŒ Π΅Ρ‰Π΅ Π΄Π²Π° соСдинСния, ΠΊΠΎΡ‚ΠΎΡ€Ρ‹Π΅ ΡΠ²Π»ΡΡŽΡ‚ΡΡ Ρ€Π΅Π΄ΠΊΠΈΠΌΠΈ: смСшанноС ΠΈ мостовоС.

ΠŸΠΎΡΠ»Π΅Π΄ΠΎΠ²Π°Ρ‚Π΅Π»ΡŒΠ½ΠΎΠ΅ соСдинСниС ΠΏΡ€ΠΎΠ²ΠΎΠ΄Π½ΠΈΠΊΠΎΠ²

ΠŸΡ€ΠΈ ΠΏΠΎΡΠ»Π΅Π΄ΠΎΠ²Π°Ρ‚Π΅Π»ΡŒΠ½ΠΎΠΌ соСдинСнии ΠΏΡ€ΠΎΠ²ΠΎΠ΄Π½ΠΈΠΊΠΎΠ² ΠΊΠΎΠ½Π΅Ρ† ΠΎΠ΄Π½ΠΎΠ³ΠΎ ΠΏΡ€ΠΎΠ²ΠΎΠ΄Π½ΠΈΠΊΠ° соСдинится с Π½Π°Ρ‡Π°Π»ΠΎΠΌ Π΄Ρ€ΡƒΠ³ΠΎΠ³ΠΎ ΠΏΡ€ΠΎΠ²ΠΎΠ΄Π½ΠΈΠΊΠ°, Π° Π΅Π³ΠΎ ΠΊΠΎΠ½Π΅Ρ† — с Π½Π°Ρ‡Π°Π»ΠΎΠΌ Ρ‚Ρ€Π΅Ρ‚ΡŒΠ΅Π³ΠΎ ΠΈ Ρ‚.Π΄. НапримСр, соСдинСниС элСктричСских Π»Π°ΠΌΠΏΠΎΡ‡Π΅ΠΊ Π² Ρ‘Π»ΠΎΡ‡Π½ΠΎΠΉ гирляндС. ΠŸΡ€ΠΈ ΠΏΠΎΡΠ»Π΅Π΄ΠΎΠ²Π°Ρ‚Π΅Π»ΡŒΠ½ΠΎΠΌ соСдинСнии ΠΏΡ€ΠΎΠ²ΠΎΠ΄Π½ΠΈΠΊΠΎΠ² Ρ‚ΠΎΠΊ ΠΏΡ€ΠΎΡ…ΠΎΠ΄ΠΈΡ‚ Ρ‡Π΅Ρ€Π΅Π· всС Π»Π°ΠΌΠΏΠΎΡ‡ΠΊΠΈ. ΠŸΡ€ΠΈ этом Ρ‡Π΅Ρ€Π΅Π· ΠΏΠΎΠΏΠ΅Ρ€Π΅Ρ‡Π½ΠΎΠ΅ сСчСниС ΠΊΠ°ΠΆΠ΄ΠΎΠ³ΠΎ ΠΏΡ€ΠΎΠ²ΠΎΠ΄Π½ΠΈΠΊΠ° Π² Π΅Π΄ΠΈΠ½ΠΈΡ†Ρƒ Π²Ρ€Π΅ΠΌΠ΅Π½ΠΈ ΠΏΡ€ΠΎΡ…ΠΎΠ΄ΠΈΡ‚ ΠΎΠ΄ΠΈΠ½Π°ΠΊΠΎΠ²Ρ‹ΠΉ заряд. Π’ΠΎ Π΅ΡΡ‚ΡŒ заряд Π½Π΅ скапливаСтся Π½ΠΈ Π² ΠΊΠ°ΠΊΠΎΠΉ части ΠΏΡ€ΠΎΠ²ΠΎΠ΄Π½ΠΈΠΊΠ°.

ΠŸΠΎΡΡ‚ΠΎΠΌΡƒ ΠΏΡ€ΠΈ ΠΏΠΎΡΠ»Π΅Π΄ΠΎΠ²Π°Ρ‚Π΅Π»ΡŒΠ½ΠΎΠΌ соСдинСнии ΠΏΡ€ΠΎΠ²ΠΎΠ΄Π½ΠΈΠΊΠΎΠ² сила Ρ‚ΠΎΠΊΠ° Π² любом участкС Ρ†Π΅ΠΏΠΈ ΠΎΠ΄ΠΈΠ½Π°ΠΊΠΎΠ²Π°: I 1 = I 2 = I .

ΠžΠ±Ρ‰Π΅Π΅ сопротивлСниС ΠΏΠΎΡΠ»Π΅Π΄ΠΎΠ²Π°Ρ‚Π΅Π»ΡŒΠ½ΠΎ соСдинённых ΠΏΡ€ΠΎΠ²ΠΎΠ΄Π½ΠΈΠΊΠΎΠ² Ρ€Π°Π²Π½ΠΎ суммС ΠΈΡ… сопротивлСний : R 1 + R 2 = R . ΠŸΠΎΡ‚ΠΎΠΌΡƒ Ρ‡Ρ‚ΠΎ ΠΏΡ€ΠΈ ΠΏΠΎΡΠ»Π΅Π΄ΠΎΠ²Π°Ρ‚Π΅Π»ΡŒΠ½ΠΎΠΌ соСдинСнии ΠΏΡ€ΠΎΠ²ΠΎΠ΄Π½ΠΈΠΊΠΎΠ² ΠΈΡ… общая Π΄Π»ΠΈΠ½Π° увСличиваСтся. Она большС, Ρ‡Π΅ΠΌ Π΄Π»ΠΈΠ½Π° ΠΊΠ°ΠΆΠ΄ΠΎΠ³ΠΎ ΠΎΡ‚Π΄Π΅Π»ΡŒΠ½ΠΎΠ³ΠΎ ΠΏΡ€ΠΎΠ²ΠΎΠ΄Π½ΠΈΠΊΠ°, соотвСтствСнно увСличиваСтся ΠΈ сопротивлСниС ΠΏΡ€ΠΎΠ²ΠΎΠ΄Π½ΠΈΠΊΠΎΠ².

По Π·Π°ΠΊΠΎΠ½Ρƒ Ома напряТСниС Π½Π° ΠΊΠ°ΠΆΠ΄ΠΎΠΌ ΠΏΡ€ΠΎΠ²ΠΎΠ΄Π½ΠΈΠΊΠ΅ Ρ€Π°Π²Π½ΠΎ: U 1 = I* R 1 , U 2 = I*R 2 . Π’ Ρ‚Π°ΠΊΠΎΠΌ случаС ΠΎΠ±Ρ‰Π΅Π΅ напряТСниС Ρ€Π°Π²Π½ΠΎ U = I ( R 1 + R 2) . ΠŸΠΎΡΠΊΠΎΠ»ΡŒΠΊΡƒ сила Ρ‚ΠΎΠΊΠ° Π²ΠΎ всСх ΠΏΡ€ΠΎΠ²ΠΎΠ΄Π½ΠΈΠΊΠ°Ρ… ΠΎΠ΄ΠΈΠ½Π°ΠΊΠΎΠ²Π°, Π° ΠΎΠ±Ρ‰Π΅Π΅ сопротивлСниС Ρ€Π°Π²Π½ΠΎ суммС сопротивлСний ΠΏΡ€ΠΎΠ²ΠΎΠ΄Π½ΠΈΠΊΠΎΠ², Ρ‚ΠΎ ΠΏΠΎΠ»Π½ΠΎΠ΅ напряТСниС Π½Π° ΠΏΠΎΡΠ»Π΅Π΄ΠΎΠ²Π°Ρ‚Π΅Π»ΡŒΠ½ΠΎ соСдинённых ΠΏΡ€ΠΎΠ²ΠΎΠ΄Π½ΠΈΠΊΠ°Ρ… Ρ€Π°Π²Π½ΠΎ суммС напряТСний Π½Π° ΠΊΠ°ΠΆΠ΄ΠΎΠΌ ΠΏΡ€ΠΎΠ²ΠΎΠ΄Π½ΠΈΠΊΠ΅ : U = U 1 + U 2 .

Из ΠΏΡ€ΠΈΠ²Π΅Π΄Ρ‘Π½Π½Ρ‹Ρ… равСнств слСдуСт, Ρ‡Ρ‚ΠΎ ΠΏΠΎΡΠ»Π΅Π΄ΠΎΠ²Π°Ρ‚Π΅Π»ΡŒΠ½ΠΎΠ΅ соСдинСниС ΠΏΡ€ΠΎΠ²ΠΎΠ΄Π½ΠΈΠΊΠΎΠ² ΠΈΡΠΏΠΎΠ»ΡŒΠ·ΡƒΠ΅Ρ‚ΡΡ Π² Ρ‚ΠΎΠΌ случаС, Ссли напряТСниС, Π½Π° ΠΊΠΎΡ‚ΠΎΡ€ΠΎΠ΅ рассчитаны ΠΏΠΎΡ‚Ρ€Π΅Π±ΠΈΡ‚Π΅Π»ΠΈ элСктричСской энСргии, мСньшС ΠΎΠ±Ρ‰Π΅Π³ΠΎ напряТСния Π² Ρ†Π΅ΠΏΠΈ.

Для ΠΏΠΎΡΠ»Π΅Π΄ΠΎΠ²Π°Ρ‚Π΅Π»ΡŒΠ½ΠΎΠ³ΠΎ соСдинСния ΠΏΡ€ΠΎΠ²ΠΎΠ΄Π½ΠΈΠΊΠΎΠ² справСдливы Π·Π°ΠΊΠΎΠ½Ρ‹ :

1) сила Ρ‚ΠΎΠΊΠ° Π²ΠΎ всСх ΠΏΡ€ΠΎΠ²ΠΎΠ΄Π½ΠΈΠΊΠ°Ρ… ΠΎΠ΄ΠΈΠ½Π°ΠΊΠΎΠ²Π°; 2) напряТСниС Π½Π° всём соСдинСнии Ρ€Π°Π²Π½ΠΎ суммС напряТСний Π½Π° ΠΎΡ‚Π΄Π΅Π»ΡŒΠ½Ρ‹Ρ… ΠΏΡ€ΠΎΠ²ΠΎΠ΄Π½ΠΈΠΊΠ°Ρ…; 3) сопротивлСниС всСго соСдинСния Ρ€Π°Π²Π½ΠΎ суммС сопротивлСний ΠΎΡ‚Π΄Π΅Π»ΡŒΠ½Ρ‹Ρ… ΠΏΡ€ΠΎΠ²ΠΎΠ΄Π½ΠΈΠΊΠΎΠ².

ΠŸΠ°Ρ€Π°Π»Π»Π΅Π»ΡŒΠ½ΠΎΠ΅ соСдинСниС ΠΏΡ€ΠΎΠ²ΠΎΠ΄Π½ΠΈΠΊΠΎΠ²

ΠŸΡ€ΠΈΠΌΠ΅Ρ€ΠΎΠΌ ΠΏΠ°Ρ€Π°Π»Π»Π΅Π»ΡŒΠ½ΠΎΠ³ΠΎ соСдинСния ΠΏΡ€ΠΎΠ²ΠΎΠ΄Π½ΠΈΠΊΠΎΠ² слуТит соСдинСниС ΠΏΠΎΡ‚Ρ€Π΅Π±ΠΈΡ‚Π΅Π»Π΅ΠΉ элСктричСской энСргии Π² ΠΊΠ²Π°Ρ€Ρ‚ΠΈΡ€Π΅. Π’Π°ΠΊ, элСктричСскиС Π»Π°ΠΌΠΏΠΎΡ‡ΠΊΠΈ, Ρ‡Π°ΠΉΠ½ΠΈΠΊ, ΡƒΡ‚ΡŽΠ³ ΠΈ ΠΏΡ€. Π²ΠΊΠ»ΡŽΡ‡Π°ΡŽΡ‚ΡΡ ΠΏΠ°Ρ€Π°Π»Π»Π΅Π»ΡŒΠ½ΠΎ.

ΠŸΡ€ΠΈ ΠΏΠ°Ρ€Π°Π»Π»Π΅Π»ΡŒΠ½ΠΎΠΌ соСдинСнии ΠΏΡ€ΠΎΠ²ΠΎΠ΄Π½ΠΈΠΊΠΎΠ² всС ΠΏΡ€ΠΎΠ²ΠΎΠ΄Π½ΠΈΠΊΠΈ ΠΎΠ΄Π½ΠΈΠΌ своим ΠΊΠΎΠ½Ρ†ΠΎΠΌ ΠΏΡ€ΠΈΡΠΎΠ΅Π΄ΠΈΠ½ΡΡŽΡ‚ΡΡ ΠΊ ΠΎΠ΄Π½ΠΎΠΉ Ρ‚ΠΎΡ‡ΠΊΠ΅ Ρ†Π΅ΠΏΠΈ. А Π²Ρ‚ΠΎΡ€Ρ‹ΠΌ ΠΊΠΎΠ½Ρ†ΠΎΠΌ ΠΊ Π΄Ρ€ΡƒΠ³ΠΎΠΉ Ρ‚ΠΎΡ‡ΠΊΠ΅ Ρ†Π΅ΠΏΠΈ. Π’ΠΎΠ»ΡŒΡ‚ΠΌΠ΅Ρ‚Ρ€, ΠΏΠΎΠ΄ΠΊΠ»ΡŽΡ‡Π΅Π½Π½Ρ‹ΠΉ ΠΊ этим Ρ‚ΠΎΡ‡ΠΊΠ°ΠΌ, ΠΏΠΎΠΊΠ°ΠΆΠ΅Ρ‚ напряТСниС ΠΈ Π½Π° ΠΏΡ€ΠΎΠ²ΠΎΠ΄Π½ΠΈΠΊΠ΅ 1, ΠΈ Π½Π° ΠΏΡ€ΠΎΠ²ΠΎΠ΄Π½ΠΈΠΊΠ΅ 2. Π’ Ρ‚Π°ΠΊΠΎΠΌ случаС напряТСниС Π½Π° ΠΊΠΎΠ½Ρ†Π°Ρ… всСх ΠΏΠ°Ρ€Π°Π»Π»Π΅Π»ΡŒΠ½ΠΎ соСдинённых ΠΏΡ€ΠΎΠ²ΠΎΠ΄Π½ΠΈΠΊΠΎΠ² ΠΎΠ΄Π½ΠΎ ΠΈ Ρ‚ΠΎ ΠΆΠ΅: U 1 = U 2 = U .

ΠŸΡ€ΠΈ ΠΏΠ°Ρ€Π°Π»Π»Π΅Π»ΡŒΠ½ΠΎΠΌ соСдинСнии ΠΏΡ€ΠΎΠ²ΠΎΠ΄Π½ΠΈΠΊΠΎΠ² элСктричСская Ρ†Π΅ΠΏΡŒ развСтвляСтся. ΠŸΠΎΡΡ‚ΠΎΠΌΡƒ Ρ‡Π°ΡΡ‚ΡŒ ΠΎΠ±Ρ‰Π΅Π³ΠΎ заряда ΠΏΡ€ΠΎΡ…ΠΎΠ΄ΠΈΡ‚ Ρ‡Π΅Ρ€Π΅Π· ΠΎΠ΄ΠΈΠ½ ΠΏΡ€ΠΎΠ²ΠΎΠ΄Π½ΠΈΠΊ, Π° Ρ‡Π°ΡΡ‚ΡŒ — Ρ‡Π΅Ρ€Π΅Π· Π΄Ρ€ΡƒΠ³ΠΎΠΉ. Π‘Π»Π΅Π΄ΠΎΠ²Π°Ρ‚Π΅Π»ΡŒΠ½ΠΎ ΠΏΡ€ΠΈ ΠΏΠ°Ρ€Π°Π»Π»Π΅Π»ΡŒΠ½ΠΎΠΌ соСдинСнии ΠΏΡ€ΠΎΠ²ΠΎΠ΄Π½ΠΈΠΊΠΎΠ² сила Ρ‚ΠΎΠΊΠ° Π² Π½Π΅Ρ€Π°Π·Π²Π΅Ρ‚Π²Π»Ρ‘Π½Π½ΠΎΠΉ части Ρ†Π΅ΠΏΠΈ Ρ€Π°Π²Π½Π° суммС силы Ρ‚ΠΎΠΊΠ° Π² ΠΎΡ‚Π΄Π΅Π»ΡŒΠ½Ρ‹Ρ… ΠΏΡ€ΠΎΠ²ΠΎΠ΄Π½ΠΈΠΊΠ°Ρ…: I = I 1 + I 2 .

Π’ соотвСтствии с Π·Π°ΠΊΠΎΠ½ΠΎΠΌ Ома I = U/R, I 1 = U 1 /R 1 , I 2 = U 2 /R 2 . ΠžΡ‚ΡΡŽΠ΄Π° слСдуСт: U/R = U 1 /R 1 + U 2 /R 2 , U = U 1 = U 2 , 1/R = 1/R 1 + 1/R 2 Π’Π΅Π»ΠΈΡ‡ΠΈΠ½Π°, обратная ΠΎΠ±Ρ‰Π΅ΠΌΡƒ ΡΠΎΠΏΡ€ΠΎΡ‚ΠΈΠ²Π»Π΅Π½ΠΈΡŽ ΠΏΠ°Ρ€Π°Π»Π»Π΅Π»ΡŒΠ½ΠΎ соСдинСнных ΠΏΡ€ΠΎΠ²ΠΎΠ΄Π½ΠΈΠΊΠΎΠ², Ρ€Π°Π²Π½Π° суммС Π²Π΅Π»ΠΈΡ‡ΠΈΠ½, ΠΎΠ±Ρ€Π°Ρ‚Π½Ρ‹Ρ… ΡΠΎΠΏΡ€ΠΎΡ‚ΠΈΠ²Π»Π΅Π½ΠΈΡŽ ΠΊΠ°ΠΆΠ΄ΠΎΠ³ΠΎ ΠΏΡ€ΠΎΠ²ΠΎΠ΄Π½ΠΈΠΊΠ°.

ΠŸΡ€ΠΈ ΠΏΠ°Ρ€Π°Π»Π»Π΅Π»ΡŒΠ½ΠΎΠΌ соСдинСнии ΠΏΡ€ΠΎΠ²ΠΎΠ΄Π½ΠΈΠΊΠΎΠ² ΠΈΡ… ΠΎΠ±Ρ‰Π΅Π΅ сопротивлСниС мСньшС, Ρ‡Π΅ΠΌ сопротивлСниС ΠΊΠ°ΠΆΠ΄ΠΎΠ³ΠΎ ΠΏΡ€ΠΎΠ²ΠΎΠ΄Π½ΠΈΠΊΠ°. Π”Π΅ΠΉΡΡ‚Π²ΠΈΡ‚Π΅Π»ΡŒΠ½ΠΎ, Ссли ΠΏΠ°Ρ€Π°Π»Π»Π΅Π»ΡŒΠ½ΠΎ соСдинСны Π΄Π²Π° ΠΏΡ€ΠΎΠ²ΠΎΠ΄Π½ΠΈΠΊΠ°, ΠΈΠΌΠ΅ΡŽΡ‰ΠΈΠ΅ ΠΎΠ΄ΠΈΠ½Π°ΠΊΠΎΠ²ΠΎΠ΅ сопротивлСниС Π³ , Ρ‚ΠΎ ΠΈΡ… ΠΎΠ±Ρ‰Π΅Π΅ сопротивлСниС Ρ€Π°Π²Π½ΠΎ: R = Π³/2 . Π­Ρ‚ΠΎ ΠΎΠ±ΡŠΡΡΠ½ΡΠ΅Ρ‚ΡΡ Ρ‚Π΅ΠΌ, Ρ‡Ρ‚ΠΎ ΠΏΡ€ΠΈ ΠΏΠ°Ρ€Π°Π»Π»Π΅Π»ΡŒΠ½ΠΎΠΌ соСдинСнии ΠΏΡ€ΠΎΠ²ΠΎΠ΄Π½ΠΈΠΊΠΎΠ² ΠΊΠ°ΠΊ Π±Ρ‹ увСличиваСтся ΠΏΠ»ΠΎΡ‰Π°Π΄ΡŒ ΠΈΡ… ΠΏΠΎΠΏΠ΅Ρ€Π΅Ρ‡Π½ΠΎΠ³ΠΎ сСчСния. Π’ Ρ€Π΅Π·ΡƒΠ»ΡŒΡ‚Π°Ρ‚Π΅ ΡƒΠΌΠ΅Π½ΡŒΡˆΠ°Π΅Ρ‚ΡΡ сопротивлСниС.

Из ΠΏΡ€ΠΈΠ²Π΅Π΄Ρ‘Π½Π½Ρ‹Ρ… Ρ„ΠΎΡ€ΠΌΡƒΠ» понятно, ΠΏΠΎΡ‡Π΅ΠΌΡƒ ΠΏΠΎΡ‚Ρ€Π΅Π±ΠΈΡ‚Π΅Π»ΠΈ элСктричСской энСргии Π²ΠΊΠ»ΡŽΡ‡Π°ΡŽΡ‚ΡΡ ΠΏΠ°Ρ€Π°Π»Π»Π΅Π»ΡŒΠ½ΠΎ. Они всС рассчитаны Π½Π° ΠΎΠΏΡ€Π΅Π΄Π΅Π»Ρ‘Π½Π½ΠΎΠ΅ ΠΎΠ΄ΠΈΠ½Π°ΠΊΠΎΠ²ΠΎΠ΅ напряТСниС, ΠΊΠΎΡ‚ΠΎΡ€ΠΎΠ΅ Π² ΠΊΠ²Π°Ρ€Ρ‚ΠΈΡ€Π°Ρ… Ρ€Π°Π²Π½ΠΎ 220 Π’. Зная сопротивлСниС ΠΊΠ°ΠΆΠ΄ΠΎΠ³ΠΎ потрСбитСля, ΠΌΠΎΠΆΠ½ΠΎ Ρ€Π°ΡΡΡ‡ΠΈΡ‚Π°Ρ‚ΡŒ силу Ρ‚ΠΎΠΊΠ° Π² ΠΊΠ°ΠΆΠ΄ΠΎΠΌ ΠΈΠ· Π½ΠΈΡ…. А Ρ‚Π°ΠΊΠΆΠ΅ соотвСтствиС суммарной силы Ρ‚ΠΎΠΊΠ° ΠΏΡ€Π΅Π΄Π΅Π»ΡŒΠ½ΠΎ допустимой силС Ρ‚ΠΎΠΊΠ°.

Для ΠΏΠ°Ρ€Π°Π»Π»Π΅Π»ΡŒΠ½ΠΎΠ³ΠΎ соСдинСния ΠΏΡ€ΠΎΠ²ΠΎΠ΄Π½ΠΈΠΊΠΎΠ² справСдливы Π·Π°ΠΊΠΎΠ½Ρ‹:

1) напряТСниС Π½Π° всСх ΠΏΡ€ΠΎΠ²ΠΎΠ΄Π½ΠΈΠΊΠ°Ρ… ΠΎΠ΄ΠΈΠ½Π°ΠΊΠΎΠ²ΠΎ; 2) сила Ρ‚ΠΎΠΊΠ° Π² мСстС соСдинСния ΠΏΡ€ΠΎΠ²ΠΎΠ΄Π½ΠΈΠΊΠΎΠ² Ρ€Π°Π²Π½Π° суммС Ρ‚ΠΎΠΊΠΎΠ² Π² ΠΎΡ‚Π΄Π΅Π»ΡŒΠ½Ρ‹Ρ… ΠΏΡ€ΠΎΠ²ΠΎΠ΄Π½ΠΈΠΊΠ°Ρ…; 3) Π²Π΅Π»ΠΈΡ‡ΠΈΠ½Π°, обратная ΡΠΎΠΏΡ€ΠΎΡ‚ΠΈΠ²Π»Π΅Π½ΠΈΡŽ всСго соСдинСния, Ρ€Π°Π²Π½Π° суммС Π²Π΅Π»ΠΈΡ‡ΠΈΠ½, ΠΎΠ±Ρ€Π°Ρ‚Π½Ρ‹Ρ… сопротивлСниям ΠΎΡ‚Π΄Π΅Π»ΡŒΠ½Ρ‹Ρ… ΠΏΡ€ΠΎΠ²ΠΎΠ΄Π½ΠΈΠΊΠΎΠ².

ΠŸΠ°Ρ€Π°Π»Π»Π΅Π»ΡŒΠ½ΠΎΠ΅ ΠΈ ΠΏΠΎΡΠ»Π΅Π΄ΠΎΠ²Π°Ρ‚Π΅Π»ΡŒΠ½ΠΎΠ΅ соСдинСниС ΠΏΡ€ΠΎΠ²ΠΎΠ΄Π½ΠΈΠΊΠΎΠ² – способы ΠΊΠΎΠΌΠΌΡƒΡ‚Π°Ρ†ΠΈΠΈ элСктричСской Ρ†Π΅ΠΏΠΈ. ЭлСктричСскиС схСмы любой слоТности ΠΌΠΎΠΆΠ½ΠΎ ΠΏΡ€Π΅Π΄ΡΡ‚Π°Π²ΠΈΡ‚ΡŒ посрСдством ΡƒΠΊΠ°Π·Π°Π½Π½Ρ‹Ρ… абстракций.

ΠžΠΏΡ€Π΅Π΄Π΅Π»Π΅Π½ΠΈΡ

БущСствуСт Π΄Π²Π° способа соСдинСния ΠΏΡ€ΠΎΠ²ΠΎΠ΄Π½ΠΈΠΊΠΎΠ², становится Π²ΠΎΠ·ΠΌΠΎΠΆΠ½Ρ‹ΠΌ ΡƒΠΏΡ€ΠΎΡΡ‚ΠΈΡ‚ΡŒ расчСт Ρ†Π΅ΠΏΠΈ ΠΏΡ€ΠΎΠΈΠ·Π²ΠΎΠ»ΡŒΠ½ΠΎΠΉ слоТности:

  • ΠšΠΎΠ½Π΅Ρ† ΠΏΡ€Π΅Π΄Ρ‹Π΄ΡƒΡ‰Π΅Π³ΠΎ ΠΏΡ€ΠΎΠ²ΠΎΠ΄Π½ΠΈΠΊΠ° соСдинСн нСпосрСдствСнно с Π½Π°Ρ‡Π°Π»ΠΎΠΌ ΡΠ»Π΅Π΄ΡƒΡŽΡ‰Π΅Π³ΠΎ β€” ΠΏΠΎΠ΄ΠΊΠ»ΡŽΡ‡Π΅Π½ΠΈΠ΅ Π½Π°Π·Ρ‹Π²Π°ΡŽΡ‚ ΠΏΠΎΡΠ»Π΅Π΄ΠΎΠ²Π°Ρ‚Π΅Π»ΡŒΠ½Ρ‹ΠΌ. ΠžΠ±Ρ€Π°Π·ΡƒΠ΅Ρ‚ΡΡ Ρ†Π΅ΠΏΠΎΡ‡ΠΊΠ°. Π§Ρ‚ΠΎΠ±Ρ‹ Π²ΠΊΠ»ΡŽΡ‡ΠΈΡ‚ΡŒ ΠΎΡ‡Π΅Ρ€Π΅Π΄Π½ΠΎΠ΅ Π·Π²Π΅Π½ΠΎ, Π½ΡƒΠΆΠ½ΠΎ ΡΠ»Π΅ΠΊΡ‚Ρ€ΠΈΡ‡Π΅ΡΠΊΡƒΡŽ схСму Ρ€Π°Π·ΠΎΡ€Π²Π°Ρ‚ΡŒ, вставив Ρ‚ΡƒΠ΄Π° Π½ΠΎΠ²Ρ‹ΠΉ ΠΏΡ€ΠΎΠ²ΠΎΠ΄Π½ΠΈΠΊ.
  • Начала ΠΏΡ€ΠΎΠ²ΠΎΠ΄Π½ΠΈΠΊΠΎΠ² соСдинСны ΠΎΠ΄Π½ΠΎΠΉ Ρ‚ΠΎΡ‡ΠΊΠΎΠΉ, ΠΊΠΎΠ½Ρ†Ρ‹ – Π΄Ρ€ΡƒΠ³ΠΎΠΉ, ΠΏΠΎΠ΄ΠΊΠ»ΡŽΡ‡Π΅Π½ΠΈΠ΅ называСтся ΠΏΠ°Ρ€Π°Π»Π»Π΅Π»ΡŒΠ½Ρ‹ΠΌ. Бвязку принято Π½Π°Π·Ρ‹Π²Π°Ρ‚ΡŒ Ρ€Π°Π·Π²Π΅Ρ‚Π²Π»Π΅Π½ΠΈΠ΅ΠΌ. ΠšΠ°ΠΆΠ΄Ρ‹ΠΉ ΠΎΡ‚Π΄Π΅Π»ΡŒΠ½Ρ‹ΠΉ ΠΏΡ€ΠΎΠ²ΠΎΠ΄Π½ΠΈΠΊ ΠΎΠ±Ρ€Π°Π·ΡƒΠ΅Ρ‚ Π²Π΅Ρ‚Π²ΡŒ. ΠžΠ±Ρ‰ΠΈΠ΅ Ρ‚ΠΎΡ‡ΠΊΠΈ ΠΈΠΌΠ΅Π½ΡƒΡŽΡ‚ΡΡ ΡƒΠ·Π»Π°ΠΌΠΈ элСктричСской сСти.

На ΠΏΡ€Π°ΠΊΡ‚ΠΈΠΊΠ΅ Ρ‡Π°Ρ‰Π΅ встрСчаСтся смСшанноС Π²ΠΊΠ»ΡŽΡ‡Π΅Π½ΠΈΠ΅ ΠΏΡ€ΠΎΠ²ΠΎΠ΄Π½ΠΈΠΊΠΎΠ², Ρ‡Π°ΡΡ‚ΡŒ соСдинСна ΠΏΠΎΡΠ»Π΅Π΄ΠΎΠ²Π°Ρ‚Π΅Π»ΡŒΠ½ΠΎ, Ρ‡Π°ΡΡ‚ΡŒ – ΠΏΠ°Ρ€Π°Π»Π»Π΅Π»ΡŒΠ½ΠΎ. НуТно Ρ€Π°Π·Π±ΠΈΡ‚ΡŒ Ρ†Π΅ΠΏΡŒ простыми сСгмСнтами, Ρ€Π΅ΡˆΠ°Ρ‚ΡŒ Π·Π°Π΄Π°Ρ‡Ρƒ для ΠΊΠ°ΠΆΠ΄ΠΎΠ³ΠΎ ΠΎΡ‚Π΄Π΅Π»ΡŒΠ½ΠΎ. Бколь ΡƒΠ³ΠΎΠ΄Π½ΠΎ ΡΠ»ΠΎΠΆΠ½ΡƒΡŽ ΡΠ»Π΅ΠΊΡ‚Ρ€ΠΈΡ‡Π΅ΡΠΊΡƒΡŽ схСму ΠΌΠΎΠΆΠ½ΠΎ ΠΎΠΏΠΈΡΠ°Ρ‚ΡŒ ΠΏΠ°Ρ€Π°Π»Π»Π΅Π»ΡŒΠ½Ρ‹ΠΌ, ΠΏΠΎΡΠ»Π΅Π΄ΠΎΠ²Π°Ρ‚Π΅Π»ΡŒΠ½Ρ‹ΠΌ соСдинСниСм ΠΏΡ€ΠΎΠ²ΠΎΠ΄Π½ΠΈΠΊΠΎΠ². Π’Π°ΠΊ дСлаСтся Π½Π° ΠΏΡ€Π°ΠΊΡ‚ΠΈΠΊΠ΅.

ИспользованиС ΠΏΠ°Ρ€Π°Π»Π»Π΅Π»ΡŒΠ½ΠΎΠ³ΠΎ ΠΈ ΠΏΠΎΡΠ»Π΅Π΄ΠΎΠ²Π°Ρ‚Π΅Π»ΡŒΠ½ΠΎΠ³ΠΎ соСдинСния ΠΏΡ€ΠΎΠ²ΠΎΠ΄Π½ΠΈΠΊΠΎΠ²

Π’Π΅Ρ€ΠΌΠΈΠ½Ρ‹, примСняСмыС ΠΊ элСктричСским цСпям

ВСория выступаСт базисом формирования ΠΏΡ€ΠΎΡ‡Π½Ρ‹Ρ… Π·Π½Π°Π½ΠΈΠΉ, Π½Π΅ΠΌΠ½ΠΎΠ³ΠΈΠ΅ Π·Π½Π°ΡŽΡ‚, Ρ‡Π΅ΠΌ напряТСниС (Ρ€Π°Π·Π½ΠΎΡΡ‚ΡŒ ΠΏΠΎΡ‚Π΅Π½Ρ†ΠΈΠ°Π»ΠΎΠ²) отличаСтся ΠΎΡ‚ падСния напряТСния. Π’ Ρ‚Π΅Ρ€ΠΌΠΈΠ½Π°Ρ… Ρ„ΠΈΠ·ΠΈΠΊΠΈ Π²Π½ΡƒΡ‚Ρ€Π΅Π½Π½Π΅ΠΉ Ρ†Π΅ΠΏΡŒΡŽ Π½Π°Π·Ρ‹Π²Π°ΡŽΡ‚ источник Ρ‚ΠΎΠΊΠ°, находящССся Π²Π½Π΅ – имСнуСтся внСшнСй. Π Π°Π·Π³Ρ€Π°Π½ΠΈΡ‡Π΅Π½ΠΈΠ΅ ΠΏΠΎΠΌΠΎΠ³Π°Π΅Ρ‚ ΠΏΡ€Π°Π²ΠΈΠ»ΡŒΠ½ΠΎ ΠΎΠΏΠΈΡΠ°Ρ‚ΡŒ распрСдСлСниС поля. Π’ΠΎΠΊ ΡΠΎΠ²Π΅Ρ€ΡˆΠ°Π΅Ρ‚ Ρ€Π°Π±ΠΎΡ‚Ρƒ. Π’ ΠΏΡ€ΠΎΡΡ‚Π΅ΠΉΡˆΠ΅ΠΌ случаС гСнСрация Ρ‚Π΅ΠΏΠ»Π° согласно Π·Π°ΠΊΠΎΠ½Ρƒ ДТоуля-Π›Π΅Π½Ρ†Π°. ЗаряТСнныС частицы, ΠΏΠ΅Ρ€Π΅Π΄Π²ΠΈΠ³Π°ΡΡΡŒ Π² сторону мСньшСго ΠΏΠΎΡ‚Π΅Π½Ρ†ΠΈΠ°Π»Π°, ΡΡ‚Π°Π»ΠΊΠΈΠ²Π°ΡŽΡ‚ΡΡ с кристалличСской Ρ€Π΅ΡˆΠ΅Ρ‚ΠΊΠΎΠΉ, ΠΎΡ‚Π΄Π°ΡŽΡ‚ ΡΠ½Π΅Ρ€Π³ΠΈΡŽ. ΠŸΡ€ΠΎΠΈΡΡ…ΠΎΠ΄ΠΈΡ‚ Π½Π°Π³Ρ€Π΅Π² сопротивлСний.

Для обСспСчСния двиТСния Π½ΡƒΠΆΠ½ΠΎ Π½Π° ΠΊΠΎΠ½Ρ†Π°Ρ… ΠΏΡ€ΠΎΠ²ΠΎΠ΄Π½ΠΈΠΊΠ° ΠΏΠΎΠ΄Π΄Π΅Ρ€ΠΆΠΈΠ²Π°Ρ‚ΡŒ Ρ€Π°Π·Π½ΠΎΡΡ‚ΡŒ ΠΏΠΎΡ‚Π΅Π½Ρ†ΠΈΠ°Π»ΠΎΠ². Π­Ρ‚ΠΎ называСтся напряТСниСм участка Ρ†Π΅ΠΏΠΈ. Если просто ΠΏΠΎΠΌΠ΅ΡΡ‚ΠΈΡ‚ΡŒ ΠΏΡ€ΠΎΠ²ΠΎΠ΄Π½ΠΈΠΊ Π² ΠΏΠΎΠ»Π΅ вдоль силовых Π»ΠΈΠ½ΠΈΠΉ, Ρ‚ΠΎΠΊ ΠΏΠΎΡ‚Π΅Ρ‡Π΅Ρ‚, Π±ΡƒΠ΄Π΅Ρ‚ ΠΎΡ‡Π΅Π½ΡŒ ΠΊΡ€Π°Ρ‚ΠΊΠΎΠ²Ρ€Π΅ΠΌΠ΅Π½Π½Ρ‹ΠΌ. ΠŸΡ€ΠΎΡ†Π΅ΡΡ Π·Π°Π²Π΅Ρ€ΡˆΠΈΡ‚ΡΡ наступлСниСм равновСсия. Π’Π½Π΅ΡˆΠ½Π΅Π΅ ΠΏΠΎΠ»Π΅ Π±ΡƒΠ΄Π΅Ρ‚ ΡƒΡ€Π°Π²Π½ΠΎΠ²Π΅ΡˆΠ΅Π½ΠΎ собствСнным ΠΏΠΎΠ»Π΅ΠΌ зарядов, ΠΏΡ€ΠΎΡ‚ΠΈΠ²ΠΎΠΏΠΎΠ»ΠΎΠΆΠ½Ρ‹ΠΌ Π½Π°ΠΏΡ€Π°Π²Π»Π΅Π½ΠΈΠ΅ΠΌ. Π’ΠΎΠΊ прСкратится. Π§Ρ‚ΠΎΠ±Ρ‹ процСсс стал Π½Π΅ΠΏΡ€Π΅Ρ€Ρ‹Π²Π½Ρ‹ΠΌ, Π½ΡƒΠΆΠ½Π° внСшняя сила.

Π’Π°ΠΊΠΈΠΌ ΠΏΡ€ΠΈΠ²ΠΎΠ΄ΠΎΠΌ двиТСния элСктричСской Ρ†Π΅ΠΏΠΈ выступаСт источник Ρ‚ΠΎΠΊΠ°. Π§Ρ‚ΠΎΠ±Ρ‹ ΠΏΠΎΠ΄Π΄Π΅Ρ€ΠΆΠΈΠ²Π°Ρ‚ΡŒ ΠΏΠΎΡ‚Π΅Π½Ρ†ΠΈΠ°Π», Π²Π½ΡƒΡ‚Ρ€ΠΈ ΡΠΎΠ²Π΅Ρ€ΡˆΠ°Π΅Ρ‚ΡΡ Ρ€Π°Π±ΠΎΡ‚Π°. Π₯имичСская рСакция, ΠΊΠ°ΠΊ Π² Π³Π°Π»ΡŒΠ²Π°Π½ΠΈΡ‡Π΅ΡΠΊΠΎΠΌ элСмСнтС, мСханичСскиС силы – Π³Π΅Π½Π΅Ρ€Π°Ρ‚ΠΎΡ€ Π“Π­Π‘. Заряды Π²Π½ΡƒΡ‚Ρ€ΠΈ источника двиТутся Π² ΠΏΡ€ΠΎΡ‚ΠΈΠ²ΠΎΠΏΠΎΠ»ΠΎΠΆΠ½ΡƒΡŽ полю сторону. Над этим ΡΠΎΠ²Π΅Ρ€ΡˆΠ°Π΅Ρ‚ΡΡ Ρ€Π°Π±ΠΎΡ‚Π° сторонних сил. МоТно ΠΏΠ΅Ρ€Π΅Ρ„Ρ€Π°Π·ΠΈΡ€ΠΎΠ²Π°Ρ‚ΡŒ ΠΏΡ€ΠΈΠ²Π΅Π΄Π΅Π½Π½Ρ‹Π΅ Π²Ρ‹ΡˆΠ΅ Ρ„ΠΎΡ€ΠΌΡƒΠ»ΠΈΡ€ΠΎΠ²ΠΊΠΈ, ΡΠΊΠ°Π·Π°Ρ‚ΡŒ:

  • Π’Π½Π΅ΡˆΠ½ΡΡ Ρ‡Π°ΡΡ‚ΡŒ Ρ†Π΅ΠΏΠΈ, Π³Π΄Π΅ заряды двиТутся, ΡƒΠ²Π»Π΅ΠΊΠ°Π΅ΠΌΡ‹Π΅ ΠΏΠΎΠ»Π΅ΠΌ.
  • ВнутрСнняя Ρ‡Π°ΡΡ‚ΡŒ Ρ†Π΅ΠΏΠΈ, Π³Π΄Π΅ заряды двиТутся ΠΏΡ€ΠΎΡ‚ΠΈΠ² напряТСнности.

Π“Π΅Π½Π΅Ρ€Π°Ρ‚ΠΎΡ€ (источник Ρ‚ΠΎΠΊΠ°) снабТСн двумя полюсами. ΠžΠ±Π»Π°Π΄Π°ΡŽΡ‰ΠΈΠΉ мСньшим ΠΏΠΎΡ‚Π΅Π½Ρ†ΠΈΠ°Π»ΠΎΠΌ называСтся ΠΎΡ‚Ρ€ΠΈΡ†Π°Ρ‚Π΅Π»ΡŒΠ½Ρ‹ΠΌ, Π΄Ρ€ΡƒΠ³ΠΎΠΉ – ΠΏΠΎΠ»ΠΎΠΆΠΈΡ‚Π΅Π»ΡŒΠ½Ρ‹ΠΌ. Π’ случаС ΠΏΠ΅Ρ€Π΅ΠΌΠ΅Π½Π½ΠΎΠ³ΠΎ Ρ‚ΠΎΠΊΠ° ΠΏΠΎΠ»ΡŽΡΡ‹ Π½Π΅ΠΏΡ€Π΅Ρ€Ρ‹Π²Π½ΠΎ ΠΌΠ΅Π½ΡΡŽΡ‚ΡΡ мСстами. НСпостоянно Π½Π°ΠΏΡ€Π°Π²Π»Π΅Π½ΠΈΠ΅ двиТСния зарядов. Π’ΠΎΠΊ Ρ‚Π΅Ρ‡Π΅Ρ‚ ΠΎΡ‚ ΠΏΠΎΠ»ΠΎΠΆΠΈΡ‚Π΅Π»ΡŒΠ½ΠΎΠ³ΠΎ полюса ΠΊ ΠΎΡ‚Ρ€ΠΈΡ†Π°Ρ‚Π΅Π»ΡŒΠ½ΠΎΠΌΡƒ. Π”Π²ΠΈΠΆΠ΅Π½ΠΈΠ΅ ΠΏΠΎΠ»ΠΎΠΆΠΈΡ‚Π΅Π»ΡŒΠ½Ρ‹Ρ… зарядов ΠΈΠ΄Π΅Ρ‚ Π² Π½Π°ΠΏΡ€Π°Π²Π»Π΅Π½ΠΈΠΈ убывания ΠΏΠΎΡ‚Π΅Π½Ρ†ΠΈΠ°Π»Π°. Богласно этому Ρ„Π°ΠΊΡ‚Ρƒ вводится понятиС падСния ΠΏΠΎΡ‚Π΅Π½Ρ†ΠΈΠ°Π»Π°:

ПадСниСм ΠΏΠΎΡ‚Π΅Π½Ρ†ΠΈΠ°Π»Π° участка Ρ†Π΅ΠΏΠΈ называСтся ΡƒΠ±Ρ‹Π»ΡŒ ΠΏΠΎΡ‚Π΅Π½Ρ†ΠΈΠ°Π»Π° Π² ΠΏΡ€Π΅Π΄Π΅Π»Π°Ρ… ΠΎΡ‚Ρ€Π΅Π·ΠΊΠ°. Π€ΠΎΡ€ΠΌΠ°Π»ΡŒΠ½ΠΎ это напряТСниС. Для Π²Π΅Ρ‚Π²Π΅ΠΉ ΠΏΠ°Ρ€Π°Π»Π»Π΅Π»ΡŒΠ½ΠΎΠΉ Ρ†Π΅ΠΏΠΈ ΠΎΠ΄ΠΈΠ½Π°ΠΊΠΎΠ²ΠΎ.

Под ΠΏΠ°Π΄Π΅Π½ΠΈΠ΅ΠΌ напряТСния понимаСтся ΠΈ Π½Π΅Ρ‡Ρ‚ΠΎ ΠΈΠ½ΠΎΠ΅. Π’Π΅Π»ΠΈΡ‡ΠΈΠ½Π°, Ρ…Π°Ρ€Π°ΠΊΡ‚Π΅Ρ€ΠΈΠ·ΡƒΡŽΡ‰Π°Ρ Ρ‚Π΅ΠΏΠ»ΠΎΠ²Ρ‹Π΅ ΠΏΠΎΡ‚Π΅Ρ€ΠΈ, числСнно Ρ€Π°Π²Π½Π° ΠΏΡ€ΠΎΠΈΠ·Π²Π΅Π΄Π΅Π½ΠΈΡŽ Ρ‚ΠΎΠΊΠ° Π½Π° Π°ΠΊΡ‚ΠΈΠ²Π½ΠΎΠ΅ сопротивлСниС участка. Π—Π°ΠΊΠΎΠ½Ρ‹ Ома, ΠšΠΈΡ€Ρ…Π³ΠΎΡ„Π°, рассмотрСнныС Π½ΠΈΠΆΠ΅, Ρ„ΠΎΡ€ΠΌΡƒΠ»ΠΈΡ€ΡƒΡŽΡ‚ΡΡ для этого случая. Π’ элСктричСских двигатСлях, трансформаторах Ρ€Π°Π·Π½ΠΈΡ†Π° ΠΏΠΎΡ‚Π΅Π½Ρ†ΠΈΠ°Π»ΠΎΠ² ΠΌΠΎΠΆΠ΅Ρ‚ Π·Π½Π°Ρ‡ΠΈΡ‚Π΅Π»ΡŒΠ½ΠΎ ΠΎΡ‚Π»ΠΈΡ‡Π°Ρ‚ΡŒΡΡ ΠΎΡ‚ падСния напряТСния. ПослСднСС Ρ…Π°Ρ€Π°ΠΊΡ‚Π΅Ρ€ΠΈΠ·ΡƒΠ΅Ρ‚ ΠΏΠΎΡ‚Π΅Ρ€ΠΈ Π½Π° Π°ΠΊΡ‚ΠΈΠ²Π½ΠΎΠΌ сопротивлСнии, Ρ‚ΠΎΠ³Π΄Π° ΠΊΠ°ΠΊ ΠΏΠ΅Ρ€Π²ΠΎΠ΅ ΡƒΡ‡ΠΈΡ‚Ρ‹Π²Π°Π΅Ρ‚ ΠΏΠΎΠ»Π½ΡƒΡŽ Ρ€Π°Π±ΠΎΡ‚Ρƒ источника Ρ‚ΠΎΠΊΠ°.

ΠŸΡ€ΠΈ Ρ€Π΅ΡˆΠ΅Π½ΠΈΠ΅ физичСских Π·Π°Π΄Π°Ρ‡ для упрощСния Π΄Π²ΠΈΠ³Π°Ρ‚Π΅Π»ΡŒ ΠΌΠΎΠΆΠ΅Ρ‚ Π²ΠΊΠ»ΡŽΡ‡Π°Ρ‚ΡŒ Π² свой состав Π­Π”Π‘, Π½Π°ΠΏΡ€Π°Π²Π»Π΅Π½ΠΈΠ΅ дСйствия ΠΊΠΎΡ‚ΠΎΡ€ΠΎΠΉ ΠΏΡ€ΠΎΡ‚ΠΈΠ²ΠΎΠΏΠΎΠ»ΠΎΠΆΠ½ΠΎ эффСкту источника питания. УчитываСтся Ρ„Π°ΠΊΡ‚ ΠΏΠΎΡ‚Π΅Ρ€ΠΈ энСргии Ρ‡Π΅Ρ€Π΅Π· Ρ€Π΅Π°ΠΊΡ‚ΠΈΠ²Π½ΡƒΡŽ Ρ‡Π°ΡΡ‚ΡŒ импСданса. Π¨ΠΊΠΎΠ»ΡŒΠ½Ρ‹ΠΉ ΠΈ вузовский курс Ρ„ΠΈΠ·ΠΈΠΊΠΈ отличаСтся ΠΎΡ‚ΠΎΡ€Π²Π°Π½Π½ΠΎΡΡ‚ΡŒΡŽ ΠΎΡ‚ Ρ€Π΅Π°Π»ΡŒΠ½ΠΎΡΡ‚ΠΈ. Π’ΠΎΡ‚ ΠΏΠΎΡ‡Π΅ΠΌΡƒ студСнты, раскрыв Ρ€ΠΎΡ‚, ΡΠ»ΡƒΡˆΠ°ΡŽΡ‚ ΠΎ явлСниях, ΠΈΠΌΠ΅ΡŽΡ‰ΠΈΡ… мСсто Π² элСктротСхникС. Π’ ΠΏΠ΅Ρ€ΠΈΠΎΠ΄, ΠΏΡ€Π΅Π΄ΡˆΠ΅ΡΡ‚Π²ΡƒΡŽΡ‰ΠΈΠΉ эпохС ΠΏΡ€ΠΎΠΌΡ‹ΡˆΠ»Π΅Π½Π½ΠΎΠΉ Ρ€Π΅Π²ΠΎΠ»ΡŽΡ†ΠΈΠΈ, ΠΎΡ‚ΠΊΡ€Ρ‹Π²Π°Π»ΠΈΡΡŒ Π³Π»Π°Π²Π½Ρ‹Π΅ Π·Π°ΠΊΠΎΠ½Ρ‹, ΡƒΡ‡Π΅Π½Ρ‹ΠΉ Π΄ΠΎΠ»ΠΆΠ΅Π½ ΠΎΠ±ΡŠΠ΅Π΄ΠΈΠ½ΡΡ‚ΡŒ Ρ€ΠΎΠ»ΡŒ Ρ‚Π΅ΠΎΡ€Π΅Ρ‚ΠΈΠΊΠ° ΠΈ Ρ‚Π°Π»Π°Π½Ρ‚Π»ΠΈΠ²ΠΎΠ³ΠΎ экспСримСнтатора. Об этом ΠΎΡ‚ΠΊΡ€Ρ‹Ρ‚ΠΎ говорят прСдисловия ΠΊ Ρ‚Ρ€ΡƒΠ΄Π°ΠΌ ΠšΠΈΡ€Ρ…Π³ΠΎΡ„Π° (Ρ€Π°Π±ΠΎΡ‚Ρ‹ Π“Π΅ΠΎΡ€Π³Π° Ома Π½Π° русский язык Π½Π΅ ΠΏΠ΅Ρ€Π΅Π²Π΅Π΄Π΅Π½Ρ‹). ΠŸΡ€Π΅ΠΏΠΎΠ΄Π°Π²Π°Ρ‚Π΅Π»ΠΈ Π±ΡƒΠΊΠ²Π°Π»ΡŒΠ½ΠΎ Π·Π°Π²Π»Π΅ΠΊΠ°Π»ΠΈ люд Π΄ΠΎΠΏΠΎΠ»Π½ΠΈΡ‚Π΅Π»ΡŒΠ½Ρ‹ΠΌΠΈ лСкциями, сдобрСнными наглядными, ΡƒΠ΄ΠΈΠ²ΠΈΡ‚Π΅Π»ΡŒΠ½Ρ‹ΠΌΠΈ экспСримСнтами.

Π—Π°ΠΊΠΎΠ½Ρ‹ Ома ΠΈ ΠšΠΈΡ€Ρ…Π³ΠΎΡ„Π° ΠΏΡ€ΠΈΠΌΠ΅Π½ΠΈΡ‚Π΅Π»ΡŒΠ½ΠΎ ΠΊ ΠΏΠΎΡΠ»Π΅Π΄ΠΎΠ²Π°Ρ‚Π΅Π»ΡŒΠ½ΠΎΠΌΡƒ ΠΈ ΠΏΠ°Ρ€Π°Π»Π»Π΅Π»ΡŒΠ½ΠΎΠΌΡƒ соСдинСнию ΠΏΡ€ΠΎΠ²ΠΎΠ΄Π½ΠΈΠΊΠΎΠ²

Для Ρ€Π΅ΡˆΠ΅Π½ΠΈΡ Ρ€Π΅Π°Π»ΡŒΠ½Ρ‹Ρ… Π·Π°Π΄Π°Ρ‡ ΠΈΡΠΏΠΎΠ»ΡŒΠ·ΡƒΡŽΡ‚ΡΡ Π·Π°ΠΊΠΎΠ½Ρ‹ Ома ΠΈ ΠšΠΈΡ€Ρ…Π³ΠΎΡ„Π°. ΠŸΠ΅Ρ€Π²Ρ‹ΠΉ Π²Ρ‹Π²ΠΎΠ΄ΠΈΠ» равСнство чисто эмпиричСским ΠΏΡƒΡ‚Π΅ΠΌ – ΡΠΊΡΠΏΠ΅Ρ€ΠΈΠΌΠ΅Π½Ρ‚Π°Π»ΡŒΠ½ΠΎ – Π²Ρ‚ΠΎΡ€ΠΎΠΉ Π½Π°Ρ‡Π°Π» матСматичСским Π°Π½Π°Π»ΠΈΠ·ΠΎΠΌ Π·Π°Π΄Π°Ρ‡ΠΈ, ΠΏΠΎΡ‚ΠΎΠΌ ΠΏΡ€ΠΎΠ²Π΅Ρ€ΠΈΠ» Π΄ΠΎΠ³Π°Π΄ΠΊΠΈ ΠΏΡ€Π°ΠΊΡ‚ΠΈΠΊΠΎΠΉ. ΠŸΡ€ΠΈΠ²Π΅Π΄Π΅ΠΌ Π½Π΅ΠΊΠΎΡ‚ΠΎΡ€Ρ‹Π΅ свСдСния, ΠΏΠΎΠΌΠΎΠ³Π°ΡŽΡ‰ΠΈΠ΅ Ρ€Π΅ΡˆΠ΅Π½ΠΈΡŽ Π·Π°Π΄Π°Ρ‡ΠΈ:

ΠŸΠΎΡΡ‡ΠΈΡ‚Π°Ρ‚ΡŒ сопротивлСния элСмСнтов ΠΏΡ€ΠΈ ΠΏΠΎΡΠ»Π΅Π΄ΠΎΠ²Π°Ρ‚Π΅Π»ΡŒΠ½ΠΎΠΌ ΠΈ ΠΏΠ°Ρ€Π°Π»Π»Π΅Π»ΡŒΠ½ΠΎΠΌ соСдинСнии

Алгоритм расчСта Ρ€Π΅Π°Π»ΡŒΠ½Ρ‹Ρ… Ρ†Π΅ΠΏΠ΅ΠΉ прост. ΠŸΡ€ΠΈΠ²Π΅Π΄Π΅ΠΌ Π½Π΅ΠΊΠΎΡ‚ΠΎΡ€Ρ‹Π΅ тСзисы ΠΊΠ°ΡΠ°Ρ‚Π΅Π»ΡŒΠ½ΠΎ рассматриваСмой Ρ‚Π΅ΠΌΠ°Ρ‚ΠΈΠΊΠΈ:

  1. ΠŸΡ€ΠΈ ΠΏΠΎΡΠ»Π΅Π΄ΠΎΠ²Π°Ρ‚Π΅Π»ΡŒΠ½ΠΎΠΌ Π²ΠΊΠ»ΡŽΡ‡Π΅Π½ΠΈΠΈ ΡΡƒΠΌΠΌΠΈΡ€ΡƒΡŽΡ‚ΡΡ сопротивлСния, ΠΏΡ€ΠΈ ΠΏΠ°Ρ€Π°Π»Π»Π΅Π»ΡŒΠ½ΠΎΠΌ β€” проводимости:
    1. Для рСзисторов Π·Π°ΠΊΠΎΠ½ пСрСписываСтся Π² Π½Π΅ΠΈΠ·ΠΌΠ΅Π½Π½ΠΎΠΉ Ρ„ΠΎΡ€ΠΌΠ΅. ΠŸΡ€ΠΈ ΠΏΠ°Ρ€Π°Π»Π»Π΅Π»ΡŒΠ½ΠΎΠΌ соСдинСнии ΠΈΡ‚ΠΎΠ³ΠΎΠ²ΠΎΠ΅ сопротивлСниС равняСтся ΠΏΡ€ΠΎΠΈΠ·Π²Π΅Π΄Π΅Π½ΠΈΡŽ исходных, Π΄Π΅Π»Π΅Π½Π½ΠΎΠΌΡƒ Π½Π° ΠΎΠ±Ρ‰ΡƒΡŽ сумму. ΠŸΡ€ΠΈ ΠΏΠΎΡΠ»Π΅Π΄ΠΎΠ²Π°Ρ‚Π΅Π»ΡŒΠ½ΠΎΠΌ – Π½ΠΎΠΌΠΈΠ½Π°Π»Ρ‹ ΡΡƒΠΌΠΌΠΈΡ€ΡƒΡŽΡ‚ΡΡ.
    2. Π˜Π½Π΄ΡƒΠΊΡ‚ΠΈΠ²Π½ΠΎΡΡ‚ΡŒ выступаСт Ρ€Π΅Π°ΠΊΡ‚ΠΈΠ²Π½Ρ‹ΠΌ сопротивлСниСм (j*Ο‰*L), Π²Π΅Π΄Π΅Ρ‚ сСбя, ΠΊΠ°ΠΊ ΠΎΠ±Ρ‹Ρ‡Π½Ρ‹ΠΉ рСзистор. Π’ ΠΏΠ»Π°Π½Π΅ написания Ρ„ΠΎΡ€ΠΌΡƒΠ»Ρ‹ Π½ΠΈΡ‡Π΅ΠΌ Π½Π΅ отличаСтся. Нюанс, для всякого чисто ΠΌΠ½ΠΈΠΌΠΎΠ³ΠΎ импСданса, Ρ‡Ρ‚ΠΎ Π½ΡƒΠΆΠ½ΠΎ ΡƒΠΌΠ½ΠΎΠΆΠΈΡ‚ΡŒ Ρ€Π΅Π·ΡƒΠ»ΡŒΡ‚Π°Ρ‚ Π½Π° ΠΎΠΏΠ΅Ρ€Π°Ρ‚ΠΎΡ€ j, ΠΊΡ€ΡƒΠ³ΠΎΠ²ΡƒΡŽ частоту Ο‰ (2*Пи*f). ΠŸΡ€ΠΈ ΠΏΠΎΡΠ»Π΅Π΄ΠΎΠ²Π°Ρ‚Π΅Π»ΡŒΠ½ΠΎΠΌ соСдинСнии ΠΊΠ°Ρ‚ΡƒΡˆΠ΅ΠΊ индуктивности Π½ΠΎΠΌΠΈΠ½Π°Π»Ρ‹ ΡΡƒΠΌΠΌΠΈΡ€ΡƒΡŽΡ‚ΡΡ, ΠΏΡ€ΠΈ ΠΏΠ°Ρ€Π°Π»Π»Π΅Π»ΡŒΠ½ΠΎΠΌ – ΡΠΊΠ»Π°Π΄Ρ‹Π²Π°ΡŽΡ‚ΡΡ ΠΎΠ±Ρ€Π°Ρ‚Π½Ρ‹Π΅ Π²Π΅Π»ΠΈΡ‡ΠΈΠ½Ρ‹.
    3. МнимоС сопротивлСниС Смкости записываСтся Π² Π²ΠΈΠ΄Π΅: -j/Ο‰*Π‘. Π›Π΅Π³ΠΊΠΎ Π·Π°ΠΌΠ΅Ρ‚ΠΈΡ‚ΡŒ: складывая Π²Π΅Π»ΠΈΡ‡ΠΈΠ½Ρ‹ ΠΏΠΎΡΠ»Π΅Π΄ΠΎΠ²Π°Ρ‚Π΅Π»ΡŒΠ½ΠΎΠ³ΠΎ соСдинСния, ΠΏΠΎΠ»ΡƒΡ‡ΠΈΠΌ Ρ„ΠΎΡ€ΠΌΡƒΠ»Ρƒ, Π² точности ΠΊΠ°ΠΊ для рСзисторов ΠΈ индуктивностСй Π±Ρ‹Π»ΠΎ ΠΏΡ€ΠΈ ΠΏΠ°Ρ€Π°Π»Π»Π΅Π»ΡŒΠ½ΠΎΠΌ. Для кондСнсаторов всС Π½Π°ΠΎΠ±ΠΎΡ€ΠΎΡ‚. ΠŸΡ€ΠΈ ΠΏΠ°Ρ€Π°Π»Π»Π΅Π»ΡŒΠ½ΠΎΠΌ Π²ΠΊΠ»ΡŽΡ‡Π΅Π½ΠΈΠΈ Π½ΠΎΠΌΠΈΠ½Π°Π»Ρ‹ ΡΠΊΠ»Π°Π΄Ρ‹Π²Π°ΡŽΡ‚ΡΡ, ΠΏΡ€ΠΈ ΠΏΠΎΡΠ»Π΅Π΄ΠΎΠ²Π°Ρ‚Π΅Π»ΡŒΠ½ΠΎΠΌ – ΡΡƒΠΌΠΌΠΈΡ€ΡƒΡŽΡ‚ΡΡ ΠΎΠ±Ρ€Π°Ρ‚Π½Ρ‹Π΅ Π²Π΅Π»ΠΈΡ‡ΠΈΠ½Ρ‹.

ВСзисы Π»Π΅Π³ΠΊΠΎ Ρ€Π°ΡΠΏΡ€ΠΎΡΡ‚Ρ€Π°Π½ΡΡŽΡ‚ΡΡ Π½Π° ΠΏΡ€ΠΎΠΈΠ·Π²ΠΎΠ»ΡŒΠ½Ρ‹Π΅ случаи. ПадСниС напряТСния Π½Π° Π΄Π²ΡƒΡ… ΠΎΡ‚ΠΊΡ€Ρ‹Ρ‚Ρ‹Ρ… ΠΊΡ€Π΅ΠΌΠ½ΠΈΠ΅Π²Ρ‹Ρ… Π΄ΠΈΠΎΠ΄Π°Ρ… Ρ€Π°Π²Π½ΠΎ суммС. На ΠΏΡ€Π°ΠΊΡ‚ΠΈΠΊΠ΅ составляСт 1 Π²ΠΎΠ»ΡŒΡ‚, Ρ‚ΠΎΡ‡Π½ΠΎΠ΅ Π·Π½Π°Ρ‡Π΅Π½ΠΈΠ΅ зависит ΠΎΡ‚ Ρ‚ΠΈΠΏΠ° ΠΏΠΎΠ»ΡƒΠΏΡ€ΠΎΠ²ΠΎΠ΄Π½ΠΈΠΊΠΎΠ²ΠΎΠ³ΠΎ элСмСнта, характСристик. Аналогичным ΠΎΠ±Ρ€Π°Π·ΠΎΠΌ Ρ€Π°ΡΡΠΌΠ°Ρ‚Ρ€ΠΈΠ²Π°ΡŽΡ‚ источники питания: ΠΏΡ€ΠΈ ΠΏΠΎΡΠ»Π΅Π΄ΠΎΠ²Π°Ρ‚Π΅Π»ΡŒΠ½ΠΎΠΌ Π²ΠΊΠ»ΡŽΡ‡Π΅Π½ΠΈΠΈ Π½ΠΎΠΌΠΈΠ½Π°Π»Ρ‹ ΡΠΊΠ»Π°Π΄Ρ‹Π²Π°ΡŽΡ‚ΡΡ. ΠŸΠ°Ρ€Π°Π»Π»Π΅Π»ΡŒΠ½ΠΎΠ΅ часто встрСчаСтся Π½Π° подстанциях, Π³Π΄Π΅ трансформаторы ставят рядком. НапряТСниС Π±ΡƒΠ΄Π΅Ρ‚ ΠΎΠ΄Π½ΠΎ (ΠΊΠΎΠ½Ρ‚Ρ€ΠΎΠ»ΠΈΡ€ΡƒΡŽΡ‚ΡΡ Π°ΠΏΠΏΠ°Ρ€Π°Ρ‚ΡƒΡ€ΠΎΠΉ), дСлятся ΠΌΠ΅ΠΆΠ΄Ρƒ вСтвями. ΠšΠΎΡΡ„Ρ„ΠΈΡ†ΠΈΠ΅Π½Ρ‚ трансформации строго Ρ€Π°Π²Π΅Π½, блокируя Π²ΠΎΠ·Π½ΠΈΠΊΠ½ΠΎΠ²Π΅Π½ΠΈΠ΅ Π½Π΅Π³Π°Ρ‚ΠΈΠ²Π½Ρ‹Ρ… эффСктов.

Π£ Π½Π΅ΠΊΠΎΡ‚ΠΎΡ€Ρ‹Ρ… Π²Ρ‹Π·Ρ‹Π²Π°Π΅Ρ‚ Π·Π°Ρ‚Ρ€ΡƒΠ΄Π½Π΅Π½ΠΈΠ΅ случай: Π΄Π²Π΅ Π±Π°Ρ‚Π°Ρ€Π΅ΠΉΠΊΠΈ Ρ€Π°Π·Π½ΠΎΠ³ΠΎ Π½ΠΎΠΌΠΈΠ½Π°Π»Π° Π²ΠΊΠ»ΡŽΡ‡Π΅Π½Ρ‹ ΠΏΠ°Ρ€Π°Π»Π»Π΅Π»ΡŒΠ½ΠΎ. Π‘Π»ΡƒΡ‡Π°ΠΉ описываСтся Π²Ρ‚ΠΎΡ€Ρ‹ΠΌ Π·Π°ΠΊΠΎΠ½ΠΎΠΌ ΠšΠΈΡ€Ρ…Π³ΠΎΡ„Π°, Π½ΠΈΠΊΠ°ΠΊΠΎΠΉ слоТности ΠΏΡ€Π΅Π΄ΡΡ‚Π°Π²ΠΈΡ‚ΡŒ Ρ„ΠΈΠ·ΠΈΠΊΡƒ Π½Π΅ ΠΌΠΎΠΆΠ΅Ρ‚. ΠŸΡ€ΠΈ нСравСнствС Π½ΠΎΠΌΠΈΠ½Π°Π»ΠΎΠ² Π΄Π²ΡƒΡ… источников бСрСтся срСднСС арифмСтичСскоС, Ссли ΠΏΡ€Π΅Π½Π΅Π±Ρ€Π΅Ρ‡ΡŒ Π²Π½ΡƒΡ‚Ρ€Π΅Π½Π½ΠΈΠΌ сопротивлСниСм ΠΎΠ±ΠΎΠΈΡ…. Π’ ΠΏΡ€ΠΎΡ‚ΠΈΠ²Π½ΠΎΠΌ случаС Ρ€Π΅ΡˆΠ°ΡŽΡ‚ΡΡ уравнСния ΠšΠΈΡ€Ρ…Π³ΠΎΡ„Π° для всСх ΠΊΠΎΠ½Ρ‚ΡƒΡ€ΠΎΠ². НСизвСстными Π±ΡƒΠ΄ΡƒΡ‚ Ρ‚ΠΎΠΊΠΈ (всСго Ρ‚Ρ€ΠΈ), ΠΎΠ±Ρ‰Π΅Π΅ количСство ΠΊΠΎΡ‚ΠΎΡ€Ρ‹Ρ… Ρ€Π°Π²Π½ΠΎ числу ΡƒΡ€Π°Π²Π½Π΅Π½ΠΈΠΉ. Для ΠΏΠΎΠ»Π½ΠΎΠ³ΠΎ понимания ΠΏΡ€ΠΈΠ²Π΅Π»ΠΈ рисунок.

ΠŸΡ€ΠΈΠΌΠ΅Ρ€ Ρ€Π΅ΡˆΠ΅Π½ΠΈΡ ΡƒΡ€Π°Π²Π½Π΅Π½ΠΈΠΉ ΠšΠΈΡ€Ρ…Π³ΠΎΡ„Π°

ΠŸΠΎΡΠΌΠΎΡ‚Ρ€ΠΈΠΌ ΠΈΠ·ΠΎΠ±Ρ€Π°ΠΆΠ΅Π½ΠΈΠ΅: ΠΏΠΎ ΡƒΡΠ»ΠΎΠ²ΠΈΡŽ Π·Π°Π΄Π°Ρ‡ΠΈ, источник Π•1 сильнСС, Π½Π΅ΠΆΠ΅Π»ΠΈ Π•2. НаправлСниС Ρ‚ΠΎΠΊΠΎΠ² Π² ΠΊΠΎΠ½Ρ‚ΡƒΡ€Π΅ Π±Π΅Ρ€Π΅ΠΌ ΠΈΠ· Π·Π΄Ρ€Π°Π²Ρ‹Ρ… сообраТСний. Но Ссли Π±Ρ‹ проставили Π½Π΅ΠΏΡ€Π°Π²ΠΈΠ»ΡŒΠ½ΠΎ, послС Ρ€Π΅ΡˆΠ΅Π½ΠΈΡ Π·Π°Π΄Π°Ρ‡ΠΈ ΠΎΠ΄ΠΈΠ½ получился Π±Ρ‹ с ΠΎΡ‚Ρ€ΠΈΡ†Π°Ρ‚Π΅Π»ΡŒΠ½Ρ‹ΠΌ Π·Π½Π°ΠΊΠΎΠΌ. Π‘Π»Π΅Π΄ΠΎΠ²Π°Π»ΠΎ Ρ‚ΠΎΠ³Π΄Π° ΠΈΠ·ΠΌΠ΅Π½ΠΈΡ‚ΡŒ Π½Π°ΠΏΡ€Π°Π²Π»Π΅Π½ΠΈΠ΅. ΠžΡ‡Π΅Π²ΠΈΠ΄Π½ΠΎ, Π²ΠΎ внСшнСй Ρ†Π΅ΠΏΠΈ Ρ‚ΠΎΠΊ Ρ‚Π΅Ρ‡Π΅Ρ‚, ΠΊΠ°ΠΊ ΠΏΠΎΠΊΠ°Π·Π°Π½ΠΎ Π½Π° рисункС. БоставляСм уравнСния ΠšΠΈΡ€Ρ…Π³ΠΎΡ„Π° для Ρ‚Ρ€Π΅Ρ… ΠΊΠΎΠ½Ρ‚ΡƒΡ€ΠΎΠ², Π²ΠΎΡ‚ Ρ‡Ρ‚ΠΎ слСдуСт:

  1. Π Π°Π±ΠΎΡ‚Π° ΠΏΠ΅Ρ€Π²ΠΎΠ³ΠΎ (сильного) источника тратится Π½Π° созданиС Ρ‚ΠΎΠΊΠ° Π²ΠΎ внСшнСй Ρ†Π΅ΠΏΠΈ, ΠΏΡ€Π΅ΠΎΠ΄ΠΎΠ»Π΅Π½ΠΈΠ΅ слабости сосСда (Ρ‚ΠΎΠΊ I2).
  2. Π’Ρ‚ΠΎΡ€ΠΎΠΉ источник Π½Π΅ ΡΠΎΠ²Π΅Ρ€ΡˆΠ°Π΅Ρ‚ ΠΏΠΎΠ»Π΅Π·Π½ΠΎΠΉ Ρ€Π°Π±ΠΎΡ‚Ρ‹ Π² Π½Π°Π³Ρ€ΡƒΠ·ΠΊΠ΅, борСтся с ΠΏΠ΅Ρ€Π²Ρ‹ΠΌ. Π˜Π½Π°Ρ‡Π΅ Π½Π΅ скаТСшь.

Π’ΠΊΠ»ΡŽΡ‡Π΅Π½ΠΈΠ΅ Π±Π°Ρ‚Π°Ρ€Π΅Π΅ΠΊ Ρ€Π°Π·Π½ΠΎΠ³ΠΎ Π½ΠΎΠΌΠΈΠ½Π°Π»Π° ΠΏΠ°Ρ€Π°Π»Π»Π΅Π»ΡŒΠ½ΠΎ являСтся бСзусловно Π²Ρ€Π΅Π΄Π½Ρ‹ΠΌ. Π§Ρ‚ΠΎ Π½Π°Π±Π»ΡŽΠ΄Π°Π΅Ρ‚ΡΡ Π½Π° подстанции ΠΏΡ€ΠΈ использовании трансформаторов с Ρ€Π°Π·Π½Ρ‹ΠΌ ΠΏΠ΅Ρ€Π΅Π΄Π°Ρ‚ΠΎΡ‡Π½Ρ‹ΠΌ коэффициСнтом. Π£Ρ€Π°Π²Π½ΠΈΡ‚Π΅Π»ΡŒΠ½Ρ‹Π΅ Ρ‚ΠΎΠΊΠΈ Π½Π΅ Π²Ρ‹ΠΏΠΎΠ»Π½ΡΡŽΡ‚ Π½ΠΈΠΊΠ°ΠΊΠΎΠΉ ΠΏΠΎΠ»Π΅Π·Π½ΠΎΠΉ Ρ€Π°Π±ΠΎΡ‚Ρ‹. Π’ΠΊΠ»ΡŽΡ‡Π΅Π½Π½Ρ‹Π΅ ΠΏΠ°Ρ€Π°Π»Π»Π΅Π»ΡŒΠ½ΠΎ Ρ€Π°Π·Π½Ρ‹Π΅ Π±Π°Ρ‚Π°Ρ€Π΅ΠΉΠΊΠΈ Π½Π°Ρ‡Π½ΡƒΡ‚ эффСктивно Ρ„ΡƒΠ½ΠΊΡ†ΠΈΠΎΠ½ΠΈΡ€ΠΎΠ²Π°Ρ‚ΡŒ, ΠΊΠΎΠ³Π΄Π° сильная просядСт Π΄ΠΎ уровня слабой.

экв. Π‘Ρ…Π΅ΠΌΡ‹

ΠŸΠ°Ρ€Π°Π»Π»Π΅Π»ΡŒΠ½Π°Ρ комбинация рСзисторов ΠΏΠΎΠΊΠ°Π·Π°Π½Π° Π½ΠΈΠΆΠ΅. Они Ρ‚Π°ΠΊΠΆΠ΅ ΠΌΠΎΠ³ΡƒΡ‚ Π±Ρ‹Ρ‚ΡŒ сводится ΠΊ Π΅Π΄ΠΈΠ½ΠΎΠΌΡƒ ΡΠΎΠΏΡ€ΠΎΡ‚ΠΈΠ²Π»Π΅Π½ΠΈΡŽ. Однако ΡƒΡ€Π°Π²Π½Π΅Π½ΠΈΠ΅ для нахоТдСния эквивалСнтноС сопротивлСниС отличаСтся ΠΎΡ‚ ΠΏΠΎΡΠ»Π΅Π΄ΠΎΠ²Π°Ρ‚Π΅Π»ΡŒΠ½ΠΎΠ³ΠΎ

Π­ΠΊΠ²ΠΈΠ²Π°Π»Π΅Π½Ρ‚Π½ΠΎΠ΅ сопротивлСниС всСгда являСтся ΠΎΠ±Ρ€Π°Ρ‚Π½ΠΎΠΉ Π²Π΅Π»ΠΈΡ‡ΠΈΠ½ΠΎΠΉ ΠΎΠ±Ρ€Π°Ρ‚Π½ΠΎΠ³ΠΎ сопротивлСния. ΠΎΡ‚Π΄Π΅Π»ΡŒΠ½Ρ‹Ρ… рСзисторов N.N {\ left ({\ frac {1} {{{R_k}}}} \ right)}}} \]

Π₯отя это ΡƒΡ€Π°Π²Π½Π΅Π½ΠΈΠ΅ Π½Π΅ Ρ‚Π°ΠΊ ΡƒΠΆ слоТно, ΠΎΠ½ΠΎ ΡƒΡ€ΠΎΠ΄Π»ΠΈΠ²Π΅Π΅, Ρ‡Π΅ΠΌ для случай сСрии. Π›ΡŽΠ΄ΠΈ Ρ€Π°Π·Ρ€Π°Π±ΠΎΡ‚Π°Π»ΠΈ нСсколько способов ΡΠ΄Π΅Π»Π°Ρ‚ΡŒ ΡƒΡ€Π°Π²Π½Π΅Π½ΠΈΠ΅ Π½Π΅ΠΌΠ½ΠΎΠ³ΠΎ ΠΏΡ€ΠΎΡ‰Π΅. ΠŸΠ΅Ρ€Π²Ρ‹ΠΉ ΠΌΠ΅Ρ‚ΠΎΠ΄ примСняСтся Ρ‚ΠΎΠ»ΡŒΠΊΠΎ Ρ‚ΠΎΠ³Π΄Π°, ΠΊΠΎΠ³Π΄Π° Π΅ΡΡ‚ΡŒ Π΄Π²Π° рСзистора ΠΏΠ°Ρ€Π°Π»Π»Π΅Π»ΡŒΠ½ΠΎ.Π’ΠΎΠ³Π΄Π° эквивалСнтноС сопротивлСниС Ρ€Π°Π²Π½ΠΎ просто ΠΏΡ€ΠΎΠΈΠ·Π²Π΅Π΄Π΅Π½ΠΈΠ΅ Π½Π° сумму сопротивлСний. Но Π±ΡƒΠ΄ΡŒ остороТСн, это Ρ€Π°Π±ΠΎΡ‚Π°Π΅Ρ‚ Ρ‚ΠΎΠ»ΡŒΠΊΠΎ для Π΄Π²ΡƒΡ… рСзисторов. Π€ΠΎΡ€ΠΌΡƒΠ»Π° Π½Π΅Π²Π΅Ρ€Π½Π° для Ρ‚Ρ€Π΅Ρ… ΠΈΠ»ΠΈ большС рСзисторов ΠΏΠ°Ρ€Π°Π»Π»Π΅Π»ΡŒΠ½ΠΎ \ [\ begin {array} {l} {R_A} = \ frac {1} {{\ frac {1} {{R1}} + \ frac {1} {{R2}}}} = \ frac {{R1 \ cdot R2}} {{R1 + R2 }} \\ {R_A} = \ frac {1} {{\ frac {1} {{R1}} + \ frac {1} {{R2}} + \ frac {1} {{R3}}}}} \ ne \ frac { {R1 \ cdot R2 \ cdot R3}} {{R1 + R2 + R3}} \\ {R_A} = \ frac {1} {{\ frac {1} {{R1}} + \ frac {1} {{R2}} + \ frac {1} {{R3}} + \ frac {1} { {R4}}}} \ ne \ frac {{R1 \ cdot R2 \ cdot R3 \ cdot R4}} {{R1 + R2 + R3 + R4}} \ ΠΊΠΎΠ½Π΅Ρ† {массив} \]

Π’Ρ‚ΠΎΡ€ΠΎΠΉ ΠΌΠ΅Ρ‚ΠΎΠ΄ Ρ€Π°Π±ΠΎΡ‚Ρ‹ с ΠΏΠ°Ρ€Π°Π»Π»Π΅Π»ΡŒΠ½Ρ‹ΠΌΠΈ рСзисторами Π·Π°ΠΊΠ»ΡŽΡ‡Π°Π΅Ρ‚ΡΡ Π² ΠΈΠ·ΠΌΠ΅Ρ€Π΅Π½ΠΈΠΈ ΠΈΡ… ΠΏΡ€ΠΎΠ²ΠΎΠ΄ΠΈΠΌΠΎΡΡ‚ΡŒ, Π° Π½Π΅ сопротивлСниС. N {{G_k}} \]

рСзисторов, Π²ΠΊΠ»ΡŽΡ‡Π΅Π½Π½Ρ‹Ρ… ΠΏΠ°Ρ€Π°Π»Π»Π΅Π»ΡŒΠ½ΠΎ | Π€ΠΎΡ€ΠΌΡƒΠ»Π° эквивалСнтного сопротивлСния

Π’Π²Π΅Π΄Π΅Π½ΠΈΠ΅

Π”Π²Π° рСзистора ΡΡ‡ΠΈΡ‚Π°ΡŽΡ‚ΡΡ ΠΏΠΎΠ΄ΠΊΠ»ΡŽΡ‡Π΅Π½Π½Ρ‹ΠΌΠΈ ΠΏΠ°Ρ€Π°Π»Π»Π΅Π»ΡŒΠ½ΠΎ, Ссли ΠΎΠ±Π° Π²Ρ‹Π²ΠΎΠ΄Π° рСзистора ΠΏΠΎΠ΄ΠΊΠ»ΡŽΡ‡Π΅Π½Ρ‹ ΠΊ ΠΊΠ°ΠΆΠ΄ΠΎΠΌΡƒ ΡΠΎΠΎΡ‚Π²Π΅Ρ‚ΡΡ‚Π²ΡƒΡŽΡ‰Π΅ΠΌΡƒ Π²Ρ‹Π²ΠΎΠ΄Ρƒ Π΄Ρ€ΡƒΠ³ΠΎΠ³ΠΎ рСзистора.Π’ сСти ΠΏΠ°Ρ€Π°Π»Π»Π΅Π»ΡŒΠ½Ρ‹Ρ… рСзисторов Ρ‚ΠΎΠΊ ΠΌΠΎΠΆΠ΅Ρ‚ ΠΏΡ€ΠΎΡ…ΠΎΠ΄ΠΈΡ‚ΡŒ ΠΏΠΎ нСскольким путям, Π² ΠΎΡ‚Π»ΠΈΡ‡ΠΈΠ΅ ΠΎΡ‚ сСти с ΠΏΠΎΡΠ»Π΅Π΄ΠΎΠ²Π°Ρ‚Π΅Π»ΡŒΠ½Ρ‹ΠΌΠΈ рСзисторами, ΠΏΠΎΡΠΊΠΎΠ»ΡŒΠΊΡƒ сущСствуСт нСсколько ΠΏΡƒΡ‚Π΅ΠΉ для прохоТдСния Ρ‚ΠΎΠΊΠ°. Π‘Π»Π΅Π΄ΠΎΠ²Π°Ρ‚Π΅Π»ΡŒΠ½ΠΎ, ΠΏΠ°Ρ€Π°Π»Π»Π΅Π»ΡŒΠ½Ρ‹Π΅ рСзистивныС Ρ†Π΅ΠΏΠΈ ΡΠ²Π»ΡΡŽΡ‚ΡΡ дСлитСлями Ρ‚ΠΎΠΊΠ°.

ΠŸΠ°Ρ€Π°Π»Π»Π΅Π»ΡŒΠ½Ρ‹Π΅ рСзисторы

Если Π΄Π²Π° ΠΈΠ»ΠΈ Π±ΠΎΠ»Π΅Π΅ рСзистора ΠΏΠΎΠ΄ΠΊΠ»ΡŽΡ‡Π΅Π½Ρ‹ ΠΏΠ°Ρ€Π°Π»Π»Π΅Π»ΡŒΠ½ΠΎ, Ρ‚ΠΎ Ρ€Π°Π·Π½ΠΎΡΡ‚ΡŒ ΠΏΠΎΡ‚Π΅Π½Ρ†ΠΈΠ°Π»ΠΎΠ² Π½Π° ΠΊΠ°ΠΆΠ΄ΠΎΠΌ рСзисторС ΠΎΠ΄ΠΈΠ½Π°ΠΊΠΎΠ²Π°. ΠŸΡ€ΠΈ ΠΏΠ°Ρ€Π°Π»Π»Π΅Π»ΡŒΠ½ΠΎΠΌ соСдинСнии рСзисторы ΠΏΠΎΠ΄ΠΊΠ»ΡŽΡ‡Π°ΡŽΡ‚ΡΡ ΠΊ ΠΎΠ΄Π½ΠΈΠΌ ΠΈ Ρ‚Π΅ΠΌ ΠΆΠ΅ ΡƒΠ·Π»Π°ΠΌ. Π­Ρ‚ΠΎ ΠΌΠΎΠΆΠ½ΠΎ ΠΎΠΏΡ€Π΅Π΄Π΅Π»ΠΈΡ‚ΡŒ ΠΏΠΎ Π½Π°Π»ΠΈΡ‡ΠΈΡŽ Π±ΠΎΠ»Π΅Π΅ Ρ‡Π΅ΠΌ ΠΎΠ΄Π½ΠΎΠ³ΠΎ ΠΏΡƒΡ‚ΠΈ прохоТдСния Ρ‚ΠΎΠΊΠ°.НапримСр, привСдСнная Π½ΠΈΠΆΠ΅ схСма прСдставляСт собой ΠΏΠ°Ρ€Π°Π»Π»Π΅Π»ΡŒΠ½ΠΎΠ΅ соСдинСниС рСзисторов. Π Π°Π·Π½ΠΎΡΡ‚ΡŒ ΠΏΠΎΡ‚Π΅Π½Ρ†ΠΈΠ°Π»ΠΎΠ² Π½Π° рСзисторС R1 такая ΠΆΠ΅, ΠΊΠ°ΠΊ ΠΈ Π½Π° рСзисторС R2, ΠΊΠΎΡ‚ΠΎΡ€Ρ‹ΠΉ Ρ€Π°Π²Π΅Π½ ΠΏΠΎΡ‚Π΅Π½Ρ†ΠΈΠ°Π»Ρƒ питания V AB .

Если V AB — это ΠΏΠΎΠ΄Π°Π²Π°Π΅ΠΌΡ‹ΠΉ ΠΏΠΎΡ‚Π΅Π½Ρ†ΠΈΠ°Π», Ρ‚ΠΎ

V R1 = V R2 = V AB

Π’ ΡΠ»Π΅Π΄ΡƒΡŽΡ‰Π΅ΠΉ схСмС рСзисторы R1, R2 ΠΈ R3 ΠΏΠΎΠ΄ΠΊΠ»ΡŽΡ‡Π΅Π½Ρ‹ ΠΏΠ°Ρ€Π°Π»Π»Π΅Π»ΡŒΠ½ΠΎ комбинация.

Π—Π΄Π΅ΡΡŒ ΠΏΠΎΡ‚Π΅Π½Ρ†ΠΈΠ°Π» питания V AB ΠΌΠ΅ΠΆΠ΄Ρƒ Ρ‚ΠΎΡ‡ΠΊΠ°ΠΌΠΈ A ΠΈ B.ΠŸΠΎΡΠΊΠΎΠ»ΡŒΠΊΡƒ рСзисторы R1, R2 ΠΈ R3 соСдинСны ΠΏΠ°Ρ€Π°Π»Π»Π΅Π»ΡŒΠ½ΠΎ, Ρ€Π°Π·Π½ΠΎΡΡ‚ΡŒ ΠΏΠΎΡ‚Π΅Π½Ρ†ΠΈΠ°Π»ΠΎΠ² Π½Π° ΠΊΠ°ΠΆΠ΄ΠΎΠΌ рСзисторС такая ΠΆΠ΅, ΠΊΠ°ΠΊ Ρƒ источника питания. Π‘Π»Π΅Π΄ΠΎΠ²Π°Ρ‚Π΅Π»ΡŒΠ½ΠΎ, V AB = V R1 = V R2 = V R3 .

Π“Π΄Π΅

Π’ R1 — ΠΏΠΎΡ‚Π΅Π½Ρ†ΠΈΠ°Π» Π½Π° рСзисторС R1.

Π’ R2 — ΠΏΠΎΡ‚Π΅Π½Ρ†ΠΈΠ°Π» Π½Π° рСзисторС R2.

Π’ R3 — ΠΏΠΎΡ‚Π΅Π½Ρ†ΠΈΠ°Π» Π½Π° рСзисторС R3.

Но Ρ‚ΠΎΠΊ, ΠΏΡ€ΠΎΡ‚Π΅ΠΊΠ°ΡŽΡ‰ΠΈΠΉ Ρ‡Π΅Ρ€Π΅Π· эти Ρ‚Ρ€ΠΈ рСзистора, Ρ€Π°Π·Π½Ρ‹ΠΉ.Если I — это Ρ‚ΠΎΠΊ, ΠΏΠΎΠΊΠΈΠ΄Π°ΡŽΡ‰ΠΈΠΉ ΡƒΠ·Π΅Π» A, Ρ‚ΠΎ ΠΎΠ½ ΠΈΠΌΠ΅Π΅Ρ‚ Ρ‚Ρ€ΠΈ ΠΏΡƒΡ‚ΠΈ, Ρ‡Ρ‚ΠΎΠ±Ρ‹ Π΄ΠΎΡΡ‚ΠΈΡ‡ΡŒ ΡƒΠ·Π»Π° B. Π’ΠΎΠΊ, ΠΏΡ€ΠΎΡ‚Π΅ΠΊΠ°ΡŽΡ‰ΠΈΠΉ Ρ‡Π΅Ρ€Π΅Π· ΠΊΠ°ΠΆΠ΄Ρ‹ΠΉ рСзистор, зависит ΠΎΡ‚ Π΅Π³ΠΎ сопротивлСния. Π‘Π»Π΅Π΄ΠΎΠ²Π°Ρ‚Π΅Π»ΡŒΠ½ΠΎ, Π² случаС ΠΏΠ°Ρ€Π°Π»Π»Π΅Π»ΡŒΠ½Ρ‹Ρ… рСзистивных Ρ†Π΅ΠΏΠ΅ΠΉ Ρ‚ΠΎΠΊ Π²ΠΎ всСх рСзисторах Π½Π΅ΠΎΠ΄ΠΈΠ½Π°ΠΊΠΎΠ². Если I1 — это Ρ‚ΠΎΠΊ, ΠΏΡ€ΠΎΡ‚Π΅ΠΊΠ°ΡŽΡ‰ΠΈΠΉ Ρ‡Π΅Ρ€Π΅Π· рСзистор R1, I2 — это Ρ‚ΠΎΠΊ, ΠΏΡ€ΠΎΡ‚Π΅ΠΊΠ°ΡŽΡ‰ΠΈΠΉ Ρ‡Π΅Ρ€Π΅Π· рСзистор R2, Π° I3 — это Ρ‚ΠΎΠΊ, ΠΏΡ€ΠΎΡ‚Π΅ΠΊΠ°ΡŽΡ‰ΠΈΠΉ Ρ‡Π΅Ρ€Π΅Π· рСзистор R3, Ρ‚ΠΎΠ³Π΄Π° Ρ‚ΠΎΠΊΠΈ I, I1, I2 ΠΈ I3 ΠΌΠΎΠ³ΡƒΡ‚ Π±Ρ‹Ρ‚ΡŒ связаны с ΠΏΠΎΠΌΠΎΡ‰ΡŒΡŽ Π·Π°ΠΊΠΎΠ½Π° Ρ‚ΠΎΠΊΠ° ΠšΠΈΡ€Ρ…Π³ΠΎΡ„Π°. . Богласно Π·Π°ΠΊΠΎΠ½Ρƒ ΠšΠΈΡ€Ρ…Π³ΠΎΡ„Π° ΠΎ Ρ‚ΠΎΠΊΠ°Ρ…, «сумма Ρ‚ΠΎΠΊΠΎΠ², входящих Π² ΡƒΠ·Π΅Π», Ρ€Π°Π²Π½Π° суммС Ρ‚ΠΎΠΊΠΎΠ², выходящих ΠΈΠ· ΡƒΠ·Π»Π°.”

Π‘Π»Π΅Π΄ΠΎΠ²Π°Ρ‚Π΅Π»ΡŒΠ½ΠΎ,

I = I1 + I2 + I3.

Π€ΠΎΡ€ΠΌΡƒΠ»Π° эквивалСнтного сопротивлСния

Π›ΡŽΠ±ΠΎΠ΅ количСство рСзисторов, Π²ΠΊΠ»ΡŽΡ‡Π΅Π½Π½Ρ‹Ρ… Π² ΠΏΠ°Ρ€Π°Π»Π»Π΅Π»ΡŒΠ½ΡƒΡŽ ΠΊΠΎΠΌΠ±ΠΈΠ½Π°Ρ†ΠΈΡŽ, ΠΌΠΎΠΆΠ½ΠΎ Π·Π°ΠΌΠ΅Π½ΠΈΡ‚ΡŒ ΠΎΠ΄Π½ΠΈΠΌ рСзистором с сопротивлСниСм, Ρ€Π°Π²Π½Ρ‹ΠΌ эквивалСнтному ΡΠΎΠΏΡ€ΠΎΡ‚ΠΈΠ²Π»Π΅Π½ΠΈΡŽ рСзисторов ΠΏΠ°Ρ€Π°Π»Π»Π΅Π»ΡŒΠ½ΠΎΠΉ ΠΊΠΎΠΌΠ±ΠΈΠ½Π°Ρ†ΠΈΠΈ.

Π‘Ρ‹Π»ΠΎ установлСно, Ρ‡Ρ‚ΠΎ напряТСниС Π½Π° ΠΊΠ°ΠΆΠ΄ΠΎΠΌ рСзисторС Π² ΠΏΠ°Ρ€Π°Π»Π»Π΅Π»ΡŒΠ½ΠΎΠΉ ΠΊΠΎΠΌΠ±ΠΈΠ½Π°Ρ†ΠΈΠΈ ΠΎΠ΄ΠΈΠ½Π°ΠΊΠΎΠ²ΠΎ, Π° ΠΎΠ±Ρ‰ΠΈΠΉ Ρ‚ΠΎΠΊ Ρ€Π°Π²Π΅Π½ суммС ΠΎΡ‚Π΄Π΅Π»ΡŒΠ½Ρ‹Ρ… Ρ‚ΠΎΠΊΠΎΠ². Рассмотрим ΡΠ»Π΅Π΄ΡƒΡŽΡ‰ΡƒΡŽ схСму.

Π—Π΄Π΅ΡΡŒ I = I1 + I2 + I3

I1 = V / R1

I2 = V / R2

I3 = V / R3

Если R T — ΠΏΠΎΠ»Π½ΠΎΠ΅ сопротивлСниС Ρ†Π΅ΠΏΠΈ, Ρ‚ΠΎ

I = V / R T

Π‘Π»Π΅Π΄ΠΎΠ²Π°Ρ‚Π΅Π»ΡŒΠ½ΠΎ, V / R T = V / R 1 + V / R 2 + V / R 3

1 / R T = 1 / R 1 + 1 / R 2 + 1 / R 3

Если R eq являСтся эквивалСнтным сопротивлСниСм Ρ†Π΅ΠΏΠΈ, Ρ‚ΠΎ ΠΎΠ½ΠΎ рассчитываСтся ΠΏΡƒΡ‚Π΅ΠΌ слоТСния ΠΎΠ±Ρ€Π°Ρ‚Π½Ρ‹Ρ… Π·Π½Π°Ρ‡Π΅Π½ΠΈΠΉ ΠΎΡ‚Π΄Π΅Π»ΡŒΠ½Ρ‹Ρ… сопротивлСний (1 / R).ΠžΠ±Ρ€Π°Ρ‚Π½Π°Ρ Π²Π΅Π»ΠΈΡ‡ΠΈΠ½Π° этой алгСбраичСской суммы даст эквивалСнтноС сопротивлСниС. Π£Ρ€Π°Π²Π½Π΅Π½ΠΈΠ΅ эквивалСнтного сопротивлСния R eq ΠΏΠΎΠΊΠ°Π·Π°Π½ΠΎ Π½ΠΈΠΆΠ΅ для ΠΏΠ°Ρ€Π°Π»Π»Π΅Π»ΡŒΠ½ΠΎΠΉ рСзистивной Ρ†Π΅ΠΏΠΈ ΠΈΠ· n рСзисторов.

(1 / R экв ) = (1 / R1) + (1 / R2) + (1 / R3) + ……… + (1 / Rn)

Из ΠΏΡ€ΠΈΠ²Π΅Π΄Π΅Π½Π½ΠΎΠ³ΠΎ Π²Ρ‹ΡˆΠ΅ уравнСния ΠΌΠΎΠΆΠ½ΠΎ ΡΠ΄Π΅Π»Π°Ρ‚ΡŒ Π²Ρ‹Π²ΠΎΠ΄, Ρ‡Ρ‚ΠΎ эквивалСнтноС сопротивлСниС рСзисторов, соСдинСнных ΠΏΠ°Ρ€Π°Π»Π»Π΅Π»ΡŒΠ½ΠΎ, всСгда мСньшС, Ρ‡Π΅ΠΌ сопротивлСниС самого малСнького рСзистора.

Если Π΄Π²Π° рСзистора ΠΏΠΎΠ΄ΠΊΠ»ΡŽΡ‡Π΅Π½Ρ‹ ΠΏΠ°Ρ€Π°Π»Π»Π΅Π»ΡŒΠ½ΠΎ, Ρ‚ΠΎ эквивалСнтноС сопротивлСниС Ρ€Π°Π²Π½ΠΎ

(1 / R eq ) = (1 / R1) + (1 / R2)

R EQ = R 1 * R 1 / (R 1 + R 2 )

Если Π΄Π²Π° рСзистора Ρ€Π°Π²Π½ΠΎΠ³ΠΎ сопротивлСния R соСдинСны ΠΏΠ°Ρ€Π°Π»Π»Π΅Π»ΡŒΠ½ΠΎ, Ρ‚ΠΎ эквивалСнтноС сопротивлСниС ΠΊΠΎΠΌΠ±ΠΈΠ½Π°Ρ†ΠΈΠΈ Π±ΡƒΠ΄Π΅Ρ‚ R / 2.

Аналогично, Ссли Ρ‚Ρ€ΠΈ рСзистора Ρ€Π°Π²Π½ΠΎΠ³ΠΎ сопротивлСния R соСдинСны Π² ΠΏΠ°Ρ€Π°Π»Π»Π΅Π»ΡŒΠ½ΠΎΠΌ соСдинСнии, Ρ‚ΠΎΠ³Π΄Π° эквивалСнтноС сопротивлСниС ΠΊΠΎΠΌΠ±ΠΈΠ½Π°Ρ†ΠΈΠΈ Π±ΡƒΠ΄Π΅Ρ‚ R / 3.

ΠŸΠ°Ρ€Π°Π»Π»Π΅Π»ΡŒΠ½ΠΎΠ΅ соСдинСниС рСзисторов Π΄Π°Π΅Ρ‚ Π·Π½Π°Ρ‡Π΅Π½ΠΈΠ΅ проводимости. ΠŸΡ€ΠΎΠ²ΠΎΠ΄ΠΈΠΌΠΎΡΡ‚ΡŒ — это Π²Π΅Π»ΠΈΡ‡ΠΈΠ½Π°, обратная ΡΠΎΠΏΡ€ΠΎΡ‚ΠΈΠ²Π»Π΅Π½ΠΈΡŽ. ΠžΠ±Ρ‹Ρ‡Π½ΠΎ ΠΎΠ½ обозначаСтся символом G. Π•Π΄ΠΈΠ½ΠΈΡ†Ρ‹ проводимости — БимСнс, прСдставлСнныС символом S. Π Π°Π½Π΅Π΅ Π΅Π΄ΠΈΠ½ΠΈΡ†Π°ΠΌΠΈ проводимости Π±Ρ‹Π»ΠΈ Mho (℧), Ρ‡Ρ‚ΠΎ ΠΎΠ·Π½Π°Ρ‡Π°Π΅Ρ‚ ΠΎΠ±Ρ€Π°Ρ‚Π½ΠΎΠ΅ написаниС Ом, Π° символ — ΠΏΠ΅Ρ€Π΅Π²Π΅Ρ€Π½ΡƒΡ‚ΠΎΠ΅ прСдставлСниС Ξ©.

НСсмотря Π½Π° Ρ‚ΠΎ, Ρ‡Ρ‚ΠΎ рСзисторы ΠΏΠ°Ρ€Π°Π»Π»Π΅Π»ΡŒΠ½ΠΎ ΠΏΠΎΠ΄ΠΊΠ»ΡŽΡ‡Π΅Π½Ρ‹ ΠΌΠ΅ΠΆΠ΄Ρƒ двумя ΡƒΠ·Π»Π°ΠΌΠΈ, прСдставлСниС этого ΠΏΠΎΠ΄ΠΊΠ»ΡŽΡ‡Π΅Π½ΠΈΡ ΠΌΠΎΠΆΠ΅Ρ‚ ΠΈΠΌΠ΅Ρ‚ΡŒ Π»ΡŽΠ±ΡƒΡŽ ΠΈΠ· ΡΠ»Π΅Π΄ΡƒΡŽΡ‰ΠΈΡ… Ρ„ΠΎΡ€ΠΌ.

ВсС Π²Ρ‹ΡˆΠ΅ΡƒΠΏΠΎΠΌΡΠ½ΡƒΡ‚Ρ‹Π΅ ΠΊΠΎΠΌΠ±ΠΈΠ½Π°Ρ†ΠΈΠΈ ΠΏΡ€Π΅Π΄ΡΡ‚Π°Π²Π»ΡΡŽΡ‚ собой ΠΏΠ°Ρ€Π°Π»Π»Π΅Π»ΡŒΠ½Ρ‹Π΅ рСзистивныС схСмы, ΠΈ всС ΠΏΡ€Π°Π²ΠΈΠ»Π° ΠΏΠ°Ρ€Π°Π»Π»Π΅Π»ΡŒΠ½Ρ‹Ρ… рСзисторов ΠΏΡ€ΠΈΠΌΠ΅Π½ΠΈΠΌΡ‹ ΠΈ ΠΊ Π²Ρ‹ΡˆΠ΅ΡƒΠΏΠΎΠΌΡΠ½ΡƒΡ‚Ρ‹ΠΌ комбинациям.

РасчСт Ρ‚ΠΎΠΊΠ°

Π’ΠΎΠΊ Π² ΠΊΠ°ΠΆΠ΄ΠΎΠΉ Π²Π΅Ρ‚Π²ΠΈ ΠΏΠ°Ρ€Π°Π»Π»Π΅Π»ΡŒΠ½ΠΎΠΉ рСзистивной Ρ†Π΅ΠΏΠΈ отличаСтся ΠΎΡ‚ Π΄Ρ€ΡƒΠ³ΠΎΠΉ. ΠŸΠΎΡΠΊΠΎΠ»ΡŒΠΊΡƒ напряТСниС Π½Π° ΠΊΠ°ΠΆΠ΄ΠΎΠΌ рСзисторС ΠΎΠ΄ΠΈΠ½Π°ΠΊΠΎΠ²ΠΎ, Ρ‚ΠΎΠΊ, ΠΏΡ€ΠΎΡ‚Π΅ΠΊΠ°ΡŽΡ‰ΠΈΠΉ Ρ‡Π΅Ρ€Π΅Π· ΠΊΠ°ΠΆΠ΄Ρ‹ΠΉ рСзистор, зависит ΠΎΡ‚ сопротивлСния этого рСзистора.Π‘Π»Π΅Π΄ΠΎΠ²Π°Ρ‚Π΅Π»ΡŒΠ½ΠΎ, Ссли Π·Π½Π°Ρ‡Π΅Π½ΠΈΠ΅ сопротивлСния Π² ΠΎΠ΄Π½ΠΎΠΉ Π²Π΅Ρ‚Π²ΠΈ отличаСтся ΠΎΡ‚ Π΄Ρ€ΡƒΠ³ΠΎΠΉ, Ρ‚ΠΎΠ³Π΄Π° Ρ‚ΠΎΠΊ Π² этих вСтвях Π±ΡƒΠ΄Π΅Ρ‚ Π΄Ρ€ΡƒΠ³ΠΈΠΌ. Π’Π΅Π»ΠΈΡ‡ΠΈΠ½Ρƒ этого Ρ‚ΠΎΠΊΠ° ΠΌΠΎΠΆΠ½ΠΎ ΠΎΠΏΡ€Π΅Π΄Π΅Π»ΠΈΡ‚ΡŒ с ΠΏΠΎΠΌΠΎΡ‰ΡŒΡŽ Π·Π°ΠΊΠΎΠ½Π° Ома.

Рассмотрим ΠΏΠ°Ρ€Π°Π»Π»Π΅Π»ΡŒΠ½ΡƒΡŽ ΡΠ΅Ρ‚ΡŒ ΠΈΠ· Π΄Π²ΡƒΡ… рСзисторов с напряТСниСм питания V ΠΌΠ΅ΠΆΠ΄Ρƒ двумя Ρ‚ΠΎΡ‡ΠΊΠ°ΠΌΠΈ A ΠΈ B.

ΠŸΡƒΡΡ‚ΡŒ I Π±ΡƒΠ΄Π΅Ρ‚ ΠΏΠΎΠ»Π½Ρ‹ΠΌ Ρ‚ΠΎΠΊΠΎΠΌ Π² ΡΠ»Π΅Π΄ΡƒΡŽΡ‰Π΅ΠΉ Ρ†Π΅ΠΏΠΈ.

ΠŸΡƒΡΡ‚ΡŒ Ρ‚ΠΎΠΊ, ΠΏΡ€ΠΎΡ‚Π΅ΠΊΠ°ΡŽΡ‰ΠΈΠΉ Ρ‡Π΅Ρ€Π΅Π· рСзистор R 1 , Ρ€Π°Π²Π΅Π½ I R1 , Π° Ρ‚ΠΎΠΊ, ΠΏΡ€ΠΎΡ‚Π΅ΠΊΠ°ΡŽΡ‰ΠΈΠΉ Ρ‡Π΅Ρ€Π΅Π· рСзистор R 2 , Ρ€Π°Π²Π΅Π½ I R2 .

Π’ΠΎΠ³Π΄Π°, согласно Π·Π°ΠΊΠΎΠ½Ρƒ ΠšΠΈΡ€Ρ…Π³ΠΎΡ„Π° ΠΎ Ρ‚ΠΎΠΊΠ°Ρ…, Β«ΠΏΠΎΠ»Π½Ρ‹ΠΉ Ρ‚ΠΎΠΊ, входящий Π² Ρ†Π΅ΠΏΡŒ, Ρ€Π°Π²Π΅Π½ Ρ‚ΠΎΠΊΡƒ Π½Π° Π²Ρ‹Ρ…ΠΎΠ΄Π΅ ΠΈΠ· Ρ†Π΅ΠΏΠΈΒ».

Если I T — ΠΎΠ±Ρ‰ΠΈΠΉ Ρ‚ΠΎΠΊ, Ρ‚ΠΎΠ³Π΄Π°

I T = I R1 + I R2

ΠŸΠΎΡΠΊΠΎΠ»ΡŒΠΊΡƒ ΠΏΠ°Π΄Π΅Π½ΠΈΠ΅ напряТСния Π½Π° ΠΊΠ°ΠΆΠ΄ΠΎΠΌ рСзисторС ΠΎΠ΄ΠΈΠ½Π°ΠΊΠΎΠ²ΠΎ

I R1 = V / R 1

И I R2 = V / R 2

Если Ρ€Π°ΡΡΠΌΠ°Ρ‚Ρ€ΠΈΠ²Π°Ρ‚ΡŒ ΠΏΠ°Ρ€Π°Π»Π»Π΅Π»ΡŒΠ½ΡƒΡŽ Ρ€Π΅Π·ΠΈΡΡ‚ΠΈΠ²Π½ΡƒΡŽ Ρ†Π΅ΠΏΡŒ, ΡΠΎΡΡ‚ΠΎΡΡ‰ΡƒΡŽ ΠΈΠ· n рСзисторов, Ρ‚ΠΎ ΠΎΠ±Ρ‰ΠΈΠΉ Ρ‚ΠΎΠΊ Π² Ρ†Π΅ΠΏΠΈ Ρ€Π°Π²Π΅Π½

I ВсСго = I R1 + Π― R2 +….+ I Rn

Если ΠΏΠΎΡΠ»Π΅Π΄ΠΎΠ²Π°Ρ‚Π΅Π»ΡŒΠ½Ρ‹Π΅ рСзистивныС Ρ†Π΅ΠΏΠΈ Π½Π°Π·Ρ‹Π²Π°ΡŽΡ‚ΡΡ цСпями дСлитСля напряТСния, Ρ‚ΠΎ Π°Π½Π°Π»ΠΎΠ³ΠΈΡ‡Π½ΠΎ ΠΏΠ°Ρ€Π°Π»Π»Π΅Π»ΡŒΠ½Ρ‹Π΅ рСзистивныС Ρ†Π΅ΠΏΠΈ Π½Π°Π·Ρ‹Π²Π°ΡŽΡ‚ΡΡ цСпями дСлитСля Ρ‚ΠΎΠΊΠ°.

Если рассматриваСтся ΠΏΠ°Ρ€Π°Π»Π»Π΅Π»ΡŒΠ½Π°Ρ рСзистивная Ρ†Π΅ΠΏΡŒ ΠΈΠ· n рСзисторов с Ρ€Π°Π·Π½Ρ‹ΠΌ сопротивлСниСм, Ρ‚ΠΎ ΠΌΠΎΠΆΠ½ΠΎ ΠΈΠΌΠ΅Ρ‚ΡŒ n Ρ€Π°Π·Π½Ρ‹Ρ… ΠΏΡƒΡ‚Π΅ΠΉ для протСкания Ρ‚ΠΎΠΊΠ° ΠΈ n Ρ€Π°Π·Π½Ρ‹Ρ… Π·Π½Π°Ρ‡Π΅Π½ΠΈΠΉ Ρ‚ΠΎΠΊΠ° Ρ‡Π΅Ρ€Π΅Π· эти ΠΏΡƒΡ‚ΠΈ. РСзисторы Π² ΠΏΠ°Ρ€Π°Π»Π»Π΅Π»ΡŒΠ½ΠΎΠΉ ΠΊΠΎΠΌΠ±ΠΈΠ½Π°Ρ†ΠΈΠΈ ΠΌΠΎΠΆΠ½ΠΎ ΠΌΠ΅Π½ΡΡ‚ΡŒ мСстами, Π½Π΅ влияя Π½Π° ΠΎΠ±Ρ‰ΠΈΠΉ Ρ‚ΠΎΠΊ ΠΈ эквивалСнтноС сопротивлСниС.

ΠŸΡ€ΠΈΠΌΠ΅Ρ€ ΠΏΠ°Ρ€Π°Π»Π»Π΅Π»ΡŒΠ½ΠΎΠ³ΠΎ ΠΏΠΎΠ΄ΠΊΠ»ΡŽΡ‡Π΅Π½ΠΈΡ рСзисторов
  1. Рассмотрим ΡΠ»Π΅Π΄ΡƒΡŽΡ‰ΡƒΡŽ схСму, Π² ΠΊΠΎΡ‚ΠΎΡ€ΠΎΠΉ Ρ‡Π΅Ρ‚Ρ‹Ρ€Π΅ рСзистора R1, R2, R3 ΠΈ R4 ΠΏΠΎΠ΄ΠΊΠ»ΡŽΡ‡Π΅Π½Ρ‹ ΠΏΠ°Ρ€Π°Π»Π»Π΅Π»ΡŒΠ½ΠΎ.

ЗначСния сопротивлСния ΠΊΠ°ΠΆΠ΄ΠΎΠ³ΠΎ рСзистора:

R1 = 10 Ом

R2 = 20 Ом

R3 = 30 Ом

R4 = 40 Ом

НапряТСниС питания V = 24 Π’

ΠžΠ±Ρ‰ΠΈΠΉ Ρ‚ΠΎΠΊ Π² схСмС ΠΌΠΎΠΆΠ½ΠΎ Ρ€Π°ΡΡΡ‡ΠΈΡ‚Π°Ρ‚ΡŒ двумя способами.

ΠŸΠ΅Ρ€Π²Ρ‹ΠΉ ΠΌΠ΅Ρ‚ΠΎΠ΄ — Π²Ρ‹Ρ‡ΠΈΡΠ»ΠΈΡ‚ΡŒ ΠΎΡ‚Π΄Π΅Π»ΡŒΠ½Ρ‹Π΅ Ρ‚ΠΎΠΊΠΈ, ΠΏΡ€ΠΎΡ‚Π΅ΠΊΠ°ΡŽΡ‰ΠΈΠ΅ Ρ‡Π΅Ρ€Π΅Π· ΠΊΠ°ΠΆΠ΄Ρ‹ΠΉ рСзистор.

Если I1 — это Ρ‚ΠΎΠΊ, ΠΏΡ€ΠΎΡ‚Π΅ΠΊΠ°ΡŽΡ‰ΠΈΠΉ Ρ‡Π΅Ρ€Π΅Π· рСзистор R1, Ρ‚ΠΎ согласно Π·Π°ΠΊΠΎΠ½Ρƒ Ома

I1 = V / R 1 = 24/10 = 2,4 А

Аналогично, Ссли I2 — это Ρ‚ΠΎΠΊ, ΠΏΡ€ΠΎΡ‚Π΅ΠΊΠ°ΡŽΡ‰ΠΈΠΉ Ρ‡Π΅Ρ€Π΅Π· рСзистор R2, Ρ‚ΠΎ согласно ΠΏΠΎ Π·Π°ΠΊΠΎΠ½Ρƒ Ома

I2 = V / R 2 = 24/20 = 1,2 A

Если I3 — это Ρ‚ΠΎΠΊ, ΠΏΡ€ΠΎΡ‚Π΅ΠΊΠ°ΡŽΡ‰ΠΈΠΉ Ρ‡Π΅Ρ€Π΅Π· рСзистор R3, Ρ‚ΠΎ ΠΏΠΎ Π·Π°ΠΊΠΎΠ½Ρƒ Ома

I3 = V / R 3 = 24 / 30 = 0,8 А

И Ссли I4 — это Ρ‚ΠΎΠΊ, ΠΏΡ€ΠΎΡ‚Π΅ΠΊΠ°ΡŽΡ‰ΠΈΠΉ Ρ‡Π΅Ρ€Π΅Π· рСзистор R4, Ρ‚ΠΎ ΠΏΠΎ Π·Π°ΠΊΠΎΠ½Ρƒ Ома

I4 = V / R 4 = 24/40 = 0.6 A

Если I Π˜Π’ΠžΠ“Πž — это ΠΎΠ±Ρ‰ΠΈΠΉ Ρ‚ΠΎΠΊ Π² Ρ†Π΅ΠΏΠΈ, Ρ‚ΠΎ согласно Π·Π°ΠΊΠΎΠ½Ρƒ ΠšΠΈΡ€Ρ…Π³ΠΎΡ„Π°

I Π˜Π’ΠžΠ“Πž = I1 + I2 + I3 + I4 = 2,4 + 1,2 + 0,8 + 0,6 = 5A

Π’Ρ‚ΠΎΡ€ΠΎΠΉ ΠΌΠ΅Ρ‚ΠΎΠ΄ расчСта Ρ‚ΠΎΠΊΠ° — это ΠΎΠΏΡ€Π΅Π΄Π΅Π»Π΅Π½ΠΈΠ΅ эквивалСнтного сопротивлСния Ρ†Π΅ΠΏΠΈ.

Π­ΠΊΠ²ΠΈΠ²Π°Π»Π΅Π½Ρ‚Π½ΠΎΠ΅ сопротивлСниС Ρ†Π΅ΠΏΠΈ составляСт

1 / R EQ = (1 / R 1 ) + (1 / R 2 ) + (1 / R 3 ) + (1 / R 4 )

1 / R EQ = (1/10) + (1/20) + (1/30) + (1/40)

R EQ = 1/2.083 = 4,8 Ом

Π­Ρ‚ΠΎΡ‚ СдинствСнный рСзистор ΠΌΠΎΠΆΠ½ΠΎ ΠΈΡΠΏΠΎΠ»ΡŒΠ·ΠΎΠ²Π°Ρ‚ΡŒ для Π·Π°ΠΌΠ΅Π½Ρ‹ всСх рСзисторов Π² ΠΏΠ°Ρ€Π°Π»Π»Π΅Π»ΡŒΠ½ΠΎΠΉ ΠΊΠΎΠΌΠ±ΠΈΠ½Π°Ρ†ΠΈΠΈ.

∴ I Π˜Π’ΠžΠ“Πž = V / R EQ = 24 / 4.8 = 5A.

Рассмотрим ΡΠ»Π΅Π΄ΡƒΡŽΡ‰ΡƒΡŽ схСму, Π² ΠΊΠΎΡ‚ΠΎΡ€ΠΎΠΉ Ρ‚Ρ€ΠΈ рСзистора R1, R2 ΠΈ R3 соСдинСны ΠΏΠ°Ρ€Π°Π»Π»Π΅Π»ΡŒΠ½ΠΎ.

Π’ΠΎΠΊ, ΠΏΡ€ΠΎΡ‚Π΅ΠΊΠ°ΡŽΡ‰ΠΈΠΉ Ρ‡Π΅Ρ€Π΅Π· R1, Ρ€Π°Π²Π΅Π½ I1 = 6A

Π’ΠΎΠΊ, ΠΏΡ€ΠΎΡ‚Π΅ΠΊΠ°ΡŽΡ‰ΠΈΠΉ Ρ‡Π΅Ρ€Π΅Π· R2, Ρ€Π°Π²Π΅Π½ I2 = 4A

Π’ΠΎΠΊ, ΠΏΡ€ΠΎΡ‚Π΅ΠΊΠ°ΡŽΡ‰ΠΈΠΉ Ρ‡Π΅Ρ€Π΅Π· R3, Ρ€Π°Π²Π΅Π½ I3 = 2A

Π’ ΠΏΠ°Ρ€Π°Π»Π»Π΅Π»ΡŒΠ½Ρ‹Ρ… рСзистивных цСпях напряТСниС Π½Π° ΠΊΠ°ΠΆΠ΄ΠΎΠΌ рСзисторС ΠΎΠ΄ΠΈΠ½Π°ΠΊΠΎΠ²ΠΎΠ΅ ΠΈ Ρ€Π°Π²Π½ΠΎ Π½Π°ΠΏΡ€ΡΠΆΠ΅Π½ΠΈΡŽ питания.

Π—Π΄Π΅ΡΡŒ напряТСниС питания V = 12 Π’.

Если V1 — это напряТСниС Π½Π° рСзисторС R1, V2 — это напряТСниС Π½Π° рСзисторС R2, Π° V3 — это напряТСниС Π½Π° рСзисторС R3, Ρ‚ΠΎΠ³Π΄Π°

V = V1 = V2 = V3 = 12 Π’

Π’ΠΎΠ³Π΄Π° ΠΏΠΎ Π·Π°ΠΊΠΎΠ½Ρƒ Ома

R1 = V 1 / I 1

R1 = 12/6

R1 = 2 Ом

R2 = V 2 / I 2

R2 = 12/4

R2 = 3 Ом

R3 = V 3 / I 3

R3 = 12/2

R3 = 6 Ом

ΠŸΡ€ΠΈΠΌΠ΅Π½Π΅Π½ΠΈΡ

ΠšΠΎΠ½Ρ†Π΅ΠΏΡ†ΠΈΡ рСзисторов Π² parallel ΠΈΡΠΏΠΎΠ»ΡŒΠ·ΡƒΠ΅Ρ‚ΡΡ ΠΏΡ€ΠΈ Π°Π½Π°Π»ΠΈΠ·Π΅ мостовой схСмы Уитстона.ΠŸΠ°Ρ€Π°Π»Π»Π΅Π»ΡŒΠ½ΠΎ соСдинСнныС рСзисторы Π΄Π΅ΠΉΡΡ‚Π²ΡƒΡŽΡ‚ ΠΊΠ°ΠΊ Ρ†Π΅ΠΏΡŒ дСлитСля Ρ‚ΠΎΠΊΠ°. Π­Ρ‚Π° тСкущая концСпция дСлитСля ΠΈΡΠΏΠΎΠ»ΡŒΠ·ΡƒΠ΅Ρ‚ΡΡ Π² Ρ‚Π°ΠΊΠΈΡ… прилоТСниях, ΠΊΠ°ΠΊ Π°Π½Π°Π»ΠΎΠ³ΠΎ-Ρ†ΠΈΡ„Ρ€ΠΎΠ²Ρ‹Π΅ ΠΏΡ€Π΅ΠΎΠ±Ρ€Π°Π·ΠΎΠ²Π°Ρ‚Π΅Π»ΠΈ ΠΈ Ρ†ΠΈΡ„Ρ€ΠΎ-Π°Π½Π°Π»ΠΎΠ³ΠΎΠ²Ρ‹Π΅ ΠΏΡ€Π΅ΠΎΠ±Ρ€Π°Π·ΠΎΠ²Π°Ρ‚Π΅Π»ΠΈ.

Π­ΠΊΠ²ΠΈΠ²Π°Π»Π΅Π½Ρ‚Π½ΠΎΠ΅ сопротивлСниС трансформатора — элСктротСхничСскоС руководство

ΠŸΠ΅Ρ€Π²ΠΈΡ‡Π½Π°Ρ ΠΈ вторичная ΠΎΠ±ΠΌΠΎΡ‚ΠΊΠΈ Ρ€Π΅Π°Π»ΡŒΠ½ΠΎΠ³ΠΎ трансформатора ΠΈΠΌΠ΅ΡŽΡ‚ Π½Π΅ΠΊΠΎΡ‚ΠΎΡ€ΠΎΠ΅ сопротивлСниС, ΠΊΠΎΡ‚ΠΎΡ€ΠΎΠ΅ прСдставлСно R 1 ΠΈ R 2 соотвСтствСнно (ΠΏΠΎΠΊΠ°Π·Π°Π½ΠΎ Π½Π° рисункС Π²Π½Π΅ ΠΎΠ±ΠΌΠΎΡ‚ΠΎΠΊ).БопротивлСния Π΄Π²ΡƒΡ… ΠΎΠ±ΠΌΠΎΡ‚ΠΎΠΊ ΠΌΠΎΠ³ΡƒΡ‚ ΠΏΠ΅Ρ€Π΅Π΄Π°Π²Π°Ρ‚ΡŒΡΡ Π½Π° Π»ΡŽΠ±ΡƒΡŽ сторону. Π­Ρ‚ΠΎ сдСлано для облСгчСния расчСтов.

Π‘ΠΎΠΏΡ€ΠΎΡ‚ΠΈΠ²Π»Π΅Π½ΠΈΠ΅ пСрСдаСтся с ΠΎΠ΄Π½ΠΎΠΉ стороны Π½Π° Π΄Ρ€ΡƒΠ³ΡƒΡŽ Ρ‚Π°ΠΊΠΈΠΌ ΠΎΠ±Ρ€Π°Π·ΠΎΠΌ, Ρ‡Ρ‚ΠΎ ΠΏΡ€ΠΎΡ†Π΅Π½Ρ‚Π½ΠΎΠ΅ ΠΏΠ°Π΄Π΅Π½ΠΈΠ΅ напряТСния остаСтся ΠΎΠ΄ΠΈΠ½Π°ΠΊΠΎΠ²Ρ‹ΠΌ с ΠΎΠ±Π΅ΠΈΡ… сторон.

ΠœΡ‹ ΠΌΠΎΠΆΠ΅ΠΌ Π½Π°ΠΉΡ‚ΠΈ эквивалСнтноС сопротивлСниС ΠΈ Ρ€Π΅Π°ΠΊΡ‚ΠΈΠ²Π½ΠΎΠ΅ сопротивлСниС трансформатора с ΠΏΠΎΠΌΠΎΡ‰ΡŒΡŽ тСста Π½Π° ΠΊΠΎΡ€ΠΎΡ‚ΠΊΠΎΠ΅ Π·Π°ΠΌΡ‹ΠΊΠ°Π½ΠΈΠ΅
.


ΠŸΡƒΡΡ‚ΡŒ
R 1 = сопротивлСниС ΠΏΠ΅Ρ€Π²ΠΈΡ‡Π½ΠΎΠΉ ΠΎΠ±ΠΌΠΎΡ‚ΠΊΠΈ
R 2 = сопротивлСниС Π²Ρ‚ΠΎΡ€ΠΈΡ‡Π½ΠΎΠΉ ΠΎΠ±ΠΌΠΎΡ‚ΠΊΠΈ
K = коэффициСнт трансформации
Π’ΠΎΠ³Π΄Π°
Π‘ΠΎΠΏΡ€ΠΎΡ‚ΠΈΠ²Π»Π΅Π½ΠΈΠ΅ Π²Ρ‚ΠΎΡ€ΠΈΡ‡Π½ΠΎΠΉ ΠΎΠ±ΠΌΠΎΡ‚ΠΊΠΈ ΠΎΡ‚Π½ΠΎΡΠΈΡ‚Π΅Π»ΡŒΠ½ΠΎ ΠΏΠ΅Ρ€Π²ΠΈΡ‡Π½ΠΎΠΉ ΠΎΠ±ΠΌΠΎΡ‚ΠΊΠΈ R 2 ’моТно Ρ€Π°ΡΡΡ‡ΠΈΡ‚Π°Ρ‚ΡŒ ΠΏΠΎ Ρ„ΠΎΡ€ΠΌΡƒΠ»Π΅, ΠΏΡ€ΠΈΠ²Π΅Π΄Π΅Π½Π½ΠΎΠΉ Π½ΠΈΠΆΠ΅.
R 2 ’= R 2 / K 2

Π­ΠΊΠ²ΠΈΠ²Π°Π»Π΅Π½Ρ‚Π½ΠΎΠ΅ сопротивлСниС трансформатора ΠΎΡ‚Π½ΠΎΡΠΈΡ‚Π΅Π»ΡŒΠ½ΠΎ ΠΏΠ΅Ρ€Π²ΠΈΡ‡Π½ΠΎΠΉ ΠΎΠ±ΠΌΠΎΡ‚ΠΊΠΈ прСдставлСно R 01 .

Π‘Π»Π΅Π΄ΠΎΠ²Π°Ρ‚Π΅Π»ΡŒΠ½ΠΎ, R 01 = R 1 + R 2 ’= R + R 2 / K 2


Π­ΠΊΠ²ΠΈΠ²Π°Π»Π΅Π½Ρ‚Π½ΠΎΠ΅ сопротивлСниС трансформатора ΠΎΡ‚Π½ΠΎΡΠΈΡ‚Π΅Π»ΡŒΠ½ΠΎ Π²Ρ‚ΠΎΡ€ΠΈΡ‡Π½ΠΎΠΉ ΠΎΠ±ΠΌΠΎΡ‚ΠΊΠΈ


Π‘ΠΎΠΏΡ€ΠΎΡ‚ΠΈΠ²Π»Π΅Π½ΠΈΠ΅ ΠΏΠ΅Ρ€Π²ΠΈΡ‡Π½ΠΎΠΉ ΠΎΠ±ΠΌΠΎΡ‚ΠΊΠΈ ΠΎΡ‚Π½ΠΎΡΠΈΡ‚Π΅Π»ΡŒΠ½ΠΎ Π²Ρ‚ΠΎΡ€ΠΈΡ‡Π½ΠΎΠΉ ΠΎΠ±ΠΌΠΎΡ‚ΠΊΠΈ обозначаСтся R 1 ’и ΠΌΠΎΠΆΠ΅Ρ‚ Π±Ρ‹Ρ‚ΡŒ рассчитано ΠΏΠΎ Ρ„ΠΎΡ€ΠΌΡƒΠ»Π΅, ΠΏΡ€ΠΈΠ²Π΅Π΄Π΅Π½Π½ΠΎΠΉ Π½ΠΈΠΆΠ΅.

R 1 ’= K 2 R 1

Π­ΠΊΠ²ΠΈΠ²Π°Π»Π΅Π½Ρ‚Π½ΠΎΠ΅ сопротивлСниС трансформатора ΠΎΡ‚Π½ΠΎΡΠΈΡ‚Π΅Π»ΡŒΠ½ΠΎ Π²Ρ‚ΠΎΡ€ΠΈΡ‡Π½ΠΎΠΉ ΠΎΠ±ΠΌΠΎΡ‚ΠΊΠΈ прСдставлСно R 02 .

Π‘Π»Π΅Π΄ΠΎΠ²Π°Ρ‚Π΅Π»ΡŒΠ½ΠΎ, R 02 = R 2 + R 1 ’= R + K 2 R 1

Π­ΠΊΠ²ΠΈΠ²Π°Π»Π΅Π½Ρ‚Π½ΠΎΠ΅ Ρ€Π΅Π°ΠΊΡ‚ΠΈΠ²Π½ΠΎΠ΅ сопротивлСниС ΡƒΡ‚Π΅Ρ‡ΠΊΠΈ трансформатора


ΠŸΠΎΡ‚ΠΎΠΊ, ΠΊΠΎΡ‚ΠΎΡ€Ρ‹ΠΉ связываСтся с ΠΎΠ±Π΅ΠΈΠΌΠΈ ΠΎΠ±ΠΌΠΎΡ‚ΠΊΠ°ΠΌΠΈ трансформатора, называСтся Π²Π·Π°ΠΈΠΌΠ½Ρ‹ΠΌ ΠΏΠΎΡ‚ΠΎΠΊΠΎΠΌ, Π° ΠΏΠΎΡ‚ΠΎΠΊ, ΠΊΠΎΡ‚ΠΎΡ€Ρ‹ΠΉ связываСтся Ρ‚ΠΎΠ»ΡŒΠΊΠΎ с ΠΎΠ΄Π½ΠΎΠΉ ΠΎΠ±ΠΌΠΎΡ‚ΠΊΠΎΠΉ трансформатора, называСтся ΠΏΠΎΡ‚ΠΎΠΊΠΎΠΌ рассСяния.

Из-Π·Π° ΠΏΠΎΡ‚ΠΎΠΊΠ° рассСяния ΠΏΠ΅Ρ€Π²ΠΈΡ‡Π½ΠΎΠΉ ΠΎΠ±ΠΌΠΎΡ‚ΠΊΠΈ ΠΈ Π²Ρ‚ΠΎΡ€ΠΈΡ‡Π½ΠΎΠΉ ΠΎΠ±ΠΌΠΎΡ‚ΠΊΠΈ Π² ΡΠΎΠΎΡ‚Π²Π΅Ρ‚ΡΡ‚Π²ΡƒΡŽΡ‰Π΅ΠΉ ΠΎΠ±ΠΌΠΎΡ‚ΠΊΠ΅ индуцируСтся Π­Π”Π‘. ΠŸΠ΅Ρ€Π²ΠΈΡ‡Π½ΠΎΠ΅ ΠΈ Π²Ρ‚ΠΎΡ€ΠΈΡ‡Π½ΠΎΠ΅ напряТСниС Π΄ΠΎΠ»ΠΆΠ½Ρ‹ ΠΏΡ€Π΅ΠΎΠ΄ΠΎΠ»Π΅Π²Π°Ρ‚ΡŒ эти Π½Π°Π²Π΅Π΄Π΅Π½Π½Ρ‹Π΅ Π­Π”Π‘. Π’Π°ΠΊΠΈΠΌ ΠΎΠ±Ρ€Π°Π·ΠΎΠΌ, эти Π½Π°Π²Π΅Π΄Π΅Π½Π½Ρ‹Π΅ Π­Π”Π‘ Ρ€Π°ΡΡΠΌΠ°Ρ‚Ρ€ΠΈΠ²Π°ΡŽΡ‚ΡΡ ΠΊΠ°ΠΊ ΠΏΠ°Π΄Π΅Π½ΠΈΠ΅ напряТСния Π½Π° искусствСнных Ρ€Π΅Π°ΠΊΡ‚ΠΈΠ²Π½Ρ‹Ρ… сопротивлСниях, Π²ΠΊΠ»ΡŽΡ‡Π΅Π½Π½Ρ‹Ρ… ΠΏΠΎΡΠ»Π΅Π΄ΠΎΠ²Π°Ρ‚Π΅Π»ΡŒΠ½ΠΎ с ΠΏΠ΅Ρ€Π²ΠΈΡ‡Π½ΠΎΠΉ ΠΈ Π²Ρ‚ΠΎΡ€ΠΈΡ‡Π½ΠΎΠΉ ΠΎΠ±ΠΌΠΎΡ‚ΠΊΠ°ΠΌΠΈ. Π­Ρ‚ΠΈ Ρ€Π΅Π°ΠΊΡ‚ΠΈΠ²Π½Ρ‹Π΅ сопротивлСния Π½Π°Π·Ρ‹Π²Π°ΡŽΡ‚ΡΡ Ρ€Π΅Π°ΠΊΡ‚ΠΈΠ²Π½Ρ‹ΠΌΠΈ сопротивлСниями ΡƒΡ‚Π΅Ρ‡ΠΊΠΈ, ΠΈ ΠΎΠ½ΠΈ ΠΏΠΎΠΊΠ°Π·Π°Π½Ρ‹ Π½Π° рисункС.

Как ΠΈ Π² случаС сопротивлСний, Ρ€Π΅Π°ΠΊΡ‚ΠΈΠ²Π½Ρ‹Π΅ сопротивлСния Ρ‚Π°ΠΊΠΆΠ΅ ΠΌΠΎΠ³ΡƒΡ‚ ΠΏΠ΅Ρ€Π΅Π΄Π°Π²Π°Ρ‚ΡŒΡΡ Π² Π»ΡŽΠ±ΡƒΡŽ сторону. Π Π΅Π°ΠΊΡ‚ΠΈΠ²Π½ΠΎΠ΅ сопротивлСниС ΠΎΡ‚ ΠΎΠ΄Π½ΠΎΠΉ стороны ΠΊ Π΄Ρ€ΡƒΠ³ΠΎΠΉ пСрСдаСтся Ρ‚Π°ΠΊΠΈΠΌ ΠΎΠ±Ρ€Π°Π·ΠΎΠΌ, Ρ‡Ρ‚ΠΎ ΠΏΡ€ΠΎΡ†Π΅Π½Ρ‚Π½ΠΎΠ΅ ΠΏΠ°Π΄Π΅Π½ΠΈΠ΅ напряТСния остаСтся ΠΎΠ΄ΠΈΠ½Π°ΠΊΠΎΠ²Ρ‹ΠΌ с ΠΎΠ±Π΅ΠΈΡ… сторон.

Π­ΠΊΠ²ΠΈΠ²Π°Π»Π΅Π½Ρ‚Π½ΠΎΠ΅ Ρ€Π΅Π°ΠΊΡ‚ΠΈΠ²Π½ΠΎΠ΅ сопротивлСниС трансформатора ΠΎΡ‚Π½ΠΎΡΠΈΡ‚Π΅Π»ΡŒΠ½ΠΎ ΠΏΠ΅Ρ€Π²ΠΈΡ‡Π½ΠΎΠΉ ΠΎΠ±ΠΌΠΎΡ‚ΠΊΠΈ


ΠŸΡƒΡΡ‚ΡŒ
X 1 = Ρ€Π΅Π°ΠΊΡ‚ΠΈΠ²Π½ΠΎΠ΅ сопротивлСниС ΠΏΠ΅Ρ€Π²ΠΈΡ‡Π½ΠΎΠΉ ΠΎΠ±ΠΌΠΎΡ‚ΠΊΠΈ
X 2 = Ρ€Π΅Π°ΠΊΡ‚ΠΈΠ²Π½ΠΎΠ΅ сопротивлСниС Π²Ρ‚ΠΎΡ€ΠΈΡ‡Π½ΠΎΠΉ ΠΎΠ±ΠΌΠΎΡ‚ΠΊΠΈ
K = коэффициСнт трансформации
Π’ΠΎΠ³Π΄Π°
Π’Ρ‚ΠΎΡ€ΠΈΡ‡Π½ΠΎΠ΅ Ρ€Π΅Π°ΠΊΡ‚ΠΈΠ²Π½ΠΎΠ΅ сопротивлСниС ΠΎΡ‚Π½ΠΎΡΠΈΡ‚Π΅Π»ΡŒΠ½ΠΎ ΠΏΠ΅Ρ€Π²ΠΈΡ‡Π½ΠΎΠΉ ΠΎΠ±ΠΌΠΎΡ‚ΠΊΠΈ X 2 ’моТно Ρ€Π°ΡΡΡ‡ΠΈΡ‚Π°Ρ‚ΡŒ ΠΏΠΎ Ρ„ΠΎΡ€ΠΌΡƒΠ»Π΅, ΠΏΡ€ΠΈΠ²Π΅Π΄Π΅Π½Π½ΠΎΠΉ Π½ΠΈΠΆΠ΅.

X 2 ’= X 2 / K 2

Π­ΠΊΠ²ΠΈΠ²Π°Π»Π΅Π½Ρ‚Π½ΠΎΠ΅ Ρ€Π΅Π°ΠΊΡ‚ΠΈΠ²Π½ΠΎΠ΅ сопротивлСниС трансформатора ΠΎΡ‚Π½ΠΎΡΠΈΡ‚Π΅Π»ΡŒΠ½ΠΎ ΠΏΠ΅Ρ€Π²ΠΈΡ‡Π½ΠΎΠΉ ΠΎΠ±ΠΌΠΎΡ‚ΠΊΠΈ прСдставлСно ΠΊΠ°ΠΊ X 01 .

Π‘Π»Π΅Π΄ΠΎΠ²Π°Ρ‚Π΅Π»ΡŒΠ½ΠΎ, X 01 = X 1 + X 2 ’= X 1 + X 2 / K 2


Π­ΠΊΠ²ΠΈΠ²Π°Π»Π΅Π½Ρ‚Π½ΠΎΠ΅ Ρ€Π΅Π°ΠΊΡ‚ΠΈΠ²Π½ΠΎΠ΅ сопротивлСниС трансформатора ΠΎΡ‚Π½ΠΎΡΠΈΡ‚Π΅Π»ΡŒΠ½ΠΎ Π²Ρ‚ΠΎΡ€ΠΈΡ‡Π½ΠΎΠΉ ΠΎΠ±ΠΌΠΎΡ‚ΠΊΠΈ


Π Π΅Π°ΠΊΡ‚ΠΈΠ²Π½ΠΎΠ΅ сопротивлСниС ΠΏΠ΅Ρ€Π²ΠΈΡ‡Π½ΠΎΠΉ ΠΎΠ±ΠΌΠΎΡ‚ΠΊΠΈ ΠΎΡ‚Π½ΠΎΡΠΈΡ‚Π΅Π»ΡŒΠ½ΠΎ Π²Ρ‚ΠΎΡ€ΠΈΡ‡Π½ΠΎΠΉ ΠΎΠ±ΠΌΠΎΡ‚ΠΊΠΈ обозначаСтся X 1 ’и ΠΌΠΎΠΆΠ΅Ρ‚ Π±Ρ‹Ρ‚ΡŒ рассчитано ΠΏΠΎ Ρ„ΠΎΡ€ΠΌΡƒΠ»Π΅, ΠΏΡ€ΠΈΠ²Π΅Π΄Π΅Π½Π½ΠΎΠΉ Π½ΠΈΠΆΠ΅.

X 1 ’= K 2 X 1

Π­ΠΊΠ²ΠΈΠ²Π°Π»Π΅Π½Ρ‚Π½ΠΎΠ΅ Ρ€Π΅Π°ΠΊΡ‚ΠΈΠ²Π½ΠΎΠ΅ сопротивлСниС трансформатора ΠΎΡ‚Π½ΠΎΡΠΈΡ‚Π΅Π»ΡŒΠ½ΠΎ Π²Ρ‚ΠΎΡ€ΠΈΡ‡Π½ΠΎΠΉ ΠΎΠ±ΠΌΠΎΡ‚ΠΊΠΈ прСдставлСно ΠΊΠ°ΠΊ X 02 .

Π‘Π»Π΅Π΄ΠΎΠ²Π°Ρ‚Π΅Π»ΡŒΠ½ΠΎ, X 02 = X 2 + X 1 ’= X 2 + K 2 X 1


ИмпСданс трансформатора


ΠŸΠ΅Ρ€Π²ΠΈΡ‡Π½Ρ‹ΠΉ импСданс Z 1 = R 1 + jX 1
И Π²Ρ‚ΠΎΡ€ΠΈΡ‡Π½Ρ‹ΠΉ импСданс Z 2 = R 2 + jX 2

ΠŸΠ΅Ρ€Π΅Π΄Π°Ρ‡Π° ΠΏΠΎΠ»Π½Ρ‹Ρ… сопротивлСний происходит ΠΏΠΎ Ρ‚Π΅ΠΌ ΠΆΠ΅ линиям, Ρ‡Ρ‚ΠΎ ΠΈ Ρ‡Ρ‚ΠΎ сопротивлСния.ΠŸΠ΅Ρ€Π΅Π½ΠΎΡ импСдансов ΠΌΠΎΠΆΠ΅Ρ‚ ΠΏΡ€ΠΎΠΈΡΡ…ΠΎΠ΄ΠΈΡ‚ΡŒ ΠΎΡ‚ ΠΏΠ΅Ρ€Π²ΠΈΡ‡Π½ΠΎΠΉ ΠΎΠ±ΠΌΠΎΡ‚ΠΊΠΈ ΠΊ Π²Ρ‚ΠΎΡ€ΠΈΡ‡Π½ΠΎΠΉ ΠΈ Π½Π°ΠΎΠ±ΠΎΡ€ΠΎΡ‚.

Z 01 = (R 01 2 + X 01 2 ) 1/2 импСданс ΠΎΡ‚Π½ΠΎΡΠΈΡ‚Π΅Π»ΡŒΠ½ΠΎ ΠΏΠ΅Ρ€Π²ΠΈΡ‡Π½ΠΎΠΉ ΠΎΠ±ΠΌΠΎΡ‚ΠΊΠΈ.

Z 02 = (R 02 2 + X 02 2 ) 1/2 импСданс ΠΎΡ‚Π½ΠΎΡΠΈΡ‚Π΅Π»ΡŒΠ½ΠΎ Π²Ρ‚ΠΎΡ€ΠΈΡ‡Π½ΠΎΠΉ стороны.

Врансформатор | ВсС сообщСния

Β© https: // yourelectricalguide.ΠΊΠΎΠΌ / эквивалСнтноС сопротивлСниС трансформатора.

Π§Ρ‚ΠΎ Ρ‚Π°ΠΊΠΎΠ΅ эквивалСнтная схСма ΠΈ ΠΊΠ°ΠΊ ΠΎΠ½Π° ΠΈΡΠΏΠΎΠ»ΡŒΠ·ΡƒΠ΅Ρ‚ΡΡ?

ΠŸΡ€ΠΈ Π°Π½Π°Π»ΠΈΠ·Π΅ элСктричСских Ρ†Π΅ΠΏΠ΅ΠΉ, Π² Ρ‚ΠΎΠΌ числС для асинхронных Π΄Π²ΠΈΠ³Π°Ρ‚Π΅Π»Π΅ΠΉ ΠΏΠ΅Ρ€Π΅ΠΌΠ΅Π½Π½ΠΎΠ³ΠΎ Ρ‚ΠΎΠΊΠ° ΠΈ Π΄Π²ΠΈΠ³Π°Ρ‚Π΅Π»Π΅ΠΉ постоянного Ρ‚ΠΎΠΊΠ°, Ссли схСма содСрТит Π΄Π²Π° ΠΈΠ»ΠΈ Π±ΠΎΠ»Π΅Π΅ ΠΎΠ΄ΠΈΠ½Π°ΠΊΠΎΠ²Ρ‹Ρ… пассивных элСмСнта (Π½Π°ΠΏΡ€ΠΈΠΌΠ΅Ρ€, рСзисторы) ΠΈ ΠΏΠΎΠ΄ΠΊΠ»ΡŽΡ‡Π΅Π½Π° ΠΈΡΠΊΠ»ΡŽΡ‡ΠΈΡ‚Π΅Π»ΡŒΠ½ΠΎ ΠΏΠΎΡΠ»Π΅Π΄ΠΎΠ²Π°Ρ‚Π΅Π»ΡŒΠ½ΠΎ ΠΈΠ»ΠΈ ΠΈΡΠΊΠ»ΡŽΡ‡ΠΈΡ‚Π΅Π»ΡŒΠ½ΠΎ ΠΏΠ°Ρ€Π°Π»Π»Π΅Π»ΡŒΠ½ΠΎ, схСму ΠΌΠΎΠΆΠ½ΠΎ Π½Π°Ρ€ΠΈΡΠΎΠ²Π°Ρ‚ΡŒ. с Π±ΠΎΠ»Π΅Π΅ простым прСдставлСниСм, содСрТащим источник напряТСния ΠΈ ΠΎΠ΄ΠΈΠ½ эквивалСнтный пассивный элСмСнт. Π­Ρ‚Π° упрощСнная вСрсия сохраняСт элСктричСскиС характСристики исходной схСмы ΠΈ называСтся эквивалСнтной схСмой .


ΠŸΠ°ΡΡΠΈΠ²Π½Ρ‹Π΅ элСмСнты — это элСмСнты, ΠΊΠΎΡ‚ΠΎΡ€Ρ‹Π΅ Ρ€Π°ΡΡΠ΅ΠΈΠ²Π°ΡŽΡ‚, Π½Π°ΠΊΠ°ΠΏΠ»ΠΈΠ²Π°ΡŽΡ‚ ΠΈΠ»ΠΈ Π²Ρ‹Π΄Π΅Π»ΡΡŽΡ‚ ΡΠ½Π΅Ρ€Π³ΠΈΡŽ Π² Π²ΠΈΠ΄Π΅ напряТСния ΠΈΠ»ΠΈ Ρ‚ΠΎΠΊΠ°. ΠŸΡ€ΠΈΠΌΠ΅Ρ€Π°ΠΌΠΈ пассивных элСмСнтов ΡΠ²Π»ΡΡŽΡ‚ΡΡ рСзисторы , кондСнсаторы ΠΈ ΠΊΠ°Ρ‚ΡƒΡˆΠΊΠΈ (Ρ‚Π°ΠΊΠΆΠ΅ извСстныС ΠΊΠ°ΠΊ ΠΈΠ½Π΄ΡƒΠΊΡ‚ΠΎΡ€Ρ‹).

АктивныС элСмСнты Π²Ρ‹Ρ€Π°Π±Π°Ρ‚Ρ‹Π²Π°ΡŽΡ‚ ΠΈΠ»ΠΈ Π²Ρ‹Ρ€Π°Π±Π°Ρ‚Ρ‹Π²Π°ΡŽΡ‚ ΡΠ½Π΅Ρ€Π³ΠΈΡŽ Π² Π²ΠΈΠ΄Π΅ напряТСния ΠΈΠ»ΠΈ Ρ‚ΠΎΠΊΠ°. Они Π²ΠΊΠ»ΡŽΡ‡Π°ΡŽΡ‚ Π² сСбя ΠΏΠΎΠ»ΡƒΠΏΡ€ΠΎΠ²ΠΎΠ΄Π½ΠΈΠΊΠΎΠ²Ρ‹Π΅ ΠΊΠΎΠΌΠΏΠΎΠ½Π΅Π½Ρ‚Ρ‹, Ρ‚Π°ΠΊΠΈΠ΅ ΠΊΠ°ΠΊ Π΄ΠΈΠΎΠ΄Ρ‹, транзисторы (ΠΏΠΎΠ»Π΅Π²Ρ‹Π΅ транзисторы, ΠΈΠ»ΠΈ ΠΏΠΎΠ»Π΅Π²Ρ‹Π΅ транзисторы, ΠΈ ΠΏΠΎΠ»Π΅Π²Ρ‹Π΅ транзисторы ΠΌΠ΅Ρ‚Π°Π»Π»-оксид-ΠΏΠΎΠ»ΡƒΠΏΡ€ΠΎΠ²ΠΎΠ΄Π½ΠΈΠΊ, ΠΈΠ»ΠΈ ΠΏΠΎΠ»Π΅Π²Ρ‹Π΅ МОП-транзисторы).


ΠŸΡ€Π°Π²ΠΈΠ»Π° комбинирования рСзисторов для создания эквивалСнтной схСмы основаны Π½Π° Π·Π°ΠΊΠΎΠ½Π΅ Ома вмСстС с Π·Π°ΠΊΠΎΠ½Π°ΠΌΠΈ ΠšΠΈΡ€Ρ…Π³ΠΎΡ„Π° для ΠΊΠΎΠ½Ρ‚ΡƒΡ€ΠΎΠ².

ΠŸΠ΅Ρ€Π²Ρ‹ΠΉ ΠΈΠ· Π·Π°ΠΊΠΎΠ½ΠΎΠ² ΠšΠΈΡ€Ρ…Π³ΠΎΡ„Π°, извСстный ΠΊΠ°ΠΊ Π—Π°ΠΊΠΎΠ½ ΠšΠΈΡ€Ρ…Π³ΠΎΡ„Π° (KCL), гласит, Ρ‡Ρ‚ΠΎ Π²Π΅Π»ΠΈΡ‡ΠΈΠ½Π° Ρ‚ΠΎΠΊΠ°, ΠΏΡ€ΠΎΡ‚Π΅ΠΊΠ°ΡŽΡ‰Π΅Π³ΠΎ Π² любой ΡƒΠ·Π΅Π» (ΠΏΠ΅Ρ€Π΅Ρ…ΠΎΠ΄) Π² Π·Π°ΠΌΠΊΠ½ΡƒΡ‚ΠΎΠΉ Ρ†Π΅ΠΏΠΈ, Ρ€Π°Π²Π½Π° Π²Π΅Π»ΠΈΡ‡ΠΈΠ½Π΅ Ρ‚ΠΎΠΊΠ°, Π²Ρ‹Ρ‚Π΅ΠΊΠ°ΡŽΡ‰Π΅Π³ΠΎ ΠΈΠ· этого ΡƒΠ·Π»Π° , Ρ‚Π°ΠΊΠΈΠΌ ΠΎΠ±Ρ€Π°Π·ΠΎΠΌ, обСспСчСниС сохранСния заряда Π² Π·Π°ΠΌΠΊΠ½ΡƒΡ‚ΠΎΠΌ ΠΊΠΎΠ½Ρ‚ΡƒΡ€Π΅.

Π’Ρ‚ΠΎΡ€ΠΎΠΉ Π·Π°ΠΊΠΎΠ½ ΠšΠΈΡ€Ρ…Π³ΠΎΡ„Π°, извСстный ΠΊΠ°ΠΊ Π·Π°ΠΊΠΎΠ½ напряТСния ΠšΠΈΡ€Ρ…Π³ΠΎΡ„Π° (KVL), гласит, Ρ‡Ρ‚ΠΎ для Π·Π°ΠΌΠΊΠ½ΡƒΡ‚ΠΎΠΉ Ρ†Π΅ΠΏΠΈ алгСбраичСская сумма всСх напряТСний Π²ΠΎΠΊΡ€ΡƒΠ³ Ρ†Π΅ΠΏΠΈ Ρ€Π°Π²Π½Π° Π½ΡƒΠ»ΡŽ. Π—Π°ΠΊΠΎΠ½ напряТСния ΠšΠΈΡ€Ρ…Π³ΠΎΡ„Π° обСспСчиваСт сохранСниС энСргии Π² Π·Π°ΠΌΠΊΠ½ΡƒΡ‚ΠΎΠΉ Ρ†Π΅ΠΏΠΈ.


ΠžΠΏΡ€Π΅Π΄Π΅Π»Π΅Π½ΠΈΠ΅ эквивалСнтного сопротивлСния для ΠΏΠΎΡΠ»Π΅Π΄ΠΎΠ²Π°Ρ‚Π΅Π»ΡŒΠ½Ρ‹Ρ… Ρ†Π΅ΠΏΠ΅ΠΉ

Для Ρ†Π΅ΠΏΠΈ, соСдинСнной ΠΏΠΎΡΠ»Π΅Π΄ΠΎΠ²Π°Ρ‚Π΅Π»ΡŒΠ½ΠΎ, Π·Π°ΠΊΠΎΠ½ ΠšΠΈΡ€Ρ…Π³ΠΎΡ„Π° ΠΏΠΎ Π½Π°ΠΏΡ€ΡΠΆΠ΅Π½ΠΈΡŽ, KVL, Π³ΠΎΠ²ΠΎΡ€ΠΈΡ‚ Π½Π°ΠΌ, Ρ‡Ρ‚ΠΎ напряТСниС Π² Ρ†Π΅ΠΏΠΈ Π±ΡƒΠ΄Π΅Ρ‚ Ρ€Π°Π²Π½ΠΎ Π½ΡƒΠ»ΡŽ. Π­Ρ‚ΠΎ ΠΎΠ·Π½Π°Ρ‡Π°Π΅Ρ‚, Ρ‡Ρ‚ΠΎ сумма ΠΏΠ°Π΄Π΅Π½ΠΈΠΉ напряТСния Π½Π° ΠΊΠ°ΠΆΠ΄ΠΎΠΌ рСзисторС Π±ΡƒΠ΄Π΅Ρ‚ Ρ€Π°Π²Π½Π° Π½Π°ΠΏΡ€ΡΠΆΠ΅Π½ΠΈΡŽ питания. Для ΠΏΠΎΡΠ»Π΅Π΄ΠΎΠ²Π°Ρ‚Π΅Π»ΡŒΠ½ΠΎΠΉ Ρ†Π΅ΠΏΠΈ с трСмя рСзисторами напряТСниС питания, Π’ с , Ρ€Π°Π²Π½ΠΎ суммС напряТСний Π½Π° Ρ‚Ρ€Π΅Ρ… рСзисторах (Π’ R1 , Π’ R2 ΠΈ Π’ R3 ):

ΠœΡ‹ Π·Π½Π°Π΅ΠΌ ΠΈΠ· Π΄Π΅ΠΉΡΡ‚Π²ΡƒΡŽΡ‰Π΅Π³ΠΎ Π·Π°ΠΊΠΎΠ½Π° ΠšΠΈΡ€Ρ…Π³ΠΎΡ„Π°, KCL, Ρ‡Ρ‚ΠΎ заряд, Ρ‚Π΅ΠΊΡƒΡ‰ΠΈΠΉ Π² любой ΡƒΠ·Π΅Π», Ρ€Π°Π²Π΅Π½ заряду, Π²Ρ‹Ρ‚Π΅ΠΊΠ°ΡŽΡ‰Π΅ΠΌΡƒ ΠΈΠ· этого ΡƒΠ·Π»Π°.ΠŸΠΎΡΠ»Π΅Π΄ΠΎΠ²Π°Ρ‚Π΅Π»ΡŒΠ½Ρ‹Π΅ Ρ†Π΅ΠΏΠΈ ΠΈΠΌΠ΅ΡŽΡ‚ Ρ‚ΠΎΠ»ΡŒΠΊΠΎ ΠΎΠ΄ΠΈΠ½ ΡƒΠ·Π΅Π» (ΠΏΠ΅Ρ€Π΅Ρ…ΠΎΠ΄), поэтому Ρ‚ΠΎΠΊ ΠΎΠ΄ΠΈΠ½Π°ΠΊΠΎΠ² Π²ΠΎ всСх Ρ‚ΠΎΡ‡ΠΊΠ°Ρ… Ρ†Π΅ΠΏΠΈ. Π­Ρ‚ΠΎ ΠΎΠ·Π½Π°Ρ‡Π°Π΅Ρ‚, Ρ‡Ρ‚ΠΎ Ρ‡Π΅Ρ€Π΅Π· ΠΊΠ°ΠΆΠ΄Ρ‹ΠΉ рСзистор ΠΏΡ€ΠΎΡ‚Π΅ΠΊΠ°Π΅Ρ‚ ΠΎΠ΄ΠΈΠ½Π°ΠΊΠΎΠ²Ρ‹ΠΉ Ρ‚ΠΎΠΊ. Π˜ΡΠΏΠΎΠ»ΡŒΠ·ΡƒΡ Π·Π°ΠΊΠΎΠ½ Ома, V = IR, Ρ‡Ρ‚ΠΎΠ±Ρ‹ Π²Ρ‹Ρ€Π°Π·ΠΈΡ‚ΡŒ напряТСниС Π½Π° ΠΊΠ°ΠΆΠ΄ΠΎΠΌ рСзисторС, ΠΌΡ‹ ΠΌΠΎΠΆΠ΅ΠΌ ΠΏΠ΅Ρ€Π΅ΠΏΠΈΡΠ°Ρ‚ΡŒ ΡƒΡ€Π°Π²Π½Π΅Π½ΠΈΠ΅ Π²Ρ‹ΡˆΠ΅:

Π’Π΅ΠΏΠ΅Ρ€ΡŒ ΠΌΡ‹ Π²ΠΈΠ΄ΠΈΠΌ, Ρ‡Ρ‚ΠΎ эквивалСнтноС сопротивлСниС — это просто сумма всСх сопротивлСний Π² ΠΏΠΎΡΠ»Π΅Π΄ΠΎΠ²Π°Ρ‚Π΅Π»ΡŒΠ½ΠΎΠΉ Ρ†Π΅ΠΏΠΈ.

А напряТСниС Ρ‚Π΅ΠΏΠ΅Ρ€ΡŒ ΠΌΠΎΠΆΠ½ΠΎ Π·Π°ΠΏΠΈΡΠ°Ρ‚ΡŒ Ρ‡Π΅Ρ€Π΅Π· эквивалСнтноС сопротивлСниС.

ΠŸΡ€ΠΈΠΌΠ΅Ρ€ Π½ΠΈΠΆΠ΅ ΠΏΠΎΠΊΠ°Π·Ρ‹Π²Π°Π΅Ρ‚ ΠΏΠΎΡΠ»Π΅Π΄ΠΎΠ²Π°Ρ‚Π΅Π»ΡŒΠ½ΡƒΡŽ схСму с трСмя рСзисторами.

Для этой Ρ†Π΅ΠΏΠΈ эквивалСнтноС сопротивлСниС:

Π’Π°ΠΊΠΈΠΌ ΠΎΠ±Ρ€Π°Π·ΠΎΠΌ, эквивалСнтная схСма Π±ΡƒΠ΄Π΅Ρ‚ ΠΈΠΌΠ΅Ρ‚ΡŒ ΠΎΠ΄ΠΈΠ½ рСзистор, эквивалСнтный 10 Ом (R eq ).

ΠœΡ‹ ΠΌΠΎΠΆΠ΅ΠΌ ΠΏΡ€ΠΎΠ²Π΅Ρ€ΠΈΡ‚ΡŒ Π½Π°ΡˆΡƒ ΡΠΊΠ²ΠΈΠ²Π°Π»Π΅Π½Ρ‚Π½ΡƒΡŽ схСму, рассчитав напряТСниС Π½Π° эквивалСнтном рСзисторС ΠΈ ΡƒΠ±Π΅Π΄ΠΈΠ²ΡˆΠΈΡΡŒ, Ρ‡Ρ‚ΠΎ ΠΎΠ½ΠΎ Ρ€Π°Π²Π½ΠΎ Π½Π°ΠΏΡ€ΡΠΆΠ΅Π½ΠΈΡŽ питания:


ΠžΠΏΡ€Π΅Π΄Π΅Π»Π΅Π½ΠΈΠ΅ эквивалСнтного сопротивлСния для ΠΏΠ°Ρ€Π°Π»Π»Π΅Π»ΡŒΠ½Ρ‹Ρ… Ρ†Π΅ΠΏΠ΅ΠΉ

Для Ρ†Π΅ΠΏΠ΅ΠΉ, соСдинСнных ΠΏΠ°Ρ€Π°Π»Π»Π΅Π»ΡŒΠ½ΠΎ, ΠΌΡ‹ ΠΌΠΎΠΆΠ΅ΠΌ Ρ€Π°ΡΡΠΌΠ°Ρ‚Ρ€ΠΈΠ²Π°Ρ‚ΡŒ ΠΊΠ°ΠΆΠ΄ΡƒΡŽ ΠΏΠ΅Ρ‚Π»ΡŽ ΠΊΠ°ΠΊ ΠΎΡ‚Π΄Π΅Π»ΡŒΠ½ΡƒΡŽ Ρ†Π΅ΠΏΡŒ, ΡΠΎΠ΅Π΄ΠΈΠ½Π΅Π½Π½ΡƒΡŽ ΠΏΠΎΡΠ»Π΅Π΄ΠΎΠ²Π°Ρ‚Π΅Π»ΡŒΠ½ΠΎ.Π—Π°ΠΊΠΎΠ½ ΠšΠΈΡ€Ρ…Π³ΠΎΡ„Π°, KVL, Π³ΠΎΠ²ΠΎΡ€ΠΈΡ‚ Π½Π°ΠΌ, Ρ‡Ρ‚ΠΎ Π² ΠΊΠ°ΠΆΠ΄ΠΎΠΌ ΠΎΡ‚Π΄Π΅Π»ΡŒΠ½ΠΎΠΌ ΠΊΠΎΠ½Ρ‚ΡƒΡ€Π΅ (ΠΏΠΎΡΠ»Π΅Π΄ΠΎΠ²Π°Ρ‚Π΅Π»ΡŒΠ½ΠΎΠΉ Ρ†Π΅ΠΏΠΈ) ΠΏΠ°Π΄Π΅Π½ΠΈΠ΅ напряТСния Π½Π° рСзисторС Ρ€Π°Π²Π½ΠΎ Π½Π°ΠΏΡ€ΡΠΆΠ΅Π½ΠΈΡŽ питания. Для ΠΏΠ°Ρ€Π°Π»Π»Π΅Π»ΡŒΠ½ΠΎΠΉ схСмы с трСмя рСзисторами ΠΏΠ°Π΄Π΅Π½ΠΈΠ΅ напряТСния Π½Π° ΠΊΠ°ΠΆΠ΄ΠΎΠΌ рСзисторС Ρ€Π°Π²Π½ΠΎ Π½Π°ΠΏΡ€ΡΠΆΠ΅Π½ΠΈΡŽ питания.

Богласно Π΄Π΅ΠΉΡΡ‚Π²ΡƒΡŽΡ‰Π΅ΠΌΡƒ Π·Π°ΠΊΠΎΠ½Ρƒ ΠšΠΈΡ€Ρ…Π³ΠΎΡ„Π°, KCL, Ρ‚ΠΎΠΊ дСлится Π½Π° ΠΊΠ°ΠΆΠ΄ΠΎΠΌ ΡƒΠ·Π»Π΅ ΠΈΠ»ΠΈ соСдинСнии, Ρ‚Π°ΠΊ Ρ‡Ρ‚ΠΎ:

Π’Ρ‹Ρ€Π°ΠΆΠ΅Π½ΠΈΠ΅ силы Ρ‚ΠΎΠΊΠ° ΠΊΠ°ΠΊ напряТСния, Π΄Π΅Π»Π΅Π½Π½ΠΎΠ³ΠΎ Π½Π° сопротивлСниС (Π’ / R), согласно Π·Π°ΠΊΠΎΠ½Ρƒ Ома:

ΠŸΠ΅Ρ€Π΅ΡΡ‚Π°Π²Π»ΡΡ ΠΏΠΎ Π½Π°ΠΏΡ€ΡΠΆΠ΅Π½ΠΈΡŽ, ΠΏΠΎΠ»ΡƒΡ‡Π°Π΅ΠΌ:

Π’Π΅ΠΏΠ΅Ρ€ΡŒ эквивалСнтноС сопротивлСниС ΠΌΠΎΠΆΠ½ΠΎ ΠΎΠΏΡ€Π΅Π΄Π΅Π»ΠΈΡ‚ΡŒ ΠΊΠ°ΠΊ:

ΠΈΠ»ΠΈ:

Π”Ρ€ΡƒΠ³ΠΈΠΌΠΈ словами, для ΠΏΠ°Ρ€Π°Π»Π»Π΅Π»ΡŒΠ½ΠΎΠΉ Ρ†Π΅ΠΏΠΈ эквивалСнтноС сопротивлСниС находится ΠΏΡƒΡ‚Π΅ΠΌ слоТСния ΠΎΠ±Ρ€Π°Ρ‚Π½Ρ‹Ρ… Π²Π΅Π»ΠΈΡ‡ΠΈΠ½ ΠΎΡ‚Π΄Π΅Π»ΡŒΠ½Ρ‹Ρ… Π·Π½Π°Ρ‡Π΅Π½ΠΈΠΉ сопротивлСния, Π° Π·Π°Ρ‚Π΅ΠΌ взятия ΠΎΠ±Ρ€Π°Ρ‚Π½ΠΎΠΉ Π²Π΅Π»ΠΈΡ‡ΠΈΠ½Ρ‹ ΠΎΡ‚ ΠΎΠ±Ρ‰Π΅Π³ΠΎ значСния.

Π’ этой ΠΏΠ°Ρ€Π°Π»Π»Π΅Π»ΡŒΠ½ΠΎΠΉ Ρ†Π΅ΠΏΠΈ с трСмя рСзисторами эквивалСнтноС сопротивлСниС составляСт:

ΠžΠΏΡΡ‚ΡŒ ΠΆΠ΅, ΠΌΡ‹ ΠΌΠΎΠΆΠ΅ΠΌ ΠΏΡ€ΠΎΠ²Π΅Ρ€ΠΈΡ‚ΡŒ Π½Π°ΡˆΡƒ ΡΠΊΠ²ΠΈΠ²Π°Π»Π΅Π½Ρ‚Π½ΡƒΡŽ схСму, ΡƒΠ±Π΅Π΄ΠΈΠ²ΡˆΠΈΡΡŒ, Ρ‡Ρ‚ΠΎ напряТСниС Π½Π° эквивалСнтном рСзисторС Ρ€Π°Π²Π½ΠΎ Π½Π°ΠΏΡ€ΡΠΆΠ΅Π½ΠΈΡŽ питания:


Π—Π°ΠΊΠΎΠ½Ρ‹ ΠšΠΈΡ€Ρ…Π³ΠΎΡ„Π° ΠΏΡ€ΠΈΠΌΠ΅Π½ΠΈΠΌΡ‹ ΠΊ ΠΏΠΎΡΠ»Π΅Π΄ΠΎΠ²Π°Ρ‚Π΅Π»ΡŒΠ½Ρ‹ΠΌ цСпям: Для Ρ†Π΅ΠΏΠΈ, соСдинСнной ΠΏΠΎΡΠ»Π΅Π΄ΠΎΠ²Π°Ρ‚Π΅Π»ΡŒΠ½ΠΎ, напряТСниС, ΠΏΡ€ΠΎΡ‚Π΅ΠΊΠ°ΡŽΡ‰Π΅Π΅ ΠΏΠΎ Ρ†Π΅ΠΏΠΈ, дСлится ΠΌΠ΅ΠΆΠ΄Ρƒ пассивными элСмСнтами, Π½ΠΎ Ρ‚ΠΎΠΊ Ρ‡Π΅Ρ€Π΅Π· ΠΊΠ°ΠΆΠ΄Ρ‹ΠΉ пассивный элСмСнт ΠΎΠ΄ΠΈΠ½Π°ΠΊΠΎΠ².

Π—Π°ΠΊΠΎΠ½Ρ‹ ΠšΠΈΡ€Ρ…Π³ΠΎΡ„Π° ΠΏΡ€ΠΈΠΌΠ΅Π½ΠΈΠΌΡ‹ ΠΊ ΠΏΠ°Ρ€Π°Π»Π»Π΅Π»ΡŒΠ½Ρ‹ΠΌ цСпям: Для Ρ†Π΅ΠΏΠΈ, ΠΏΠΎΠ΄ΠΊΠ»ΡŽΡ‡Π΅Π½Π½ΠΎΠΉ ΠΏΠ°Ρ€Π°Π»Π»Π΅Π»ΡŒΠ½ΠΎ, напряТСниС, ΠΏΡ€ΠΎΡ‚Π΅ΠΊΠ°ΡŽΡ‰Π΅Π΅ ΠΏΠΎ Ρ†Π΅ΠΏΠΈ, ΠΎΠ΄ΠΈΠ½Π°ΠΊΠΎΠ²ΠΎ для ΠΊΠ°ΠΆΠ΄ΠΎΠ³ΠΎ пассивного элСмСнта, Π½ΠΎ Ρ‚ΠΎΠΊ дСлится ΠΌΠ΅ΠΆΠ΄Ρƒ пассивными элСмСнтами.


Π—Π°Π³Π°Π΄ΠΊΠ° эквивалСнтного сопротивлСния рСзисторного ΠΊΡƒΠ±Π°

<ΠŸΡ€Π΅Π΄Ρ‹Π΄ΡƒΡ‰Π°Ρ Π”Π°Π»Π΅Π΅>

Π—Π°Π³Π°Π΄ΠΊΠ° эквивалСнтного сопротивлСния рСзисторного ΠΊΡƒΠ±Π°

Π’Ρ‹, Π½Π°Π²Π΅Ρ€Π½ΠΎΠ΅, Π²ΠΈΠ΄Π΅Π»ΠΈ Π³Π΄Π΅-Ρ‚ΠΎ Π½Π° Π»ΠΈΠ½ΠΈΠΈ Π² вашСй ΠΊΠ°Ρ€ΡŒΠ΅Ρ€Π΅ элСктронщика ΠΏΡ€ΠΎΠ±Π»Π΅ΠΌΠ° рСзисторного ΠΊΡƒΠ±Π°.12 Π³Ρ€Π°Π½Π΅ΠΉ ΠΊΡƒΠ±Π° ΠΊΠ°ΠΆΠ΄Ρ‹ΠΉ содСрТат рСзистор 1 Ом, ΠΈ Π·Π°Π΄Π°Ρ‡Π° состоит Π² Ρ‚ΠΎΠΌ, Ρ‡Ρ‚ΠΎΠ±Ρ‹ Π²Ρ‹Ρ‡ΠΈΡΠ»ΠΈΡ‚ΡŒ, ΠΊΠ°ΠΊΠΎΠΉ эквивалСнт сопротивлСниС ΠΌΠ΅ΠΆΠ΄Ρƒ двумя ΠΏΡ€ΠΎΡ‚ΠΈΠ²ΠΎΠΏΠΎΠ»ΠΎΠΆΠ½Ρ‹ΠΌΠΈ ΡƒΠ³Π»Π°ΠΌΠΈ. ИспользованиС прямого схСмотСхничСский Π°Π½Π°Π»ΠΈΠ·, ΠΏΠΎΡΠΊΠΎΠ»ΡŒΠΊΡƒ ΠΎΠ½ Ρ‚Ρ€Π΅Π±ΡƒΠ΅Ρ‚ написания ΠΈ Ρ€Π΅ΡˆΠ΅Π½ΠΈΡ Π½Π΅ΡΠΊΠΎΠ»ΡŒΠΊΠΈΡ… ΡƒΡ€Π°Π²Π½Π΅Π½ΠΈΠΉ сСтки. Π•ΡΡ‚ΡŒ ΠΌΠ½ΠΎΠ³ΠΎ возмоТностСй ΡΠΎΠ²Π΅Ρ€ΡˆΠΈΡ‚ΡŒ ΠΎΡˆΠΈΠ±ΠΊΡƒ.

Один ΠΈΠ· Π²Π°Ρ€ΠΈΠ°Π½Ρ‚ΠΎΠ², Ссли Ρƒ вас Π΅ΡΡ‚ΡŒ врСмя ΠΈ возмоТности, — это ΠΏΠΎΡΡ‚Ρ€ΠΎΠΈΡ‚ΡŒ модСль Π² симулятор схСмы, ΠΈ ΠΏΡƒΡΡ‚ΡŒ ΠΎΠ½ опрСдСляСт Ρ€Π΅Π·ΡƒΠ»ΡŒΡ‚Π°Ρ‚.Однако ΠΎΠ±Ρ‹Ρ‡Π½ΠΎ ΠΊΡƒΠ± Π½Π°Ρ‚ΠΎΠ»ΠΊΠ½ΡƒΡ‚ΡŒ Π½Π° вас Π² компромиссной ситуации, ΠΊΠ°ΠΊ Π½Π° собСсСдовании. Если Ρ‚Ρ‹ ΠΈΠ½ΠΆΠ΅Π½Π΅Ρ€-элСктрик ΠΈ Π½Π΅ ΠΌΠΎΠΆΠ΅Ρ‚ ΠΏΠΎΠ½ΡΡ‚ΡŒ это Π½Π° мСстС, Π·Π°Π±ΡƒΠ΄ΡŒΡ‚Π΅ эту схСму дизайнСрская Ρ€Π°Π±ΠΎΡ‚Π°. Если Π²Ρ‹ спСциалист ΠΏΠΎ элСктроникС, вас простят Π·Π° Ρ‚ΠΎ, Ρ‡Ρ‚ΠΎ Π²Ρ‹ Π½Π΅ Ρ€Π΅ΡˆΠΈΠ»ΠΈ это, Π½ΠΎ Π²Π°ΠΌ Π»ΡƒΡ‡ΡˆΠ΅ ΠΏΡ€ΠΎΠ΄Π΅ΠΌΠΎΠ½ΡΡ‚Ρ€ΠΈΡ€ΠΎΠ²Π°Ρ‚ΡŒ ΠΏΠΎΠ½ΠΈΠΌΠ°Π½ΠΈΠ΅ ΠΌΠ΅Ρ‚ΠΎΠ΄Π°, ΠΊΠ°ΠΊ Ρ‚ΠΎΠ»ΡŒΠΊΠΎ ΠΎΠ½ Π±ΡƒΠ΄Π΅Ρ‚ прСдставлСн.

Как оказалось, сущСствуСт ΠΎΡ‚Π½ΠΎΡΠΈΡ‚Π΅Π»ΡŒΠ½ΠΎ простой Π°Π½Π°Π»ΠΈΠ·, основанный Π½Π° симмСтрии ΠΈ Ρ„ΡƒΠ½Π΄Π°ΠΌΠ΅Π½Ρ‚Π°Π»ΡŒΠ½Ρ‹ΠΉ ΡƒΡ€ΠΎΠ²Π΅Π½ΡŒ понимания Ρ‚ΠΎΠΊΠΎΠ² ΠΈ напряТСний.

Π˜ΡΠΏΠΎΠ»ΡŒΠ·ΡƒΠ΅ΠΌΡ‹ΠΉ Ρ‚Ρ€Π°Π΄ΠΈΡ†ΠΈΠΎΠ½Π½Ρ‹ΠΉ ΠΌΠ΅Ρ‚ΠΎΠ΄ Π²ΠΊΠ»ΡŽΡ‡Π°Π΅Ρ‚ распознаваниС Π½Π°Π±ΠΎΡ€ΠΎΠ² ΡΠΊΠ²ΠΈΠΏΠΎΡ‚Π΅Π½Ρ†ΠΈΠ°Π»ΡŒΠ½Ρ‹Ρ… Ρ‚ΠΎΡ‡Π΅ΠΊ Π² Π²Π΅Ρ€ΡˆΠΈΠ½Π°Ρ… ΠΊΡƒΠ±Π°, Π·Π°Ρ‚Π΅ΠΌ Π·Π°ΠΌΡ‹ΠΊΠ°Π½ΠΈΠ΅ ΠΈΡ… вмСстС, Ρ‡Ρ‚ΠΎΠ±Ρ‹ ΠΌΠΎΠΆΠ½ΠΎ Π±Ρ‹Π»ΠΎ Ρ€Π°ΡΡΡ‡ΠΈΡ‚Π°Ρ‚ΡŒ ΠΏΠ°Ρ€Π°Π»Π»Π΅Π»ΡŒΠ½Ρ‹Π΅ сопротивлСния. НаконСц, эти сопротивлСния Π΄ΠΎΠ±Π°Π²Π»ΡΡŽΡ‚ΡΡ ΠΏΠΎΡΠ»Π΅Π΄ΠΎΠ²Π°Ρ‚Π΅Π»ΡŒΠ½ΠΎ, Ρ‡Ρ‚ΠΎΠ±Ρ‹ ΠΏΠΎΠ»ΡƒΡ‡ΠΈΡ‚ΡŒ эквивалСнтноС сопротивлСниС. ΠŸΡ€ΠΎΡ†Π΅ΡΡ ΠΏΡ€ΠΎΠΈΠ»Π»ΡŽΡΡ‚Ρ€ΠΈΡ€ΠΎΠ²Π°Π½ΠΎ Π½ΠΈΠΆΠ΅.

ПослС объяснСния Ρ‚Ρ€Π°Π΄ΠΈΡ†ΠΈΠΎΠ½Π½ΠΎΠ³ΠΎ ΠΌΠ΅Ρ‚ΠΎΠ΄Π° я ΠΏΡ€Π΅Π΄ΡΡ‚Π°Π²Π»ΡŽ ΠΌΠΎΠ΅ Ρ€Π΅ΡˆΠ΅Π½ΠΈΠ΅, ΠΊΠΎΡ‚ΠΎΡ€ΠΎΠ΅ Π½Π΅ΠΌΠ½ΠΎΠ³ΠΎ большС ΠΈΠ½Ρ‚ΡƒΠΈΡ‚ΠΈΠ²Π½ΠΎ понятный ΠΈ прямой ΠΌΠ΅Ρ‚ΠΎΠ΄ получСния ΠΎΠ΄Π½ΠΎΠ³ΠΎ ΠΈ Ρ‚ΠΎΠ³ΠΎ ΠΆΠ΅ ΠΎΡ‚Π²Π΅Ρ‚Π°.РСшСниС с ΠΏΠΎΠΌΠΎΡ‰ΡŒΡŽ Ρ‚Ρ€Π°Π΄ΠΈΡ†ΠΈΠΎΠ½Π½Ρ‹Ρ… ΠΌΠ΅Ρ‚ΠΎΠ΄ фактичСски Ρ‚Ρ€Π΅Π±ΡƒΠ΅Ρ‚ Ρ‚ΠΎΠ³ΠΎ ΠΆΠ΅ знания Ρ‚ΠΎΠ³ΠΎ, ΠΊΠ°ΠΊ Ρ‚ΠΎΠΊΠΈ дСлятся Π² ΡƒΠ·Π»Π°Ρ….

НаконСц, LTSpice ΠΈΡΠΏΠΎΠ»ΡŒΠ·ΡƒΠ΅Ρ‚ΡΡ для получСния ΠΎΡ‚Π²Π΅Ρ‚Π° Ρ‡Π΅Ρ€Π΅Π· схСмотСхничСский симулятор.

Π’ΠΎΡ‚ ΠŸΡ€ΠΎΠ±Π»Π΅ΠΌΠ° с ΡˆΠ΅ΡΡ‚ΠΈΠ³Ρ€Π°Π½Π½ΠΎΠΉ рСзисторной сСткой (# 4) для вас ΠΎΡ‚ Popular Electronics ΠΆΡƒΡ€Π½Π°Π».


Π’Ρ€Π°Π΄ΠΈΡ†ΠΈΠΎΠ½Π½Ρ‹ΠΉ ΠΌΠ΅Ρ‚ΠΎΠ΄ Ρ€Π΅ΡˆΠ΅Π½ΠΈΡ ΠŸΡ€ΠΎΠ±Π»Π΅ΠΌΠ° ΠšΡƒΠ±ΠΈΠΊΠ° РСзистора

Π­Ρ‚ΠΎ кубичСская структура, состоящая ΠΈΠ· 12 рСзисторов, элСктричСски соСдинСнных ΠΌΠ΅ΠΆΠ΄Ρƒ 8 Π²Π΅Ρ€ΡˆΠΈΠ½Π°ΠΌΠΈ.ΠšΠ°ΠΆΠ΄Ρ‹ΠΉ рСзистор ΠΈΠΌΠ΅Π΅Ρ‚ сопротивлСниС 1 Ом, Π½ΠΎ ΠΌΠΎΠΆΠ½ΠΎ ΠΈΡΠΏΠΎΠ»ΡŒΠ·ΠΎΠ²Π°Ρ‚ΡŒ любоС Π·Π½Π°Ρ‡Π΅Π½ΠΈΠ΅, поэтому ΠΏΠΎΠΊΠ° ΠΎΠ½ΠΈ всС ΠΎΠ΄ΠΈΠ½Π°ΠΊΠΎΠ²Ρ‹Π΅.

Π—Π΄Π΅ΡΡŒ Π² ΠΈΠ³Ρ€Ρƒ вступаСт интуиция. ЦвСтовая ΠΊΠΎΠ΄ΠΈΡ€ΠΎΠ²ΠΊΠ° ΠΈΡΠΏΠΎΠ»ΡŒΠ·ΡƒΠ΅Ρ‚ΡΡ, Ρ‡Ρ‚ΠΎΠ±Ρ‹ ΠΏΠΎΠΌΠΎΡ‡ΡŒ ΡΠΎΡ…Ρ€Π°Π½ΠΈΡ‚ΡŒ Π΄ΠΎΡ€ΠΎΠΆΠΊΠ° рСзисторов ΠΈ связанных ΡƒΠ·Π»ΠΎΠ² (Π½ΠΈΠΆΠ΅). Благодаря симмСтрии ΠΏΠΎΡ‚Π΅Π½Ρ†ΠΈΠ°Π» (напряТСниС) Π² Ρ‚Ρ€Π΅Ρ… ΡƒΠ·Π»Π°Ρ…, ΠΎΠ±ΠΎΠ·Π½Π°Ρ‡Π΅Π½Π½Ρ‹Ρ… «α», Ρ€Π°Π²Π½Ρ‹. ΠŸΠΎΡΠΊΠΎΠ»ΡŒΠΊΡƒ ΠΌΠ΅ΠΆΠ΄Ρƒ ΡƒΠ·Π»Π°ΠΌΠΈ с Ρ€Π°Π·Π½ΠΎΡΡ‚ΡŒΡŽ ΠΏΠΎΡ‚Π΅Π½Ρ†ΠΈΠ°Π»ΠΎΠ² 0 Π’ Π½Π΅Ρ‚ Ρ‚ΠΎΠΊΠ°, ΠΎΠ½ΠΈ ΠΌΠΎΠ³ΡƒΡ‚ Π·Π°ΠΊΠΎΡ€ΠΎΡ‚ΠΈΡ‚ΡŒ вмСстС, Π½Π΅ Π½Π°Ρ€ΡƒΡˆΠ°Ρ цСлостности Ρ†Π΅ΠΏΠΈ.Π’ΠΎ ΠΆΠ΅ самоС ΠΌΠΎΠΆΠ½ΠΎ ΡΠ΄Π΅Π»Π°Ρ‚ΡŒ для ΡƒΠ·Π»ΠΎΠ², ΠΏΠΎΠΌΠ΅Ρ‡Π΅Π½Π½Ρ‹Ρ… «β».

Π—Π°ΠΌΡ‹ΠΊΠ½ΡƒΠ² эти ΡƒΠ·Π»Ρ‹, Π²Ρ‹ ΠΏΠΎΠ»ΡƒΡ‡ΠΈΡ‚Π΅ ΡΠΊΠ²ΠΈΠ²Π°Π»Π΅Π½Ρ‚Π½ΡƒΡŽ схСму, ΠΏΠΎΠΊΠ°Π·Π°Π½Π½ΡƒΡŽ Π½ΠΈΠΆΠ΅. Π’ Π²ΠΈΠ΄Π΅ Π’Ρ‹ ΠΌΠΎΠΆΠ΅Ρ‚Π΅ Π²ΠΈΠ΄Π΅Ρ‚ΡŒ, Ρ‡Ρ‚ΠΎ Π΅ΡΡ‚ΡŒ Π΄Π²Π° Π½Π°Π±ΠΎΡ€Π° ΠΈΠ· Ρ‚Ρ€Π΅Ρ… рСзисторов ΠΏΠ°Ρ€Π°Π»Π»Π΅Π»ΡŒΠ½ΠΎ, ΠΏΠΎΡΠ»Π΅Π΄ΠΎΠ²Π°Ρ‚Π΅Π»ΡŒΠ½ΠΎ с ΠΎΠ΄Π½ΠΈΠΌ Π½Π°Π±ΠΎΡ€ ΠΈΠ· ΡˆΠ΅ΡΡ‚ΠΈ рСзисторов, Π²ΠΊΠ»ΡŽΡ‡Π΅Π½Π½Ρ‹Ρ… ΠΏΠ°Ρ€Π°Π»Π»Π΅Π»ΡŒΠ½ΠΎ. Π˜Ρ‚Π°ΠΊ, Ρƒ вас Π΅ΡΡ‚ΡŒ 1/3 Ом ΠΏΠΎΡΠ»Π΅Π΄ΠΎΠ²Π°Ρ‚Π΅Π»ΡŒΠ½ΠΎ с 1/6 Ом. ΠΏΠΎΡΠ»Π΅Π΄ΠΎΠ²Π°Ρ‚Π΅Π»ΡŒΠ½ΠΎ с 1/3 Ом, Ρ‡Ρ‚ΠΎ Ρ€Π°Π²Π½ΠΎ 5/6 Ом.


RF Cafe Бпособ Ρ€Π΅ΡˆΠ΅Π½ΠΈΡ ΠΏΡ€ΠΎΠ±Π»Π΅ΠΌΡ‹ РСзистор ΠšΡƒΠ± ΠŸΡ€ΠΎΠ±Π»Π΅ΠΌΠ°

Π’Π΅ΠΏΠ΅Ρ€ΡŒ я ΠΏΡ€Π΅Π΄ΡΡ‚Π°Π²Π»ΡŽ свой ΠΌΠ΅Ρ‚ΠΎΠ΄ Ρ€Π΅ΡˆΠ΅Π½ΠΈΡ Π·Π°Π΄Π°Ρ‡ΠΈ ΠΊΡƒΠ±ΠΈΠΊΠ° рСзистора.Π‘Ρ‚Ρ€ΡƒΠΊΡ‚ΡƒΡ€Π° повторяСтся здСсь снова.

Π’Π΅ΠΊΡƒΡ‰ΠΈΠΉ Π·Π°ΠΊΠΎΠ½ ΠšΠΈΡ€Ρ…Π³ΠΎΡ„Π°, ΠΊΠΎΡ‚ΠΎΡ€Ρ‹ΠΉ гласит, Ρ‡Ρ‚ΠΎ сумма входящСго ΠΈ Π²Ρ‹Ρ…ΠΎΠ΄ ΠΈΠ· ΡƒΠ·Π»Π° Ρ€Π°Π²Π΅Π½ Π½ΡƒΠ»ΡŽ, Π²Π°ΠΆΠ΅Π½ для Π°Π½Π°Π»ΠΈΠ·Π°.

ΠŸΠ΅Ρ€Π²Ρ‹ΠΉ шаг — ΠΏΡ€ΠΈΠ·Π½Π°Ρ‚ΡŒ, Ρ‡Ρ‚ΠΎ Π² ΡƒΠ·Π»Π΅, Π³Π΄Π΅ ΡΡƒΡ‰Π΅ΡΡ‚Π²ΡƒΡŽΡ‚ Ρ€Π°Π²Π½Ρ‹Π΅ сопротивлСния, Ρ‚ΠΎΠΊ, ΠΏΠΎΡΡ‚ΡƒΠΏΠ°ΡŽΡ‰ΠΈΠΉ Π² ΡƒΠ·Π΅Π», Π±ΡƒΠ΄Π΅Ρ‚ Ρ€Π°Π²Π½ΠΎΠΌΠ΅Ρ€Π½ΠΎ Ρ€Π°ΡΠΏΡ€Π΅Π΄Π΅Π»ΡΡ‚ΡŒΡΡ ΠΌΠ΅ΠΆΠ΄Ρƒ Π½ΠΎΠΌΠ΅Ρ€Π°ΠΌΠΈ Π²Ρ‹Ρ…ΠΎΠ΄Π½Ρ‹Ρ… Π²Π΅Ρ‚ΠΎΠΊ — Π² Π΄Π°Π½Π½ΠΎΠΌ случаС Ρ‚Ρ€ΠΈ. Для удобства я Π½Π°Π·Π½Π°Ρ‡ΠΈΠ» Π²Ρ…ΠΎΠ΄Π½ΠΎΠΉ Ρ‚ΠΎΠΊ 3 Π°ΠΌΠΏΠ΅Ρ€Π° Π² ΡƒΠ³Π»Ρƒ с надписью «A», Ρ‡Ρ‚ΠΎΠ±Ρ‹ Ρ‡Π΅Ρ€Π΅Π· ΠΊΠ°ΠΆΠ΄Ρ‹ΠΉ Π²Ρ‹Ρ…ΠΎΠ΄ ΠΏΡ€ΠΎΡ…ΠΎΠ΄ΠΈΠ» 1 Π°ΠΌΠΏΠ΅Ρ€. Π²Π΅Ρ‚Π²ΡŒ.ΠžΠ±Ρ€Π°Ρ‚ΠΈΡ‚Π΅ Π²Π½ΠΈΠΌΠ°Π½ΠΈΠ΅, Ρ‡Ρ‚ΠΎ Ρ‡Π΅Ρ€Π΅Π· ΠΊΠ°ΠΆΠ΄ΡƒΡŽ Π²Π΅Ρ‚Π²ΡŒ ΠΏΡ€ΠΎΡ‚Π΅ΠΊΠ°Π΅Ρ‚ Ρ‚ΠΎΠΊ 1 А.

На дальнСй сторонС ΠΊΠ°ΠΆΠ΄ΠΎΠΉ ΠΈΠ· этих Π²Π΅Ρ‚Π²Π΅ΠΉ находится Π΅Ρ‰Π΅ ΠΎΠ΄ΠΈΠ½ ΡƒΠ·Π΅Π» с двумя Π²Ρ‹Ρ…ΠΎΠ΄Π½Ρ‹ΠΌΠΈ вСтвями. ΠžΠΏΡΡ‚ΡŒ ΠΆΠ΅, ΠΈΠ·-Π·Π° симмСтрии, Π²Ρ…ΠΎΠ΄Π½ΠΎΠΉ Ρ‚ΠΎΠΊ Π±ΡƒΠ΄Π΅Ρ‚ Π΄Π΅Π»ΠΈΡ‚ΡŒΡΡ Ρ€Π°Π²Π½ΠΎΠΌΠ΅Ρ€Π½ΠΎ, Ρ‚Π°ΠΊ Ρ‡Ρ‚ΠΎ Β½ A Ρ‚Π΅Ρ‡Π΅Ρ‚ Π² каТдая Π²Π΅Ρ‚ΠΊΠ°. Глядя Π½Π° Π²Ρ‹Ρ…ΠΎΠ΄Π½ΠΎΠΉ ΡƒΠ·Π΅Π» ΠΊΡƒΠ±Π°, ΠΏΠΎΠΌΠ΅Ρ‡Π΅Π½Π½Ρ‹ΠΉ «B», становится ΠΎΡ‡Π΅Π²ΠΈΠ΄Π½Ρ‹ΠΌ, Ρ‡Ρ‚ΠΎ такая ΠΆΠ΅ ситуация, ΠΊΠ°ΠΊ ΠΈ с «А».

НайдитС ΠΌΠΎΠΌΠ΅Π½Ρ‚, Ρ‡Ρ‚ΠΎΠ±Ρ‹ ΠΏΡ€ΠΎΡΡƒΠΌΠΌΠΈΡ€ΠΎΠ²Π°Ρ‚ΡŒ Ρ‚ΠΎΠΊΠΈ Π½Π° Π²Ρ…ΠΎΠ΄Π΅ ΠΈ Π²Ρ‹Ρ…ΠΎΠ΄Π΅ ΠΈΠ· ΠΊΠ°ΠΆΠ΄ΠΎΠ³ΠΎ ΡƒΠ·Π»Π°, Ρ‡Ρ‚ΠΎΠ±Ρ‹ ΡƒΠ±Π΅Π΄ΠΈΡ‚ΡŒΡΡ, Ρ‡Ρ‚ΠΎ ΠΎΠ½ΠΈ всС ΡΠΊΠ»Π°Π΄Ρ‹Π²Π°ΡŽΡ‚ΡΡ ΠΏΠΎ ΠΌΠ΅Ρ€Π΅ нСобходимости.

Π’Π΅ΠΏΠ΅Ρ€ΡŒ, ΠΊΠΎΠ³Π΄Π° Π²Ρ‹ Π·Π½Π°Π΅Ρ‚Π΅ Ρ‚ΠΎΠΊ Ρ‡Π΅Ρ€Π΅Π· ΠΊΠ°ΠΆΠ΄ΡƒΡŽ Π²Π΅Ρ‚Π²ΡŒ, ΠΈ Π²Ρ‹ Π·Π½Π°Π΅Ρ‚Π΅, Ρ‡Ρ‚ΠΎ каТдая Π²Π΅Ρ‚Π²ΡŒ Π² Π½Π΅ΠΌ Π΅ΡΡ‚ΡŒ СдинствСнный рСзистор 1 Ом, Π·Π°ΠΊΠΎΠ½ Ома позволяСт Ρ€Π°ΡΡΡ‡ΠΈΡ‚Π°Ρ‚ΡŒ напряТСниС Π½Π° ΠΊΠ°ΠΆΠ΄ΠΎΠΌ рСзисторС.

Π‘Π»Π΅Π΄ΡƒΡŽΡ‰ΠΈΠΌ шагом являСтся суммированиС напряТСния ΠΎΡ‚ Π²Ρ…ΠΎΠ΄Π½ΠΎΠ³ΠΎ ΡƒΠ·Π»Π° Β«AΒ» ΠΊ Π²Ρ‹Ρ…ΠΎΠ΄Π½ΠΎΠΌΡƒ ΡƒΠ·Π»Ρƒ Β«BΒ». Π›ΡŽΠ±ΠΎΠΉ ΠŸΡƒΡ‚ΡŒ, ΠΏΠΎ ΠΊΠΎΡ‚ΠΎΡ€ΠΎΠΌΡƒ Π²Ρ‹ ΠΈΠ΄Π΅Ρ‚Π΅, ΠΏΡ€ΠΎΡ…ΠΎΠ΄ΠΈΡ‚ ΠΏΠΎ Ρ‚Ρ€Π΅ΠΌ краям, ΠΈ всС ΠΎΠ½ΠΈ ΡΠΎΡΡ‚Π°Π²Π»ΡΡŽΡ‚ 2Β½ Π²ΠΎΠ»ΡŒΡ‚Π°.

НаконСц, ΠΏΡ€ΠΈΠΌΠ΅Π½ΠΈΡ‚Π΅ Π·Π°ΠΊΠΎΠ½ Ома, ΠΊΠΎΡ‚ΠΎΡ€Ρ‹ΠΉ гласит, Ρ‡Ρ‚ΠΎ сопротивлСниС Ρ€Π°Π²Π½ΠΎ Π½Π°ΠΏΡ€ΡΠΆΠ΅Π½ΠΈΡŽ дСлится Π½Π° Ρ‚Π΅ΠΊΡƒΡ‰Π΅Π΅. Как ΠΈ Π² случаС с Π΄Ρ€ΡƒΠ³ΠΈΠΌ ΠΌΠ΅Ρ‚ΠΎΠ΄ΠΎΠΌ Π°Π½Π°Π»ΠΈΠ·Π°, ΠΏΠΎΠ»ΡƒΡ‡Π΅Π½Π½Ρ‹ΠΉ эквивалСнт сопротивлСниС 5/6 Ом.

Π’Ρ‹ ΠΌΠΎΠΆΠ΅Ρ‚Π΅ Π²ΠΈΠ΄Π΅Ρ‚ΡŒ, Ρ‡Ρ‚ΠΎ Π½Π° самом Π΄Π΅Π»Π΅, имСя Π²ΠΎΠ·ΠΌΠΎΠΆΠ½ΠΎΡΡ‚ΡŒ Π΄Π΅Π»Π°Ρ‚ΡŒ прСдполоТСния Π² Ρ‚Ρ€Π°Π΄ΠΈΡ†ΠΈΠΎΠ½Π½Ρ‹Ρ… РСшСниС Ρ‚Ρ€Π΅Π±ΡƒΠ΅Ρ‚ понимания Ρ‚Π΅ΠΊΡƒΡ‰ΠΈΡ… ΠΏΡ€ΠΈΠ½Ρ†ΠΈΠΏΠΎΠ² раздСлСния Π² ΠΌΠΎΠ΅ΠΌ ΠΌΠ΅Ρ‚ΠΎΠ΄Π΅.Π˜Ρ‚Π°ΠΊ, ИМΠ₯О, ΠΏΡ€ΠΎΡ‰Π΅ Π΄ΠΎΠ±Π°Π²ΠΈΡ‚ΡŒ напряТСния, Π° Π·Π°Ρ‚Π΅ΠΌ ΠΏΠΎΠ΄ΠΊΠ»ΡŽΡ‡ΠΈΡ‚ΡŒ напряТСниС ΠΈ Ρ‚ΠΎΠΊ ΠΊ ΠžΠΌΡƒ. Π·Π°ΠΊΠΎΠ½, Ρ‡Ρ‚ΠΎΠ±Ρ‹ ΠΏΡ€ΠΈΠΉΡ‚ΠΈ ΠΊ ΠΎΡ‚Π²Π΅Ρ‚Ρƒ, Ρ‡Π΅ΠΌ Ρ€ΠΈΡΠΊΠΎΠ²Π°Ρ‚ΡŒ, закорачивая ΡƒΠ·Π»Ρ‹ Π½Π΅ΠΏΡ€Π°Π²ΠΈΠ»ΡŒΠ½ΠΎ. Π­Ρ‚ΠΎ Ρ‚Π°ΠΊ просто ΠΏΠ΅Ρ‰Π΅Ρ€Π½Ρ‹ΠΉ Ρ‡Π΅Π»ΠΎΠ²Π΅ΠΊ ΠΌΠΎΠ³ это ΡΠ΄Π΅Π»Π°Ρ‚ΡŒ.


ΠœΠ΅Ρ‚ΠΎΠ΄ модСлирования Ρ†Π΅ΠΏΠΈ Ρ€Π΅ΡˆΠ΅Π½ΠΈΡ Π·Π°Π΄Π°Ρ‡ΠΈ рСзисторного ΠΊΡƒΠ±Π°

Для ΠΏΡ€ΠΎΠ²Π΅Ρ€ΠΊΠΈ Ρ€Π΅Π·ΡƒΠ»ΡŒΡ‚Π°Ρ‚Π° Π±Ρ‹Π»Π° смодСлирована схСма рСзисторного ΠΊΡƒΠ±Π° с использованиСм бСсплатная ΠΏΡ€ΠΎΠ³Ρ€Π°ΠΌΠΌΠ° LTSpice ΠΎΡ‚ Linear Technology. РСзисторы ΠΎΠ±ΠΎΠ·Π½Π°Ρ‡Π΅Π½Ρ‹ Π² соотвСтствии с этикСтками Π² Ρ‚Ρ€Π°Π΄ΠΈΡ†ΠΈΠΎΠ½Π½Ρ‹Ρ… ΠΌΠ΅Ρ‚ΠΎΠ΄ Π°Π½Π°Π»ΠΈΠ·Π°.Π˜ΡΡ‚ΠΎΡ‡Π½ΠΈΠΊ Ρ‚ΠΎΠΊΠ° Π½Π° 3 А помСщаСтся Π²ΠΎ Π²Ρ…ΠΎΠ΄Π½ΠΎΠΉ ΡƒΠ·Π΅Π» N001 . Π Π΅Π·ΡƒΠ»ΡŒΡ‚ΠΈΡ€ΡƒΡŽΡ‰Π΅Π΅ напряТСниС составляСт ΠΏΡ€ΠΎΠ³Π½ΠΎΠ·ΠΈΡ€ΡƒΠ΅ΠΌΡ‹Π΅ 2,5 Π’. ΠžΠΏΡΡ‚ΡŒ ΠΆΠ΅, Π·Π°ΠΊΠΎΠ½ Ома для 3 Π°ΠΌΠΏΠ΅Ρ€ ΠΈ 2,5 Π’. Π²ΠΎΠ»ΡŒΡ‚ Π΄Π°Π΅Ρ‚ сопротивлСниС 5/6 Ом.

RΒ§b N001 N003 1 ΠžΡ‚Ρ‡Π΅Ρ‚ ΠΎΠ± Π°Π½Π°Π»ΠΈΠ·Π΅: Π’ (n001): 2,5 напряТСния
R§d N002 N004 1
R§a N002 N001 1
R§f N004 N003 1
R§h N005 N007 1
R§l N006 0 1
R§i N006 N005 1
R§j 0 N007 1
R§g N003 N007 1
R§k N004 0 1
R§e N002 N006 1
R§c N001 N005 1
I1 0 N001 3
.op
.backanno
.end

ΠžΠΏΡƒΠ±Π»ΠΈΠΊΠΎΠ²Π°Π½ΠΎ 22 ноября 2019 Π³. (ΠΎΡ€ΠΈΠ³ΠΈΠ½Π°Π» 6/11/2010)


Бпасибо ΠΏΠΎΡΠ΅Ρ‚ΠΈΡ‚Π΅Π»ΡŽ RF Cafe Les Carpenter Π·Π° Ρ‚ΠΎ, Ρ‡Ρ‚ΠΎ прислали ΠΌΠ½Π΅ это Ρ€Π΅ΡˆΠ΅Π½ΠΈΠ΅, ΠΊΠΎΡ‚ΠΎΡ€ΠΎΠ΅ мСняСт рСзисторы Π² ΠΊΠΎΠ½Ρ„ΠΈΠ³ΡƒΡ€Π°Ρ†ΠΈΠΈ Ρ‚Ρ€Π΅ΡƒΠ³ΠΎΠ»ΡŒΠ½ΠΈΠΊΠΎΠΌ ΠΈ Π·Π²Π΅Π·Π΄ΠΎΠΉ Ρ‡Ρ‚ΠΎΠ±Ρ‹ Β«ΡƒΠΏΡ€ΠΎΡΡ‚ΠΈΡ‚ΡŒΒ» Ρ€Π΅ΡˆΠ΅Π½ΠΈΠ΅. Моя Π³ΠΎΠ»ΠΎΠ²Π° всС Π΅Ρ‰Π΅ Π±ΠΎΠ»ΠΈΡ‚, глядя Π½Π° это.

ΠžΠΏΡƒΠ±Π»ΠΈΠΊΠΎΠ²Π°Π½ΠΎ: 27 июня, 2010 Π³.

Как Ρ€Π°ΡΡΡ‡ΠΈΡ‚Π°Ρ‚ΡŒ ΠΏΠ°Π΄Π΅Π½ΠΈΠ΅ напряТСния Π½Π° рСзисторС Π² ΠΏΠ°Ρ€Π°Π»Π»Π΅Π»ΡŒΠ½ΠΎΠΉ Ρ†Π΅ΠΏΠΈ

ОбновлСно 28 дСкабря 2020 Π³.

Автор: S.Hussain Ather

β€’β€’β€’ Syed Hussain Ather

TL; DR (слишком Π΄ΠΎΠ»Π³ΠΎ; Π½Π΅ Ρ‡ΠΈΡ‚Π°Π»)

На ΠΏΡ€ΠΈΠ²Π΅Π΄Π΅Π½Π½ΠΎΠΉ Π²Ρ‹ΡˆΠ΅ схСмС ΠΏΠ°Ρ€Π°Π»Π»Π΅Π»ΡŒΠ½ΠΎΠΉ Ρ†Π΅ΠΏΠΈ ΠΏΠ°Π΄Π΅Π½ΠΈΠ΅ напряТСния ΠΌΠΎΠΆΠ½ΠΎ Π½Π°ΠΉΡ‚ΠΈ, слоТив сопротивлСния ΠΊΠ°ΠΆΠ΄ΠΎΠ³ΠΎ рСзистора ΠΈ ΠΎΠΏΡ€Π΅Π΄Π΅Π»ΠΈΠ², ΠΊΠ°ΠΊΠΎΠ΅ напряТСниС Π²Ρ‹Ρ‚Π΅ΠΊΠ°Π΅Ρ‚ ΠΈΠ· Ρ‚ΠΎΠΊΠ° Π² этой ΠΊΠΎΠ½Ρ„ΠΈΠ³ΡƒΡ€Π°Ρ†ΠΈΠΈ. Π­Ρ‚ΠΈ ΠΏΡ€ΠΈΠΌΠ΅Ρ€Ρ‹ ΠΏΠ°Ρ€Π°Π»Π»Π΅Π»ΡŒΠ½Ρ‹Ρ… Ρ†Π΅ΠΏΠ΅ΠΉ ΠΈΠ»Π»ΡŽΡΡ‚Ρ€ΠΈΡ€ΡƒΡŽΡ‚ ΠΊΠΎΠ½Ρ†Π΅ΠΏΡ†ΠΈΠΈ Ρ‚ΠΎΠΊΠ° ΠΈ напряТСния Π² Ρ€Π°Π·Π½Ρ‹Ρ… вСтвях.

На схСмС ΠΏΠ°Ρ€Π°Π»Π»Π΅Π»ΡŒΠ½ΠΎΠΉ Ρ†Π΅ΠΏΠΈ, ΠΏΠ°Π΄Π΅Π½ΠΈΠ΅ напряТСния Π½Π° рСзисторС Π² ΠΏΠ°Ρ€Π°Π»Π»Π΅Π»ΡŒΠ½ΠΎΠΉ Ρ†Π΅ΠΏΠΈ ΠΎΠ΄ΠΈΠ½Π°ΠΊΠΎΠ²ΠΎ для всСх рСзисторов Π² ΠΊΠ°ΠΆΠ΄ΠΎΠΉ Π²Π΅Ρ‚Π²ΠΈ ΠΏΠ°Ρ€Π°Π»Π»Π΅Π»ΡŒΠ½ΠΎΠΉ Ρ†Π΅ΠΏΠΈ.НапряТСниС, Π²Ρ‹Ρ€Π°ΠΆΠ΅Π½Π½ΠΎΠ΅ Π² Π²ΠΎΠ»ΡŒΡ‚Π°Ρ…, измСряСт ΡΠ»Π΅ΠΊΡ‚Ρ€ΠΎΠ΄Π²ΠΈΠΆΡƒΡ‰ΡƒΡŽ силу ΠΈΠ»ΠΈ Ρ€Π°Π·Π½ΠΎΡΡ‚ΡŒ ΠΏΠΎΡ‚Π΅Π½Ρ†ΠΈΠ°Π»ΠΎΠ² Π² Ρ†Π΅ΠΏΠΈ.

Если Ρƒ вас Π΅ΡΡ‚ΡŒ Ρ†Π΅ΠΏΡŒ с извСстной Π²Π΅Π»ΠΈΡ‡ΠΈΠ½ΠΎΠΉ Ρ‚ΠΎΠΊΠ°, , ΠΏΠΎΡ‚ΠΎΠΊΠ° элСктричСского заряда, Π²Ρ‹ ΠΌΠΎΠΆΠ΅Ρ‚Π΅ Ρ€Π°ΡΡΡ‡ΠΈΡ‚Π°Ρ‚ΡŒ ΠΏΠ°Π΄Π΅Π½ΠΈΠ΅ напряТСния Π² схСмах ΠΏΠ°Ρ€Π°Π»Π»Π΅Π»ΡŒΠ½ΠΎΠΉ Ρ†Π΅ΠΏΠΈ ΡΠ»Π΅Π΄ΡƒΡŽΡ‰ΠΈΠΌ ΠΎΠ±Ρ€Π°Π·ΠΎΠΌ:

  1. ΠžΠΏΡ€Π΅Π΄Π΅Π»ΠΈΡ‚Π΅ объСдинСнноС сопротивлСниС ΠΈΠ»ΠΈ сопротивлСниС ΠΊ ΠΏΠΎΡ‚ΠΎΠΊΡƒ заряда ΠΏΠ°Ρ€Π°Π»Π»Π΅Π»ΡŒΠ½Ρ‹Ρ… рСзисторов. Π‘ΡƒΠΌΠΌΠΈΡ€ΡƒΠΉΡ‚Π΅ ΠΈΡ… ΠΊΠ°ΠΊ 1/ Ρ€ΡƒΠΏΠΈΠΉ всСго = 1/ 1 + 1/ 2 … для ΠΊΠ°ΠΆΠ΄ΠΎΠ³ΠΎ рСзистора. Для ΠΏΡ€ΠΈΠ²Π΅Π΄Π΅Π½Π½ΠΎΠΉ Π²Ρ‹ΡˆΠ΅ ΠΏΠ°Ρ€Π°Π»Π»Π΅Π»ΡŒΠ½ΠΎΠΉ Ρ†Π΅ΠΏΠΈ ΠΏΠΎΠ»Π½ΠΎΠ΅ сопротивлСниС ΠΌΠΎΠΆΠ½ΠΎ Π½Π°ΠΉΡ‚ΠΈ ΠΊΠ°ΠΊ:
    1. 1 / R всСго = 1/5 Ом + 1/6 Ом + 1/10 Ом
    2. 1 / R всСго = 6/30 Ом + 5/30 Ом + 3/30 Ом
    3. 1 / R всСго = 14/30 Ом
    4. R всСго = 30/14 Ом = 15/7 Ом
  2. Π£ΠΌΠ½ΠΎΠΆΡŒΡ‚Π΅ Ρ‚ΠΎΠΊ Π½Π° ΠΎΠ±Ρ‰Π΅Π΅ сопротивлСниС, Ρ‡Ρ‚ΠΎΠ±Ρ‹ ΠΏΠΎΠ»ΡƒΡ‡ΠΈΡ‚ΡŒ ΠΏΠ°Π΄Π΅Π½ΠΈΠ΅ напряТСния Π² соотвСтствии с Π·Π°ΠΊΠΎΠ½ΠΎΠΌ Ома Π’ = IR .Π­Ρ‚ΠΎ Ρ€Π°Π²Π½ΠΎ падСнию напряТСния Π²ΠΎ всСй ΠΏΠ°Ρ€Π°Π»Π»Π΅Π»ΡŒΠ½ΠΎΠΉ Ρ†Π΅ΠΏΠΈ ΠΈ Π½Π° ΠΊΠ°ΠΆΠ΄ΠΎΠΌ рСзисторС Π² ΠΏΠ°Ρ€Π°Π»Π»Π΅Π»ΡŒΠ½ΠΎΠΉ Ρ†Π΅ΠΏΠΈ. Π’ этом ΠΏΡ€ΠΈΠΌΠ΅Ρ€Π΅ ΠΏΠ°Π΄Π΅Π½ΠΈΠ΅ напряТСния Ρ€Π°Π²Π½ΠΎ Π’ = 5 А x 15/7 Ом = 75/7 Π’.

Π­Ρ‚ΠΎΡ‚ ΠΌΠ΅Ρ‚ΠΎΠ΄ Ρ€Π΅ΡˆΠ΅Π½ΠΈΡ ΡƒΡ€Π°Π²Π½Π΅Π½ΠΈΠΉ Ρ€Π°Π±ΠΎΡ‚Π°Π΅Ρ‚, ΠΏΠΎΡ‚ΠΎΠΌΡƒ Ρ‡Ρ‚ΠΎ Ρ‚ΠΎΠΊ, входящий Π² Π»ΡŽΠ±ΡƒΡŽ Ρ‚ΠΎΡ‡ΠΊΡƒ ΠΏΠ°Ρ€Π°Π»Π»Π΅Π»ΡŒΠ½ΠΎΠΉ Ρ†Π΅ΠΏΠΈ, Π΄ΠΎΠ»ΠΆΠ΅Π½ Π±Ρ‹Ρ‚ΡŒ Ρ€Π°Π²Π΅Π½ Ρ‚Π΅ΠΊΡƒΡ‰ΠΈΠΉ ΡƒΡ…ΠΎΠ΄. Π­Ρ‚ΠΎ происходит ΠΈΠ·-Π·Π° Ρ‚Π΅ΠΊΡƒΡ‰Π΅Π³ΠΎ Π·Π°ΠΊΠΎΠ½Π° ΠšΠΈΡ€Ρ…Π³ΠΎΡ„Π°, ΠΊΠΎΡ‚ΠΎΡ€Ρ‹ΠΉ гласит: «алгСбраичСская сумма Ρ‚ΠΎΠΊΠΎΠ² Π² сСти ΠΏΡ€ΠΎΠ²ΠΎΠ΄Π½ΠΈΠΊΠΎΠ², Π²ΡΡ‚Ρ€Π΅Ρ‡Π°ΡŽΡ‰ΠΈΡ…ΡΡ Π² ΠΎΠ΄Π½ΠΎΠΉ Ρ‚ΠΎΡ‡ΠΊΠ΅, Ρ€Π°Π²Π½Π° Π½ΡƒΠ»ΡŽ.»ΠšΠ°Π»ΡŒΠΊΡƒΠ»ΡΡ‚ΠΎΡ€ ΠΏΠ°Ρ€Π°Π»Π»Π΅Π»ΡŒΠ½ΠΎΠΉ Ρ†Π΅ΠΏΠΈ ΠΌΠΎΠ³ Π±Ρ‹ ΠΈΡΠΏΠΎΠ»ΡŒΠ·ΠΎΠ²Π°Ρ‚ΡŒ этот Π·Π°ΠΊΠΎΠ½ Π² вСтвях ΠΏΠ°Ρ€Π°Π»Π»Π΅Π»ΡŒΠ½ΠΎΠΉ Ρ†Π΅ΠΏΠΈ.

Если ΠΌΡ‹ сравним Ρ‚ΠΎΠΊ, входящий Π² Ρ‚Ρ€ΠΈ Π²Π΅Ρ‚Π²ΠΈ ΠΏΠ°Ρ€Π°Π»Π»Π΅Π»ΡŒΠ½ΠΎΠΉ Ρ†Π΅ΠΏΠΈ, ΠΎΠ½ Π΄ΠΎΠ»ΠΆΠ΅Π½ Ρ€Π°Π²Π½ΡΡ‚ΡŒΡΡ ΠΎΠ±Ρ‰Π΅ΠΌΡƒ Ρ‚ΠΎΠΊΡƒ, выходящСму ΠΈΠ· Π²Π΅Ρ‚Π²Π΅ΠΉ. ΠŸΠΎΡΠΊΠΎΠ»ΡŒΠΊΡƒ ΠΏΠ°Π΄Π΅Π½ΠΈΠ΅ напряТСния остаСтся постоянная Π½Π° ΠΊΠ°ΠΆΠ΄ΠΎΠΌ ΠΏΠ°Ρ€Π°Π»Π»Π΅Π»ΡŒΠ½ΠΎΠΌ рСзисторС, это ΠΏΠ°Π΄Π΅Π½ΠΈΠ΅ напряТСния, Π²Ρ‹ ΠΌΠΎΠΆΠ΅Ρ‚Π΅ ΡΡƒΠΌΠΌΠΈΡ€ΠΎΠ²Π°Ρ‚ΡŒ сопротивлСниС ΠΊΠ°ΠΆΠ΄ΠΎΠ³ΠΎ рСзистора, Ρ‡Ρ‚ΠΎΠ±Ρ‹ ΠΏΠΎΠ»ΡƒΡ‡ΠΈΡ‚ΡŒ ΠΎΠ±Ρ‰Π΅Π΅ сопротивлСниС ΠΈ ΠΎΠΏΡ€Π΅Π΄Π΅Π»ΠΈΡ‚ΡŒ напряТСниС ΠΏΠΎ этому Π·Π½Π°Ρ‡Π΅Π½ΠΈΡŽ. ΠŸΡ€ΠΈΠΌΠ΅Ρ€Ρ‹ ΠΏΠ°Ρ€Π°Π»Π»Π΅Π»ΡŒΠ½ΠΎΠΉ Ρ†Π΅ΠΏΠΈ ΠΏΠΎΠΊΠ°Π·Ρ‹Π²Π°ΡŽΡ‚ это.

ПадСниС напряТСния Π² ΠΏΠΎΡΠ»Π΅Π΄ΠΎΠ²Π°Ρ‚Π΅Π»ΡŒΠ½ΠΎΠΉ Ρ†Π΅ΠΏΠΈ

β€’β€’ β€’ Syed Hussain Ather

Π’ ΠΏΠΎΡΠ»Π΅Π΄ΠΎΠ²Π°Ρ‚Π΅Π»ΡŒΠ½ΠΎΠΉ Ρ†Π΅ΠΏΠΈ, с Π΄Ρ€ΡƒΠ³ΠΎΠΉ стороны, Π²Ρ‹ ΠΌΠΎΠΆΠ΅Ρ‚Π΅ Ρ€Π°ΡΡΡ‡ΠΈΡ‚Π°Ρ‚ΡŒ ΠΏΠ°Π΄Π΅Π½ΠΈΠ΅ напряТСния Π½Π° ΠΊΠ°ΠΆΠ΄ΠΎΠΌ рСзисторС, зная, Ρ‡Ρ‚ΠΎ Π² ΠΏΠΎΡΠ»Π΅Π΄ΠΎΠ²Π°Ρ‚Π΅Π»ΡŒΠ½ΠΎΠΉ Ρ†Π΅ΠΏΠΈ Ρ‚ΠΎΠΊ постоянСн Π½Π° всСм протяТСнии.Π­Ρ‚ΠΎ ΠΎΠ·Π½Π°Ρ‡Π°Π΅Ρ‚, Ρ‡Ρ‚ΠΎ ΠΏΠ°Π΄Π΅Π½ΠΈΠ΅ напряТСния Π½Π° ΠΊΠ°ΠΆΠ΄ΠΎΠΌ рСзисторС Ρ€Π°Π·Π½ΠΎΠ΅ ΠΈ зависит ΠΎΡ‚ сопротивлСния Π² соотвСтствии с Π·Π°ΠΊΠΎΠ½ΠΎΠΌ Ома V = IR . Π’ ΠΏΡ€ΠΈΠ²Π΅Π΄Π΅Π½Π½ΠΎΠΌ Π²Ρ‹ΡˆΠ΅ ΠΏΡ€ΠΈΠΌΠ΅Ρ€Π΅ ΠΏΠ°Π΄Π΅Π½ΠΈΠ΅ напряТСния Π½Π° ΠΊΠ°ΠΆΠ΄ΠΎΠΌ рСзисторС составляСт:

V_1 = R_1I = 3 \ times 3 = 9 \ text {V} \\ V_2 = R_2I = 10 \ times 3 = 30 \ text {V} \\ V_3 = R_3I = 5 \ times 3 = 15 \ text {V}

Π‘ΡƒΠΌΠΌΠ° ΠΊΠ°ΠΆΠ΄ΠΎΠ³ΠΎ падСния напряТСния Π΄ΠΎΠ»ΠΆΠ½Π° Π±Ρ‹Ρ‚ΡŒ Ρ€Π°Π²Π½Π° Π½Π°ΠΏΡ€ΡΠΆΠ΅Π½ΠΈΡŽ Π±Π°Ρ‚Π°Ρ€Π΅ΠΈ Π² ΠΏΠΎΡΠ»Π΅Π΄ΠΎΠ²Π°Ρ‚Π΅Π»ΡŒΠ½ΠΎΠΉ Ρ†Π΅ΠΏΠΈ. Π­Ρ‚ΠΎ ΠΎΠ·Π½Π°Ρ‡Π°Π΅Ρ‚, Ρ‡Ρ‚ΠΎ наша батарСя ΠΈΠΌΠ΅Π΅Ρ‚ напряТСниС 54 Π’.

Π­Ρ‚ΠΎΡ‚ ΠΌΠ΅Ρ‚ΠΎΠ΄ Ρ€Π΅ΡˆΠ΅Π½ΠΈΡ ΡƒΡ€Π°Π²Π½Π΅Π½ΠΈΠΉ Ρ€Π°Π±ΠΎΡ‚Π°Π΅Ρ‚, ΠΏΠΎΡ‚ΠΎΠΌΡƒ Ρ‡Ρ‚ΠΎ ΠΏΠ°Π΄Π΅Π½ΠΈΠ΅ напряТСния Π½Π° всСх рСзисторах, установлСнных ΠΏΠΎΡΠ»Π΅Π΄ΠΎΠ²Π°Ρ‚Π΅Π»ΡŒΠ½ΠΎ, Π΄ΠΎΠ»ΠΆΠ½ΠΎ Π² суммС Ρ€Π°Π²Π½ΡΡ‚ΡŒΡΡ ΠΎΠ±Ρ‰Π΅ΠΌΡƒ Π½Π°ΠΏΡ€ΡΠΆΠ΅Π½ΠΈΡŽ ΠΏΠΎΡΠ»Π΅Π΄ΠΎΠ²Π°Ρ‚Π΅Π»ΡŒΠ½ΠΎΠΉ Ρ†Π΅ΠΏΠΈ.Π­Ρ‚ΠΎ происходит ΠΈΠ·-Π·Π° Π·Π°ΠΊΠΎΠ½Π° напряТСния ΠšΠΈΡ€Ρ…Π³ΠΎΡ„Π°, ΠΊΠΎΡ‚ΠΎΡ€Ρ‹ΠΉ гласит, Ρ‡Ρ‚ΠΎ «направлСнная сумма разностСй ΠΏΠΎΡ‚Π΅Π½Ρ†ΠΈΠ°Π»ΠΎΠ² (напряТСний) Π²ΠΎΠΊΡ€ΡƒΠ³ любого Π·Π°ΠΌΠΊΠ½ΡƒΡ‚ΠΎΠ³ΠΎ ΠΊΠΎΠ½Ρ‚ΡƒΡ€Π° Ρ€Π°Π²Π½Π° Π½ΡƒΠ»ΡŽΒ». Π­Ρ‚ΠΎ ΠΎΠ·Π½Π°Ρ‡Π°Π΅Ρ‚, Ρ‡Ρ‚ΠΎ Π² любой Ρ‚ΠΎΡ‡ΠΊΠ΅ Π·Π°ΠΌΠΊΠ½ΡƒΡ‚ΠΎΠΉ ΠΏΠΎΡΠ»Π΅Π΄ΠΎΠ²Π°Ρ‚Π΅Π»ΡŒΠ½ΠΎΠΉ Ρ†Π΅ΠΏΠΈ ΠΏΠ°Π΄Π΅Π½ΠΈΠ΅ напряТСния Π½Π° ΠΊΠ°ΠΆΠ΄ΠΎΠΌ рСзисторС Π΄ΠΎΠ»ΠΆΠ½ΠΎ Ρ€Π°Π²Π½ΡΡ‚ΡŒΡΡ ΠΎΠ±Ρ‰Π΅ΠΌΡƒ Π½Π°ΠΏΡ€ΡΠΆΠ΅Π½ΠΈΡŽ Ρ†Π΅ΠΏΠΈ. ΠŸΠΎΡΠΊΠΎΠ»ΡŒΠΊΡƒ Ρ‚ΠΎΠΊ Π² ΠΏΠΎΡΠ»Π΅Π΄ΠΎΠ²Π°Ρ‚Π΅Π»ΡŒΠ½ΠΎΠΉ Ρ†Π΅ΠΏΠΈ постоянный, ΠΏΠ°Π΄Π΅Π½ΠΈΠ΅ напряТСния Π΄ΠΎΠ»ΠΆΠ½ΠΎ Ρ€Π°Π·Π»ΠΈΡ‡Π°Ρ‚ΡŒΡΡ Π½Π° ΠΊΠ°ΠΆΠ΄ΠΎΠΌ рСзисторС.

ΠŸΠ°Ρ€Π°Π»Π»Π΅Π»ΡŒΠ½Ρ‹Π΅ ΠΈ ΠΏΠΎΡΠ»Π΅Π΄ΠΎΠ²Π°Ρ‚Π΅Π»ΡŒΠ½Ρ‹Π΅ схСмы

Π’ ΠΏΠ°Ρ€Π°Π»Π»Π΅Π»ΡŒΠ½ΠΎΠΉ схСмС всС ΠΊΠΎΠΌΠΏΠΎΠ½Π΅Π½Ρ‚Ρ‹ схСмы ΠΏΠΎΠ΄ΠΊΠ»ΡŽΡ‡Π°ΡŽΡ‚ΡΡ ΠΌΠ΅ΠΆΠ΄Ρƒ ΠΎΠ΄Π½ΠΈΠΌΠΈ ΠΈ Ρ‚Π΅ΠΌΠΈ ΠΆΠ΅ Ρ‚ΠΎΡ‡ΠΊΠ°ΠΌΠΈ Π½Π° схСмС.Π­Ρ‚ΠΎ Π΄Π°Π΅Ρ‚ ΠΈΠΌ ΠΈΡ… Ρ€Π°Π·Π²Π΅Ρ‚Π²Π»Π΅Π½Π½ΡƒΡŽ структуру, Π² ΠΊΠΎΡ‚ΠΎΡ€ΠΎΠΉ Ρ‚ΠΎΠΊ раздСляСтся ΠΌΠ΅ΠΆΠ΄Ρƒ ΠΊΠ°ΠΆΠ΄ΠΎΠΉ Π²Π΅Ρ‚Π²ΡŒΡŽ, Π½ΠΎ ΠΏΠ°Π΄Π΅Π½ΠΈΠ΅ напряТСния Π½Π° ΠΊΠ°ΠΆΠ΄ΠΎΠΉ Π²Π΅Ρ‚Π²ΠΈ остаСтся Π½Π΅ΠΈΠ·ΠΌΠ΅Π½Π½Ρ‹ΠΌ. Π‘ΡƒΠΌΠΌΠ° ΠΊΠ°ΠΆΠ΄ΠΎΠ³ΠΎ рСзистора Π΄Π°Π΅Ρ‚ ΠΎΠ±Ρ‰Π΅Π΅ сопротивлСниС, ΠΎΠ±Ρ€Π°Ρ‚Π½ΠΎΠ΅ ΠΊΠ°ΠΆΠ΄ΠΎΠΌΡƒ ΡΠΎΠΏΡ€ΠΎΡ‚ΠΈΠ²Π»Π΅Π½ΠΈΡŽ ( 1 / R всСго = 1 / R 1 + 1 / R 2 для ΠΊΠ°ΠΆΠ΄ΠΎΠ³ΠΎ рСзистора).

Π’ ΠΏΠΎΡΠ»Π΅Π΄ΠΎΠ²Π°Ρ‚Π΅Π»ΡŒΠ½ΠΎΠΉ Ρ†Π΅ΠΏΠΈ, Π½Π°ΠΏΡ€ΠΎΡ‚ΠΈΠ², Π΅ΡΡ‚ΡŒ Ρ‚ΠΎΠ»ΡŒΠΊΠΎ ΠΎΠ΄ΠΈΠ½ ΠΏΡƒΡ‚ΡŒ для прохоТдСния Ρ‚ΠΎΠΊΠ°. Π­Ρ‚ΠΎ ΠΎΠ·Π½Π°Ρ‡Π°Π΅Ρ‚, Ρ‡Ρ‚ΠΎ Ρ‚ΠΎΠΊ остаСтся постоянным Π½Π° всСм протяТСнии, Π° ΠΏΠ°Π΄Π΅Π½ΠΈΠ΅ напряТСния Π½Π° ΠΊΠ°ΠΆΠ΄ΠΎΠΌ рСзисторС отличаСтся.Π‘ΡƒΠΌΠΌΠ° ΠΊΠ°ΠΆΠ΄ΠΎΠ³ΠΎ рСзистора Π΄Π°Π΅Ρ‚ ΠΎΠ±Ρ‰Π΅Π΅ сопротивлСниС ΠΏΡ€ΠΈ Π»ΠΈΠ½Π΅ΠΉΠ½ΠΎΠΌ суммировании ( R всСго = R 1 + R 2 для ΠΊΠ°ΠΆΠ΄ΠΎΠ³ΠΎ рСзистора).

ΠŸΠΎΡΠ»Π΅Π΄ΠΎΠ²Π°Ρ‚Π΅Π»ΡŒΠ½ΠΎ-ΠΏΠ°Ρ€Π°Π»Π»Π΅Π»ΡŒΠ½Ρ‹Π΅ Ρ†Π΅ΠΏΠΈ

Π’Ρ‹ ΠΌΠΎΠΆΠ΅Ρ‚Π΅ ΠΈΡΠΏΠΎΠ»ΡŒΠ·ΠΎΠ²Π°Ρ‚ΡŒ ΠΎΠ±Π° Π·Π°ΠΊΠΎΠ½Π° ΠšΠΈΡ€Ρ…Π³ΠΎΡ„Π° для любой Ρ‚ΠΎΡ‡ΠΊΠΈ ΠΈΠ»ΠΈ ΠΏΠ΅Ρ‚Π»ΠΈ Π² любой Ρ†Π΅ΠΏΠΈ ΠΈ ΠΏΡ€ΠΈΠΌΠ΅Π½ΡΡ‚ΡŒ ΠΈΡ… для опрСдСлСния напряТСния ΠΈ Ρ‚ΠΎΠΊΠ°. Π—Π°ΠΊΠΎΠ½Ρ‹ ΠšΠΈΡ€Ρ…Π³ΠΎΡ„Π° Π΄Π°ΡŽΡ‚ Π²Π°ΠΌ ΠΌΠ΅Ρ‚ΠΎΠ΄ опрСдСлСния Ρ‚ΠΎΠΊΠ° ΠΈ напряТСния Π² ситуациях, ΠΊΠΎΠ³Π΄Π° ΠΏΡ€ΠΈΡ€ΠΎΠ΄Π° Ρ†Π΅ΠΏΠΈ ΠΊΠ°ΠΊ ΠΏΠΎΡΠ»Π΅Π΄ΠΎΠ²Π°Ρ‚Π΅Π»ΡŒΠ½ΠΎΠΉ ΠΈ ΠΏΠ°Ρ€Π°Π»Π»Π΅Π»ΡŒΠ½ΠΎΠΉ ΠΌΠΎΠΆΠ΅Ρ‚ Π±Ρ‹Ρ‚ΡŒ Π½Π΅ Ρ‚Π°ΠΊΠΎΠΉ простой.

Как ΠΏΡ€Π°Π²ΠΈΠ»ΠΎ, для Ρ†Π΅ΠΏΠ΅ΠΉ, ΠΊΠΎΡ‚ΠΎΡ€Ρ‹Π΅ ΠΈΠΌΠ΅ΡŽΡ‚ ΠΊΠ°ΠΊ ΠΏΠΎΡΠ»Π΅Π΄ΠΎΠ²Π°Ρ‚Π΅Π»ΡŒΠ½Ρ‹Π΅, Ρ‚Π°ΠΊ ΠΈ ΠΏΠ°Ρ€Π°Π»Π»Π΅Π»ΡŒΠ½Ρ‹Π΅ ΠΊΠΎΠΌΠΏΠΎΠ½Π΅Π½Ρ‚Ρ‹, Π²Ρ‹ ΠΌΠΎΠΆΠ΅Ρ‚Π΅ Ρ€Π°ΡΡΠΌΠ°Ρ‚Ρ€ΠΈΠ²Π°Ρ‚ΡŒ ΠΎΡ‚Π΄Π΅Π»ΡŒΠ½Ρ‹Π΅ части схСмы ΠΊΠ°ΠΊ ΠΏΠΎΡΠ»Π΅Π΄ΠΎΠ²Π°Ρ‚Π΅Π»ΡŒΠ½Ρ‹Π΅ ΠΈΠ»ΠΈ ΠΏΠ°Ρ€Π°Π»Π»Π΅Π»ΡŒΠ½Ρ‹Π΅ ΠΈ соотвСтствСнно ΠΊΠΎΠΌΠ±ΠΈΠ½ΠΈΡ€ΠΎΠ²Π°Ρ‚ΡŒ ΠΈΡ….

Π­Ρ‚ΠΈ слоТныС ΠΏΠΎΡΠ»Π΅Π΄ΠΎΠ²Π°Ρ‚Π΅Π»ΡŒΠ½ΠΎ-ΠΏΠ°Ρ€Π°Π»Π»Π΅Π»ΡŒΠ½Ρ‹Π΅ схСмы ΠΌΠΎΠΆΠ½ΠΎ Ρ€Π΅ΡˆΠΈΡ‚ΡŒ нСсколькими способами. Один ΠΈΠ· ΠΌΠ΅Ρ‚ΠΎΠ΄ΠΎΠ² — Ρ€Π°ΡΡΠΌΠ°Ρ‚Ρ€ΠΈΠ²Π°Ρ‚ΡŒ ΠΈΡ… части ΠΊΠ°ΠΊ ΠΏΠ°Ρ€Π°Π»Π»Π΅Π»ΡŒΠ½Ρ‹Π΅ ΠΈΠ»ΠΈ ΠΏΠΎΡΠ»Π΅Π΄ΠΎΠ²Π°Ρ‚Π΅Π»ΡŒΠ½Ρ‹Π΅. Π”Ρ€ΡƒΠ³ΠΎΠΉ ΠΌΠ΅Ρ‚ΠΎΠ΄ — использованиС Π·Π°ΠΊΠΎΠ½ΠΎΠ² ΠšΠΈΡ€Ρ…Π³ΠΎΡ„Π° для опрСдСлСния ΠΎΠ±ΠΎΠ±Ρ‰Π΅Π½Π½Ρ‹Ρ… Ρ€Π΅ΡˆΠ΅Π½ΠΈΠΉ, ΠΈΡΠΏΠΎΠ»ΡŒΠ·ΡƒΡŽΡ‰ΠΈΡ… систСму ΡƒΡ€Π°Π²Π½Π΅Π½ΠΈΠΉ. ΠšΠ°Π»ΡŒΠΊΡƒΠ»ΡΡ‚ΠΎΡ€ ΠΏΠΎΡΠ»Π΅Π΄ΠΎΠ²Π°Ρ‚Π΅Π»ΡŒΠ½ΠΎ-ΠΏΠ°Ρ€Π°Π»Π»Π΅Π»ΡŒΠ½Ρ‹Ρ… Ρ†Π΅ΠΏΠ΅ΠΉ ΡƒΡ‡ΠΈΡ‚Ρ‹Π²Π°Π΅Ρ‚ Ρ€Π°Π·Π»ΠΈΡ‡Π½Ρ‹ΠΉ Ρ…Π°Ρ€Π°ΠΊΡ‚Π΅Ρ€ Ρ†Π΅ΠΏΠ΅ΠΉ.

β€’β€’β€’ Syed Hussain Ather

Π’ ΠΏΡ€ΠΈΠ²Π΅Π΄Π΅Π½Π½ΠΎΠΌ Π²Ρ‹ΡˆΠ΅ ΠΏΡ€ΠΈΠΌΠ΅Ρ€Π΅ тСкущая Ρ‚ΠΎΡ‡ΠΊΠ° Π²Ρ‹Ρ…ΠΎΠ΄Π° A Π΄ΠΎΠ»ΠΆΠ½Π° Ρ€Π°Π²Π½ΡΡ‚ΡŒΡΡ Ρ‚Π΅ΠΊΡƒΡ‰Π΅ΠΉ Ρ‚ΠΎΡ‡ΠΊΠ΅ Π²Ρ‹Ρ…ΠΎΠ΄Π° A. Π­Ρ‚ΠΎ ΠΎΠ·Π½Π°Ρ‡Π°Π΅Ρ‚, Ρ‡Ρ‚ΠΎ Π²Ρ‹ ΠΌΠΎΠΆΠ΅Ρ‚Π΅ Π½Π°ΠΏΠΈΡΠ°Ρ‚ΡŒ:

(1). I_1 = I_2 + I_3 \ text {ΠΈΠ»ΠΈ} I_1-I_2-I_3 = 0

Если Ρ€Π°ΡΡΠΌΠ°Ρ‚Ρ€ΠΈΠ²Π°Ρ‚ΡŒ Π²Π΅Ρ€Ρ…Π½ΠΈΠΉ ΠΊΠΎΠ½Ρ‚ΡƒΡ€ ΠΊΠ°ΠΊ Π·Π°ΠΌΠΊΠ½ΡƒΡ‚ΡƒΡŽ ΠΏΠΎΡΠ»Π΅Π΄ΠΎΠ²Π°Ρ‚Π΅Π»ΡŒΠ½ΡƒΡŽ Ρ†Π΅ΠΏΡŒ ΠΈ Ρ€Π°ΡΡΠΌΠ°Ρ‚Ρ€ΠΈΠ²Π°Ρ‚ΡŒ ΠΏΠ°Π΄Π΅Π½ΠΈΠ΅ напряТСния Π½Π° ΠΊΠ°ΠΆΠ΄ΠΎΠΌ рСзисторС, ΠΈΡΠΏΠΎΠ»ΡŒΠ·ΡƒΡ Π·Π°ΠΊΠΎΠ½ Ома с ΡΠΎΠΎΡ‚Π²Π΅Ρ‚ΡΡ‚Π²ΡƒΡŽΡ‰ΠΈΠΌ сопротивлСниСм, Π²Ρ‹ ΠΌΠΎΠΆΠ΅Ρ‚Π΅ Π½Π°ΠΏΠΈΡΠ°Ρ‚ΡŒ:

(2). V_1-R_1I_1-R_2I_2 = 0

ΠΈ, ΠΏΡ€ΠΎΠ΄Π΅Π»Π°Π² Ρ‚ΠΎ ΠΆΠ΅ самоС для Π½ΠΈΠΆΠ½Π΅Π³ΠΎ ΠΊΠΎΠ½Ρ‚ΡƒΡ€Π°, Π²Ρ‹ ΠΌΠΎΠΆΠ΅Ρ‚Π΅ ΠΎΠ±Ρ€Π°Π±Π°Ρ‚Ρ‹Π²Π°Ρ‚ΡŒ ΠΊΠ°ΠΆΠ΄ΠΎΠ΅ ΠΏΠ°Π΄Π΅Π½ΠΈΠ΅ напряТСния Π² Π½Π°ΠΏΡ€Π°Π²Π»Π΅Π½ΠΈΠΈ Ρ‚ΠΎΠΊΠ° Π² зависимости ΠΎΡ‚ Ρ‚ΠΎΠΊΠ° ΠΈ сопротивлСния, Ρ‡Ρ‚ΠΎΠ±Ρ‹ Π·Π°ΠΏΠΈΡΠ°Ρ‚ΡŒ:

(3).V_1 + V_2 + R_3I_3-R_2I_2 = 0

Π­Ρ‚ΠΎ Π΄Π°Π΅Ρ‚ Π²Π°ΠΌ Ρ‚Ρ€ΠΈ уравнСния, ΠΊΠΎΡ‚ΠΎΡ€Ρ‹Π΅ ΠΌΠΎΠΆΠ½ΠΎ Ρ€Π΅ΡˆΠΈΡ‚ΡŒ нСсколькими способами. Π’Ρ‹ ΠΌΠΎΠΆΠ΅Ρ‚Π΅ ΠΏΠ΅Ρ€Π΅ΠΏΠΈΡΠ°Ρ‚ΡŒ ΠΊΠ°ΠΆΠ΄ΠΎΠ΅ ΠΈΠ· ΡƒΡ€Π°Π²Π½Π΅Π½ΠΈΠΉ (1) — (3) Ρ‚Π°ΠΊ, Ρ‡Ρ‚ΠΎΠ±Ρ‹ напряТСниС Π±Ρ‹Π»ΠΎ с ΠΎΠ΄Π½ΠΎΠΉ стороны, Π° Ρ‚ΠΎΠΊ ΠΈ сопротивлСниС — с Π΄Ρ€ΡƒΠ³ΠΎΠΉ. Π’Π°ΠΊΠΈΠΌ ΠΎΠ±Ρ€Π°Π·ΠΎΠΌ, Π²Ρ‹ ΠΌΠΎΠΆΠ΅Ρ‚Π΅ Ρ€Π°ΡΡΠΌΠ°Ρ‚Ρ€ΠΈΠ²Π°Ρ‚ΡŒ Ρ‚Ρ€ΠΈ уравнСния ΠΊΠ°ΠΊ зависимыС ΠΎΡ‚ Ρ‚Ρ€Π΅Ρ… ΠΏΠ΅Ρ€Π΅ΠΌΠ΅Π½Π½Ρ‹Ρ… I 1 , I 2 ΠΈ I 3 с коэффициСнтами ΠΊΠΎΠΌΠ±ΠΈΠ½Π°Ρ†ΠΈΠΉ R 1 , R 2 ΠΈ R 3 .

\ begin {align} & (1).I_1-I_2-I_3 = 0 \\ & (2). R_1I_1 + R_2I_2 + 0 \ times I_3 = V_1 \\ & (3). 0 \ times I_1 + R_2I_2-R_3I_3 = V_1 + V_2 \ end {Π²Ρ‹Ρ€ΠΎΠ²Π½Π΅Π½ΠΎ}

Π­Ρ‚ΠΈ Ρ‚Ρ€ΠΈ уравнСния Π΄Π΅ΠΌΠΎΠ½ΡΡ‚Ρ€ΠΈΡ€ΡƒΡŽΡ‚, ΠΊΠ°ΠΊ напряТСниС Π² ΠΊΠ°ΠΆΠ΄ΠΎΠΉ Ρ‚ΠΎΡ‡ΠΊΠ΅ Ρ†Π΅ΠΏΠΈ ΠΊΠ°ΠΊΠΈΠΌ-Ρ‚ΠΎ ΠΎΠ±Ρ€Π°Π·ΠΎΠΌ зависит ΠΎΡ‚ Ρ‚ΠΎΠΊΠ° ΠΈ сопротивлСния. Если Π²Ρ‹ ΠΏΠΎΠΌΠ½ΠΈΡ‚Π΅ Π·Π°ΠΊΠΎΠ½Ρ‹ ΠšΠΈΡ€Ρ…Π³ΠΎΡ„Π°, Π²Ρ‹ ΠΌΠΎΠΆΠ΅Ρ‚Π΅ ΡΠΎΠ·Π΄Π°Ρ‚ΡŒ эти ΠΎΠ±ΠΎΠ±Ρ‰Π΅Π½Π½Ρ‹Π΅ Ρ€Π΅ΡˆΠ΅Π½ΠΈΡ схСмных Π·Π°Π΄Π°Ρ‡ ΠΈ ΠΈΡΠΏΠΎΠ»ΡŒΠ·ΠΎΠ²Π°Ρ‚ΡŒ ΠΌΠ°Ρ‚Ρ€ΠΈΡ‡Π½ΡƒΡŽ Π½ΠΎΡ‚Π°Ρ†ΠΈΡŽ для ΠΈΡ… Ρ€Π΅ΡˆΠ΅Π½ΠΈΡ. Π’Π°ΠΊΠΈΠΌ ΠΎΠ±Ρ€Π°Π·ΠΎΠΌ, Π²Ρ‹ ΠΌΠΎΠΆΠ΅Ρ‚Π΅ ΠΏΠΎΠ΄ΡΡ‚Π°Π²ΠΈΡ‚ΡŒ значСния для Π΄Π²ΡƒΡ… Π²Π΅Π»ΠΈΡ‡ΠΈΠ½ (срСди ΠΊΠΎΡ‚ΠΎΡ€Ρ‹Ρ… напряТСниС, Ρ‚ΠΎΠΊ, сопротивлСниС), Ρ‡Ρ‚ΠΎΠ±Ρ‹ Π½Π°ΠΉΡ‚ΠΈ Ρ‚Ρ€Π΅Ρ‚ΡŒΡŽ.

ΠžΠ±ΡŠΡΡΠ½ΠΈΡ‚Π΅ ΠΈ Π²Ρ‹Π²Π΅Π΄ΠΈΡ‚Π΅ ΡΠΊΠ²ΠΈΠ²Π°Π»Π΅Π½Ρ‚Π½ΡƒΡŽ Ρ„ΠΎΡ€ΠΌΡƒΠ»Ρƒ сопротивлСния для класса 12 ΠΏΠΎ Ρ„ΠΈΠ·ΠΈΠΊΠ΅ CBSE

Подсказка: Когда Ρ‚Ρ€ΠΈ рСзистора скрыты Π² Π²Ρ‹Π²ΠΎΠ΄Π΅, Ρ‚ΠΎ ΠΎΠ΄ΠΈΠ½Π°ΠΊΠΎΠ²Ρ‹ΠΉ Ρ‚ΠΎΠΊ ΠΏΡ€ΠΎΡ…ΠΎΠ΄ΠΈΡ‚ Ρ‡Π΅Ρ€Π΅Π· ΠΊΠ°ΠΆΠ΄Ρ‹ΠΉ рСзистор, Π½ΠΎ ΠΏΠ°Π΄Π΅Π½ΠΈΠ΅ напряТСния Π½Π° ΠΊΠ°ΠΆΠ΄ΠΎΠΌ рСзисторС Ρ€Π°Π·Π½ΠΎΠ΅.

ПолноС пошаговоС Ρ€Π΅ΡˆΠ΅Π½ΠΈΠ΅:
По Ρ„ΠΎΡ€ΠΌΡƒΠ»Π΅ Ома V = IR.
Когда Ρ‚Ρ€ΠΈ рСзистора соСдинСны ΠΏΠΎΡΠ»Π΅Π΄ΠΎΠ²Π°Ρ‚Π΅Π»ΡŒΠ½ΠΎ, Ρ‚ΠΎ ΠΎΠ΄ΠΈΠ½Π°ΠΊΠΎΠ²Ρ‹ΠΉ Ρ‚ΠΎΠΊ ΠΏΡ€ΠΎΡ…ΠΎΠ΄ΠΈΡ‚ Ρ‡Π΅Ρ€Π΅Π· ΠΊΠ°ΠΆΠ΄Ρ‹ΠΉ рСзистор, Π½ΠΎ ΠΏΠ°Π΄Π΅Π½ΠΈΠ΅ напряТСния Π½Π° ΠΊΠ°ΠΆΠ΄ΠΎΠΌ рСзисторС Ρ€Π°Π·Π½ΠΎΠ΅.Если V — ΠΏΡ€ΠΈΠ»ΠΎΠΆΠ΅Π½Π½ΠΎΠ΅ напряТСниС, Π° \ [{V_1}, \, \, \, {V_2} \] ΠΈ \ [{V_3} \] — напряТСниС Π½Π° сопротивлСнии \ [{R_1}, \, \, {R_2} \ , \, ΠΈ \, \, {R_3} \] соотвСтствСнно.
\ [V = \, \, {V_1} \, \, + \, \, {V_2} \, \, + \, \, {V_3} \]… (1)

По Π·Π°ΠΊΠΎΠ½Ρƒ Ома
Π’ = IR
\ [\ поэтому \] \ [I {R_ {eq}} \, \, = \, \, I {R_1} + I {R_2} + I {R_3} \]
\ [I {R_ { eq}} \, \, = \, \, I \ left ({{R_1} + {R_2} + {R_3}} \ right) \]
\ [{R_ {eq}} \, \, = \, \, {R_1} + {R_2} + {R_3} \]

Π’Π°ΠΊΠΈΠΌ ΠΎΠ±Ρ€Π°Π·ΠΎΠΌ, эквивалСнтноС сопротивлСниС ΠΈΠ»ΠΈ ΠΏΠΎΠ»Π½ΠΎΠ΅ сопротивлСниС Ρ†Π΅ΠΏΠΈ ΠΌΠΎΠΆΠ½ΠΎ ΠΎΠΏΡ€Π΅Π΄Π΅Π»ΠΈΡ‚ΡŒ ΠΊΠ°ΠΊ ΠΎΠ΄Π½ΠΎ Π·Π½Π°Ρ‡Π΅Π½ΠΈΠ΅ сопротивлСния, ΠΊΠΎΡ‚ΠΎΡ€ΠΎΠ΅ ΠΌΠΎΠΆΠ΅Ρ‚ Π·Π°ΠΌΠ΅Π½ΠΈΡ‚ΡŒ любоС количСство рСзисторов, соСдинСнных ΠΏΠΎΡΠ»Π΅Π΄ΠΎΠ²Π°Ρ‚Π΅Π»ΡŒΠ½ΠΎ, Π±Π΅Π· измСнСния Π·Π½Π°Ρ‡Π΅Π½ΠΈΠ΅ Ρ‚ΠΎΠΊΠ° ΠΈΠ»ΠΈ напряТСния Ρ†Π΅ΠΏΠΈ.
Если Ρƒ нас Π΅ΡΡ‚ΡŒ n ΠΏΠΎΡΠ»Π΅Π΄ΠΎΠ²Π°Ρ‚Π΅Π»ΡŒΠ½ΠΎ соСдинСнных сопротивлСний, Ρ‚ΠΎ обобщСнная Ρ„ΠΎΡ€ΠΌΡƒΠ»Π° эквивалСнтного сопротивлСния Π±ΡƒΠ΄Π΅Ρ‚:
\ [{R_ {eq}} \, \, = \, \, {R_1} + {R_2} + {R_3} …. … + {R_n} \]


ΠœΡ‹ Π·Π½Π°Π΅ΠΌ, Ρ‡Ρ‚ΠΎ ΠΏΡ€ΠΈΠ»ΠΎΠΆΠ΅Π½Π½ΠΎΠ΅ напряТСниС Ρ€Π°Π²Π½ΠΎ (v)
\ [\ поэтому \, \, V = {V_1} + {V_2} + {V_3} \]
ΠœΡ‹ Ρ‚Π°ΠΊΠΆΠ΅ Π·Π½Π°Π΅ΠΌ, Ρ‡Ρ‚ΠΎ
V = IR (из закона Ома)
\ [\ поэтому \, I \, {R_ {eq}} = I {R_1} + I {R_2} + I {R_1} \]
\ [\ Rightarrow I \, {R_ {eq}} = I \ left ({{R_1} + {R_2} + {R_3}} \ right) \]
\ [\ Rightarrow {R_1} + {R_2} + {R_3} \]
Π˜Ρ‚Π°ΠΊ, Π­ΠΊΠ²ΠΈΠ²Π°Π»Π΅Π½Ρ‚Π½ΠΎΠ΅ сопротивлСниС ΠΈΠ»ΠΈ ΠΏΠΎΠ»Π½ΠΎΠ΅ сопротивлСниС Ρ†Π΅ΠΏΠΈ ΠΌΠΎΠΆΠ½ΠΎ ΠΎΠΏΡ€Π΅Π΄Π΅Π»ΠΈΡ‚ΡŒ ΠΊΠ°ΠΊ ΠΎΠ΄Π½ΠΎ Π·Π½Π°Ρ‡Π΅Π½ΠΈΠ΅ рСзистора, ΠΏΠΎΠ΄ΠΊΠ»ΡŽΡ‡Π΅Π½Π½ΠΎΠ³ΠΎ ΠΏΠΎΡΠ»Π΅Π΄ΠΎΠ²Π°Ρ‚Π΅Π»ΡŒΠ½ΠΎ, с ΠΈΠ·ΠΌΠ΅Π½Π΅Π½ΠΈΠ΅ΠΌ Π·Π½Π°Ρ‡Π΅Π½ΠΈΠΉ Ρ‚ΠΎΠΊΠ° ΠΈΠ»ΠΈ напряТСния Π² Ρ†Π΅ΠΏΠΈ.

Π Π°Π·Π½ΠΎΠ΅

Π”ΠΎΠ±Π°Π²ΠΈΡ‚ΡŒ ΠΊΠΎΠΌΠΌΠ΅Π½Ρ‚Π°Ρ€ΠΈΠΉ

Π’Π°Ρˆ адрСс email Π½Π΅ Π±ΡƒΠ΄Π΅Ρ‚ ΠΎΠΏΡƒΠ±Π»ΠΈΠΊΠΎΠ²Π°Π½. ΠžΠ±ΡΠ·Π°Ρ‚Π΅Π»ΡŒΠ½Ρ‹Π΅ поля ΠΏΠΎΠΌΠ΅Ρ‡Π΅Π½Ρ‹ *